Sie sind auf Seite 1von 406

13/07/2015

Reference ranges

Question 1 of 343

Next

A 40 year old women presented with a mass on her forehead. On examination, she had a
fluctuant pulsatile mass on her head. Examination of her neck revealed a mass inferior to the
hyoid with a positive Berry's sign. What is the most likely underlying diagnosis?

Question stats

End and review

Score: 0%

39.3%

17.5%

16.6%

16.2%

10.5%

A. Follicular thyroid cancer


39.3% of users answered this
question correctly

B. Medullary thyroid cancer


C. Papillary thyroid cancer
D. Anaplastic thyroid cancer

Search

E. Parathyroid gland cancer

Go

Next question

Berrys sign= Absence of carotid pulse due to malignant thyromegaly.

Theme from April 2013 Exam


Papillary thyroid cancers will tend to spread via lymphatics and present with disease that is
nearly always confined to the neck. Follicular carcinomas may metastasise haematogenously
and the skull may be the presenting site of disease in between 2 and 8% of patients.
Thyroid malignancy
Papillary carcinoma
Commonest sub-type
Accurately diagnosed on fine needle aspiration cytology
Histologically, they may demonstrate psammoma bodies (areas of calcification) and so
called 'orphan Annie' nuclei
They typically metastasise via the lymphatics and thus laterally located apparently
ectopic thyroid tissue is usually a metastasis from a well differentiated papillary
carcinoma

Follicular carcinoma
Are less common than papillary lesions
Like papillary tumours, they may present as a discrete nodule. Although they appear
to be well encapsulated macroscopically there is invasion on microscopic evaluation
Lymph node metastases are uncommon and these tumours tend to spread
haematogenously. This translates into a higher mortality rate
Follicular lesions cannot be accurately diagnosed on fine needle aspiration cytology
and thus all follicular FNA's (THY 3) will require at least a hemi thyroidectomy

Anaplastic carcinoma
Less common and tend to occur in elderly females
Disease is usually advanced at presentation and often only palliative decompression
and radiotherapy can be offered.

Medullary carcinoma
These are tumours of the parafollicular cells ( C Cells) and are of neural crest origin.
The serum calcitonin may be elevated which is of use when monitoring for recurrence.
They may be familial and occur as part of the MEN -2A disease spectrum.
Spread may be either lymphatic or haematogenous and as these tumours are not
derived primarily from thyroid cells they are not responsive to radioiodine.

Lymphoma
These respond well to radiotherapy
Radical surgery is unnecessary once the disease has been diagnosed on biopsy
material. Such biopsy material is not generated by an FNA and thus a core biopsy has
to be obtained (with care!).

Rate question:

Next question

Comment on this question

http://www.emrcs.com/question/question.php?q=0

1/2

13/07/2015
Comment on this question

All contents of this site are 2012 E-Medical Revision Ltd

http://www.emrcs.com/question/question.php?q=0

Terms and Conditions

Privacy policy

2/2

13/07/2015

Reference ranges

Previous

Question 2 of 343

Next

What is the most common cause of mesenteric infarction?

Question stats

Score: 0%

18.9%

49.8%

17.7%

A. Mesenteric vein thrombosis

5.9%

B. Acute embolism affecting the superior mesenteric artery

7.8%

C. Acute on chronic thrombus of the superior mesenteric artery

49.8% of users answered this


question correctly

D. Sub intimal dissection of the superior mesenteric artery

End and review

1
2

E. Proximal migration of abdominal aortic aneurysm


Search
Next question
Go

Theme from January 2013 Exam


Acute embolic events account for up to 50% of cases of mesenteric infarcts. These may
occur as a result of long standing atrial fibrillation, ventricular anuerysms and post
myocardial infarction.
Mesenteric vessel disease
Mesenteric ischaemia accounts for 1 in 1000 acute surgical admissions. It is primarily caused
by arterial embolism resulting in infarction of the colon. It is more likely to occur in areas such
as the splenic flexure that are located at the borders of the territory supplied by the superior
and inferior mesenteric arteries.
Types
Acute
mesenteric
embolus
(commonest
50%)

Acute on
chronic
mesenteric
ischaemia

Mesenteric
vein
thrombosis

Low flow
mesenteric
infarction

Sudden onset abdominal pain followed by profuse diarrhoea.


May be associated with vomiting.
Rapid clinical deterioration.
Serological tests: WCC, lactate, amylase may all be abnormal
particularly in established disease. These can be normal in the early
phases.

Usually longer prodromal history.


Post prandial abdominal discomfort and weight loss are dominant
features. Patients will usually present with an acute on chronic event,
but otherwise will tend not to present until mesenteric flow is reduced by
greater than 80%.
When acute thrombosis occurs presentation may be as above. In the
chronic setting the symptoms will often be those of ischaemic colitis
(mucosa is the most sensitive area to this insult).

Usually a history over weeks.


Overt abdominal signs and symptoms will not occur until venous
thrombosis has reached a stage to compromise arterial inflow.
Thrombophilia accounts for 60% of cases.

This occurs in patients with multiple co morbidities in whom mesenteric


perfusion is significantly compromised by overuse of inotropes or
background cardiovascular compromise.
The end result is that the bowel is not adequately perfused and infarcts
occur from the mucosa outwards.

Diagnosis
Serological tests: WCC, lactate, CRP, amylase (can be normal in early disease).
Cornerstone for diagnosis of arterial AND venous mesenteric disease is CT
angiography scanning in the arterial phase with thin slices (<5mm). Venous phase
contrast is not helpful.
SMA duplex USS is useful in the evaluation of proximal SMA disease in patients with
chronic mesenteric ischaemia.
MRI is of limited use due to gut peristalsis and movement artefact.

Management
Overt signs of peritonism: Laparotomy
Mesenteric vein thrombosis: If no peritonism: Medical management with IV heparin
At operation limited resection of frankly necrotic bowel with view to relook laparotomy
at 24-48h. In the interim urgent bowel revascularisation via endovascular (preferred)

http://www.emrcs.com/question/question.php?q=0

1/2

13/07/2015

or surgery.

Prognosis
Overall poor. Best outlook is from an acute ischaemia from an embolic event where surgery
occurs within 12h. Survival may be 50%. This falls to 30% with treatment delay. The other
conditions carry worse survival figures.
Rate question:

Next question

Comment on this question

All contents of this site are 2012 E-Medical Revision Ltd

http://www.emrcs.com/question/question.php?q=0

Terms and Conditions

Privacy policy

2/2

13/07/2015

Reference ranges

Previous

Question 3 of 343

Next

A 38 year old lady presents with a recent episode of renal colic. As part of her investigations
the following results are obtained:

Question stats

Score: 33.3%
8%

18.6%

2
3

56.5%

10.2%

PTH

6.7%

88pg/ml (increased)

Corrected Calcium 3.84 mmol/l

Her serum urea and electrolytes are normal.


What is the most likely diagnosis?

End and review

56.5% of users answered this


question correctly

A. Carcinoma of the bronchus


Search

B. Secondary hyperparathyroidism
C. Primary hyperparathyroidism

Go

D. Tertiary hyperparathyroidism
E. Carcinoma of the breast
Next question

Theme from September 2012 exam


Theme from September 2011 exam
In this situation the most likely diagnosis is primary hyperparathyroidism. The question
mentions that serum urea and electrolytes are normal, which makes tertiary
hyperparathyroidism unlikely.
Primary hyperparathyroidism
In exams primary hyperparathyroidism is stereotypically seen in elderly females with an
unquenchable thirst and an inappropriately normal or raised parathyroid hormone level. It is
most commonly due to a solitary adenoma
Causes of primary hyperparathyroidism
80%: solitary adenoma
15%: hyperplasia
4%: multiple adenoma
1%: carcinoma

Features - 'bones, stones, abdominal groans and psychic moans'


Polydipsia, polyuria
Peptic ulceration/constipation/pancreatitis
Bone pain/fracture
Renal stones
Depression
Hypertension

Associations
Hypertension
Multiple endocrine neoplasia: MEN I and II

Investigations
Raised calcium, low phosphate
PTH may be raised or normal
Technetium-MIBI subtraction scan

Treatment
Parathyroidectomy, if imaging suggests target gland then a focused approach may be
used

Rate question:

Next question

Comment on this question

All contents of this site are 2012 E-Medical Revision Ltd

http://www.emrcs.com/question/question.php?q=0

Terms and Conditions

Privacy policy

1/2

13/07/2015

http://www.emrcs.com/question/question.php?q=0

2/2

13/07/2015

Reference ranges

Previous0 / 3

Question 4-6 of 343

Next

Question stats

End and review

Score: 16.7%

Average score for registered users:

1
2

Theme: Head and neck lumps


A. Branchial cyst

42%

76.2%

38.3%

3
4-6 0 / 3

B. Cystic hygroma
C. Carotid body tumour
D. Lymphadenopathy

Search

E. Adenolymphoma of the parotid

Go

F. Pleomorphic adenoma of the parotid


G. Submandibular tumour
H. Thyroglossal cyst
I. Thoracic outlet syndrome
J. Submandibular gland calculus
Please select the most likely lesion to account for the clinical scenario given. Each option
may be used once, more than once or not at all.

4.

A 60 year old Tibetan immigrant is referred to the surgical clinic with a painless
neck swelling. On examination, it is located on the left side immediately posterior
to the sternocleidomastoid muscle. There are no other abnormalities to find.
You answered Branchial cyst
The correct answer is Carotid body tumour
Carotid body tumours typically present as painless masses. They may compress
the vagus or hypoglossal nerves with symptoms attributable to these structures.
Over 90% occur spontaneously and are more common in people living at high
altitude. In familial cases up to 30% may be bilateral. Treatment is with excision.

5.

A 40 year old women presents as an emergency with a painful mass underneath


her right mandible. The mass has appeared over the previous week with the
pain worsening as the lump has increased in size. On examination, there is a
4cm mass underneath her mandible, there is no associated lymphadenopathy.
You answered Carotid body tumour
The correct answer is Submandibular gland calculus
The sub mandibular gland is the most common site for salivary calculi. Patients
will usually complain of pain, which is worse on eating. When the lesion is
located distally the duct may be laid open and the stone excised. Otherwise the
gland will require removal.

6.

A 73 year old male smoker is referred to the clinic by his GP. On examination he
has a 3cm soft mass immediately anterior to his ear. It has been present for the
past five years and is otherwise associated with no symptoms.
You answered Pleomorphic adenoma of the parotid
The correct answer is Adenolymphoma of the parotid
Warthins tumours (a.k.a. adenolymphoma) are commoner in older men
(especially smokers). They are the second commonest benign tumour of the
parotid gland, they may be bilateral. They are soft and slow growing and
relatively easy to excise. Pleomorphic adenomas typically present in females
aged between 40 - 60 years.

Next question

Neck lumps

The table below gives characteristic exam question features for conditions causing neck
lumps:
Reactive
lymphadenopathy

By far the most common cause of neck swellings. There may be a history
of local infection or a generalised viral illness

http://www.emrcs.com/question/question.php?q=0

1/2

13/07/2015

Lymphoma

Rubbery, painless lymphadenopathy


The phenomenon of pain whilst drinking alcohol is very uncommon
There may be associated night sweats and splenomegaly

Thyroid swelling

May be hypo-, eu- or hyperthyroid symptomatically


Moves upwards on swallowing

Thyroglossal cyst

More common in patients < 20 years old


Usually midline, between the isthmus of the thyroid and the hyoid bone
Moves upwards with protrusion of the tongue
May be painful if infected

Pharyngeal
pouch

More common in older men


Represents a posteromedial herniation between thyropharyngeus and
cricopharyngeus muscles
Usually not seen, but if large then a midline lump in the neck that gurgles
on palpation
Typical symptoms are dysphagia, regurgitation, aspiration and chronic
cough

Cystic hygroma

A congenital lymphatic lesion (lymphangioma) typically found in the neck,


classically on the left side
Most are evident at birth, around 90% present before 2 years of age

Branchial cyst

An oval, mobile cystic mass that develops between the


sternocleidomastoid muscle and the pharynx
Develop due to failure of obliteration of the second branchial cleft in
embryonic development
Usually present in early adulthood

Cervical rib

More common in adult females


Around 10% develop thoracic outlet syndrome

Carotid aneurysm

Pulsatile lateral neck mass which doesn't move on swallowing

Rate question:

Next question

Comment on this question

All contents of this site are 2012 E-Medical Revision Ltd

http://www.emrcs.com/question/question.php?q=0

Terms and Conditions

Privacy policy

2/2

13/07/2015

Reference ranges

Previous

Question 7 of 343

Next

A 12 year old child is admitted with a 12 hour history of colicky right upper quadrant pain. On
examination the child is afebrile and is jaundiced. The abdomen is soft and non tender at the
time of examination. What is the most likely cause?

Question stats

End and review

Score: 14.3%

13.4%

12.8%

9.5%

38.7%

25.6%

1
2
3
4-6 0 / 3
7

A. Infectious hepatitis
38.7% of users answered this
question correctly

B. Acute cholecystitis
C. Cholangitis
D. Hereditary spherocytosis

Search

E. Gilberts syndrome

Go

Next question

Theme from September 2012 Exam


The child is most likely to have hereditary spherocytosis. In these individuals there may be
disease flares precipitated by acute illness. They form small pigment stones. These may
cause biliary colic and some may require cholecystectomy.
Gilbert's syndrome is an inherited condition causing unconjugated hyperbilirubinaemia.
Patients may have jaundice or be asymptomatic. The other LFTs are normal and Gilbert's
may be confirmed with a fasting test or Nicotinic acid test. There is no need for treatment as
it is a benign condition.
Hereditary Spherocytosis
Most common disorder of the red cell membrane, it has an incidence of 1 in 5000. The
abnormally shaped erythrocytes are prone to splenic sequestration and destruction. This
can result in hyperbilirubinaemia, jaundice and splenomegaly. In older patients an
intercurrent illness may increase the rate of red cell destruction resulting in more acute
symptoms.
Severe cases may benefit from splenectomy.
Rate question:

Next question

Comment on this question

All contents of this site are 2012 E-Medical Revision Ltd

http://www.emrcs.com/question/question.php?q=0

Terms and Conditions

Privacy policy

1/1

13/07/2015

Reference ranges

Previous

Question 8 of 343

Next

A 28 year old man has a carcinoid tumour identified in his appendix. Blood testing for which
of the substances listed below is likely to be helpful during follow up?

A. CA19-9
B. Alkaline phosphatase

Question stats

Score: 25%

20.3%

9.3%

10.4%

19.7%

40.3%

40.3% of users answered this


question correctly

C. AFP

End and review

1
2
3
4-6 0 / 3
7
8

D. CEA
E. Chromogranin A

Search
Go

Next question

Theme from September 2013 Exam


It is important to distinguish between blood and urine tests for carcinoid syndrome. Blood
tests usually measure chromogranin A,neuron-specific enolase (NSE), substance P, and
gastrin. Urine tests usually measure 5 HIAA, which is a metabolite of serotonin. Sometimes
blood tests for 5 hydroxytryptamine (serotonin) are also performed.
Carcinoid syndrome

Carcinoid tumours secrete serotonin


Originate in neuroendocrine cells mainly in the intestine (midgut-distal ileum/appendix)
Can occur in the rectum, bronchi
Hormonal symptoms mainly occur when disease spreads outside the bowel

Clinical features
Onset: years
Flushing face
Palpitations
Pulmonary valve stenosis and tricuspid regurgitation causing dyspnoea
Asthma
Severe diarrhoea (secretory, persists despite fasting)

Investigation
5-HIAA in a 24-hour urine collection
Somatostatin receptor scintigraphy
CT scan
Blood testing for chromogranin A

Treatment
Octreotide
Surgical removal

Rate question:

Next question

Comment on this question

All contents of this site are 2012 E-Medical Revision Ltd

http://www.emrcs.com/question/question.php?q=0

Terms and Conditions

Privacy policy

1/1

13/07/2015

Reference ranges

Previous

Question 9 of 343

Next

A 40 year old man presents with obstructive jaundice and dysphagia. Twenty years
previously he underwent a right hemicolectomy for a mucinous right sided colonic carcinoma.
He was subsequently diagnosed as having Lynch syndrome. A recent colonoscopy was
normal. What is the most likely cause of his jaundice?

Question stats

Score: 33.3%

11.9%

21.2%

20.2%

19.2%

27.4%

27.4% of users answered this


question correctly

A. Hepatocellular carcinoma

End and review

1
2
3
4-6 0 / 3
7
8
9

B. Liver metastasis from colonic cancer


C. Pancreatic carcinoma
Search

D. Duodenal carcinoma
E. Gastric carcinoma

Go

Next question

Theme from April 2013


Lynch syndrome usually results in colonic cancer which is right sided and mucinous. The
next most common site to be affected is the uterus. The stomach is at particular risk and this
risk is up to 10 times greater in HNPCC (Lynch) patients than the general population.
Duodenal adenomas (and rarely carcinoma) are usually seen in association with FAP. Whilst
pancreatic carcinoma is associated with HNPCC it is far less likely to occur than gastric
cancer.
Genetics of colorectal cancer
The lifetime risk of colorectal cancer in the UK population is 5%. Up to 5% of newly
diagnosed bowel cancers will be in those individuals who have a high genetically acquired
risk of bowel cancer. Cancers arising in the low-moderate genetic risk group comprise
approximately 30% of newly diagnosed bowel cancer.
Genetics of inherited colorectal cancer syndromes
Syndrome

Features

Genes
implicated

FAP

More than 100 adenomatous polyps affecting the colon and


rectum. Duodenal and fundic glandular polyps

APC (over 90%)

Gardner
syndrome

As FAP but with desmoid tumours and mandibular


osteomas

APC

Turcots
syndrome

Polyposis and colonic tumours and CNS tumours

APC +MLH1
and PMS2

HNPCC

Colorectal cancer without extensive polyposis. Endometrial


cancer, renal and CNS

MSH2, MLH1,
PMS2 and
GTBP

PeutzJeghers
syndrome

Hamartomatous polyps in GI tract and increased risk of GI


malignancy

LKB1
andSTK11 (in
up to 70%)

Cowden
disease

Multiple hamartomas (see below)

PTEN (85%)

MYH
associated
polyposis

Autosomal recessive, multiple adenomatous polyps in GI


tract, those in colon having somatic KRAS mutations

MYH

FAP
Autosomal dominant condition, affects 1 in 12,000. Accounts for 0.5% of all CRCs. Lifetime
incidence of colorectal cancer in untreated FAP =100%. Up to 25% cases are caused by denovo germ line mutations and show no prior family history. The APC tumour suppressor gene
is affected in most cases.
APC in non inherited colorectal cancer
Up to 80% of sporadic colorectal cancers will have somatic mutations that inactivate APC[1].
Both alleles are usually affected. Although the APC protein more than likely has multiple
critical cellular functions, the best-established role for APC in the cancer process is as a
major binding partner and regulator of the - catenin protein in the so-called canonical or catenin dependent Wnt signaling pathway.
HNPCC

http://www.emrcs.com/question/question.php?q=0

1/2

13/07/2015

HNPCC cancers differ from conventional tumours in a number of respects. In the colon the
tumours are more likely to be right sided, histologically they are more likely to be mucinous
and have dense lymphocytic infiltrates. To be diagnosed as having HNPCC individuals must
show typically HNPCC tumours in at least three individuals, (one of whom must be a first
degree relative to the other two). In at least two successive generations. At least one cancer
must be diagnosed under the age of 50. FAP must be excluded and tumours should be
verified by pathological identification (Amsterdam criteria). The genetic changes in HNPCC
stem primarily from microsatellite instability affecting DNA mismatch repair genes. In HNPCC
the mismatch repair genes most commonly implicated include; MSH2 and MLH1 and these
occur in up to 70% of people with HNPCC. The finding of microsatellite instability is unusual
in sporadic colorectal cancers. Approximately 60% of individuals who fulfill the Amsterdam
criteria will not be found to have evidence of mismatch repair gene defects on genetic
testing. The risk of developing colorectal cancer in those who have not demonstrated
mutation of the mis match repair genes is increased if they fulfill the Amsterdam criteria, but
not
the extent that it is increased in those who fulfill the criteria AND have evidence of mis match
repair gene defects.
KRAS Mutations
The RAS family of small G proteins act as molecular switches downstream of growth factor
receptors. KRAS and the other two members of the family; HRAS and NRAS, are the site of
mutation in approximately 40% of colorectal cancers. When adenomas are examined the
proportion of adenomas less than 1cm showing KRAS mutations was only 10% which
contrasts with 50% in those lesions greater than 1cm.
p53 mutations
The p53 protein functions as a key transcriptional regulator of genes that encode proteins
with functions in cell-cycle checkpoints at the G1/S and G2/M boundaries, in promoting
apoptosis, and in restricting angiogenesis . As such, selection for p53 defects at the
adenoma-carcinoma transition may reflect the fact that stresses on tumor cells activate cellcycle arrest, apoptotic, and antiangiogenic pathways in cells with wild-type p53 function.
Many colonic tumours will demonstrate changes in the p53 gene that may facilitate tumour
progression through from adenoma to carcinoma.
Cowden syndrome
Also known as multiple hamartoma syndrome. Rare autosomal dominant condition with
incidence of 1 in 200,000.. It is characterised by multiple mucocutaneous lesions,
trichilemmomas, oral papillomas and acral keratosis. Most often diagnosed in third decade of
life. Breast carcinoma may occur in up to 50% of patients and conditions such as fibrocystic
disease of the breast may occur in 75% of women. Thyroid disease occurs in 75% and may
include malignancy. Endoscopic screening will identify disease in up to 85% although the
small bowel is rarely involved. There is a 15-20% risk of developing colorectal cancer and
regular colonoscopic screening from age 45 is recommended.
Terminology
Oncogene

Oncogenes are genes which have the potential to induce cellular proliferation and
avoid apoptosis. Oncogene mutations are general gain of function and are
therefore dominant. Increased expression of oncogenes are found in most tumours

Tumour
suppressor
gene

These genes generally inhibit cellular proliferation or induce apoptosis. Mutations


in tumour suppressor genes are generally loss of function mutations, and are
therefore recessive. Mutations in both tumour suppressor gene alleles allow cells
to proliferate without restraint

References
1. Fearon, E.R. and B. Vogelstein, A genetic model for colorectal tumorigenesis. Cell, 1990.
61(5): p. 759-67.
Rate question:

Next question

Comment on this question

All contents of this site are 2012 E-Medical Revision Ltd

http://www.emrcs.com/question/question.php?q=0

Terms and Conditions

Privacy policy

2/2

13/07/2015

Reference ranges

Previous3 / 3

Question 10-12 of 343

Next

Question stats

End and review

Score: 50%

Average score for registered users:

1
2

Theme: Breast disease


A. Tuberculosis

10

82.5%

11

62.8%

12

68.6%

3
4-6 0 / 3

B. Actinomycosis

C. Duct ectasia

D. Fibroadenoma

Search

E. Fat necrosis

Go

10-12 3 / 3

F. Intraductal papilloma
G. Breast abscess
H. Breast cancer
What is the most likely diagnosis for each scenario given? Each diagnosis may be used
once, more than once or not at all.

10.

A 32 year old woman presents with a tender breast lump. She has a 2 month
old child. Clinically there is a tender, fluctuant mass of the breast.
Breast abscess
Theme from January 2013 Exam
Theme from September 2011 Exam
This lady is likely to be breast feeding and is at risk of mastitis. This may lead
to an abscess if not treated. Staphylococcus aureus is usually the causative
organism.

11.

A 53 year old lady presents with a creamy nipple discharge. On examination


she has discharge originating from multiple ducts and associated nipple
inversion.
Duct ectasia
Duct ectasia is common during the period of breast involution that occurs
during the menopausal period. As the ducts shorten they may contain
insipiated material. The discharge will often discharge from several ducts.

12.

A 52 year old lady presents with an episode of nipple discharge. It is usually


clear in nature. On examination the discharge is seen to originate from a single
duct and although it appears clear, when the discharge is tested with a labstix it
is shown to contain blood. Imaging and examination shows no obvious mass
lesion.
Intraductal papilloma
Intraductal papilloma usually cause single duct discharge. The fluid is often
clear, although it may be blood stained. If the fluid is tested with a labstix (little
point in routine practice) then it will usually contain small amounts of blood. A
microdocechtomy may be performed.

Next question

Non malignant breast disease

Duct ectasia
Mammary duct ectasia may be seen in up to 25% of normal female breasts
Patients usually present with nipple discharge, which may be from single or multiple
ducts (usually present age >50 years)
The discharge is often thick and green
Duct ectasia is a normal variant of breast involution and is not the same condition as
periductal mastitis

Periductal mastitis
Present at younger age than duct ectasia
May present with features of inflammation, abscess or mammary duct fistula
Strongly associated with smoking

http://www.emrcs.com/question/question.php?q=0

1/2

13/07/2015

Usually treated with antibiotics, abscess will require drainage

Intraductal papilloma
Growth of papilloma in a single duct
Usually presents with clear or blood stained discharge originating from a single duct
No increase in risk of malignancy

Breast abscess
Lactational mastitis is common
Infection is usually with Staphylococcus aureus
On examination there is usually a tender fluctuant mass
Treatment is with antibiotics and ultrasound guided aspiration
Overlying skin necrosis is an indication for surgical debridement, which may be
complicated by the development of a subsequent mammary duct fistula.

Tuberculosis
Rare in western countries, usually secondary TB
Affects women later in child bearing period
Chronic breast or axillary sinus is present in up to 50% cases
Diagnosis is by biopsy culture and histology

Rate question:

Next question

Comment on this question

All contents of this site are 2012 E-Medical Revision Ltd

http://www.emrcs.com/question/question.php?q=0

Terms and Conditions

Privacy policy

2/2

13/07/2015

Reference ranges

Question 13 of 343

Previous

Next

A 34 year old women who has previously undergone a colectomy for familial adenomatous
polyposis coli presents with a firm lesion at the inferior aspect of her rectus abdominis
muscle. Which cell type is most typically associated with such tumours?

Question stats

Score: 53.8%

13.7%

16.6%

9.1%

11.4%

49.3%

A. Myocytes
49.3% of users answered this
question correctly

B. Proliferation of apocrine glands

End and review

C. Chondrocytes

1
2
3
4-6 0 / 3
7
8
9
10-12 3 / 3

D. Lipoblasts

13
Search

E. Myofibroblasts

Go

Next question

Desmoid tumours would be the most likely differential here and consist of a clonal
proliferation of myofibroblasts.
Desmoid tumours

Desmoid tumours are fibrous neoplasms arising from musculoaponeurotic structures.


They are typically contain clonal proliferations of myofibroblasts.
They are usually firm overgrowths of tissue with a propensity to local infiltration.
They occur in up to 15% of patients with familial adenomatous polyposis coli.
Desmoid tumours usually show bi allelic APC mutations
They occur most commonly in women after childbirth in the rectus abdominis muscle.
They are usually treated (regardless of site) by radical surgical resection, in some
patients radiotherapy and chemotherapy may be considered. The results of non
surgical therapy is inferior to surgical resection. In selected cases of abdominal
desmoids a period of observation may be preferred as some may spontaneously
regress.
Desmoids have a high tendency to local recurrence.
Desmoids consist of sheets of differentiated fibroblasts

Image sourced from Wikipedia

Rate question:

Next question

Comment on this question

All contents of this site are 2012 E-Medical Revision Ltd

http://www.emrcs.com/question/question.php?q=0

Terms and Conditions

Privacy policy

1/1

13/07/2015

Reference ranges

Previous

Question 20 of 343

Next

A 38 year old man has been suffering from mechanical back pain for several years. One
morning he awakes from sleep and feels a sudden onset of pain in his back radiating down
his left leg. Which of the following events is most likely to account for his symptoms?

Question stats

Score: 50%

10.5%

16.3%

53.7%

10.3%

9.3%

A. Prolapse of inner annulus fibrosus


53.7% of users answered this
question correctly

B. Prolapse of outer annulus fibrosus

End and review

C. Prolapse of nucleus pulposus

1
2
3
4-6 0 / 3
7
8
9
10-12 3 / 3

D. Rupture of the ligamentum flavum

13
Search

E. None of the above

14
Go

Next question

15-17 1 / 3
18
19

Theme from 2009 Exam


Theme from September 2012 Exam
The symptoms would be most likely the result of intervertebral disk prolapse. In disk prolapse
the nucleus pulposus is the structure which usually herniates.

20

Intervertebral discs

Consist of an outer annulus fibrosus and an inner nucleus pulposus.


The anulus fibrosus consists of several layers of fibrocartilage.
The nucleus pulposus contains loose fibres suspended in a mucoprotein gel with the
consistency of jelly. The nucleus of the disc acts as a shock absorber.
Pressure on the disc causes posterior protrusion of the nucleus pulposus. Most
commonly in the lumbrosacral and lower cervical areas.
The discs are separated by hyaline cartilage.
There is one disc between each pair of vertebrae, except for C1/2 and the
sacrococcygeal vertebrae.

Rate question:

Next question

Comment on this question

All contents of this site are 2012 E-Medical Revision Ltd

http://www.emrcs.com/question/question.php?q=0

Terms and Conditions

Privacy policy

1/1

13/07/2015

Reference ranges

Previous1 / 3

Question 21-23 of 343

Next

Question stats

End and review

Score: 47.8%

Average score for registered users:

1
2

Theme: Paediatric neck masses


A. Cystic hygroma

21

42.8%

22

70.1%

23

60%

3
4-6 0 / 3

B. Thyroglossal cyst

C. Rhabdomyosarcoma

D. Branchial cyst

Search

E. Dermoid cyst

Go

10-12 3 / 3
13

Please select the most likely underlying diagnosis for the situation that is described. Each
option may be used once, more than once, or not at all.

14
15-17 1 / 3
18

21.

A 2 year old boy is brought to the clinic by his mother who has noticed that he
has developed a small mass. On examination; a small smooth cyst is identified
which is located above the hyoid bone. On ultrasound the lesion appears to be
a heterogenous and multiloculated mass.

19
20
21-23 1 / 3

You answered Cystic hygroma


The correct answer is Dermoid cyst
Dermoid cysts are usually multiloculated and heterogeneous. Most are located
above the hyoid, and their appearances on imaging differentiate them from
thyroglossal cysts.
22.

A 22 month old baby is brought to the clinic by her mother who is concerned
that she has developed a swelling in her neck. On examination; she has a soft
lesion located in the posterior triangle that transilluminates.
You answered Rhabdomyosarcoma
The correct answer is Cystic hygroma
Cystic hygromas are soft and transilluminate. Most are located in the posterior
triangle.

23.

A 3 year old boy is brought to the clinic by his mother who has noticed a mass
in his neck. On examination; he has a smooth mass located on the lateral
aspect of his anterior triangle, near to the angle of the mandible. On
ultrasound; it has a fluid filled, anechoic, appearance.
Branchial cyst
Branchial cysts are usually located laterally and derived from the second
branchial cleft. Unless infection has occurred they will usually have an anechoic
appearance on ultrasound.

Next question

Neck Masses in Children

Thyroglossal cyst

Branchial cyst

Located in the anterior triangle, usually in the midline and below


the hyoid (65% cases)
Derived from remnants of the thyroglossal duct
Thin walled and anechoic on USS (echogenicity suggests infection
of cyst)

Six branchial arches separated by branchial clefts


Incomplete obliteration of the branchial apparatus may result in
cysts, sinuses or fistulae
75% of branchial cysts originate from the second branchial cleft
Usually located anterior to the sternocleidomastoid near the angle
of the mandible
Unless infected the fluid of the cyst has a similar consistency to
water and is anechoic on USS

Dermoids

http://www.emrcs.com/question/question.php?q=0

1/2

13/07/2015

Dermoids

Thyroid gland

Lymphatic
malformations

Infantile
haemangioma

Lymphadenopathy

Derived from pleuripotent stem cells and are located in the midline
Most commonly in a suprahyoid location
They have heterogeneous appearances on imaging and contain
variable amounts of calcium and fat

True thyroid lesions are rare in children and usually represent


thyroglossal cysts or tumours like lymphoma

Usually located posterior to the sternocleidomastoid


Cystic hygroma result from occlusion of lymphatic channels
The painless, fluid filled, lesions usually present prior to the age of
2
They are often closely linked to surrounding structures and
surgical removal is difficult
They are typically hypoechoic on USS

May present in either triangle of the neck


Grow rapidly initially and then will often spontaneously regress
Plain x-rays will show a mass lesion, usually containing calcified
phleboliths
As involution occurs the fat content of the lesions increases

Located in either triangle of the neck


May be reactive or neoplastic
Generalised lymphadenopathy usually secondary to infection in
children (very common)

Rate question:

Next question

Comment on this question

All contents of this site are 2012 E-Medical Revision Ltd

http://www.emrcs.com/question/question.php?q=0

Terms and Conditions

Privacy policy

2/2

13/07/2015

Reference ranges

Previous

Question 24 of 343

Next

A 34 year old man presents to the surgical clinic 8 months following a laparotomy for a
ruptured spleen. He complains of a nodule in the centre of his laparotomy wound. This is
explored surgically and a stitch granuloma is found and excised. From which of the following
cell types do granulomata arise?

Question stats

Score: 45.8%

13.2%

10.2%

9%

8.1%

59.5%

59.5% of users answered this


question correctly

A. Polymorpho nucleocytes

End and review

B. Plasma cells

1
2
3
4-6 0 / 3
7
8
9
10-12 3 / 3

C. Reed- Sternberg cells

13
Search

D. Platelets

14

E. Macrophages

Go

15-17 1 / 3
18

Next question

19
20

Granulomas are organised collections of macrophages

21-23 1 / 3
24

Theme from 2011 Exam


Macrophages give origin to granulomas.
Chronic inflammation
Overview
Chronic inflammation may occur secondary to acute inflammation.In most cases chronic
inflammation occurs as a primary process. These may be broadly viewed as being one of
three main processes:
Persisting infection with certain organisms such as Mycobacterium tuberculosis which
results in delayed type hypersensitivity reactions and inflammation.
Prolonged exposure to non-biodegradable substances such as silica or suture
materials which may induce an inflammatory response.
Autoimmune conditions involving antibodies formed against host antigens.

Acute vs. Chronic inflammation


Acute inflammation

Chronic inflammation

Changes to existing vascular structure and increased


permeability of endothelial cells

Angiogenesis predominates

Infiltration of neutrophils

Macrophages, plasma cells and


lymphocytes predominate

Process may resolve with:

Healing by fibrosis is the main result

Suppuration
Complete resolution
Abscess formation
Progression to chronic inflammation
Healing by fibrosis

Granulomatous inflammation
A granuloma consists of a microscopic aggregation of macrophages (with epithelial type
arrangement =epitheliod). Large giant cells may be found at the periphery of granulomas.
Mediators
Growth factors released by activated macrophages include agents such as interferon and
fibroblast growth factor (plus many more). Some of these such as interferons may have
systemic features resulting in systemic symptoms and signs, which may be present in
individuals with long standing chronic inflammation.
The finding of granulomas is pathognomonic of chronic inflammation, as illustrated in this
biopsy from a patient with colonic Crohns disease

http://www.emrcs.com/question/question.php?q=0

1/2

13/07/2015

Image sourced from Wikipedia

Rate question:

Next question

Comment on this question

All contents of this site are 2012 E-Medical Revision Ltd

http://www.emrcs.com/question/question.php?q=0

Terms and Conditions

Privacy policy

2/2

13/07/2015

Reference ranges

Question 31 of 343

Previous

Next

As a busy surgical trainee on the colorectal unit you are given the unenviable task of
reviewing the unit's histopathology results for colonic polyps. Which of the polyp types
described below has the greatest risk of malignant transformation?

Question stats

Score: 45.2%

19.3%

12.5%

46.8%

12.5%

8.9%

A. Hyperplastic polyp
46.8% of users answered this
question correctly

B. Tubular adenoma

End and review

C. Villous adenoma

1
2
3
4-6 0 / 3
7
8
9
10-12 3 / 3

D. Hamartomatous polyp

13
Search

E. Serrated polyp

14
Go

Next question

15-17 1 / 3
18
19

Villous adenomas carry the highest risk of malignant transformation. Hyperplastic polyps
carry little in the way of increased risk. Although, patients with hamartomatous polyp
syndromes may have a high risk of malignancy, the polyps themselves have little malignant
potential.

20
21-23 1 / 3
24
25

Colonic polyps

26
27

Colonic Polyps
May occur in isolation, or greater numbers as part of the polyposis syndromes. In FAP
greater than 100 polyps are typically present. The risk of malignancy in association with
adenomas is related to size, and is the order of 10% in a 1cm adenoma. Isolated adenomas
seldom give risk of symptoms (unless large and distal). Distally sited villous lesions may
produce mucous and if very large, electrolyte disturbances may occur.

28
29
30
31

Follow up of colonic polyps


Group

Features

Action

Low risk

1 or 2 adenomas less than 1cm

No follow up or recolonoscopy at 5 years

Moderate
risk

3 or 4 small adenomas or 1 adenoma greater than


1cm

Re-scope at 3 years

High risk

More than 5 small adenomas or more than 3 with 1


of them greater than 1cm

Re scope at 1 year

From Atkins and Saunders Gut 2002 51 (suppl V:V6-V9). It is important to stratify patients
appropriately and ensure that a complete colonoscopy with good views was performed.
Segmental resection or complete colectomy should be considered when:
1. Incomplete excision of malignant polyp
2. Malignant sessile polyp
3. Malignant pedunculated polyp with submucosal invasion
4. Polyps with poorly differentiated carcinoma
5. Familial polyposis coli
-Screening from teenager up to 40 years by 2 yearly sigmoidoscopy/colonoscopy
-Panproctocolectomy and Ileostomy or Restorative Panproctocolectomy.
Rectal polypoidal lesions may be amenable to trans anal endoscopic microsurgery.
References
Cairns S et al. Guidelines for colorectal cancer screening and surveillance in moderate and
high risk groups (update from 2002). Gut 2010;59:666-690.
Rate question:

Next question

Comment on this question

All contents of this site are 2012 E-Medical Revision Ltd

http://www.emrcs.com/question/question.php?q=0

Terms and Conditions

Privacy policy

1/1

13/07/2015

Reference ranges

Previous

Question 32 of 343

Next

A 23 year old man presents to the surgical clinic with an inguinal hernia. On examination he
has a small direct hernia. However, you also notice that he has pigmented spots around his
mouth, on his palms and soles. In his history he underwent a reduction of an intussusception
aged 12 years. Which of the following lesions is most likely to be identified if a colonoscopy
were performed?

Question stats

Score: 43.8%

51.7%

12.7%

8.2%

13.8%

13.6%

51.7% of users answered this


question correctly

A. Hamartomas

End and review

1
2
3
4-6 0 / 3
7
8
9
10-12 3 / 3

B. Tubulovillous adenoma

13

C. Colorectal cancer

Search
14

D. Crohns disease

Go

E. Hyperplastic polyps

15-17 1 / 3
18
19

Next question

20
21-23 1 / 3

Theme from April 2014 Exam


He is most likely to have Peutz-Jeghers syndrome which is associated with Hamartomas.

24
25

Peutz-Jeghers syndrome

26
27

Peutz-Jeghers syndrome is an autosomal dominant condition characterised by numerous


benign hamartomatous polyps in the gastrointestinal tract. It is also associated with
pigmented freckles on the lips, face, palms and soles. Around 50% of patients will have died
from a gastrointestinal tract cancer by the age of 60 years.

28
29
30

Genetics

31
32

Autosomal dominant
Responsible gene encodes serine threonine kinase LKB1 or STK11

Features
Hamartomatous polyps in GI tract (mainly small bowel)
Pigmented lesions on lips, oral mucosa, face, palms and soles
Intestinal obstruction e.g. intussusception (which may lead to diagnosis)
Gastrointestinal bleeding

Management
Conservative unless complications develop

Rate question:

Next question

Comment on this question

All contents of this site are 2012 E-Medical Revision Ltd

http://www.emrcs.com/question/question.php?q=0

Terms and Conditions

Privacy policy

1/1

13/07/2015

Reference ranges

Previous

Question 33 of 343

Next

A 56 year old surgeon has been successfully operating for many years. Over the past few
weeks she has begun to notice that her hands are becoming blistering and weepy. A latex
allergy is diagnosed. Which of the following pathological processes accounts for this
scenario?

Question stats

Score: 45.5%

18.2%

12.7%

48.9%

13.2%

7%

48.9% of users answered this


question correctly

A. Type 1 hypersensitivity reaction

End and review

B. Type 2 hypersensitivity reaction

1
2
3
4-6 0 / 3
7
8
9
10-12 3 / 3

C. Type 4 hypersensitivity reaction

13
Search

D. Type 3 hypersensitivity reaction

14

E. None of the above

Go

15-17 1 / 3
18

Next question

19
20

Hypersensitivity reactions: ACID

21-23 1 / 3
24

type 1 --Anaphylactic
type 2 --Cytotoxic
type 3 --Immune complex
type 4 --Delayed hypersensitivity

25
26
27
28

Theme from 2012 Exam


Contact dermatitis of a chronic nature is an example of a type 4 hypersensitivity reaction.
Type 4 hypersensitivity reactions are cell mediated rather than antibody mediated.

29
30
31

Hypersensitivity reactions

32
33

The Gell and Coombs classification divides hypersensitivity reactions into 4 types
Type I

Type II

Type III

Type IV

Description

Anaphylactic

Cytotoxic

Immune
complex

Delayed type

Mediator

IgE

IgG, IgM

IgG, Ig A,
IgM

T-cells

Antigen

Exogenous

Cell surface

Soluble

Tissues

Response
time

Minutes

Hours

Hours

2-3 days

Examples

Asthma
Hay fever

Autoimmune haemolytic
anaemia
Pemphigus
Goodpasture's

Serum
sickness
SLE
Aspergillosis

Graft versus host


disease
Contact dermatitis

Rate question:

Next question

Comment on this question

All contents of this site are 2012 E-Medical Revision Ltd

http://www.emrcs.com/question/question.php?q=0

Terms and Conditions

Privacy policy

1/1

13/07/2015

Reference ranges

Question 36 of 343

Previous

Next

A 43 year old man presents with haemoptysis and is diagnosed as having tuberculosis.
Which of the cell types listed below will usually internalise the tubercule bacullis?

A. Fibroblast
B. Neutrophil

Question stats

Score: 50%

7.6%

10.3%

7.6%

63.9%

10.6%

63.9% of users answered this


question correctly

C. Erythrocyte

End and review

1
2
3
4-6 0 / 3
7
8
9
10-12 3 / 3

D. Macrophage

13

E. Eosinophil

Search
14
Go

Next question

15-17 1 / 3
18

Theme from January 2013 Exam


M. Tuberculosis will reside in macrophages where it will often survive

19
20
21-23 1 / 3

Tuberculosis pathology

24
25

Is a form of primary chronic inflammation, caused by the inability of macrophages to kill


the Mycobacterium tuberculosis.
The macrophages often migrate to regional lymph nodes, the lung lesion plus affected
lymph nodes is referred to as a Ghon complex.
This leads to the formation of a granuloma which is a collection of epithelioid
histiocytes.
There is the presence of caseous necrosis in the centre.
The inflammatory response is mediated by a type 4 hypersensitivity reaction.
In healthy individuals the disease may be contained, in the immunocompromised
disseminated (miliary TB) may occur.

26
27
28
29
30
31
32
33
34

Diagnosis

35

Waxy membrane of mycobacteria prevents binding with normal stains. Ziehl - Neelsen
staining is typically used.
Culture based methods take far longer.

36

Image showing acid- alcohol fast mycobacteria stained using the Ziehl- Neelsen method

Image sourced from Wikipedia

Rate question:

Next question

Comment on this question

All contents of this site are 2012 E-Medical Revision Ltd

http://www.emrcs.com/question/question.php?q=0

Terms and Conditions

Privacy policy

1/1

13/07/2015

Reference ranges

Previous

Question 38 of 343

Next

A 39 year old man has suffered from terminal ileal Crohns disease for the past 20 years.
Which condition is he least likely to develop?

A. Gallstones
B. Malabsorption

Question stats

Score: 50%

17.3%

13.6%

12.3%

17.6%

39.3%

39.3% of users answered this


question correctly

C. Pyoderma gangrenosum

End and review

1
2
3
4-6 0 / 3
7
8
9
10-12 3 / 3

D. Amyloidosis

13

E. Feltys syndrome

Search
14
Next question

Go

15-17 1 / 3
18
19

Felteys syndrome:

20

Rheumatoid disease
Splenomegaly
Neutropenia

21-23 1 / 3
24
25
26

Feltys syndrome is associated with rheumatoid disease. Individuals with long standing
Crohns disease are at risk of gallstones because of impairment of the enterohepatic
recycling of bile salts. Formation of entero-enteric fistulation may produce malabsorption.
Amyloidosis may complicate chronic inflammatory states.

27
28
29
30

Crohns disease

31

Crohns disease is a chronic transmural inflammation of a segment(s) of the gastrointestinal


tract and may be associated with extra intestinal manifestations. Frequent disease patterns
observed include ileal, ileocolic and colonic disease. Peri-anal disease may occur in
association with any of these. The disease is often discontinuous in its distribution.
Inflammation may cause ulceration, fissures, fistulas and fibrosis with stricturing. Histology
reveals a chronic inflammatory infiltrate that is usually patchy and transmural.

32
33
34
35
36
37

Ulcerative colitis Vs Crohns


38

Crohn's disease

Ulcerative colitis

Distribution

Mouth to anus

Rectum and colon

Macroscopic
changes

Cobblestone appearance, apthoid ulceration

Contact bleeding

Depth of
disease

Transmural inflammation

Superficial inflammation

Distribution
pattern

Patchy

Continuous

Histological
features

Granulomas (non caseating epithelioid cell


aggregates with Langerhans' giant cells)

Crypt abscesses, Inflammatory


cells in the lamina propria

Extraintestinal manifestations of Crohns


Related to disease extent

Unrelated to disease extent

Aphthous ulcers (10%)

Sacroiliiitis (10-15%)

Erythema nodosum (5-10%)

Ankylosing spondylitis (1-2%)

Pyoderma gangrenosum (0.5%)

Primary sclerosing cholangitis (Rare)

Acute arthropathy (6-12%)

Gallstones (up to 30%)

Ocular complications (up to 10%)

Renal calculi (up to 10%)

Diarrhoea in Crohns
Diarrhoea in Crohns may be multifactorial since actual inflammation of the colon is not
common. Causes therefore include the following:
Bile salt diarrhoea secondary to terminal ileal disease
Entero-colic fistula

http://www.emrcs.com/question/question.php?q=0

1/2

13/07/2015

Short bowel due to multiple resections


Bacterial overgrowth

Surgical interventions in Crohns disease


The commonest disease pattern in Crohns is stricturing terminal ileal disease and this often
culminates in an ileocaecal resection. Other procedures performed include segmental small
bowel resections and stricturoplasty. Colonic involvement in patients with Crohns is not
common and, where found, distribution is often segmental. However, despite this distribution
segmental resections of the colon in patients with Crohns disease are generally not
advocated because the recurrence rate in the remaining colon is extremely high, as a result
the standard options of colonic surgery in Crohns patients are generally; sub total colectomy,
panproctocolectomy and staged sub total colectomy and proctectomy. Restorative
procedures such as ileoanal pouch have no role in therapy.
Crohns disease is notorious for the developmental of intestinal fistulae; these may form
between the rectum and skin (peri anal) or the small bowel and skin. Fistulation between
loops of bowel may also occur and result in bacterial overgrowth and malabsorption.
Management of enterocutaneous fistulae involves controlling sepsis, optimising nutrition,
imaging the disease and planning definitive surgical management.
Rate question:

Next question

Comment on this question

All contents of this site are 2012 E-Medical Revision Ltd

http://www.emrcs.com/question/question.php?q=0

Terms and Conditions

Privacy policy

2/2

13/07/2015

Reference ranges

3/3

Question 1-3 of 304

Next

Question stats
Average score for registered users:

Theme: Renal stones


A. Calcium oxalate

69%

46.2%

67.4%

End and review

Score: 100%
1-3 3 / 3

B. Uric acid
C. Cystine
D. Struvite

Search

E. Calcium phosphate

Go

Please select the most likely stone type for each of the following urinary tract stone
scenarios. Each option may be used once, more than once or not at all.

1.

A 73 year old lady is undergoing chemotherapy for treatment of acute


leukaemia. She develops symptoms of renal colic. Her urine tests positive for
blood. A KUB x-ray shows no evidence of stones.
Uric acid
Chemotherapy and cell death can increase uric acid levels. In this acute setting
the uric acid stones are unlikely to be coated with calcium and will therefore be
radiolucent.

2.

A 16 year old boy presents with renal colic. His parents both have a similar
history of the condition. His urine tests positive for blood. A KUB style x-ray
shows a relatively radiodense stone in the region of the mid ureter.
Cystine
Cystine stones are associated with an inherited metabolic disorder.

3.

A 43 year old lady with episodes of recurrent urinary tract sepsis presents with a
staghorn calculus of the left kidney. Her urinary pH is 7.3. A KUB x-ray shows a
faint outline of the calculus.
Struvite
Theme from April 2012 Exam
Chronic infection with urease producing enzymes can produce an alkaline urine
with formation of struvite stone.

Next question

Renal stones

Type of
stones

Features

Percentage
of all
calculi

Calcium
oxalate

Hypercalciuria is a major risk factor (various causes)


Hyperoxaluria may also increase risk
Hypocitraturia increases risk because citrate forms complexes with
calcium making it more soluble
Stones are radio-opaque (though less than calcium phosphate
stones)
Hyperuricosuria may cause uric acid stones to which calcium
oxalate binds

85%

Cystine

Inherited recessive disorder of transmembrane cystine transport


leading to decreased absorption of cystine from intestine and renal
tubule
Multiple stones may form
Relatively radiodense because they contain sulphur

1%

Uric acid

Uric acid is a product of purine metabolism


May precipitate when urinary pH low
May be caused by diseases with extensive tissue breakdown e.g.
malignancy
More common in children with inborn errors of metabolism
Radiolucent

5-10%

http://www.emrcs.com/question/question.php?q=0

1/2

13/07/2015

Calcium
phosphate

May occur in renal tubular acidosis, high urinary pH increases


supersaturation of urine with calcium and phosphate
Renal tubular acidosis types 1 and 3 increase risk of stone
formation (types 2 and 4 do not)
Radio-opaque stones (composition similar to bone)

10%

Struvite

Stones formed from magnesium, ammonium and phosphate


Occur as a result of urease producing bacteria (and are thus
associated with chronic infections)
Under the alkaline conditions produced, the crystals can
precipitate
Slightly radio-opaque

2-20%

Effect of urinary pH on stone formation


Urine pH will show individual variation (from pH 5-7). Post prandially the pH falls as purine
metabolism will produce uric acid. Then the urine becomes more alkaline (alkaline tide).
When the stone is not available for analysis the pH of urine may help to determine which
stone was present.
Stone type

Urine acidity

Mean urine pH

Calcium phosphate

Normal- alkaline

>5.5

Calcium oxalate

Variable

Uric acid

Acid

5.5

Struvate

Alkaline

>7.2

Cystine

Normal

6.5

Rate question:

Next question

Comment on this question

All contents of this site are 2012 E-Medical Revision Ltd

http://www.emrcs.com/question/question.php?q=0

Terms and Conditions

Privacy policy

2/2

13/07/2015

Reference ranges

Question 4 of 304

Previous

Next

A pathologist is examining a histological section and identifies Hassall's corpuscles. With


what are they most commonly associated?

A. Follicular carcinoma of the thyroid


B. Medulla of the thymus

Question stats

End and review

Score: 75%
1-3 3 / 3

17.2%

40%

21.9%

12.3%

8.6%

40% of users answered this


question correctly

C. Medulla of the spleen


D. Medulla of the kidney
E. Fundus of the stomach

Search
Go

Next question

Theme from September 2014 Exam


Hassall's corpuscles are the concentric ring of epithelial cells seen in the medulla of the
thymus.
Thymus
The thymus develops from the third and fourth pharyngeal pouches. It descends to lie in the
anterior superior mediastinum. It is encapsulated and is subdivided into lobules, these
consist of a cortex and a medulla. The cortex is composed of tightly packed lymphocytes, the
medulla consists largely of epithelial cells. The medullary epithelial cells are concentrically
arranged and may surround a keratinised centre, known as Hassall's corpuscles.
The inferior parathyroid glands also develop from the third pharyngeal pouch and may also
be located with the thymus gland.
Its arterial supply is from the internal mammary artery or pericardiophrenic arteries. Venous
drainage is to the left brachiocephalic vein.
Hassall's corpuscles stained with H+E

Image sourced from Wikipedia

Rate question:

Next question

Comment on this question

All contents of this site are 2012 E-Medical Revision Ltd

http://www.emrcs.com/question/question.php?q=0

Terms and Conditions

Privacy policy

1/1

13/07/2015

Reference ranges

Previous

Question 5 of 304

Next

A 64 year old man presents to the clinic with right upper quadrant discomfort. He has never
attended the hospital previously and is usually well. He has just retired from full time
employment as a machinist in a PVC factory. CT scanning shows a large irregular tumour in
the right lobe of his liver. Which of the following lesions is the most likely?

Question stats

End and review

Score: 80%
1-3 3 / 3

13.5%

45.8%

16.5%

12.7%

11.4%

4
5

45.8% of users answered this


question correctly

A. Liposarcoma
B. Angiosarcoma
C. Hamartoma

Search

D. Hyatid liver disease


E. Benign angioma

Go

Next question

Angiosarcoma of the liver is a rare tumour. However, it is linked to working with vinyl chloride,
as in this case. Although modern factories minimise the exposure to this agent, this has not
always been the case.
Occupational cancers
Occupational cancers accounted for 5.3% cancer deaths in 2005.
In men the main cancers include:
Mesothelioma
Bladder cancer
Non melanoma skin cancer
Lung cancer
Sino nasal cancer

Occupations with high levels of occupational tumours include:


Construction industry
Working with coal tar and pitch
Mining
Metalworkers
Working with asbestos (accounts for 98% of all mesotheliomas)
Working in rubber industry

Shift work has been linked to breast cancer in women (Health and safety executive report
RR595).
The latency between exposure and disease is typically 15 years for solid tumours and 20 for
leukaemia.
Many occupational cancers are otherwise rare. For example sino nasal cancer is an
uncommon tumour, 50% will be SCC. They are linked to conditions such as wood dust
exposure and unlike lung cancer is not strongly linked to cigarette smoking. Another typical
occupational tumour is angiosarcoma of the liver which is linked to working with vinyl chloride.
Again in the non occupational context this is an extremely rare sporadic tumour.
Rate question:

Next question

Comment on this question

All contents of this site are 2012 E-Medical Revision Ltd

http://www.emrcs.com/question/question.php?q=0

Terms and Conditions

Privacy policy

1/1

13/07/2015

Reference ranges

Previous

Question 6 of 304

Next

A 32 year old man is involved in a house fire and sustains extensive partial thickness burns
to his torso and thigh. Two weeks post operatively he develops oedema of both lower legs.
The most likely cause of this is:

Question stats

End and review

Score: 83.3%
1-3 3 / 3

10.5%

20.1%

53.4%

8.1%

7.9%

4
5
6

A. Iliofemoral deep vein thrombosis


53.4% of users answered this
question correctly

B. Venous obstruction due to scarring


C. Hypoalbuminaemia
D. Excessive administration of intravenous fluids

Search

E. None of the above

Go

Next question

Theme from 2009 Exam


Loss of plasma proteins is the most common cause of oedema developing in this time frame.
Burns pathology
Extensive burns
Haemolysis due to damage of erythrocytes by heat and microangiopathy
Loss of capillary membrane integrity causing plasma leakage into interstitial space
Extravasation of fluids from the burn site causing hypovolaemic shock (up to 48h after
injury)- decreased blood volume and increased haematocrit
Protein loss
Secondary infection e.g. Staphylococcus aureus
ARDS
Risk of Curlings ulcer (acute peptic stress ulcers)
Danger of full thickness circumferential burns in an extremity as these may develop
compartment syndrome

Healing
Superficial burns: keratinocytes migrate to form a new layer over the burn site
Full thickness burns: dermal scarring. Usually need keratinocytes from skin grafts to
provide optimal coverage.

Rate question:

Next question

Comment on this question

All contents of this site are 2012 E-Medical Revision Ltd

http://www.emrcs.com/question/question.php?q=0

Terms and Conditions

Privacy policy

1/1

13/07/2015

Reference ranges

Previous

Question 7 of 304

Next

A 42 year old man from Southern India presents with chronic swelling of both lower legs, they
are brawny and indurated with marked skin trophic changes. Which of the following
organisms is the most likely origin of this disease process?

Question stats

End and review

Score: 85.7%
1-3 3 / 3

11.9%

48.9%

18.7%

12.9%

7.6%

4
5

A. Loa loa
48.9% of users answered this
question correctly

B. Wuchereria bancrofti
C. Trypanosoma cruzi
D. Trypanosoma gambiense

Search

E. None of the above

Go

Next question

W. Bancrofti is the commonest cause of filariasis leading to lymphatic obstruction. Infection


with Loa loa typically occurs in the African sub continent and usually results in generalised
sub cutaneous infections without lymphatic obstruction. Trypanosomal infections would not
produce this clinical picture.
Wuchereria bancrofti

Parasitic filarial nematode


Accounts for 90% of cases of filariasis
Usually diagnosed by blood smears
Usually transmitted by mosquitos
Treatment is with diethylcarbamazine

Rate question:

Next question

Comment on this question

All contents of this site are 2012 E-Medical Revision Ltd

http://www.emrcs.com/question/question.php?q=0

Terms and Conditions

Privacy policy

1/1

13/07/2015

Reference ranges

Previous

Question 8 of 304

Next

A 45 year old lady has recently undergone a thyroidectomy for treatment of medullary thyroid
cancer. Which of the following tumour markers is used clinically to screen for recurrence?

A. Free T3
B. Thyroglobulin

Question stats

Score: 87.5%
1-3 3 / 3

10.9%

17.7%

53.1%

9.1%

9.2%

53.1% of users answered this


question correctly

C. Calcitonin

End and review

4
5

D. Free T4
E. Thyroid stimulating hormone

Search
Go

Next question

Theme from 2011 Exam


Calcitonin is clinically utilised to screen for medullary thyroid cancer recurrence. Thyroid
function testing does not form part of either diagnosis or follow up from a malignancy
perspective. However, routine assessment of TSH may be needed in patients on thyroxine.
Thyroid malignancy
Papillary carcinoma
Commonest sub-type
Accurately diagnosed on fine needle aspiration cytology
Histologically, they may demonstrate psammoma bodies (areas of calcification) and so
called 'orphan Annie' nuclei
They typically metastasise via the lymphatics and thus laterally located apparently
ectopic thyroid tissue is usually a metastasis from a well differentiated papillary
carcinoma

Follicular carcinoma
Are less common than papillary lesions
Like papillary tumours, they may present as a discrete nodule. Although they appear
to be well encapsulated macroscopically there is invasion on microscopic evaluation
Lymph node metastases are uncommon and these tumours tend to spread
haematogenously. This translates into a higher mortality rate
Follicular lesions cannot be accurately diagnosed on fine needle aspiration cytology
and thus all follicular FNA's (THY 3) will require at least a hemi thyroidectomy

Anaplastic carcinoma
Less common and tend to occur in elderly females
Disease is usually advanced at presentation and often only palliative decompression
and radiotherapy can be offered.

Medullary carcinoma
These are tumours of the parafollicular cells ( C Cells) and are of neural crest origin.
The serum calcitonin may be elevated which is of use when monitoring for recurrence.
They may be familial and occur as part of the MEN -2A disease spectrum.
Spread may be either lymphatic or haematogenous and as these tumours are not
derived primarily from thyroid cells they are not responsive to radioiodine.

Lymphoma
These respond well to radiotherapy
Radical surgery is unnecessary once the disease has been diagnosed on biopsy
material. Such biopsy material is not generated by an FNA and thus a core biopsy has
to be obtained (with care!).

Rate question:

Next question

Comment on this question

All contents of this site are 2012 E-Medical Revision Ltd

http://www.emrcs.com/question/question.php?q=0

Terms and Conditions

Privacy policy

1/1

13/07/2015

Reference ranges

Previous

Question 9 of 304

Next

A 22 year old man is kicked in the head during a rugby match. He is temporarily concussed,
but then regains consciousness. Half an hour later he develops slurred speech, ataxia and
loses consciousnesses. On arrival in hospital he is intubated and ventilated. A CT Scan is
performed which shows an extradural haematoma. What is the most likely cause?

Question stats

Score: 88.9%
1-3 3 / 3

9.2%

64.6%

8.5%

7.9%

9.8%

64.6% of users answered this


question correctly

A. Basilar artery laceration

End and review

4
5

8
9

B. Middle meningeal artery laceration


C. Laceration of the sigmoid sinus
Search

D. Laceration of the anterior cerebral artery


E. Laceration of the middle cerebral artery

Go

Next question

Theme from April 2014 Exam


The most likely vessel from those in the list to cause an acute extra dural haemorrhage is the
middle meningeal artery. The anterior and middle cerebral arteries may cause acute sub
dural haemorrhage. Acute sub dural haemorrhages usually take slightly longer to evolve
than acute extra dural haemorrhages.
Middle meningeal artery

Middle meningeal artery is typically the third branch of the first part of the maxillary
artery, one of the two terminal branches of the external carotid artery. After branching
off the maxillary artery in the infratemporal fossa, it runs through the foramen
spinosum to supply the dura mater (the outermost meninges) .
The middle meningeal artery is the largest of the three (paired) arteries which supply
the meninges, the others being the anterior meningeal artery and the posterior
meningeal artery.
The middle meningeal artery runs beneath the pterion. It is vulnerable to injury at this
point, where the skull is thin. Rupture of the artery may give rise to an extra dural
hematoma.
In the dry cranium, the middle meningeal, which runs within the dura mater
surrounding the brain, makes a deep indention in the calvarium.
The middle meningeal artery is intimately associated with the auriculotemporal nerve
which wraps around the artery making the two easily identifiable in the dissection of
human cadavers and also easily damaged in surgery.

Rate question:

Next question

Comment on this question

All contents of this site are 2012 E-Medical Revision Ltd

http://www.emrcs.com/question/question.php?q=0

Terms and Conditions

Privacy policy

1/1

13/07/2015

Reference ranges

Previous

Question 10 of 304

Next

Question stats

End and review

Score: 80%
1-3 3 / 3

15.8%

13.3%

20.7%

A. Altered macrophages

9.5%

B. Fused macrophages

40.8%

C. Epithelioid cells

40.8% of users answered this


question correctly

Which of the following is not characteristic of a granuloma?

D. Mixture of chronic inflammatory cells

4
5

8
9
10

E. Polymorphnuclear leucocytes, cellular debris and fibrin


Search
Next question
Go

These are typical components of an abscess cavity. Polymorphonuclear leucocytes may be


found in a granuloma if there is a focus of suppuration.
Chronic inflammation
Overview
Chronic inflammation may occur secondary to acute inflammation.In most cases chronic
inflammation occurs as a primary process. These may be broadly viewed as being one of
three main processes:
Persisting infection with certain organisms such as Mycobacterium tuberculosis which
results in delayed type hypersensitivity reactions and inflammation.
Prolonged exposure to non-biodegradable substances such as silica or suture
materials which may induce an inflammatory response.
Autoimmune conditions involving antibodies formed against host antigens.

Acute vs. Chronic inflammation


Acute inflammation

Chronic inflammation

Changes to existing vascular structure and increased


permeability of endothelial cells

Angiogenesis predominates

Infiltration of neutrophils

Macrophages, plasma cells and


lymphocytes predominate

Process may resolve with:

Healing by fibrosis is the main result

Suppuration
Complete resolution
Abscess formation
Progression to chronic inflammation
Healing by fibrosis

Granulomatous inflammation
A granuloma consists of a microscopic aggregation of macrophages (with epithelial type
arrangement =epitheliod). Large giant cells may be found at the periphery of granulomas.
Mediators
Growth factors released by activated macrophages include agents such as interferon and
fibroblast growth factor (plus many more). Some of these such as interferons may have
systemic features resulting in systemic symptoms and signs, which may be present in
individuals with long standing chronic inflammation.
The finding of granulomas is pathognomonic of chronic inflammation, as illustrated in this
biopsy from a patient with colonic Crohns disease

Image sourced from Wikipedia

http://www.emrcs.com/question/question.php?q=0

1/2

13/07/2015

Rate question:

Next question

Comment on this question

All contents of this site are 2012 E-Medical Revision Ltd

http://www.emrcs.com/question/question.php?q=0

Terms and Conditions

Privacy policy

2/2

13/07/2015

Reference ranges

Previous

Question 11 of 304

Next

A 42 year old man presents with a painless lump in the left testicle that he noticed on self
examination. Clinically there is a firm nodule in the left testicle, ultrasound appearances show
an irregular mass lesion. His serum AFP and HCG levels are both within normal limits. What
is the most likely diagnosis?

Question stats

Score: 72.7%
1-3 3 / 3

11.5%

45.6%

17.8%

7.8%

17.3%

45.6% of users answered this


question correctly

A. Yolk sack tumour

End and review

4
5

8
9

B. Seminoma
10

C. Testicular teratoma
11

Search

D. Epididymo-orchitis
E. Adenomatoid tumour

Go

Next question

Seminomas typically have normal AFP and HCG. These are usually raised in teratomas
and yolk sac tumours

This man's age, presenting symptoms and normal tumour markers make a seminoma the
most likely diagnosis. Epididymo-orchitis does not produce irregular mass lesions which are
painless.
Testicular disorders
Testicular cancer
Testicular cancer is the most common malignancy in men aged 20-30 years. Around 95% of
cases of testicular cancer are germ-cell tumours. Germ cell tumours may essentially be
divided into:
Tumour type

Seminoma

Non seminomatous
germ cell tumours
(42%)
Teratoma
Yolk sac tumour
Choriocarcinoma
Mixed germ cell
tumours (10%)

Key features

Commonest
subtype
(50%)
Average
age at
diagnosis =
40
Even
advanced
disease
associated
with 5 year
survival of
73%
Younger age at
presentation =2030 years
Advanced
disease carries
worse prognosis
(48% at 5 years)
Retroperitoneal
lymph node
dissection may be
needed for residual
disease after
chemotherapy

Tumour
markers

Pathology

AFP usually
normal
HCG elevated
in 10%
seminomas
Lactate
dehydrogenase;
elevated in 1020%
seminomas (but
also in many
other
conditions)

Sheet like lobular patterns


of cells with substantial
fibrous component. Fibrous
septa contain lymphocytic
inclusions and granulomas
may be seen.

AFP elevated
in up to 70% of
cases
HCG elevated
in up to 40% of
cases
Other
markers rarely
helpful

Heterogenous texture with


occasional ectopic tissue
such as hair

Image demonstrating a classical seminoma, these tumours are typically more uniform than
teratomas

http://www.emrcs.com/question/question.php?q=0

1/3

13/07/2015

Image sourced from Wikipedia

Risk factors for testicular cancer


Cryptorchidism
Infertility
Family history
Klinefelter's syndrome
Mumps orchitis

Features
A painless lump is the most common presenting symptom
Pain may also be present in a minority of men
Other possible features include hydrocele, gynaecomastia

Diagnosis
Ultrasound is first-line
CT scanning of the chest/ abdomen and pelvis is used for staging
Tumour markers (see above) should be measured

Management
Orchidectomy (Inguinal approach)
Chemotherapy and radiotherapy may be given depending on staging
Abdominal lesions >1cm following chemotherapy may require retroperitoneal lymph
node dissection.

Prognosis is generally excellent


5 year survival for seminomas is around 95% if Stage I
5 year survival for teratomas is around 85% if Stage I

Benign disease
Epididymo-orchitis
Acute epididymitis is an acute inflammation of the epididymis, often involving the testis and
usually caused by bacterial infection.
Infection spreads from the urethra or bladder. In men <35 years, gonorrhoea or
chlamydia are the usual infections.
Amiodarone is a recognised non infective cause of epididymitis, which resolves on
stopping the drug.
Tenderness is usually confined to the epididymis, which may facilitate differentiating it
from torsion where pain usually affects the entire testis.

Testicular torsion

http://www.emrcs.com/question/question.php?q=0

2/3

13/07/2015

Twist of the spermatic cord resulting in testicular ischaemia and necrosis.


Most common in males aged between 10 and 30 (peak incidence 13-15 years)
Pain is usually severe and of sudden onset.
Cremasteric reflex is lost and elevation of the testis does not ease the pain.
Treatment is with surgical exploration. If a torted testis is identified then both testis
should be fixed as the condition of bell clapper testis is often bilateral.

Hydrocele
Presents as a mass that transilluminates, usually possible to "get above" it on
examination.
In younger men it should be investigated with USS to exclude tumour.
In children it may occur as a result of a patent processus vaginalis.
Treatment in adults is with a Lords or Jabouley procedure.
Treatment in children is with trans inguinal ligation of PPV.

Rate question:

Next question

Comment on this question

All contents of this site are 2012 E-Medical Revision Ltd

http://www.emrcs.com/question/question.php?q=0

Terms and Conditions

Privacy policy

3/3

13/07/2015

Reference ranges

Previous

Question 12 of 304

Next

A baby is born by normal vaginal delivery at 39 weeks gestation. Initially all appears well and
then the clinical staff become concerned because the baby develops recurrent episodes of
cyanosis. These are worse during feeding and improve dramatically when the baby cries.
The most likely underlying diagnosis is:

Question stats

Score: 75%
1-3 3 / 3

32.8%

8.5%

25.5%

20.6%

12.6%

32.8% of users answered this


question correctly

A. Choanal atresia

End and review

4
5

8
9

B. Oesophageal reflux
10

C. Tetralogy of Fallot
11

Search

D. Oesophageal atresia

12

E. Congenital diaphragmatic hernia

Go

Next question

Theme from 2011 exam


Theme from April 2013 Exam
In Choanal atresia the episodes of cyanosis are usually worst during feeding. Improvement
may be seen when the baby cries as the oropharyngeal airway is used.
Choanal atresia

Congenital disorder with an incidence of 1 in 7000 births.


Posterior nasal airway occluded by soft tissue or bone.
Associated with other congenital malformations e.g. coloboma
Babies with unilateral disease may go unnoticed.
Babies with bilateral disease will present early in life as they are obligate mouth
breathers.
Treatment is with fenestration procedures designed to restore patency.

Rate question:

Next question

Comment on this question

All contents of this site are 2012 E-Medical Revision Ltd

http://www.emrcs.com/question/question.php?q=0

Terms and Conditions

Privacy policy

1/1

13/07/2015

Reference ranges

Previous

Question 13 of 304

Next

A 28 year old lady presents with a pigmented lesion on her calf. Excisional biopsy confirms a
diagnosis of melanoma measuring 1cm in diameter with a Breslow thickness of 0.1mm. The
lesion is less than 1 mm at all resection margins. Which of the following surgical resection
margins is acceptable for this lesion?

Question stats

Score: 71.4%
1-3 3 / 3

9.7%

41.8%

18.2%

18.9%

11.3%

41.8% of users answered this


question correctly

A. 5 cm

End and review

4
5

8
9

B. 1 cm
10

C. 0.5 cm
11

Search

D. 2 cm

12

E. 3 cm

Go

13
Next question

Malignant melanoma

The main diagnostic features (major criteria):

Secondary features (minor criteria)

Change in size
Change in shape
Change in colour

Diameter >6mm
Inflammation
Oozing or bleeding
Altered sensation

Treatment
Suspicious lesions should undergo excision biopsy. The lesion should be removed in
completely as incision biopsy can make subsequent histopathological assessment
difficult.
Once the diagnosis is confirmed the pathology report should be reviewed to determine
whether further re-excision of margins is required (see below):

Margins of excision-Related to Breslow thickness


Lesions 0-1mm thick

1cm

Lesions 1-2mm thick

1- 2cm (Depending upon site and pathological features)

Lesions 2-4mm thick

2-3 cm (Depending upon site and pathological features)

Lesions >4 mm thick

3cm

Marsden J et al Revised UK guidelines for management of Melanoma. Br J Dermatol 2010


163:238-256.
Further treatments such as sentinel lymph node mapping, isolated limb perfusion and block
dissection of regional lymph node groups should be selectively applied.
Rate question:

Next question

Comment on this question

All contents of this site are 2012 E-Medical Revision Ltd

http://www.emrcs.com/question/question.php?q=0

Terms and Conditions

Privacy policy

1/1

13/07/2015

Reference ranges

Question 1 of 291

Next

A 20 year old man is involved in a road traffic accident. Following the incident he is unable to
extend his wrist. However, this improves over the following weeks. Which type of injury is he
most likely to have sustained?

Question stats

End and review

Score: 100%

12.7%

65%

6.6%

8.2%

7.6%

A. Radial nerve neurotmesis


65% of users answered this
question correctly

B. Radial nerve neuropraxia


C. Axillary nerve axonotmesis
D. Ulnar nerve neuropraxia

Search

E. Ulnar nerve axonotmesis

Go

Next question

Theme from April 2011 Exam


Transient loss of function makes neuropraxia the most likely injury. The wrist extensors are
innervated by the radial nerve making this the most likely site of injury.
Neuropraxia

Nerve intact but electrical conduction is affected


Myelin sheath integrity is preserved
Full recovery
Autonomic function preserved
Wallerian degeneration does not occur

Rate question:

Next question

Comment on this question

All contents of this site are 2012 E-Medical Revision Ltd

http://www.emrcs.com/question/question.php?q=0

Terms and Conditions

Privacy policy

1/1

13/07/2015

Reference ranges

Previous

Question 2 of 291

Next

A 53 year old lady has undergone a bilateral breast augmentation procedure many years
previously. The implants are tense and uncomfortable and are removed. During their
removal the surgeon encounters a dense membrane surrounding the implants, it has a
coarse granular appearance. The tissue is sent for histology and it demonstrates fibrosis
with the presence of calcification. The underlying process responsible for these changes is:

Question stats

End and review

Score: 100%

11.2%

10.7%

10.8%

58.2%

9.1%

1
2

58.2% of users answered this


question correctly

A. Hyperplasia
B. Dysplasia
C. Metastatic calcification

Search

D. Dystrophic calcification

Go

E. Necrosis
Next question

Breast implants often become surrounded by a pseudocapsule and this may secondarily
then be subjected to a process of dystrophic calcification.
Pathological calcification

Dystrophic
calcification

Deposition of calcium deposits in tissues that have undergone degeneration,


damage or disease in the presence of normal serum calcium levels

Metastatic
calcification

Deposition of calcium deposits in tissues that are otherwise normal in the


presence of increased serum calcium levels

Rate question:

Next question

Comment on this question

All contents of this site are 2012 E-Medical Revision Ltd

http://www.emrcs.com/question/question.php?q=0

Terms and Conditions

Privacy policy

1/1

13/07/2015

Reference ranges

Previous

Question 3 of 291

Next

A 4 year old girl presents with symptoms of right sided loin pain, lethargy and haematuria. On
examination she is pyrexial and has a large mass in the right upper quadrant. The most likely
underlying diagnosis is:

Question stats

End and review

Score: 66.7%
1

23.6%

54.1%

5.9%

9.7%

6.7%

A. Perinephric abscess
54.1% of users answered this
question correctly

B. Nephroblastoma
C. Renal cortical adenoma
D. Grawitz tumour

Search

E. Squamous cell carcinoma of the kidney

Go

Next question

In a child of this age, with the symptoms described a nephroblastoma is the most likely
diagnosis. A perinephric abscess is most unlikely. If an abscess were to occur it would be
confined to Gerotas fascia in the first instance, and hence anterior extension would be
unlikely.
Nephroblastoma
Nephroblastoma (Wilms tumours)
Usually present in first 4 years of life
May often present as a mass associated with haematuria (pyrexia may occur in 50%)
Often metastasise early (usually to lung)
Treated by nephrectomy
Younger children have better prognosis (<1 year of age =80% overall 5 year survival)

Rate question:

Next question

Comment on this question

All contents of this site are 2012 E-Medical Revision Ltd

http://www.emrcs.com/question/question.php?q=0

Terms and Conditions

Privacy policy

1/1

13/07/2015

Reference ranges

Previous1 / 3

Question 4-6 of 291

Next

Question stats

End and review

Score: 44.4%

Average score for registered users:

1
2

Theme: Thyroid neoplasms


A. Follicular carcinoma

84.5%

42.8%

52.7%

3
4-6 1 / 3

B. Anaplastic carcinoma
C. Medullary carcinoma
D. Papillary carcinoma

Search

E. Lymphoma

Go

F. Hashimotos thyroiditis
G. Graves disease
For the following histological descriptions please select the most likely underlying thyroid
neoplasm. Each option may be used once, more than once or not at all.

4.

A 22 year old female undergoes a thyroidectomy. The resected specimen shows


a non encapsulated tumour with papillary projections and pale empty nuclei.
Papillary carcinoma
Theme from April 2012
The presence of papillary structures together with the cytoplasmic features
described is strongly suggestive of papillary carcinoma. They are seldom
encapsulated.

5.

A thyroidectomy specimen from a 43 year old lady shows a mass with prominent
oxyphil cells and scanty thyroid colloid.
You answered Medullary carcinoma
The correct answer is Follicular carcinoma
Hurthle cell tumours are a variant of follicular neoplasms in which oxyphil cells
predominate. They have a poorer prognosis than conventional follicular
neoplasms

6.

A 32 year old lady undergoes a thyroidectomy for a mild goitre. The resected
specimen shows an intense lymphocytic infiltrate with acinar destruction and
fibrosis.
You answered Anaplastic carcinoma
The correct answer is Hashimotos thyroiditis
Lymphocytic infiltrates and fibrosis are typically seen in Hashimotos thyroiditis. In
Lymphoma only dense lymphatic type tissue is usually present.

Next question

Thyroid neoplasms

Lesion
Follicular
adenoma

Papillary
carcinoma

Follicular
carcinoma

Common features
Usually present as a solitary thyroid nodule
Malignancy can only be excluded on formal histological assessment

Usually contain a mixture of papillary and colloidal filled follicles


Histologically tumour has papillary projections and pale empty nuclei
Seldom encapsulated
Lymph node metastasis predominate
Haematogenous metastasis rare
Account for 60% of thyroid cancers

May appear macroscopically encapsulated, microscopically capsular


invasion is seen. Without this finding the lesion is a follicular adenoma.
Vascular invasion predominates

http://www.emrcs.com/question/question.php?q=0

1/2

13/07/2015

Multifocal disease rare


Account for 20% of all thyroid cancers
Anaplastic
carcinoma

Medullary
carcinoma

Most common in elderly females


Local invasion is a common feature
Account for 10% of thyroid cancers
Treatment is by resection where possible, palliation may be achieved
through isthmusectomy and radiotherapy. Chemotherapy is ineffective.

Tumours of the parafollicular cells (C Cells)


C cells derived from neural crest and not thyroid tissue
Serum calcitonin levels often raised
Familial genetic disease accounts for up to 20% cases
Both lymphatic and haematogenous metastasis are recognised, nodal
disease is associated with a very poor prognosis.

Rate question:

Next question

Comment on this question

All contents of this site are 2012 E-Medical Revision Ltd

http://www.emrcs.com/question/question.php?q=0

Terms and Conditions

Privacy policy

2/2

13/07/2015

Reference ranges

Question 7 of 291

Previous

Next

Question stats

Score: 50%

13.4%

10.8%

11.1%

A. Neutrophils

52.9%

B. Myofibroblasts

11.8%

C. Fibroblasts

52.9% of users answered this


question correctly

From which of the following cell types do giant cells most commonly originate?

D. Macrophages

End and review

1
2
3
4-6 1 / 3
7

E. Goblet cells
Search
Next question
Go

Theme from September 2011 and 2009 Exam


Although many cell types may give rise to giant cells, macrophages remain the most
common.
Giant cells

A giant cell is a mass formed by the union of several distinct types of cells
They are most commonly comprised of macrophages
They are different to granulomas although causative agents may overlap

Section stained using haematoxylin and eosin showing giant cell reaction to suture material

Image sourced from Wikipedia

Rate question:

Next question

Comment on this question

All contents of this site are 2012 E-Medical Revision Ltd

http://www.emrcs.com/question/question.php?q=0

Terms and Conditions

Privacy policy

1/1

13/07/2015

Reference ranges

Question 1 of 284

Next

A 43 year old lady with hypertension is suspected of having a phaeochromocytoma. Which of


the following investigations is most likely to be beneficial in this situation?

A. Dexamethasone suppression test


B. Urinary 5-Hydroxyindoleacetic Acid (5-HIAA)

Question stats

End and review

Score: 100%

11.7%

24.5%

6.3%

7.8%

49.7%

49.7% of users answered this


question correctly

C. Histamine provocation test


D. Tyramine provocation test
E. Urinary vanillymandelic acid measurements

Search
Next question

Go

Theme from September 2011 Exam


Theme from September 2012 Exam
Urinary VMA measurements are not completely specific but constitute first line assessment.
Stimulation tests of any sort are not justified in first line assessments.
Phaeochromocytoma and adrenal lesions
Phaeochromocytoma
Neuroendocrine tumour of the chromaffin cells of the adrenal medulla. Hypertension and
hyperglycaemia are often found.
10% of cases are bilateral.
10% occur in children.
11% are malignant (higher when tumour is located outside the adrenal).
10% will not be hypertensive.

Familial cases are usually linked to the Multiple endocrine neoplasia syndromes (considered
under its own heading).
Most tumours are unilateral (often right sided) and smaller than 10cm.
Diagnosis
Urine analysis of vanillymandelic acid (VMA) is often used (false positives may occur e.g. in
patients eating vanilla ice cream!)
Blood testing for plasma metanephrine levels.
CT and MRI scanning are both used to localise the lesion.
Treatment
Patients require medical therapy first. An irreversible alpha adrenoreceptor blocker should
be given, although minority may prefer reversible blockade(1). Labetolol may be coadministered for cardiac chronotropic control. Isolated beta blockade should not be
considered as it will lead to unopposed alpha activity.
These patients are often volume depleted and will often require moderate volumes of intra
venous normal saline perioperatively.
Once medically optimised the phaeochromocytoma should be removed. Most
adrenalectomies can now be performed using a laparoscopic approach(2). The adrenals are
highly vascular structures and removal can be complicated by catastrophic haemorrhage in
the hands of the inexperienced. This is particularly true of right sided resections where the
IVC is perilously close. Should the IVC be damaged a laparotomy will be necessary and the
defect enclosed within a Satinsky style vascular clamp and the defect closed with prolene
sutures. Attempting to interfere with the IVC using any instruments other than vascular
clamps will result in vessel trauma and make a bad situation much worse.
Incidental adrenal lesions
Adrenal lesions may be identified on CT scanning performed for other reasons(3). Factors
suggesting benign disease on CT include(4):
Size less than 3cm
Homogeneous texture
Lipid rich tissue
Thin wall to lesion

All patients with incidental lesions should be managed jointly with an endocrinologist and full

http://www.emrcs.com/question/question.php?q=0

1/2

13/07/2015

work up as described above. Patients with functioning lesions or those with adverse
radiological features (Particularly size >3cm) should proceed to surgery.

References
1. Weingarten TN, Cata JP, O'Hara JF, Prybilla DJ, Pike TL, Thompson GB, et al.
Comparison of two preoperative medical management strategies for laparoscopic resection
of pheochromocytoma. Urology. 2010 Aug;76(2):508 e6-11.
2. Nguyen PH, Keller JE, Novitsky YW, Heniford BT, Kercher KW. Laparoscopic approach to
adrenalectomy: review of perioperative outcomes in a single center. Am Surg. 2011
May;77(5):592-6.
3. Ng VW, Ma RC, So WY, Choi KC, Kong AP, Cockram CS, et al. Evaluation of functional
and malignant adrenal incidentalomas. Arch Intern Med. 2010 Dec 13;170(22):2017-20.
4. Muth A, Hammarstedt L, Hellstrom M, Sigurjonsdottir HA, Almqvist E, Wangberg B. Cohort
study of patients with adrenal lesions discovered incidentally. Br J Surg. 2011 May 27.
Rate question:

Next question

Comment on this question

All contents of this site are 2012 E-Medical Revision Ltd

http://www.emrcs.com/question/question.php?q=0

Terms and Conditions

Privacy policy

2/2

13/07/2015

Reference ranges

Previous

Question 2 of 284

Next

A 46 year old lady presents with symptoms of diarrhoea, weight loss of 10 Kg and a skin rash
of erythematous blisters involving the abdomen and buttocks. The blisters have an irregular
border and both intact and ruptured vesicles. What is the most likely diagnosis?

Question stats

End and review

Score: 50%

11.6%

15.6%

27.3%

32.1%

13.4%

1
2

A. Colonic adenocarcinoma
32.1% of users answered this
question correctly

B. Pancreatic adenocarcinoma
C. Tropical sprue
D. Glucagonoma

Search

E. Insulinoma

Go

Next question

Theme from September 2011 Exam


Theme from September 2012 Exam
Theme from September 2013 Exam
Glucagonoma is strongly associated with necrolytic migratory erythema.
Glucagonoma

Rare pancreatic tumours arising from the alpha cells of the pancreas.
Glucagon levels markedly elevated.
Symptoms include diarrhoea, weight loss and necrolytic migratory erythema.
A serum level of glucagon >1000pg/ml usually suggests the diagnosis, imaging with
CT scanning is also required.
Treatment is with surgical resection. However, careful staging is required for these
tumours are usually malignant and non resectable.

Rate question:

Next question

Comment on this question

All contents of this site are 2012 E-Medical Revision Ltd

http://www.emrcs.com/question/question.php?q=0

Terms and Conditions

Privacy policy

1/1

13/07/2015

Reference ranges

Previous

Question 3 of 284

Next

A 56 year old man presents with symptoms of neuropathic facial pain and some weakness of
the muscles of facial expression on the right side. On examination he has a hard mass
approximately 6cm anterior to the right external auditory meatus. What is the most likely
diagnosis?

Question stats

End and review

Score: 66.7%
1

26.5%

20.5%

13.5%

33.1%

6.5%

33.1% of users answered this


question correctly

A. Pleomorphic adenoma
B. Adenocarcinoma
C. Mucoepidermoid carcinoma

Search

D. Adenoid cystic carcinoma


E. Lymphoma

Go

Next question

Theme from September 2011 Exam


The patient is most likely to have a malignant lesion within the parotid. Of the malignancies
listed; adenoid cystic carcinoma has the greatest tendency to perineural invasion.
Parotid gland malignancy

Most parotid neoplasms (80%) are benign lesions


Most commonly present with painless mass around the mandible
Up to 30% may present with pain, when this is associated with a discrete mass lesion
in the parotid it usually indicates perineural invasion.
Perineural invasion is very unlikely to occur in association with benign lesions
80% of patients with facial nerve weakness caused by parotid malignancies will have
nodal metastasis and a 5 year survival of 25%

Types of malignancy
Mucoepidermoid
carcinoma

30% of all parotid malignancies


Usually low potential for local invasiveness and metastasis (depends
mainly on grade)

Adenoid cystic
carcinoma

Unpredictable growth pattern


Tendency for perineural spread
Nerve growth may display skip lesions resulting in incomplete excision
Distant metastasis more common (visceral rather than nodal spread)
5 year survival 35%

Mixed tumours

Often a malignancy occurring in a previously benign parotid lesion

Acinic cell
carcinoma

Intermediate grade malignancy


May show perineural invasion
Low potential for distant metastasis
5 year survival 80%

Adenocarcinoma

Develops from secretory portion of gland


Risk of regional nodal and distant metastasis
5 year survival depends upon stage at presentation, may be up to 75% with
small lesions with no nodal involvement

Lymphoma

Large rubbery lesion, may occur in association with Warthins tumours


Diagnosis should be based on regional nodal biopsy rather than parotid
resection Treatment is with chemotherapy (and radiotherapy)

Rate question:

Next question

Comment on this question

All contents of this site are 2012 E-Medical Revision Ltd

http://www.emrcs.com/question/question.php?q=0

Terms and Conditions

Privacy policy

1/1

13/07/2015

Reference ranges

Previous

Question 4 of 284

Next

A 20 year old African lady undergoes an open appendicectomy. She is reviewed for an
unrelated problem 8 months later. On abdominal inspection the wound site is covered by
shiny dark protuberant scar tissue that projects beyond the limits of the skin incision. Which
of the following is the most likely underlying process?

Question stats

End and review

Score: 50%
1

11.1%

67.8%

7.6%

7.3%

6.2%

67.8% of users answered this


question correctly

A. Hypertrophic scar
B. Keloid scar
C. Marjolins ulcer

Search

D. Repeated episodes of wound sepsis


E. Mycosis fungoides

Go

Next question

Keloid scars extend beyond the limits of the incision. Mycosis fungoides is a cutaneous T cell
lymphoma.
Wound healing
Surgical wounds are either incisional or excisional and either clean, clean contaminated or
dirty. Although the stages of wound healing are broadly similar their contributions will vary
according to the wound type.
The main stages of wound healing include:
Haemostasis
Minutes to hours following injury
Vasospasm in adjacent vessels, platelet plug formation and generation of fibrin rich
clot.

Inflammation
Typically days 1-5
Neutrophils migrate into wound (function impaired in diabetes).
Growth factors released, including basic fibroblast growth factor and vascular
endothelial growth factor.
Fibroblasts replicate within the adjacent matrix and migrate into wound.
Macrophages and fibroblasts couple matrix regeneration and clot substitution.

Regeneration
Typically days 7 to 56
Platelet derived growth factor and transformation growth factors stimulate fibroblasts
and epithelial cells.
Fibroblasts produce a collagen network.
Angiogenesis occurs and wound resembles granulation tissue.

Remodeling
From 6 weeks to 1 year
Longest phase of the healing process and may last up to one year (or longer).
During this phase fibroblasts become differentiated (myofibroblasts) and these
facilitate wound contraction.
Collagen fibres are remodeled.
Microvessels regress leaving a pale scar.

The above description represents an idealised scenario. A number of diseases may distort
this process. Neovascularisation is an important early process. Endothelial cells may
proliferate in the wound bed and recanalise to form a vessel. Vascular disease, shock and
sepsis can all compromise microvascular flow and impair healing.
Conditions such as jaundice will impair fibroblast synthetic function and immunity with a
detrimental effect in most parts of the healing process.
Problems with scars:
Hypertrophic scars

http://www.emrcs.com/question/question.php?q=0

1/2

13/07/2015

Excessive amounts of collagen within a scar. Nodules may be present histologically


containing randomly arranged fibrils within and parallel fibres on the surface. The tissue itself
is confined to the extent of the wound itself and is usually the result of a full thickness dermal
injury. They may go on to develop contractures.
Image of hypertrophic scarring. Note that it remains confined to the boundaries of the original
wound:

Image sourced from Wikipedia

Keloid scars
Excessive amounts of collagen within a scar. Typically a keloid scar will pass beyond the
boundaries of the original injury. They do not contain nodules and may occur following even
trivial injury. They do not regress over time and may recur following removal.
Image of a keloid scar. Note the extension beyond the boundaries of the original incision:

Image sourced from Wikipedia

Drugs which impair wound healing:


Non steroidal anti inflammatory drugs
Steroids
Immunosupressive agents
Anti neoplastic drugs

Closure
Delayed primary closure is the anatomically precise closure that is delayed for a few days but
before granulation tissue becomes macroscopically evident.
Secondary closure refers to either spontaneous closure or to surgical closure after
granulation tissue has formed.
Rate question:

Next question

Comment on this question

All contents of this site are 2012 E-Medical Revision Ltd

http://www.emrcs.com/question/question.php?q=0

Terms and Conditions

Privacy policy

2/2

13/07/2015

Reference ranges

Question 5 of 284

Previous

Next

Question stats

End and review

Score: 60%
1

26.7%

12.8%

42.6%

A. Necrosis caused by expanding granulomas

9%

B. Ability to multiply within fibroblasts

8.9%

C. Delayed hypersensitivity reaction against bacteria

42.6% of users answered this


question correctly

The pathogenicity of the tubercle bacillus is due to which of the following?

D. Effect of antibody response


E. Direct toxic effect on host cells

Search
Next question
Go

Mycobacteria stimulate a specific T cell response of cell mediated immunity. This is effective
in reducing the infection, the delayed hypersensitivity also damages tissues. Necrosis occurs
in TB but is usually within the granuloma.
Tuberculosis pathology

Is a form of primary chronic inflammation, caused by the inability of macrophages to kill


the Mycobacterium tuberculosis.
The macrophages often migrate to regional lymph nodes, the lung lesion plus affected
lymph nodes is referred to as a Ghon complex.
This leads to the formation of a granuloma which is a collection of epithelioid
histiocytes.
There is the presence of caseous necrosis in the centre.
The inflammatory response is mediated by a type 4 hypersensitivity reaction.
In healthy individuals the disease may be contained, in the immunocompromised
disseminated (miliary TB) may occur.

Diagnosis
Waxy membrane of mycobacteria prevents binding with normal stains. Ziehl - Neelsen
staining is typically used.
Culture based methods take far longer.

Image showing acid- alcohol fast mycobacteria stained using the Ziehl- Neelsen method

Image sourced from Wikipedia

Rate question:

Next question

Comment on this question

All contents of this site are 2012 E-Medical Revision Ltd

http://www.emrcs.com/question/question.php?q=0

Terms and Conditions

Privacy policy

1/1

13/07/2015

Reference ranges

Question 6 of 284

Previous

Next

A 45 year old women with a thyroid carcinoma undergoes a total thyroidectomy. The post
operative histology report shows a final diagnosis of medullary type thyroid cancer. Which of
the tests below is most likely to be of clinical use in screening for disease recurrence?

Question stats

Score: 57.1%
1

7.7%

10%

7.5%

68.8%

6%

68.8% of users answered this


question correctly

A. Serum CA 19-9 Levels


B. Serum thyroglobulin levels

End and review

C. Serum PTH levels


D. Serum calcitonin levels
Search

E. Serum TSH levels

Go

Next question

Theme from September 2012 Exam


Medullary thyroid cancers often secrete calcitonin and monitoring the serum levels of this
hormone is useful in detecting sub clinical recurrence.
Thyroid neoplasms

Lesion

Common features

Follicular
adenoma

Papillary
carcinoma

Follicular
carcinoma

Anaplastic
carcinoma

Medullary
carcinoma

Usually present as a solitary thyroid nodule


Malignancy can only be excluded on formal histological assessment

Usually contain a mixture of papillary and colloidal filled follicles


Histologically tumour has papillary projections and pale empty nuclei
Seldom encapsulated
Lymph node metastasis predominate
Haematogenous metastasis rare
Account for 60% of thyroid cancers

May appear macroscopically encapsulated, microscopically capsular


invasion is seen. Without this finding the lesion is a follicular adenoma.
Vascular invasion predominates
Multifocal disease rare
Account for 20% of all thyroid cancers

Most common in elderly females


Local invasion is a common feature
Account for 10% of thyroid cancers
Treatment is by resection where possible, palliation may be achieved
through isthmusectomy and radiotherapy. Chemotherapy is ineffective.

Tumours of the parafollicular cells (C Cells)


C cells derived from neural crest and not thyroid tissue
Serum calcitonin levels often raised
Familial genetic disease accounts for up to 20% cases
Both lymphatic and haematogenous metastasis are recognised, nodal
disease is associated with a very poor prognosis.

Rate question:

Next question

Comment on this question

All contents of this site are 2012 E-Medical Revision Ltd

http://www.emrcs.com/question/question.php?q=0

Terms and Conditions

Privacy policy

1/1

13/07/2015

Reference ranges

Question 7 of 284

Previous

Next

A 15 year old boy undergoes an emergency splenectomy for trauma. He makes a full
recovery and is discharged home. Eight weeks post operatively the general practitioner
performs a full blood count with a blood film. Which of the following is most likely to be
present?

Question stats

Score: 62.5%
1

8.8%

63.7%

10.4%

10.1%

7.1%

63.7% of users answered this


question correctly

A. Myofibroblasts

End and review

B. Howell-Jolly bodies

6
7

C. Multinucleate giant cells


Search

D. Reed Sternberg Cells


E. None of the above

Go

Next question

Post splenectomy blood film features:


Howell- Jolly bodies
Pappenheimer bodies
Target cells
Irregular contracted erythrocytes

As the filtration function is the spleen is no longer present Howell-Jolly bodies are found.
Post splenectomy blood film changes
The loss of splenic tissue results in the inability to readily remove immature or abnormal red
blood cells from the circulation. The red cell count does not alter significantly. However,
cytoplasmic inclusions may be seen e.g. Howell-Jolly bodies.
In the first few days after splenectomy target cells, siderocytes and reticulocytes will appear
in the circulation. Immediately following splenectomy a granulocytosis (mainly composed of
neutrophils) is seen, this is replaced by a lymphocytosis and monocytosis over the following
weeks.
The platelet count is usually increased and this may be persistent, oral antiplatelet agents
may be needed in some patients.
Image showing Howell Jolly bodies (arrowed)

Image sourced from Wikipedia

Rate question:

Next question

Comment on this question

All contents of this site are 2012 E-Medical Revision Ltd

http://www.emrcs.com/question/question.php?q=0

Terms and Conditions

Privacy policy

1/1

13/07/2015

Reference ranges

Previous

Question 8 of 284

Next

A 43 year old women is identified as being a carrier of a BRCA 1 mutation. Apart from breast
cancer, which of the following malignancies is she at greatest risk of developing?

A. Colonic cancer
B. Ovarian cancer
C. Follicular carcinoma of the thyroid

Question stats

End and review

Score: 66.7%
1

13%

65.1%

8.2%

7.6%

6%

65.1% of users answered this


question correctly

D. Pituitary adenoma

7
8

E. Phaeochromocytoma

Search
Next question

Go

BRCA 1 mutation patients are 55% more likely to get ovarian cancer. Those with BRCA 2 are
25% more likely. The risk of developing other malignancies is slightly increased but not to the
same extent, and not enough to justify screening.
Genetics and surgical disease
Some of the more commonly occurring genetic conditions occurring in surgical patients are
presented here.
Li-Fraumeni Syndrome
Autosomal dominant
Consists of germline mutations to p53 tumour suppressor gene
High incidence of malignancies particularly sarcomas and leukaemias
Diagnosed when:

*Individual develops sarcoma under 45 years


*First degree relative diagnosed with any cancer below age 45 years and another family
member develops malignancy under 45 years or sarcoma at any age
BRCA 1 and 2
Carried on chromosome 17 (BRCA 1) and Chromosome 13 (BRCA 2)
Linked to developing breast cancer (60%) risk.
Associated risk of developing ovarian cancer (55% with BRCA 1 and 25% with BRCA
2).

Lynch Syndrome
Autosomal dominant
Develop colonic cancer and endometrial cancer at young age
80% of affected individuals will get colonic and/ or endometrial cancer
High risk individuals may be identified using the Amsterdam criteria

Amsterdam criteria
Three or more family members with a confirmed diagnosis of colorectal cancer, one of whom
is a first degree (parent, child, sibling) relative of the other two.
Two successive affected generations.
One or more colon cancers diagnosed under age 50 years.
Familial adenomatous polyposis (FAP) has been excluded.
Gardners syndrome
Autosomal dominant familial colorectal polyposis
Multiple colonic polyps
Extra colonic diseases include: skull osteoma, thyroid cancer and epidermoid cysts
Desmoid tumours are seen in 15%
Mutation of APC gene located on chromosome 5
Due to colonic polyps most patients will undergo colectomy to reduce risk of colorectal
cancer
Now considered a variant of familial adenomatous polyposis coli

Rate question:

Next question

Comment on this question

http://www.emrcs.com/question/question.php?q=0

1/2

13/07/2015

All contents of this site are 2012 E-Medical Revision Ltd

http://www.emrcs.com/question/question.php?q=0

Terms and Conditions

Privacy policy

2/2

13/07/2015

Reference ranges

Previous

Question 9 of 284

Next

A 53 year old man is due to undergo a splenectomy as a treatment for refractory haemolytic
anaemia. The underlying pathological basis for haemolytic anaemia is thought to be a Type
2 hypersensitivity response. Which of the following mechanisms best describes this process

Question stats

Score: 72.7%
1

13.1%

29.1%

10.1%

42.5%

5.2%

A. Deposition of immune complexes


42.5% of users answered this
question correctly

B. Cell mediated immune response

End and review

6
7

C. IgE mediated response


8

D. Formation of autoantibodies against cell surface antigens


9

Search

E. None of the above

Go

Next question

Mnemonic for the reactions and the mediators involved


ACID EGG-T
Type 1 Anaphylactic
Type 2 Cytotoxic
Type 3 Immune complex
Type 4 Delayed type
EGG T (mediators)
IgE
IgG
IgG
T cells

Type 2 hypersensitivity reactions (which includes haemolytic anaemia) are associated with
formation of antibody against cell surface antigens.
Hypersensitivity reactions
The Gell and Coombs classification divides hypersensitivity reactions into 4 types
Type I

Type II

Type III

Type IV

Description

Anaphylactic

Cytotoxic

Immune
complex

Delayed type

Mediator

IgE

IgG, IgM

IgG, Ig A,
IgM

T-cells

Antigen

Exogenous

Cell surface

Soluble

Tissues

Response
time

Minutes

Hours

Hours

2-3 days

Examples

Asthma
Hay fever

Autoimmune haemolytic
anaemia
Pemphigus
Goodpasture's

Serum
sickness
SLE
Aspergillosis

Graft versus host


disease
Contact dermatitis

Rate question:

Next question

Comment on this question

All contents of this site are 2012 E-Medical Revision Ltd

http://www.emrcs.com/question/question.php?q=0

Terms and Conditions

Privacy policy

1/1

13/07/2015

Reference ranges

Previous

Question 10 of 284

Next

Question stats

End and review

Score: 75%
1

10.7%

12%

20.3%

A. ras

39.8%

B. myc

17.2%

C. sis

39.8% of users answered this


question correctly

Which of the following is not an oncogene?

D. Ki 67
E. erb-B

6
7
8
9

Search
Next question
Go

10

Ki 67 is a nuclear proliferation marker (used in immunohistochemistry). Although, Ki67


positivity is a marker of malignancy, it is not itself, an oncogene.
Oncogenes
Oncogenes are cancer promoting genes that are derived from normal genes (protooncogenes). Proto-oncogenes play an important physiological role in cellular growth. They
are implicated in the development of up to 20% of human cancers.
Proto-oncogenes may become oncogenes via the following processes:
Mutation (point mutation)
Chromosomal translocation
Increased protein expression
Only one mutated copy of the gene is needed for cancer to occur - a dominant effect
Classification of oncogenes
Growth factors e.g. Sis
Transcription factors e.g. Myc
Receptor tyrosine kinase e.g. RET
Cytoplasmic tyrosine kinase e.g. Src
Regulatory GTPases e.g. Ras

Tumour supressor genes


Tumour supressor genes restrict or repress cellular proliferation in normal cells. Their
inactivation through mutation or germ line incorporation is implicated in renal, colonic, breast,
bladder and many other cancers. One of the best known tumour supressor genes is p53.
p53 gene offers protection by causing apoptosis of damaged cells. Other well known genes
include BRCA 1 and 2.
Rate question:

Next question

Comment on this question

All contents of this site are 2012 E-Medical Revision Ltd

http://www.emrcs.com/question/question.php?q=0

Terms and Conditions

Privacy policy

1/1

13/07/2015

Reference ranges

Previous

Question 11 of 284

Next

A 25 year old man is injured in a road traffic accident. His right tibia is fractured and is
managed by fasciotomies and application of an external fixator. Over the next 48 hours his
serum creatinine rises and urine is sent for microscopy, muddy brown casts are identified.
What is the most likely underlying diagnosis?

Question stats

Score: 71.4%
1

11.3%

63.6%

9.1%

8.3%

7.8%

63.6% of users answered this


question correctly

A. Acute interstitial nephritis

End and review

B. Acute tubular necrosis

6
7
8

C. Glomerulonephritis

Search

D. IgA Nephropathy
E. Thin basement membrane disease

Go

10
11

Next question

This patient is likely to have had compartment syndrome (tibial fracture + fasciotomies) which
may produce myoglobinuria. The presence of worsening renal function, together with muddy
brown casts is strongly suggestive of acute tubular necrosis. Acute interstitial nephritis
usually arises from drug toxicity and does not usually produce urinary muddy brown casts.
Thin basement membrane disease is an autosomal dominant condition that causes
persistent microscopic haematuria, but not worsening renal function.
Acute Renal Failure

Final pathway is tubular cell death.


Renal medulla is a relatively hypoxic environment making it susceptible to renal tubular
hypoxia.
Renovascular autoregulation maintains renal blood flow across a range of arterial
pressures.
Estimates of GFR are best indices of level of renal function. Useful clinical estimates
can be obtained by considering serum creatinine, age, race, gender and body size.
eGFR calculations such as the Cockcroft and Gault equation are less reliable in
populations with high GFR's.
Nephrotoxic stimuli such as aminoglycosides and radiological contrast media induce
apoptosis. Myoglobinuria and haemolysis result in necrosis. Overlap exists and
proinflammatory cytokines play and important role in potentiating ongoing damage.
Post-operative renal failure is more likely to occur in patients who are elderly, have
peripheral vascular disease, high BMI, have COPD, receive vasopressors, are on
nephrotoxic medication or undergo emergency surgery.
Avoiding hypotension will reduce risk of renal tubular damage.
There is no evidence that administration of ACE inhibitors or dopamine reduces the
incidence of post-operative renal failure.

Rate question:

Next question

Comment on this question

All contents of this site are 2012 E-Medical Revision Ltd

http://www.emrcs.com/question/question.php?q=0

Terms and Conditions

Privacy policy

1/1

13/07/2015

Reference ranges

Previous

Question 12 of 284

Next

A 56 year old man has undergone a radical nephrectomy. The pathologist bisects the kidney
and identifies a pink fleshy tumour in the renal pelvis. What is the most likely disease?

A. Renal cell carcinoma


B. Transitional cell carcinoma

Question stats

Score: 68.8%
1

25.7%

35.8%

21.3%

8%

9.2%

35.8% of users answered this


question correctly

C. Angiomyolipoma

End and review

D. Phaeochromocytoma

6
7
8

E. Renal adenoma

Search
Next question

Go

10
11
12

Most renal tumours are yellow or brown in colour. TCC's are one of the few tumours to
appear pink.

Theme from April 2012


The finding of a TCC in the renal pelvis mandates a nephroureterectomy.
Renal lesions

Lesion
Renal cell
carcinoma

Nephroblastoma

Neuroblastoma

Transitional cell
carcinoma

Angiomyolipoma

Disease specific features


Most present with haematuria
(50%)
Common renal tumour (85%
cases)
Paraneoplastic features include
hypertension and polycythaemia
Most commonly has
haematogenous mestastasis

Rare childhood tumour


It accounts for 80% of all
genitourinary malignancies in
those under the age of 15 years
Up to 90% will have a mass
50% will be hypertensive
Diagnostic work up includes
ultrasound and CT scanning

Most common extracranial


tumour of childhood
80% occur in those under 4
years of age
Tumour of neural crest origin (up
to 50% occur in the adrenal
gland)
The tumour is usually calcified
and may be diagnosed using
MIBG scanning
Staging is with CT

Accounts for 90% of lower urinary


tract tumour, but only 10% of
renal tumours
Males affected 3x more than
females
Occupational exposure to
industrial dyes and rubber
chemicals may increase risk
Up to 80% present with painless
haematuria
Diagnosis and staging is with CT
IVU

80% of these hamartoma type


lesions occur sporadically, the
remainder are seen in those with
tuberous sclerosis

http://www.emrcs.com/question/question.php?q=0

Treatment
Usually radical or partial
nephrectomy

Surgical resection combined with


chemotherapy (usually
vincristine, actinomycin D and
doxorubicin)

Surgical resection, radiotherapy


and chemotherapy

Radical nephroureterectomy

50% of patients with lesions


>4cm will have symptoms and
will require surgical resection

1/2

13/07/2015

Tumour is composed of blood


vessels, smooth muscle and fat
Massive bleeding may occur in
10% of cases

Rate question:

Next question

Comment on this question

All contents of this site are 2012 E-Medical Revision Ltd

http://www.emrcs.com/question/question.php?q=0

Terms and Conditions

Privacy policy

2/2

13/07/2015

Reference ranges

Previous

Question 13 of 284

Next

A 65 year old lady presents with a lesion affecting her right breast. On examination she has a
weeping, crusting lesion overlying the right nipple, the areolar region is not involved. There is
no palpable mass lesion in the breast, there is a palpable axillary lymph node. The patient's
general practitioner has tried treating the lesion with 1% hydrocortisone cream, with no
success. What is the most likely diagnosis?

Question stats

Score: 66.7%
1

11.5%

59.3%

11%

9.3%

8.8%

59.3% of users answered this


question correctly

A. Infection with Staphylococcus aureus

End and review

6
7
8

B. Pagets disease of the nipple


C. Phyllodes tumour

Search

D. Nipple eczema

Go

E. Basal cell carcinoma

10
11
12

Next question

13

A weeping, crusty lesion such as this is most likely to represent Pagets disease of the nipple
(especially since the areolar region is spared). Although no mass lesion is palpable, a
proportion of patients will still have an underlying invasive malignancy (hence the
lymphadenopathy).
Pagets disease of the nipple
Pagets disease is an eczematoid change of the nipple associated with an underlying breast
malignancy and it is present in 1-2% of patients with breast cancer. In half of these patients,
it is associated with an underlying mass lesion and 90% of such patients will have an invasive
carcinoma. 30% of patients without a mass lesion will still be found to have an underlying
carcinoma. The remainder will have carcinoma in situ.
Pagets disease differs from eczema of the nipple in that it involves the nipple primarily and
only latterly spreads to the areolar (the opposite occurs in eczema).
Diagnosis is made by punch biopsy, mammography and ultrasound of the breast.
Treatment will depend on the underlying lesion.
Rate question:

Next question

Comment on this question

All contents of this site are 2012 E-Medical Revision Ltd

http://www.emrcs.com/question/question.php?q=0

Terms and Conditions

Privacy policy

1/1

13/07/2015

Reference ranges

Previous

Question 14 of 284

Next

A 73 year old man presents with haemoptysis and is suspected of suffering from lung cancer.
On examination he has an enlarged supraclavicular lymph node. Which of the following
features is most likely to be present on histological examination?

Question stats

Score: 68.4%
1

56.2%

10.1%

11.2%

13.3%

9.3%

A. Increased mitoses
56.2% of users answered this
question correctly

B. Apoptosis

End and review

6
7

C. Barr Bodies
8

D. Multinucleate giant cells


9

Search

E. Granuloma

Go

10
11

Next question

12

Theme from 2011 Exam

13
14

Increased mitoses are commonly seen in association with malignant transformation of cells.
Apoptosis is not a common feature of metastatic cancer. Barr Bodies are formed during X
chromosome inactivation in female somatic cells.
Histopathology of malignancy

Abnormal tissue architecture


Coarse chromatin
Invasion of basement membrane*
Abnormal mitoses
Angiogenesis
De-differentiation
Areas of necrosis
Nuclear pleomorphism

*= Those features that distinguish invasive malignancy from in situ disease


Rate question:

Next question

Comment on this question

All contents of this site are 2012 E-Medical Revision Ltd

http://www.emrcs.com/question/question.php?q=0

Terms and Conditions

Privacy policy

1/1

13/07/2015

Reference ranges

Question 15 of 284

Previous

Next

Which of the following pathological explanations best describes the initial pathological
processes occurring in an abdominal aortic aneurysm in an otherwise well 65 year old,
hypertensive male?

Question stats

Score: 70%
1

16.1%

13.4%

21.1%

38.1%

11.3%

A. Loss of elastic fibres from the adventitia


38.1% of users answered this
question correctly

B. Loss of collagen from the adventitia

End and review

6
7

C. Loss of collagen from the media


8

D. Loss of elastic fibres from the media


9

Search

E. Decreased matrix metalloproteinases in the adventitia

Go

10
11

Next question

12

Theme from April 2012 Exam


Theme from April 2013 Exam
In established aneurysmal disease there is dilation of all layers of the arterial wall and loss of
both elastin and collagen. The primary event is loss of elastic fibres with subsequent
degradation of collagen fibres.

13
14
15

Pathology of abdominal aortic aneurysm


Abdominal aortic aneurysms occur primarily as a result of the failure of elastic proteins within
the extracellular matrix. Aneurysms typically represent dilation of all layers of the arterial wall.
Most aneurysms are caused by degenerative disease. After the age of 50 years the normal
diameter of the infrarenal aorta is 1.5cm in females and 1.7cm in males. Diameters of 3cm
and greater, are considered aneurysmal. The pathophysiology involved in the development
of aneurysms is complex and the primary event is loss of the intima with loss of elastic fibres
from the media. This process is associated with, and potentiated by, increased proteolytic
activity and lymphocytic infiltration.
Major risk factors for the development of aneurysms include smoking and hypertension. Rare
but important causes include syphilis and connective tissues diseases such as Ehlers Danlos
type 1 and Marfans syndrome.
Layers of arterial wall

Image sourced from Wikipedia

Rate question:

Next question

Comment on this question

All contents of this site are 2012 E-Medical Revision Ltd

http://www.emrcs.com/question/question.php?q=0

Terms and Conditions

Privacy policy

1/1

13/07/2015

Reference ranges

Question 17 of 284

Previous

Next

A 34 year old lady undergoes an elective cholecystectomy for attacks of recurrent


cholecystitis due to gallstones. Microscopic assessment of the gallbladder is most likely to
show which of the following?

Question stats

Score: 72.7%
1

14.5%

10.9%

41.2%

20.8%

12.6%

A. Dysplasia of the fundus


41.2% of users answered this
question correctly

B. Widespread necrosis

End and review

6
7

C. Ashoff-Rokitansky sinuses
8

D. Metaplasia of the fundus


9

Search

E. None of the above

Go

Next question

10
11
12

Aschoff-Rokitansky sinuses are the result of hyperplasia and herniation of epithelial cells
through the fibromuscular layer of the gallbladder wall. They may be macroscopic or
microscopic. Ashoff-Rokitansky sinuses may be identified in cases of chronic cholecystitis
and gallstones. Although gallstones may predispose to the development of gallbladder
cancer the actual incidence of dysplasia and metaplastic change is rare. In the elective
setting described above necrosis would be rare.

13
14
15
16
17

Gallbladder

Fibromuscular sac with capacity of 50ml


Columnar epithelium

Relations of the gallbladder


Anterior

Liver

Posterior

Covered by peritoneum
Transverse colon
1st part of the duodenum

Laterally

Right lobe of liver

Medially

Quadrate lobe of liver

Arterial supply
Cystic artery (branch of Right hepatic artery)
Venous drainage
Directly to the liver
Nerve supply
Sympathetic- mid thoracic spinal cord, Parasympathetic- anterior vagal trunk
Common bile duct
Origin

Confluence of cystic and common hepatic ducts

Relations at
origin

Medially - Hepatic artery


Posteriorly- Portal vein

Relations distally

Arterial supply

Duodenum - anteriorly
Pancreas - medially and laterally
Right renal vein - posteriorly
Branches of hepatic artery and retroduodenal branches of gastroduodenal
artery

Hepatobiliary triangle
Medially

Common hepatic duct

Inferiorly

Cystic duct

http://www.emrcs.com/question/question.php?q=0

1/2

13/07/2015

Superiorly

Inferior edge of liver

Contents

Cystic artery

Rate question:

Next question

Comment on this question

All contents of this site are 2012 E-Medical Revision Ltd

http://www.emrcs.com/question/question.php?q=0

Terms and Conditions

Privacy policy

2/2

13/07/2015

Reference ranges

Previous

Question 18 of 284

Next

Question stats

End and review

Score: 69.6%
1

11.3%

14.3%

17.3%

A. It consists of mutations to the p53 tumour suppressor gene

41.7%

B. Is likely to be present in a teenager presenting with a liposarcoma

15.5%

C. It has an autosomal dominant inheritance pattern

41.7% of users answered this


question correctly

Which of the following are not true of Li-Fraumeni syndrome?

D. Affected individuals are unlikely to develop acute myeloid leukaemia


E. Adrenal malignancies are more common than in normal population

6
7
8
9

Search
Next question
Go

10
11

They are at high risk of developing leukaemia.

12

Genetics and surgical disease

13
14

Some of the more commonly occurring genetic conditions occurring in surgical patients are
presented here.

15
16

Li-Fraumeni Syndrome

17
18

Autosomal dominant
Consists of germline mutations to p53 tumour suppressor gene
High incidence of malignancies particularly sarcomas and leukaemias
Diagnosed when:

*Individual develops sarcoma under 45 years


*First degree relative diagnosed with any cancer below age 45 years and another family
member develops malignancy under 45 years or sarcoma at any age
BRCA 1 and 2
Carried on chromosome 17 (BRCA 1) and Chromosome 13 (BRCA 2)
Linked to developing breast cancer (60%) risk.
Associated risk of developing ovarian cancer (55% with BRCA 1 and 25% with BRCA
2).

Lynch Syndrome
Autosomal dominant
Develop colonic cancer and endometrial cancer at young age
80% of affected individuals will get colonic and/ or endometrial cancer
High risk individuals may be identified using the Amsterdam criteria

Amsterdam criteria
Three or more family members with a confirmed diagnosis of colorectal cancer, one of whom
is a first degree (parent, child, sibling) relative of the other two.
Two successive affected generations.
One or more colon cancers diagnosed under age 50 years.
Familial adenomatous polyposis (FAP) has been excluded.
Gardners syndrome
Autosomal dominant familial colorectal polyposis
Multiple colonic polyps
Extra colonic diseases include: skull osteoma, thyroid cancer and epidermoid cysts
Desmoid tumours are seen in 15%
Mutation of APC gene located on chromosome 5
Due to colonic polyps most patients will undergo colectomy to reduce risk of colorectal
cancer
Now considered a variant of familial adenomatous polyposis coli

Rate question:

Next question

Comment on this question

All contents of this site are 2012 E-Medical Revision Ltd

http://www.emrcs.com/question/question.php?q=0

Terms and Conditions

Privacy policy

1/2

13/07/2015

http://www.emrcs.com/question/question.php?q=0

2/2

13/07/2015

Reference ranges

Previous

Question 19 of 284

Next

A 35 year old type 1 diabetic presents with difficulty mobilising and back pain radiating to the
thigh. He has a temperature of 39 oC and has pain on extension of the hip. He is diagnosed
with an iliopsoas abscess. Which of the following statements is false in relation to his
diagnosis?

Question stats

Score: 64%
1

20.2%

26.5%

12.2%

24.2%

16.9%

26.5% of users answered this


question correctly

A. Staphylococcus aureus is the most likely primary cause

End and review

B. Recurrence occurs in 60% cases

6
7
8

C. More common in males

Search

D. Crohn's is the most likely secondary cause

Go

E. CT guided drainage is preferable first line management

10
11

Next question

12
13

Classical features include: a limp, back pain and fever. Recurrence rates are about 15-20%.
Staphylococcus is the commonest primary cause, others include Streptococcus and E.coli.
Management is ideally by CT guided drainage.

14
15
16

Iliopsoas abscess

17
18

Collection of pus in iliopsoas compartment (iliopsoas and iliacus)


Causes:

19

Primary

Haematogenous spread of bacteria


Staphylococcus aureus: most common

Secondary

Crohn's (commonest cause in this category)


Diverticulitis, Colorectal cancer
UTI, GU cancers
Vertebral osteomyelitis
Femoral catheter, lithotripsy
Endocarditis

Note the mortality rate can be up to 19-20% in secondary iliopsoas abscesses compared with
2.4% in primary abscesses.
Clinical features

Fever
Back/flank pain
Limp
Weight loss

Clinical examination

Patient in the supine position with the knee flexed and the hip mildly externally rotated

Specific tests to diagnose iliopsoas inflammation:

Place hand proximal to the patient's ipsilateral knee and ask patient to lift thigh against your
hand. This will cause pain due to contraction of the psoas muscle.
Lie the patient on the normal side and hyperextend the affected hip. In inflammation this
should elicit pain as the psoas muscle is stretched.
Investigation

http://www.emrcs.com/question/question.php?q=0

1/2

13/07/2015

CT is gold standard

Management

Antibiotics
Percutaneous drainage
Surgery is indicated if:

1. Failure of percutaneous drainage


2. Presence of an another intra-abdominal pathology which requires surgery
Surgical approach
The authors technique for draining these collections is given here.
Review the CT scans and plan surgical approach. An extraperitoneal approach is important.
The collection usually extends inferiorly and can be accessed from an incision at a level of
L4 on the affected side.
GA
Transverse laterally placed incision.
Incise external oblique.
Split the subsequent muscle layers.
As you approach the peritoneum use blunt dissection to pass laterally around it.
Remember the ureter and gonadal veins lie posterior at this level.
Eventually you will enter the abscess cavity, a large amount of pus is usually released at this
point. Drain the area with suction and washout with saline.
Place a corrugated drain well into the abscess cavity.
If you have made a small skin incision it is reasonable to bring the drain up through the skin
wound. Otherwise place a lateral exit site and close the skin and external oblique. If you do
this ensure that you use interrupted sutures.
Anchor the drain with strong securely tied silk sutures (it is extremely tiresome if it falls out!)
Reference
Iliopsoas abscesses
I H Mallick, M H Thoufeeq, T P Rajendran
Postgrad Med J 2004;80:459-462
Rate question:

Next question

Comment on this question

All contents of this site are 2012 E-Medical Revision Ltd

http://www.emrcs.com/question/question.php?q=0

Terms and Conditions

Privacy policy

2/2

13/07/2015

Reference ranges

Previous

Question 20 of 284

Next

Question stats

End and review

Score: 61.5%
1

29.1%

12.4%

23.4%

A. 15% of cases are due to parathyroid carcinoma

16.4%

B. 80% of cases are due to parathyroid adenomas

18.7%

C. Parathyroid adenomas are often encapsulated

29.1% of users answered this


question correctly

Which of the following statements relating to parathyroid neoplasms is incorrect?

D. 10% of parathyroid adenomas develop in ectopically located glands


E. 85% of cases of primary hyperparathyroidism are due to solitary
adenomas

6
7
8
9

Search
Next question

Go

10
11

Parathyroid carcinomas account for up to 5% of tumours. Adenomas are often encapsulated.


Lesions that are fibrotic and densely adherent to the gland may be a carcinoma. 85% cases
of primary hyperparathyroidism are due to a single adenoma and this is the reason some
surgeons favour a focused parathyroidectomy.

12
13
14
15

Parathyroid glands and disorders of calcium metabolism

16
17

Hyperparathyroidism
18

Disease type

Hormone profile

Clinical
features

Cause

May be
asymptomatic if
mild
Recurrent
abdominal pain
(pancreatitis,
renal colic)
Changes to
emotional or
cognitive state

Most cases due to solitary


adenoma (80%), multifocal
disease occurs in 10-15%
and parathyroid carcinoma in
1% or less

May have few


symptoms
Eventually
may develop
bone disease,
osteitis fibrosa
cystica and soft
tissue
calcifications

Parathyroid gland hyperplasia


occurs as a result of low
calcium, almost always in a
setting of chronic renal failure

Metastatic
calcification
Bone pain
and / or fracture

Occurs as a result of ongoing


hyperplasia of the parathyroid
glands after correction of
underlying renal disorder,
hyperplasia of all 4 glands is
usually the cause

19
20

Primary
hyperparathyroidism

Secondary
hyperparathyroidism

Tertiary
hyperparathyroidism

PTH
(Elevated)
Ca2+
(Elevated)
Phosphate
(Low)
Urine
calcium :
creatinine
clearance
ratio > 0.01

PTH
(Elevated)
Ca2+ (Low or
normal)
Phosphate
(Elevated)
Vitamin D
levels (Low)

Ca2+
(Normal or
high)
PTH
(Elevated)
Phosphate
levels
(Decreased
or Normal)
Vitamin D
(Normal or
decreased)
Alkaline
phosphatase
(Elevated)

Nephrolithiasis
Pancreatitis

Differential diagnoses
It is important to consider the rare but relatively benign condition of benign familial
hypocalciuric hypercalcaemia, caused by an autosomal dominant genetic disorder. Diagnosis
is usually made by genetic testing and concordant biochemistry (urine calcium : creatinine
clearance ratio <0.01-distinguished from primary hyperparathyroidism).
Treatment
Primary hyperparathyroidism
Indications for surgery

http://www.emrcs.com/question/question.php?q=0

1/2

13/07/2015

Elevated serum Calcium > 1mg/dL above normal


Hypercalciuria > 400mg/day
Creatinine clearance < 30% compared with normal
Episode of life threatening hypercalcaemia
Nephrolithiasis
Age < 50 years
Neuromuscular symptoms
Reduction in bone mineral density of the femoral neck, lumbar spine, or distal radius of
more than 2.5 standard deviations below peak bone mass (T score lower than -2.5)

Secondary hyperparathyroidism
Usually managed with medical therapy.
Indications for surgery in secondary (renal) hyperparathyroidism:
Bone pain
Persistent pruritus
Soft tissue calcifications

Tertiary hyperparathyroidism
Allow 12 months to elapse following transplant as many cases will resolve
The presence of an autonomously functioning parathyroid gland may require surgery. If the
culprit gland can be identified then it should be excised. Otherwise total parathyroidectomy
and re-implantation of part of the gland may be required.
References
1. Pitt S et al. Secondary and Tertiary Hyperparathyroidism, State of the Art Surgical
Management. Surg Clin North Am 2009 Oct;89(5):1227-39.
2. MacKenzie-Feder J et al. Primary Hyperparathyroidism: An Overview. Int J Endocrinol
2011; 2011: 251410.
Rate question:

Next question

Comment on this question

All contents of this site are 2012 E-Medical Revision Ltd

http://www.emrcs.com/question/question.php?q=0

Terms and Conditions

Privacy policy

2/2

13/07/2015

Reference ranges

Previous

Question 21 of 284

Next

A 20 year old girl presents with a thyroid cancer, she is otherwise well with no significant
family history. On examination she has a nodule in the left lobe of the thyroid with a small
discrete mass separate from the gland itself. Which of the following is the most likely cause?

Question stats

Score: 59.3%
1

20.5%

7.1%

11.2%

48.7%

12.4%

A. Follicular carcinoma
48.7% of users answered this
question correctly

B. Anaplastic carcinoma

End and review

6
7

C. Medullary carcinoma
8

D. Papillary carcinoma
9

Search

E. B Cell Lymphoma

Go

Next question

10
11
12

Theme from September 2011 Exam

13

Papillary carcinoma is the most common subtype and may cause lymph node metastasis
(mass separate from the gland itself) that is rare with follicular tumours. Anaplastic carcinoma
would cause more local symptoms and would be rare in this age group.

14
15
16
17

Thyroid malignancy

18
19

Papillary carcinoma
Commonest sub-type
Accurately diagnosed on fine needle aspiration cytology
Histologically, they may demonstrate psammoma bodies (areas of calcification) and so
called 'orphan Annie' nuclei
They typically metastasise via the lymphatics and thus laterally located apparently
ectopic thyroid tissue is usually a metastasis from a well differentiated papillary
carcinoma

20
21

Follicular carcinoma
Are less common than papillary lesions
Like papillary tumours, they may present as a discrete nodule. Although they appear
to be well encapsulated macroscopically there is invasion on microscopic evaluation
Lymph node metastases are uncommon and these tumours tend to spread
haematogenously. This translates into a higher mortality rate
Follicular lesions cannot be accurately diagnosed on fine needle aspiration cytology
and thus all follicular FNA's (THY 3) will require at least a hemi thyroidectomy

Anaplastic carcinoma
Less common and tend to occur in elderly females
Disease is usually advanced at presentation and often only palliative decompression
and radiotherapy can be offered.

Medullary carcinoma
These are tumours of the parafollicular cells ( C Cells) and are of neural crest origin.
The serum calcitonin may be elevated which is of use when monitoring for recurrence.
They may be familial and occur as part of the MEN -2A disease spectrum.
Spread may be either lymphatic or haematogenous and as these tumours are not
derived primarily from thyroid cells they are not responsive to radioiodine.

Lymphoma
These respond well to radiotherapy
Radical surgery is unnecessary once the disease has been diagnosed on biopsy
material. Such biopsy material is not generated by an FNA and thus a core biopsy has
to be obtained (with care!).

Rate question:

Next question

Comment on this question

http://www.emrcs.com/question/question.php?q=0

1/2

13/07/2015
All contents of this site are 2012 E-Medical Revision Ltd

http://www.emrcs.com/question/question.php?q=0

Terms and Conditions

Privacy policy

2/2

13/07/2015

Reference ranges

Previous

Question 22 of 284

Next

A 28 year old lady is breast feeding her first child. She presents with discomfort of the right
breast. Clinical examination demonstrates erythema and an area that is fluctuant. Aspiration
and culture of the fluid is most likely to demonstrate infection with which of the following
organisms?

Question stats

Score: 60.7%
1

7.2%

64.1%

7.9%

13.1%

7.7%

64.1% of users answered this


question correctly

A. Clostridium perfringens

End and review

B. Staphylococcus aureus

6
7
8

C. Streptococcus pyogenes

Search

D. Staphylococcus epidermidis
E. Actinomycosis

Go

10
11

Next question

12
13

Theme from April 2014 Exam

14
15

Staphylococcus aureus is the commonest cause. The infants mouth is usually the source as
it damages the nipple areolar complex allowing entry of bacteria.

16
17

Breast abscess

18
19

In lactational women Staphylococcus aureus is the most common cause


Typical presentation is with a tender, fluctuant mass in a lactating women
Diagnosis and treatment is performed using USS and associated drainage of the
abscess cavity. Antibiotics should also be administered
Where there is necrotic skin overlying the abscess, the patient should undergo
surgery

Rate question:

20
21
22

Next question

Comment on this question

All contents of this site are 2012 E-Medical Revision Ltd

http://www.emrcs.com/question/question.php?q=0

Terms and Conditions

Privacy policy

1/1

13/07/2015

Reference ranges

Previous

Question 23 of 284

Next

A 20 year old male presents with a tense, swollen knee joint. There is no history of
antecedent trauma. On examination the joint is tense and swollen but there is no sign of
injury. Plain x-rays show no fracture or arthritis. What is the most likely explanation?

Question stats

Score: 62.1%
1

9.9%

7.5%

7.4%

60.6%

14.6%

A. Rupture of the anterior cruciate ligament


60.6% of users answered this
question correctly

B. Rupture of the medial collateral ligament

End and review

6
7

C. Tibial plateau fracture


8

D. Haemophilia A
9

Search

E. von Willebrands disease

Go

10
11

Next question

12
13

Haemarthrosis without trauma is typically a feature of haemophilia A and B

14
15

Theme from 2014 Exam


Without a history of trauma, ligamentous rupture or tibial plateau fractures would be unusual.

16
17

Abnormal coagulation

18
19

Cause

Factors affected

20

Heparin

Prevents activation factors 2,9,10,11

21

Warfarin

Affects synthesis of factors 2,7,9,10

DIC

Factors 1,2,5,8,11

Liver disease

Factors 1,2,5,7,9,10,11

22
23

Interpretation blood clotting test results


Disorder

APTT

PT

Bleeding time

Haemophilia

Increased

Normal

Normal

von Willebrand's disease

Increased

Normal

Increased

Vitamin K deficiency

Increased

Increased

Normal

Rate question:

Next question

Comment on this question

All contents of this site are 2012 E-Medical Revision Ltd

http://www.emrcs.com/question/question.php?q=0

Terms and Conditions

Privacy policy

1/1

13/07/2015

Reference ranges

Previous

Question 24 of 284

Next

An 18 year old rock climber falls onto his left arm and sustains a large haematoma of the left
upper arm. Unfortunately the wound associated with the injury is neglected and it becomes
infected. Which of these changes is least likely to occur?

Question stats

Score: 63.3%
1

13%

61%

7.9%

8.4%

9.7%

A. Axillary lymphadenopathy
61% of users answered this
question correctly

B. Leucopenia

End and review

6
7

C. Tenderness
8

D. Mild pyrexia
9

Search

E. Local formation of yellow pus

Go

10
11

Next question

12

Leucopenia would be unusual and should prompt a search for another cause.

13
14

Acute inflammation

15
16

Inflammation is the reaction of the tissue elements to injury. Vascular changes occur,
resulting in the generation of a protein rich exudate. So long as the injury does not totally
destroy the existing tissue architecture, the episode may resolve with restoration of original
tissue architecture.

17
18
19

Vascular changes

20
21

Vasodilation occurs and persists throughout the inflammatory phase.


Inflammatory cells exit the circulation at the site of injury.
The equilibrium that balances Starlings forces within capillary beds is disrupted and a
protein rich exudate will form as the vessel walls also become more permeable to
proteins.
The high fibrinogen content of the fluid may form a fibrin clot. This has several
important immunomodulatory functions.

22
23
24

Sequelae
Resolution

Organisation

Suppuration

Progression to chronic
inflammation

Typically occurs with minimal initial injury


Stimulus removed and normal tissue architecture results

Delayed removal of exudate


Tissues undergo organisation and usually fibrosis

Typically formation of an abscess or an empyema


Sequestration of large quantities of dead neutrophils

Coupled inflammatory and reparative activities


Usually occurs when initial infection or suppuration has
been inadequately managed

Causes
Infections e.g. Viruses, exotoxins or endotoxins released by bacteria
Chemical agents
Physical agents e.g. Trauma
Hypersensitivity reactions
Tissue necrosis

Presence of neutrophil polymorphs is a histological diagnostic feature of acute


inflammation
Rate question:

Next question

Comment on this question

All contents of this site are 2012 E-Medical Revision Ltd

http://www.emrcs.com/question/question.php?q=0

Terms and Conditions

Privacy policy

1/1

13/07/2015

Reference ranges

Previous2 / 3

Question 25-27 of 284

Next

Question stats

End and review

Score: 63.6%

Average score for registered users:

1
2

Theme: Tumour markers


A. Invasive ductal carcinoma of the breast

25

59.6%

26

64.1%

27

78.4%

B. Prostate cancer

3
4
5

C. Gastric cancer

D. Ovarian cancer

Search

E. Colorectal cancer

7
Go

F. Pancreatic adenocarcinoma

8
9

G. Seminoma testicular cancer


10

H. Non-seminomatous testicular cancer


11

I. Hepatocellular carcinoma

12

For each tumour marker please select the most likely underlying malignancy. Each option
may be used once, more than once or not at all.

13
14
15

25.

Raised beta-human chorionic gonadotropin with a raised alpha-feto protein


level

16
17
18

You answered Prostate cancer

19
20

The correct answer is Non-seminomatous testicular cancer

21

Theme from April 2012 Exam


A raised alpha-feto protein level excludes a seminoma

22
23

26.

Elevated CA 19-9

24
25-27 2 / 3

Pancreatic adenocarcinoma

27.

Raised alpha-feto protein level in a 54-year-old woman


Hepatocellular carcinoma

Next question

Tumour markers

Theme from January 2013 exam


Tumour markers may be divided into:
monoclonal antibodies against carbohydrate or glycoprotein tumour antigens
tumour antigens
enzymes (alkaline phosphatase, neurone specific enolase)
hormones (e.g. calcitonin, ADH)

It should be noted that tumour markers usually have a low specificity


Monoclonal antibodies
Tumour marker

Association

CA 125

Ovarian cancer

CA 19-9

Pancreatic cancer

CA 15-3

Breast cancer

NB: The breast cancer tumour marker is not specific or sensitive enough to be used
routinely.
Tumour antigens
Tumour marker

Association

http://www.emrcs.com/question/question.php?q=0

1/2

13/07/2015

Prostate specific antigen (PSA)

Prostatic carcinoma

Alpha-feto protein (AFP)

Hepatocellular carcinoma, teratoma

Carcinoembryonic antigen (CEA)

Colorectal cancer

Rate question:

Next question

Comment on this question

All contents of this site are 2012 E-Medical Revision Ltd

http://www.emrcs.com/question/question.php?q=0

Terms and Conditions

Privacy policy

2/2

13/07/2015

Reference ranges

Previous

Question 31 of 284

Next

A 17 year old man is identified as having a Meckels diverticulum. From which of the following
embryological structures is it derived?

A. Foregut
B. Hindgut

Question stats

Score: 56.8%
1

12.1%

10.8%

10.8%

9.5%

56.8%

56.8% of users answered this


question correctly

C. Urachus

End and review

D. Cloaca

6
7
8

E. Vitello-intestinal duct

Search
Go

Next question

10
11
12

Rule of 2's

13
14

2% of population
2 inches (5cm) long
2 feet (60 cm) from the ileocaecal valve
2 x's more common in men
2 tissue types involved

15
16
17
18
19

The Meckels diverticulum is a persistence of the vitello-intestinal duct.

20

Meckel's diverticulum

21
22

Congenital abnormality resulting in incomplete obliteration of the vitello-intestinal duct


Normally, in the foetus, there is an attachment between the vitello-intestinal duct and
the yolk sac.This disappears at 6 weeks gestation.
The tip is free in majority of cases.
Associated with enterocystomas, umbilical sinuses, and omphaloileal fistulas.
Arterial supply: omphalomesenteric artery.
2% of population, 2 inches long, 2 feet from the ileocaecal valve.
Typically lined by ileal mucosa but ectopic gastric mucosa can occur, with the risk of
peptic ulceration. Pancreatic and jejunal mucosa can also occur.

23
24
25-27 2 / 3
28-30 0 / 3
31

Clinical
Normally asymptomatic and an incidental finding.
Complications are the result of obstruction, ectopic tissue, or inflammation.
Removal if narrow neck or symptomatic. Options are between wedge excision or formal
small bowel resection and anastomosis.

Rate question:

Next question

Comment on this question

All contents of this site are 2012 E-Medical Revision Ltd

http://www.emrcs.com/question/question.php?q=0

Terms and Conditions

Privacy policy

1/1

13/07/2015

Reference ranges

Previous0 / 3

Question 32-34 of 284

Next

Question stats

End and review

Score: 52.5%

Average score for registered users:

1
2

Theme: Rectal bleeding


A. Solitary rectal ulcer syndrome

32

85.5%

33

77.7%

34

45.6%

B. Haemorrhoidal disease

3
4
5

C. Fissure in ano

D. Fistula in ano

Search

E. Anal cancer

7
Go

F. Ulcerative colitis

8
9

Please select the most likely diagnosis for the scenario given. Each option may be used
once, more than once or not at all.

10
11
12
13

32.

A 22 year old man presents with a 6 day history of passage of bloody


diarrhoea with passage of mucous and slime. He is passing an average of 8 to
9 bowel movements per day. On digital rectal examination there is no discrete
abnormality to feel, but there is some blood stained mucous on the glove.

14
15
16
17

You answered Solitary rectal ulcer syndrome

18

The correct answer is Ulcerative colitis


19

33.

The passage of bloody diarrhoea together with mucous and a short history
makes this a likely first presentation of inflammatory bowel disease.

20

A 17 year old man presents with a 2 week history of significant pain on


defecation accompanied by the presence of a small amount of blood which is
noticed on toilet paper.

22

21

23
24
25-27 2 / 3
28-30 0 / 3

You answered Haemorrhoidal disease


The correct answer is Fissure in ano

31

Young patients with painful rectal bleeding may have a fissure. Treatment is
with stool softeners and either GTN or Diltiazem initially.
34.

32-34 0 / 3

A 24 year old woman presents with a long history of obstructed defecation and
chronic constipation. She often strains to open her bowels for long periods and
occasionally notices that she has passed a small amount of blood. On
examination she has an indurated area located anteriorly approximately 3cm
proximal to the anal verge.
You answered Fissure in ano
The correct answer is Solitary rectal ulcer syndrome
Solitary rectal ulcers are associated with chronic constipation and straining. It
will need to be biopsied to exclude malignancy (the histological appearances
are characteristic). Diagnostic work up should include endoscopy and probably
defecating proctogram and ano-rectal manometry studies.

Next question

Rectal bleeding

Rectal bleeding is a common cause for patients to be referred to the surgical clinic. In the
clinical history it is useful to try and localise the anatomical source of the blood. Bright red
blood is usually of rectal anal canal origin, whilst dark red blood is more suggestive of a
proximally sited bleeding source. Blood which has entered the GI tract from a gastroduodenal source will typically resemble malaena due to the effects of the digestive enzymes
on the blood itself.
In the table below we give some typical bleeding scenarios together with physical examination
findings and causation.
Cause

Type of
bleeding

Features in history

http://www.emrcs.com/question/question.php?q=0

Examination findings

1/3

13/07/2015

Fissure in
ano

Bright red
rectal
bleeding

Painful bleeding that occurs


post defecation in small
volumes. Usually antecedent
features of constipation

Muco-epithelial defect usually in


the midline posteriorly (anterior
fissures more likely to be due to
underlying disease)

Haemorroids

Bright red
rectal
bleeding

Post defecation bleeding noted


both on toilet paper and drips
into pan. May be alteration of
bowel habit and history of
straining. No blood mixed with
stool. No local pain.

Normal colon and rectum.


Proctoscopy may show internal
haemorrhoids. Internal
haemorrhoids are usually
impalpable.

Crohns
disease

Bright red
or mixed
blood

Bleeding that is accompanied


by other symptoms such as
altered bowel habit, malaise,
history of fissures (especially
anterior) and abscesses.

Perineal inspection may show


fissures or fistulae. Proctoscopy
may demonstrate indurated
mucosa and possibly strictures.
Skip lesions may be noted at
colonoscopy.

Ulcerative
colitis

Bright red
bleeding
often
mixed
with stool

Diarrhoea, weight loss,


nocturnal incontinence,
passage of mucous PR.

Proctitis is the most marked


finding. Peri anal disease is
usually absent. Colonoscopy
will show continuous mucosal
lesion.

Rectal
cancer

Bright red
blood
mixed
volumes

Alteration of bowel habit.


Tenesmus may be present.
Symptoms of metastatic
disease.

Usually obvious mucosal


abnormality. Lesion may be
fixed or mobile depending upon
disease extent. Surrounding
mucosa often normal, although
polyps may be present.

Image showing a fissure in ano. Typically these are located posteriorly and in the midline.
Fissures at other sites may be associated with underlying disease.

Image sourced from Wikipedia

Colonoscopic image of internal haemorroids. Note these may often be impalpable.

Image sourced from Wikipedia

Investigation
All patients presenting with rectal bleeding require digital rectal examination and
procto-sigmoidoscopy as a minimal baseline.
Remember that haemorrhoids are typically impalpable and to attribute bleeding to
these in the absence of accurate internal inspection is unsatisfactory.
In young patients with no other concerning features in the history a carefully
performed sigmoidoscopy that demonstrates clear haemorrhoidal disease may be
sufficient. If clear views cannot be obtained then patients require bowel preparation

http://www.emrcs.com/question/question.php?q=0

2/3

13/07/2015

with an enema and a flexible sigmoidscopy performed.


In those presenting with features of altered bowel habit or suspicion of inflammatory
bowel disease a colonoscopy is the best test.
Patients with excessive pain who are suspected of having a fissure may require an
examination under general or local anaesthesia.
In young patients with external stigmata of fissure and a compatible history it is
acceptable to treat medically and defer internal examination until the fissure is healed.
If the fissure fails to heal then internal examination becomes necessary along the lines
suggested above to exclude internal disease.

Special tests
In patients with a malignancy of the rectum the staging investigations comprise an MRI
of the rectum to identify circumferential resection margin compromise and to identify
mesorectal nodal disease. In addition to this CT scanning of the chest abdomen and
pelvis is necessary to stage for more distant disease. Some centres will still stage the
mesorectum with endo rectal ultrasound but this is becoming far less common.

Patients with fissure in ano who are being considered for surgical sphincterotomy and
are females who have an obstetric history should probably have ano rectal manometry
testing performed together with endo anal ultrasound. As this service is not universally
available it is not mandatory but in the absence of such information there are
continence issues that may arise following sphincterotomy.

Management
Disease

Management

Fissure in
ano

GTN ointment 0.2% or diltiazem cream applied topically is the usual first line
treatment. Botulinum toxin for those who fail to respond. Internal sphincterotomy
for those who fail with botox, can be considered earlier in males.

Haemorroids

Lifestyle advice, for small internal haemorrhoids can consider injection


sclerotherapy or rubber band ligation. For external haemorrhoids consider
haemorrhoidectomy. Modern options include HALO procedure and stapled
haemorrhoidectomy.

Inflammatory
bowel
disease

Medical management- although surgery may be needed for fistulating Crohns


(setons).

Rectal
cancer

Anterior resection or abdomino-perineal excision of the colon and rectum. Total


mesorectal excision is now standard of care. Most resections below the
peritoneal reflection will require defunctioning ileostomy. Most patients will
require preoperative radiotherapy.

Rate question:

Next question

Comment on this question

All contents of this site are 2012 E-Medical Revision Ltd

http://www.emrcs.com/question/question.php?q=0

Terms and Conditions

Privacy policy

3/3

13/07/2015

Reference ranges

Previous

Question 35 of 284

Next

Question stats

End and review

Score: 50%
1

21.7%

29.2%

31.1%

A. Glioblastoma multiforme

7.4%

B. Astrocytoma

10.6%

C. Medulloblastoma

29.2% of users answered this


question correctly

Which of the following is the most common childhood brain tumour?

D. Ependymoma
E. Meningioma

6
7
8
9

Search
Next question
Go

10
11

Glioblastoma multiforme is rare in childhood. In contrast, astrocytoma is the commonest brain


tumour in children. Medulloblastoma is no longer the commonest CNS tumour in children
(Cancer research UK)

12
13
14

CNS tumours

15
16

60% = Glioma and metastatic disease


20% = Meningioma
10% = Pituitary lesions

17
18
19

In paediatric practice medulloblastomas (neuroectodermal tumours) were the commonest


lesions, astrocytomas now account for the majority.
Tumours arising in right temporal and frontal lobe may reach considerable size before
becoming symptomatic. Whereas tumours in the speech and visual areas will typically
produce early symptoms.

20
21
22
23

Diagnosis
MRI Scanning provides the best resolution.

24
25-27 2 / 3
28-30 0 / 3

Treatment
Usually surgery, even if tumour cannot be completely resected conditions such as rising ICP
can be addressed with tumour debulking and survival and quality of life prolonged.
Curative surgery can usually be undertaken with lesions such as meningiomas. Gliomas
have a marked propensity to invade normal brain and resection of these lesions is nearly
always incomplete.
Rate question:

31
32-34 0 / 3
35

Next question

Comment on this question

All contents of this site are 2012 E-Medical Revision Ltd

http://www.emrcs.com/question/question.php?q=0

Terms and Conditions

Privacy policy

1/1

13/07/2015

Reference ranges

Previous

Question 36 of 284

Next

A keen surgical trainee is about to embark on her first hemi arthroplasty for a fractured neck
of femur. In the anaesthetic room the patient is given 1.2g intravenous co-amoxiclav. There
is a possible history of penicillin allergy but the patient is demented and the history is not
checked. The patient then develops severe respiratory compromise and haemodynamic
collapse. Which pathological process accounts for this event?

Question stats

Score: 48.8%
1

13.2%

62.8%

8.5%

9.3%

6.2%

62.8% of users answered this


question correctly

A. Binding of the drug to circulating IgG class antibodies

End and review

6
7
8

B. Recognition of the drug by IgE receptors on mast cells


C. Drug initiated formation of hapten-protein complexes

Search

D. Binding of the drug to circulating IgM class antibodies

Go

E. None of the above

10
11
12

Next question

13
14

Anaphylactic shock:
Antigen recognised by IgE molecules on the surface of mast cells resulting in rapid
degranulation with release of histamine and other inflammatory cytokines.

15
16
17
18

This is a case of anaphylactic shock. In anaphylaxis the mast cells degranulate.

19
20

Anaphylactic shock

21
22

Suspect if there has been exposure to an allergen

23
24

Management
- Remove allergen
- ABCD
- Drugs:

25-27 2 / 3
28-30 0 / 3
31
32-34 0 / 3

Adrenaline 1:1000 0.5ml INTRAMUSCULARLY (not IV). Repeat after 5 mins if no response.
Then Chlorpheniramine 10mg IV
Then Hydrocortisone 100-200mg IV

35
36

Reference
Emergency treatment of anaphylactic reactions. Guidelines for healthcare providers.
Working Group of the Resuscitation Council (UK).2008
Rate question:

Next question

Comment on this question

All contents of this site are 2012 E-Medical Revision Ltd

http://www.emrcs.com/question/question.php?q=0

Terms and Conditions

Privacy policy

1/1

13/07/2015

Reference ranges

Previous

Question 37 of 284

Next

Which of the following hepatobiliary disorders are most classically associated with ulcerative
colitis?

A. Gallstones
B. Primary sclerosing cholangitis

Question stats

Score: 47.7%
1

13.9%

62.4%

8.3%

8.4%

7.1%

62.4% of users answered this


question correctly

C. Bile duct stones

End and review

D. Liver hamartomas

6
7
8

E. Hepatocellular carcinoma

Search
Next question

Go

10
11

Primary sclerosing cholangitis is an idiopathic inflammation of the bile ducts. It may result in
episodes of cholestasis and cholangitis and ultimately result in the need for liver
transplantation. It carries a 10% risk of malignant transformation. Crohns disease is
associated with gallstones due to impaired entero-hepatic circulation. Apart from PSC,
ulcerative colitis does not increase the risk of other liver lesions.

12
13
14
15
16

Ulcerative colitis

17

Ulcerative colitis is a form of inflammatory bowel disease. Inflammation always starts at


rectum, does not spread beyond ileocaecal valve (although backwash ileitis may occur) and
is continuous. The peak incidence of ulcerative colitis is in people aged 15-25 years and in
those aged 55-65 years. It is less common in smokers.
The initial presentation is usually following insidious and intermittent symptoms. Features
include:
bloody diarrhoea
urgency
tenesmus
abdominal pain, particularly in the left lower quadrant
extra-intestinal features (see below)

18
19
20
21
22
23
24
25-27 2 / 3
28-30 0 / 3
31
32-34 0 / 3
35

Questions regarding the 'extra-intestinal' features of inflammatory bowel disease are


common. Extra-intestinal features include sclerosing cholangitis, iritis and ankylosing
spondylitis.
Common to both Crohn's disease
(CD) and Ulcerative colitis (UC)

Notes

Related to
disease
activity

Arthritis: pauciarticular, asymmetric


Erythema nodosum
Episcleritis
Osteoporosis

Arthritis is the most common extraintestinal feature in both CD and UC


Episcleritis is more common in
Crohns disease

Unrelated to
disease
activity

Arthritis: polyarticular, symmetric


Uveitis
Pyoderma gangrenosum
Clubbing
Primary sclerosing cholangitis

Primary sclerosing cholangitis is


much more common in UC
Uveitis is more common in UC

36
37

Pathology
Red, raw mucosa, bleeds easily
No inflammation beyond submucosa (unless fulminant disease)
Widespread superficial ulceration with preservation of adjacent mucosa which has the
appearance of polyps ('pseudopolyps')
Inflammatory cell infiltrate in lamina propria
Neutrophils migrate through the walls of glands to form crypt abscesses
Depletion of goblet cells and mucin from gland epithelium
Granulomas are infrequent

Barium enema
Loss of haustrations
Superficial ulceration, 'pseudopolyps'
Long standing disease: colon is narrow and short -'drainpipe colon'

http://www.emrcs.com/question/question.php?q=0

1/2

13/07/2015

Endoscopy
Superficial inflammation of the colonic and rectal mucosa
Continuous disease from rectum proximally
Superficial ulceration, mucosal islands, loss of vascular definition and continuous
ulceration pattern.

Management
Patients with long term disease are at increased risk of development of malignancy
Acute exacerbations are generally managed with steroids, in chronic patients agents
such as azathioprine and infliximab may be used
Individuals with medically unresponsive disease usually require surgery- in the acute
phase a sub total colectomy and end ileostomy. In the longer term a proctectomy will
be required. An ileoanal pouch is an option for selected patients

References
Ford A et al. Ulcerative colitis. BMJ 2013 (346):29-34.
Rate question:

Next question

Comment on this question

All contents of this site are 2012 E-Medical Revision Ltd

http://www.emrcs.com/question/question.php?q=0

Terms and Conditions

Privacy policy

2/2

13/07/2015

Reference ranges

Previous

Question 38 of 284

Next

Question stats

End and review

Score: 48.9%
1

7.3%

14.4%

7%

A. Endothelial cell damage

61.3%

B. Use of tourniquets in surgery

10.1%

C. Formation of platelet aggregates

61.3% of users answered this


question correctly

Which of the following is least associated with thrombosis?

D. Thrombocytopenia
E. Carcinoma of the stomach

6
7
8
9

Search
Next question
Go

10
11

All the other options either act directly to promote thrombosis e.g. endothelial cell damage or
via changes in consistency or flow of blood.

12
13

Abnormal coagulation

14
15

Cause

Factors affected

Heparin

Prevents activation factors 2,9,10,11

Warfarin

Affects synthesis of factors 2,7,9,10

DIC

Factors 1,2,5,8,11

20

Liver disease

Factors 1,2,5,7,9,10,11

21

16
17
18
19

22
23

Interpretation blood clotting test results


Disorder

APTT

PT

24

Bleeding time

Haemophilia

Increased

Normal

Normal

von Willebrand's disease

Increased

Normal

Increased

25-27 2 / 3
28-30 0 / 3
31
32-34 0 / 3

Vitamin K deficiency

Increased

Increased

35

Normal

36
37

Rate question:

38

Next question

Comment on this question

All contents of this site are 2012 E-Medical Revision Ltd

http://www.emrcs.com/question/question.php?q=0

Terms and Conditions

Privacy policy

1/1

13/07/2015

Reference ranges

Previous

Question 40 of 284

Next

Infection with which of the following micro-organisms may result in a clinical picture
resembling achalasia of the oesphagus?

A. Epstein Barr virus


B. Wuchereria Bancrofti

Question stats

Score: 46.8%
1

11%

13%

19.9%

40%

16.1%

40% of users answered this


question correctly

C. Candida Spp

End and review

D. Trypanosoma Cruzi

6
7
8

E. Helicobacter Pylori

Search
Go

Next question

10
11
12

Infection with Trypanosoma Cruzi may result in destruction of the ganglion cells of the
myenteric plexus, resulting in a clinical picture similar to achalasia.

13
14

Trypanosoma Cruzi

15
16

Protozoan
Causes Chagas disease
Carried by bugs which infect the skin whilst feeding
Penetrate through open wounds and mucous membranes
Intracellular proliferation
Major infective sites include CNS, intestinal myenteric plexus, spleen, lymph nodes and
cardiac muscle
Chronic disease is irreversible, nifurtimox is used to treat acute infection

17
18
19
20
21
22
23
24

Rate question:

25-27 2 / 3
28-30 0 / 3

Next question

Comment on this question

31
32-34 0 / 3
35
36
37
38
39
40

All contents of this site are 2012 E-Medical Revision Ltd

http://www.emrcs.com/question/question.php?q=0

Terms and Conditions

Privacy policy

1/1

13/07/2015

Reference ranges

Previous

Question 41 of 284

Next

A 45-year-old man presents to surgical outpatients with a long history of recurrent abdominal
pain and vomiting. He is noted to have a peripheral motor neuropathy on examination. What
is the most likely diagnosis?

Question stats

Score: 45.8%
1

14.1%

10.8%

46.5%

14.5%

14.2%

A. Huntington's disease
46.5% of users answered this
question correctly

B. Myeloma

End and review

6
7

C. Acute intermittent porphyria


8

D. Lawrence-Moon-Biedl syndrome
9

Search

E. Friedreich's ataxia

Go

10
11

Next question

12

Neurological signs combined with abdominal pain is acute intermittent porphyria or lead
poisoning until proven otherwise.

13

Acute intermittent porphyria

15

14

16

Acute intermittent porphyria (AIP) is a rare autosomal dominant condition caused by a defect
in porphobilinogen deaminase, an enzyme involved in the biosynthesis of haem. The results
in the toxic accumulation of delta aminolaevulinic acid and porphobilinogen. It
characteristically presents with abdominal and neuropsychiatric symptoms in 20-40 year
olds. AIP is more common in females (5:1)

17
18
19
20
21

Features

22

abdominal: abdominal pain, vomiting


neurological: motor neuropathy
psychiatric: e.g. depression
hypertension and tachycardia common

23
24
25-27 2 / 3
28-30 0 / 3
31

Diagnosis

32-34 0 / 3

classically urine turns deep red on standing


raised urinary porphobilinogen (elevated between attacks and to a greater extent
during acute attacks)
assay of red cells for porphobilinogen deaminase
raised serum levels of delta aminolaevulinic acid and porphobilinogen

35
36
37
38
39
40

Rate question:

Next question

41

Comment on this question

All contents of this site are 2012 E-Medical Revision Ltd

http://www.emrcs.com/question/question.php?q=0

Terms and Conditions

Privacy policy

1/1

13/07/2015

Reference ranges

Previous

Question 42 of 284

Next

A 56 year old man presents with episodic facial pain and discomfort whilst eating. He has
suffered from halitosis recently and he frequently complains of a dry mouth. He has a smooth
swelling underneath his right mandible. What is the most likely underlying diagnosis?

Question stats

Score: 46.9%
1

46.2%

22.5%

11.2%

12%

8.2%

A. Stone impacted in Whartons duct


46.2% of users answered this
question correctly

B. Stone impacted in Stensens duct

End and review

6
7

C. Benign adenoma of the submandibular gland


8

D. Adenocarcinoma of the submandibular gland


9

Search

E. Squamous cell carcinoma of the submandibular gland

Go

10
11

Next question

12

The symptoms are typical for sialolithiasis. The stones most commonly form in the
submandibular gland and therefore may occlude Whartons duct. Stensens duct drains the
parotid gland.

13
14
15

Submandibular glands- disease

16
17

Physiology
The submandibular glands secrete approximately 800- 1000ml saliva per day. They typically
produce mixed seromucinous secretions. When parasympathetic activity is dominant; the
secretions will be more serous. The parasympathetic fibres are derived from the chorda
tympani nerves and the submandibular ganglion. Sensory fibres are conveyed by the lingual
branch of the mandibular nerve.

18
19
20
21
22

Sialolithiasis

23
24

80% of all salivary gland calculi occur in the submandibular gland


70% of the these calculi are radio-opaque
Stones are usually composed of calcium phosphate or calcium carbonate
Patients typically develop colicky pain and post prandial swelling of the gland
Investigation involves sialography to demonstrate the site of obstruction and
associated other stones
Stones impacted in the distal aspect of Whartons duct may be removed orally, other
stones and chronic inflammation will usually require gland excision

25-27 2 / 3
28-30 0 / 3
31
32-34 0 / 3
35
36
37

Sialadenitis

38
39

Usually occurs as a result of Staphylococcus aureus infection


Pus may be seen leaking from the duct, erythema may also be noted
Development of a sub mandibular abscess is a serious complication as it may spread
through the other deep fascial spaces and occlude the airway

40
41
42

Submandibular tumours
Only 8% of salivary gland tumours affect the sub mandibular gland
Of these 50% are malignant (usually adenoid cystic carcinoma)
Diagnosis usually involves fine needle aspiration cytology
Imaging is with CT and MRI
In view of the high prevalence of malignancy, all masses of the submandibular glands
should generally be excised.

Rate question:

Next question

Comment on this question

All contents of this site are 2012 E-Medical Revision Ltd

http://www.emrcs.com/question/question.php?q=0

Terms and Conditions

Privacy policy

1/1

13/07/2015

Reference ranges

Previous

Question 43 of 284

Next

Question stats

End and review

Score: 46%
1

62.4%

7.2%

8.5%

A. Reed Sternberg Cells

12.1%

B. T lymphocytes

9.8%

C. Macrophages

62.4% of users answered this


question correctly

Which of the following cellular types or features is not seen in sarcoidosis?

D. Asteroid bodies
E. B lymphocytes

6
7
8
9

Search
Next question
Go

Reed Sternberg cells are seen in Hodgkins disease. All of the other cell types are seen in
sarcoid.

10
11
12
13

Chronic inflammation

14

Overview
Chronic inflammation may occur secondary to acute inflammation.In most cases chronic
inflammation occurs as a primary process. These may be broadly viewed as being one of
three main processes:
Persisting infection with certain organisms such as Mycobacterium tuberculosis which
results in delayed type hypersensitivity reactions and inflammation.
Prolonged exposure to non-biodegradable substances such as silica or suture
materials which may induce an inflammatory response.
Autoimmune conditions involving antibodies formed against host antigens.

15
16
17
18
19
20
21
22
23

Acute vs. Chronic inflammation

24

Acute inflammation

Chronic inflammation

Changes to existing vascular structure and increased


permeability of endothelial cells

Angiogenesis predominates

25-27 2 / 3
28-30 0 / 3
31
32-34 0 / 3

Infiltration of neutrophils

Macrophages, plasma cells and


lymphocytes predominate

35

Process may resolve with:

Healing by fibrosis is the main result

37

Suppuration
Complete resolution
Abscess formation
Progression to chronic inflammation
Healing by fibrosis

36

38
39
40
41
42

Granulomatous inflammation
A granuloma consists of a microscopic aggregation of macrophages (with epithelial type
arrangement =epitheliod). Large giant cells may be found at the periphery of granulomas.

43

Mediators
Growth factors released by activated macrophages include agents such as interferon and
fibroblast growth factor (plus many more). Some of these such as interferons may have
systemic features resulting in systemic symptoms and signs, which may be present in
individuals with long standing chronic inflammation.
The finding of granulomas is pathognomonic of chronic inflammation, as illustrated in this
biopsy from a patient with colonic Crohns disease

Image sourced from Wikipedia

http://www.emrcs.com/question/question.php?q=0

1/2

13/07/2015

Rate question:

Next question

Comment on this question

All contents of this site are 2012 E-Medical Revision Ltd

http://www.emrcs.com/question/question.php?q=0

Terms and Conditions

Privacy policy

2/2

13/07/2015

Reference ranges

Question 44 of 284

Previous

Next

Question stats

End and review

Score: 44.2%
1

15%

9.5%

9.6%

A. Polya gastrectomy for antral ulcer

8.4%

B. Atrophic gastritis

57.5%

C. Intestinal metaplasia of columnar type at the gastric cardia

57.5% of users answered this


question correctly

Which of the following diseases is not considered a risk factor for gastric cancer?

D. Patient with polyp showing medium grade dysplasia


E. Long term therapy with H2 blockers

6
7
8
9

Search
Next question
Go

10
11

Although some acid lowering procedures increase the risk of gastric cancer the use of H2
blockers does not, at the present time, seem to increase the risk.

12
13

Gastric cancer

14
15

Overview
There are 700,000 new cases of gastric cancer worldwide each year. It is most common in
Japan and less common in western countries. It is more common in men and incidence rises
with increasing age. The exact cause of many sporadic cancer is not known, however,
familial cases do occur in HNPCC families. In addition, smoking and smoked or preserved
foods increase the risk. Japanese migrants retain their increased risk (decreased in
subsequent generations). The distribution of the disease in western countries is changing
towards a more proximal location (perhaps due to rising obesity).
Pathology
There is some evidence of support a stepwise progression of the disease through intestinal
metaplasia progressing to atrophic gastritis and subsequent dysplasia, through to cancer.
The favoured staging system is TNM. The risk of lymph node involvement is related to size
and depth of invasion; early cancers confined to submucosa have a 20% incidence of lymph
node metastasis. Tumours of the gastro-oesophageal junction are classified as below:

16
17
18
19
20
21
22
23
24
25-27 2 / 3
28-30 0 / 3
31
32-34 0 / 3

Type
1

True oesophageal cancers and may be associated with Barrett's oesophagus.

Type
2

Carcinoma of the cardia, arising from cardiac type epithelium


or short segments with intestinal metaplasia at the oesophagogastric junction.

Type
3

Sub cardial cancers that spread across the junction. Involve similar nodal stations to
gastric cancer.

35
36
37
38
39
40
41

Groups for close endoscopic monitoring


Intestinal metaplasia of columnar type
Atrophic gastritis
Low to medium grade dysplasia
Patients who have previously undergone resections for benign peptic ulcer disease
(except highly selective vagotomy).

42
43
44

Referral to endoscopy
Patients of any age with
dyspepsia and any of the
following

Patients without
dyspepsia

Worsening dyspepsia

Chronic gastrointestinal bleeding

Dysphagia

Barretts oesophagus

Dysphagia

Unexplained
abdominal pain or
weight loss

Intestinal metaplasia

Weight loss

Vomiting

Dysplasia

Iron deficiency anaemia

Upper abdominal
mass

Atrophic gastritis

Upper abdominal mass

Jaundice

Patient aged over 55 years with


unexplained or persistent
dyspepsia

http://www.emrcs.com/question/question.php?q=0

1/3

13/07/2015

Upper GI endoscopy performed for dyspepsia. The addition of dye spraying (as shown in the
bottom right) may facilitate identification of smaller tumours

Image sourced from Wikipedia

Staging
CT scanning of the chest abdomen and pelvis is the routine first line staging
investigation in most centres.
Laparoscopy to identify occult peritoneal disease
PET CT (particularly for junctional tumours)

Treatment
Proximally sited disease greater than 5-10cm from the OG junction may be treated by
sub total gastrectomy
Total gastrectomy if tumour is <5cm from OG junction
For type 2 junctional tumours (extending into oesophagus) oesophagogastrectomy is
usual
Endoscopic sub mucosal resection may play a role in early gastric cancer confined to
the mucosa and perhaps the sub mucosa (this is debated)
Lymphadenectomy should be performed. A D2 lymphadenectomy is widely advocated
by the Japanese, the survival advantages of extended lymphadenectomy have been
debated. However, the overall recommendation is that a D2 nodal dissection be
undertaken.
Most patients will receive chemotherapy either pre or post operatively.

Prognosis
UK Data
Disease extent

Percentage 5 year survival

All RO resections

54%

Early gastric cancer

91%

Stage 1

87%

Stage 2

65%

Stage 3

18%

Operative procedure
Total Gastrectomy , lymphadenectomy and Roux en Y anastomosis
General anaesthesia
Prophylactic intravenous antibiotics
Incision: Rooftop.
Perform a thorough laparotomy to identify any occult disease.
Mobilise the left lobe of the liver off the diaphragm and place a large pack over it. Insert a
large self retaining retractor e.g. omnitract or Balfour (take time with this, the set up should
be perfect). Pack the small bowel away.
Begin by mobilising the omentum off the transverse colon.
Proceed to detach the short gastric vessels.
Mobilise the pylorus and divide it at least 2cm distally using a linear cutter stapling device.
Continue the dissection into the lesser sac taking the lesser omentum and left gastric artery
flush at its origin.
http://www.emrcs.com/question/question.php?q=0

2/3

13/07/2015

The lymph nodes should be removed en bloc with the specimen where possible.
Place 2 stay sutures either side of the distal oesophagus. Ask the anaesthetist to pull back
on the nasogastric tube. Divide the distal oesophagus and remove the stomach.
The oesphago jejunal anastomosis should be constructed. Identify the DJ flexure and bring a
loop of jejunum up to the oesophagus (to check it will reach). Divide the jejunum at this point.
Bring the divided jejunum either retrocolic or antecolic to the oesophagus. Anastamose the
oesophagus to the jejunum, using either interrupted 3/0 vicryl or a stapling device. Then
create the remainder of the Roux en Y reconstruction distally.
Place a jejunostomy feeding tube.
Wash out the abdomen and insert drains (usually the anastomosis and duodenal stump).
Help the anaesthetist insert the nasogastric tube (carefully!)
Close the abdomen and skin.
Enteral feeding may commence on the first post-operative day. However, most surgeons will
leave patients on free NG drainage for several days and keep them nil by mouth.

Rate question:

Next question

Comment on this question

All contents of this site are 2012 E-Medical Revision Ltd

http://www.emrcs.com/question/question.php?q=0

Terms and Conditions

Privacy policy

3/3

13/07/2015

Reference ranges

Previous

Question 45 of 284

Next

A 56 year old man is diagnosed as having a glioma. From which of the following cell types do
these tumours usually originate?

A. Astrocytes
B. Oligodendrocytes

Question stats

Score: 43.4%
1

24.5%

19%

10%

5.9%

40.6%

40.6% of users answered this


question correctly

C. Ependymal cells

End and review

D. Squamous cells

6
7
8

E. Neuroglial cells

Search
Go

Next question

10
11
12

Theme from January 2012 Exam


Gliomas originate from glial (otherwise known as neuroglial) cells. These serve a structural
function in the CNS. The tumours produced may resemble a number of CNS cell types.
Tumours are therefore named according to the cells they resemble rather than the origin.
Where this is not possible they are termed gliomas.

13
14
15
16

Glioma

17
18

Glioma is a tumour that is typically found in the CNS. These tumours arise from glial cells.
They are sub categorised according to the cell type they most closely resemble.

19
20

Glioma sub types

21

Ependymomas- Ependymal cells


Astocytomas- Astrocytes (including glioblastoma)
Oligodendrogliomas- Oligodendrocytes
Mixed- e.g. oligoastrocytomas

22
23
24
25-27 2 / 3
28-30 0 / 3

Gliomas are categorised as being either high or low grade lesions (the former has the worse
prognosis). They may be either supra or infra tentorial. Their symptoms will typically reflect
their site of origin. Glioblastoma multiforme has the worst prognosis and few patients will
survive beyond 12 months.

31
32-34 0 / 3
35
36

Rate question:

Next question

37
38

Comment on this question

39
40
41
42
43
44
45

All contents of this site are 2012 E-Medical Revision Ltd

http://www.emrcs.com/question/question.php?q=0

Terms and Conditions

Privacy policy

1/1

13/07/2015

Reference ranges

Previous

Question 46 of 284

Next

A 78 year old man presents with unilateral deafness which has been present for the past 3
months. On examination, Webers test localises to the contralateral side and a CT scan of his
head shows a thickened calvarium with areas of sclerosis and radiolucency. His blood tests
show an elevated alkaline phosphatase, normal serum calcium and normal PTH levels.
Which of the following is the most likely underlying diagnosis?

Question stats

Score: 42.6%
1

21.6%

7.8%

46%

8.1%

16.4%

46% of users answered this


question correctly

A. Multiple myeloma with skull involvement

End and review

6
7
8

B. Osteoporosis
C. Pagets disease with skull involvement

Search

D. Lung cancer with skull metastasis

Go

E. Osteopetrosis with skull involvement

10
11
12

Next question

13
14

Of the conditions listed Pagets disease is the most likely diagnosis (skull vault expansion and
sensorineural hearing loss). Multiple myeloma would typically result in multiple areas of
radiolucency and usually raised calcium in this setting. Osteopetrosis is a recognised cause
of the features described. However, it is a rare inherited disorder and usually presents in
children in young adults. Presentation at this stage with no prior symptoms would be
extremely rare and therefore this is not the most likely diagnosis.

15
16
17
18
19

Pagets disease

20
21

Paget's disease is a disease of increased but uncontrolled bone turnover and is


characterised by architecturally abnormal bones. It is thought to be primarily a disorder of
osteoclasts, with excessive osteoclastic resorption followed by increased osteoblastic activity
causing areas of sclerosis and deformity. Paget's disease is common (UK prevalence 5%)
but symptomatic in only 1 in 20 patients

22
23
24
25-27 2 / 3
28-30 0 / 3

Predisposing factors

31

increasing age
male sex
northern latitude
family history

32-34 0 / 3
35
36
37

Clinical features

38
39

bone pain (e.g. pelvis, lumbar spine, femur)


classical, untreated features: bowing of tibia, bossing of skull
raised alkaline phosphatase (ALP) - calcium* and phosphate are typically normal
skull x-ray: thickened vault, osteoporosis circumscripta

40
41
42
43

Indications for treatment include bone pain, skull or long bone deformity, fracture,
periarticular Paget's

44
45

bisphosphonate (either oral risedronate or IV zoledronate)


calcitonin is less commonly used now

46

Complications
deafness (cranial nerve entrapment)
bone sarcoma (1% if affected for > 10 years)
fractures
skull thickening
high-output cardiac failure

*usually normal in this condition but hypercalcaemia may occur with prolonged immobilisation
Rate question:

Next question

Comment on this question

All contents of this site are 2012 E-Medical Revision Ltd

http://www.emrcs.com/question/question.php?q=0

Terms and Conditions

Privacy policy

1/1

13/07/2015

Reference ranges

Previous2 / 3

Question 47-49 of 284

Next

Question stats

End and review

Score: 43.9%

Average score for registered users:

1
2

Theme: Genetic causes of cancer


A. Multiple endocrine neoplasia type I

47

75.6%

48

65.2%

49

73.3%

B. Multiple endocrine neoplasia type II

3
4
5

C. Gardner's syndrome

D. Lynch Syndrome

Search

E. Kartagener's syndrome

7
Go

F. Von Recklinghausen's disease

8
9

Please select the most likely condition for the disease process described. Each option may
be used once, more than once or not at all

10
11
12
13

47.

A 5 year old boy presents with recurrent episodes of sinusitis. The casualty
staff are surprised to find his liver lying in the left upper quadrant of the
abdomen

14
15
16

Kartagener's syndrome
17

This is a case of Kartagener's syndrome. The primary problem is of immotile


cilia syndrome. When associated with situs inversus Kartagener's syndrome is
diagnosed.

18
19
20

48.

A 22 year old man presents with carcinoma of the caecum. His brother died
from colorectal cancer aged 25 and his mother died from endometrial cancer
aged 38.

21
22
23

Lynch Syndrome

24

This is a case of Lynch syndrome HNPCC. It is transmitted in an autosomal


dominant fashion. Cancer of the uterine body is more common in HNPCC than
Gardners syndrome
49.

A tall 32 year old lady presents with a diffuse neck swelling a carcinoma of the
thyroid medullary type is diagnosed.

25-27 2 / 3
28-30 0 / 3
31
32-34 0 / 3
35
36

You answered Multiple endocrine neoplasia type I

37
38

The correct answer is Multiple endocrine neoplasia type II


This is a case MEN type IIb. It is associated with phaeochromocytomas and is
transmitted in an autosomal dominant pattern if inherited. All MEN II tend to
have medullary carcinoma of the thyroid as a presenting feature

39
40
41
42

Next question

43
44
45

Genetics and surgical disease

46
47-49 2 / 3

Some of the more commonly occurring genetic conditions occurring in surgical patients are
presented here.
Li-Fraumeni Syndrome
Autosomal dominant
Consists of germline mutations to p53 tumour suppressor gene
High incidence of malignancies particularly sarcomas and leukaemias
Diagnosed when:

*Individual develops sarcoma under 45 years


*First degree relative diagnosed with any cancer below age 45 years and another family
member develops malignancy under 45 years or sarcoma at any age
BRCA 1 and 2
Carried on chromosome 17 (BRCA 1) and Chromosome 13 (BRCA 2)
Linked to developing breast cancer (60%) risk.
Associated risk of developing ovarian cancer (55% with BRCA 1 and 25% with BRCA

http://www.emrcs.com/question/question.php?q=0

1/2

13/07/2015

2).

Lynch Syndrome
Autosomal dominant
Develop colonic cancer and endometrial cancer at young age
80% of affected individuals will get colonic and/ or endometrial cancer
High risk individuals may be identified using the Amsterdam criteria

Amsterdam criteria
Three or more family members with a confirmed diagnosis of colorectal cancer, one of whom
is a first degree (parent, child, sibling) relative of the other two.
Two successive affected generations.
One or more colon cancers diagnosed under age 50 years.
Familial adenomatous polyposis (FAP) has been excluded.
Gardners syndrome
Autosomal dominant familial colorectal polyposis
Multiple colonic polyps
Extra colonic diseases include: skull osteoma, thyroid cancer and epidermoid cysts
Desmoid tumours are seen in 15%
Mutation of APC gene located on chromosome 5
Due to colonic polyps most patients will undergo colectomy to reduce risk of colorectal
cancer
Now considered a variant of familial adenomatous polyposis coli

Rate question:

Next question

Comment on this question

All contents of this site are 2012 E-Medical Revision Ltd

http://www.emrcs.com/question/question.php?q=0

Terms and Conditions

Privacy policy

2/2

13/07/2015

Reference ranges

Previous

Question 50 of 284

Next

A 45 year old man presents with symptoms of urinary colic. In the history he has suffered
from recurrent episodes of frank haematuria over the past week or so. On examination he
has a left loin mass and a varicocele. The most likely diagnosis is:

Question stats

Score: 43.1%
1

54.4%

9.2%

18%

8.8%

9.5%

A. Renal adenocarcinoma
54.4% of users answered this
question correctly

B. Renal cortical adenoma

End and review

6
7

C. Squamous cell carcinoma of the renal pelvis


8

D. Retroperitoneal fibrosis
9

Search

E. Nephroblastoma

Go

Next question

10
11
12

Renal adenocarcinoma are the most common renal malignancy and account for 75%
cases.
Patients may develop frank haematuria and have episodes of clot colic.
A Grawitz tumour is an eponymous name for Renal Adenocarcinoma.
May metastasise to bone.

13
14
15
16
17
18

Renal tumours

19
20

Renal cell carcinoma


Renal cell carcinoma is an adenocarcinoma of the renal cortex and is believed to arise from
the proximal convoluted tubule. They are usually solid lesions, up to 20% may be multifocal,
20% may be calcified and 20% may have either a cystic component or be wholly cystic. They
are often circumscribed by a pseudocapsule of compressed normal renal tissue. Spread may
occur either by direct extension into the adrenal gland, renal vein or surrounding fascia.
More distant disease usually occurs via the haematogenous route to lung, bone or brain.
Renal cell carcinoma comprise up to 85% of all renal malignancies. Males are more
commonly affected than females and sporadic tumours typically affect patients in their sixth
decade.
Patients may present with a variety of symptoms including; haematuria (50%), loin pain
(40%), mass (30%) and up to 25% may have symptoms of metastasis.Less than 10% have
the classic triad of haematuria, pain and mass.

21
22
23
24
25-27 2 / 3
28-30 0 / 3
31
32-34 0 / 3
35
36
37

Investigation
Many cases will present as haematuria and be discovered during diagnostic work up. Benign
renal tumours are rare, so renal masses should be investigated with multislice CT scanning.
Some units will add and arterial and venous phase to the scan to demonstrate vascularity
and evidence of caval ingrowth.
CT scanning of the chest and abdomen to detect distant disease should also be undertaken.

38
39
40
41
42
43

Routine bone scanning is not indicated in the absence of symptoms.


Biopsy should not be performed when a nephrectomy is planned but is mandatory before
any ablative therapies are undertaken.

44
45
46
47-49 2 / 3

Assessment of the functioning of the contra lateral kidney.

50

Management
T1 lesions may be managed by partial nephrectomy and this gives equivalent oncological
results to total radical nephrectomy. Partial nephrectomy may also be performed when there
is inadequate reserve in the remaining kidney.
For T2 lesions and above a radical nephrectomy is standard practice and this may be
performed via a laparoscopic or open approach. Preoperative embolisation is not indicated
nor is resection of uninvolved adrenal glands. During surgery early venous control is
mandatory to avoid shedding of tumour cells into the circulation.
Patients with completely resected disease do not benefit from adjuvant therapy with
either chemotherapy or biological agents. These should not be administered outside the
setting of clinical trials.
Patients with transitional cell cancer will require a nephroureterectomy with disconnection of
the ureter at the bladder.

http://www.emrcs.com/question/question.php?q=0

1/2

13/07/2015

References
Lungberg B et al. EAU guidelines on renal cell carcinoma: The 2010 update. European
Urology 2010 (58): 398-406.
Rate question:

Next question

Comment on this question

All contents of this site are 2012 E-Medical Revision Ltd

http://www.emrcs.com/question/question.php?q=0

Terms and Conditions

Privacy policy

2/2

13/07/2015

Reference ranges

Previous

Question 51 of 284

Next

A 63 year old man finds that he has to stop walking after 100 yards due to bilateral calf pain.
He finds that bending forwards and walking up hill helps. He is able to ride a bike without any
pain. What is the most likely underlying cause?

Question stats

Score: 44.1%
1

59.7%

7.2%

11.6%

12.9%

8.5%

A. Lumbar canal stenosis


59.7% of users answered this
question correctly

B. Diabetic neuropathy

End and review

6
7

C. Aorto-iliac occlusion
8

D. Occlusion of the superficial femoral artery


9

Search

E. Pelvic rheumatoid arthritis

Go

10
11

Next question

12

Theme from April 2012 Exam


Theme from April 2013 Exam
The positional nature of the pain and the fact that improves with walking uphill makes an
underlying vascular aetiology far less likely.

13
14
15
16

Lumbar spinal stenosis

17
18

Lumbar spinal stenosis is a condition in which the central canal is narrowed by tumour, disk
prolapse or other similar degenerative changes.
Patients may present with a combination of back pain, neuropathic pain and symptoms
mimicking claudication. One of the main features that may help to differentiate it from true
claudication in the history is the positional element to the pain. Sitting is better than standing
and patients may find it easier to walk uphill rather than downhill. The neurogenic
claudication type history makes lumbar spinal stenosis a likely underlying diagnosis, the
absence of such symptoms makes it far less likely.

19
20
21
22
23
24
25-27 2 / 3
28-30 0 / 3

Pathology
Degenerative disease is the commonest underlying cause. Degeneration is believed to begin
in the intervertebral disk where biochemical changes such as cell death and loss of
proteoglycan and water content lead to progressive disk bulging and collapse. This process
leads to an increased stress transfer to the posterior facet joints, which accelerates
cartilaginous degeneration, hypertrophy, and osteophyte formation; this is associated with
thickening and distortion of the ligamentum flavum. The combination of the ventral disk
bulging, osteophyte formation at the dorsal facet, and ligamentum flavum hyptertrophy
combine to circumferentially narrow the spinal canal and the space available for the neural
elements. The compression of the nerve roots of the cauda equina leads to the
characteristic clinical signs and symptoms of lumbar spinal stenosis.

31
32-34 0 / 3
35
36
37
38
39
40

Diagnosis
MRI scanning is the best modality for demonstrating the canal narrowing. Historically a
bicycle test was used as true vascular claudicants could not complete the test.

41
42
43

Treatment
Laminectomy

44
45

Rate question:

46

Next question

47-49 2 / 3
Comment on this question

50
51

All contents of this site are 2012 E-Medical Revision Ltd

http://www.emrcs.com/question/question.php?q=0

Terms and Conditions

Privacy policy

1/1

13/07/2015

Reference ranges

Previous

Question 52 of 284

Next

A 73 year old lady is admitted for a laparoscopic cholecystectomy. During her pre-operative
assessment it is noted that she is receiving furosemide for the treatment of hypertension.
Approximately what proportion of the sodium that is filtered at the glomerulus will be
subsequently excreted?

Question stats

Score: 43.3%
1

40.2%

17.1%

20.3%

12.1%

10.2%

40.2% of users answered this


question correctly

A. Up to 25%

End and review

B. Upt to 75%

6
7
8

C. Between 3 and 5%

Search

D. <2%
E. Between 1 and 2%

Go

10
11

Next question

12
13

Theme from 2010 Exam


The loop diuretics can lead to marked increases in the amount of sodium excreted. They act
in the medullary and cortical aspects of the thick ascending limb of the loop of Henle. This
results in a decreased medullary osmolal gradient and increases free water excretion (as well
as loss of sodium). Because loop diuretics result in the loss of both sodium and water they
are less frequently associated with hyponatraemia than thiazide diuretics (these latter agents
act in the cortex and do not affect urine concentrating ability).

14
15
16
17
18
19

Diuretic agents

20
21

The diuretic drugs are divided into three major classes, which are distinguished according to
the site at which they impair sodium reabsorption: loop diuretics in the thick ascending loop
of Henle, thiazide type diuretics in the distal tubule and connecting segment; and potassium
sparing diuretics in the aldosterone - sensitive principal cells in the cortical collecting tubule.

22
23
24

In the kidney, sodium is reabsorbed through Na+/ K+ ATPase pumps located on the
basolateral membrane. These pumps return reabsorbed sodium to the circulation and
maintain low intracellular sodium levels. This latter effect ensures a constant concentration
gradient.

25-27 2 / 3
28-30 0 / 3
31
32-34 0 / 3

Physiological effects of commonly used diuretics


Site of action

Diuretic

35

Carrier or
channel
inhibited

Percentage of filtered
sodium excreted

36
37
38

Ascending limb of loop of


Henle

Frusemide

Na+/K+ 2Cl carrier

Upt to 25%

Distal tubule and


connecting segment

Thiazides

Na+Cl- carrier

Between 3 and 5%

Cortical collecting tubule

Spironolactone

Na+/K+ ATP ase


pump

Between 1 and 2%

39
40
41
42
43
44
45
46

Rate question:

Next question

47-49 2 / 3
50

Comment on this question

51
52

All contents of this site are 2012 E-Medical Revision Ltd

http://www.emrcs.com/question/question.php?q=0

Terms and Conditions

Privacy policy

1/1

13/07/2015

Reference ranges

Question 53 of 284

Previous

Next

A 59 year old man presents with recurrent episodes of urinary sepsis. In his history he
mentions that he has suffered from recurrent attacks of left iliac fossa pain over the past few
months. He has also notices bubbles in his urine. He undergoes a CT scan which shows a
large inflammatory mass in the left iliac fossa. No other abnormality is detected. The most
likely diagnosis is:

Question stats

Score: 44.3%
1

11.3%

18.5%

6.5%

54.1%

9.7%

54.1% of users answered this


question correctly

A. Ulcerative colitis

End and review

6
7
8

B. Crohns disease
C. Mesenteric ischaemia

Search

D. Diverticular disease

Go

E. Rectal cancer

10
11
12

Next question

13
14

Diverticular disease is one of the commonest causes of colovesical fistula

15
16

Theme from 2009 and 2011 Exam


Recurrent attacks of diverticulitis may cause the development of local abscesses which may
erode into the bladder resulting in urinary sepsis and pneumaturia. This would be an unusual
presentation from Crohns disease and rectal cancer would be more distally sited and
generally evidence of extra colonic disease would be present if the case were malignant and
this advanced.

17
18
19
20
21
22

Diverticular disease

23
24

Diverticular disease is a common surgical problem. It consists of herniation of colonic mucosa


through the muscular wall of the colon. The usual site is between the taenia coli where
vessels pierce the muscle to supply the mucosa. For this reason, the rectum, which lacks
taenia, is often spared.

25-27 2 / 3
28-30 0 / 3
31
32-34 0 / 3

Symptoms
Altered bowel habit
Bleeding
Abdominal pain

35
36
37
38
39

Complications
Diverticulitis
Haemorrhage
Development of fistula
Perforation and faecal peritonitis
Perforation and development of abscess
Development of diverticular phlegmon

40
41
42
43
44
45
46

Diagnosis
Patients presenting in clinic will typically undergo either a colonoscopy, CT cologram or
barium enema as part of their diagnostic work up. All tests can identify diverticular disease. It
can be far more difficult to confidently exclude cancer, particularly in diverticular strictures.

47-49 2 / 3
50
51
52

Acutely unwell surgical patients should be investigated in a systematic way. Plain abdominal
films and an erect chest x-ray will identify perforation. An abdominal CT scan (not a CT
cologram) with oral and intravenous contrast will help to identify whether acute inflammation
is present but also the presence of local complications such as abscess formation.

53

Severity Classification- Hinchey


I

Para-colonic abscess

II

Pelvic abscess

III

Purulent peritonitis

IV

Faecal peritonitis

Treatment

http://www.emrcs.com/question/question.php?q=0

1/2

13/07/2015

Increase dietary fibre intake.


Mild attacks of diverticulitis may be managed conservatively with antibiotics.
Peri colonic abscesses should be drained either surgically or radiologically.
Recurrent episodes of acute diverticulitis requiring hospitalisation are a relative
indication for a segmental resection.
Hinchey IV perforations (generalised faecal peritonitis) will require a resection and
usually a stoma. This group have a very high risk of post operative complications and
usually require HDU admission. Less severe perforations may be managed by
laparoscopic washout and drain insertion.

Rate question:

Next question

Comment on this question

All contents of this site are 2012 E-Medical Revision Ltd

http://www.emrcs.com/question/question.php?q=0

Terms and Conditions

Privacy policy

2/2

13/07/2015

Reference ranges

Previous

Question 54 of 284

Next

Question stats

End and review

Score: 45.2%
1

9.6%

10%

10.4%

A. Gastric ulcers

34.7%

B. Gastric cancer

35.4%

C. Anaemia

34.7% of users answered this


question correctly

Which of the following is least likely to occur in association with severe atrophic gastritis?

D. Duodenal ulcers
E. Gastric polyps

6
7
8
9

Search
Next question
Go

10
11

Achlorhydria would make the formation of duodenal ulcers unlikely. Note the question
states "least likely".

12
13
14

Due to the loss of gastric acid a duodenal ulcer is unlikely. Note that gastric polyps may form
(see below).

15
16
17

Gastritis

18

Type of
gastritis

Features

Type A

Autoimmune
Circulating antibodies to parietal cells, causes reduction in cell mass and
hypochlorhydria
Loss of parietal cells = loss of intrinsic factor = B12 malabsorption
Absence of antral involvement
Hypochlorhydria causes elevated gastrin levels- stimulating enterochromaffin
cells and adenomas may form

Type B

Reflux
gastritis

19
20
21
22
23
24
25-27 2 / 3
28-30 0 / 3

Antral gastritis
Associated with infection with helicobacter pylori infection
Intestinal metaplasia may occur in stomach and require surveillance endoscopy
Peptic ulceration may occur

31
32-34 0 / 3
35
36

Bile refluxes into stomach, either post surgical or due to failure of pyloric
function
Histologically, evidence of chronic inflammation, and foveolar hyperplasia
May respond to therapy with prokinetics

37
38
39

Erosive
gastritis

Stress
ulceration

Agents disrupt the gastric mucosal barrier


Most commonly due to NSAIDs and alcohol
With NSAIDs the effects occur secondary to COX 1 inhibition

40
41
42

This occurs as a result of mucosal ischaemia during hypotension or


hypovolaemia
The stomach is the most sensitive organ in the GI tract to ischaemia following
hypovolaemia
Diffuse ulceration may occur
Prophylaxis with acid lowering therapy and sucralfate may minimise
complications

43
44
45
46
47-49 2 / 3

Menetriers
disease

Gross hypertrophy of the gastric mucosal folds, excessive mucous production


and hypochlorhydria
Pre malignant condition

50
51
52

References
Whiting J et al. The long term results of endoscopic surveillance of premalignant gastric
lesions. Gut 2002; 50 :378381.

53
54

Dixon M et al. Reflux gastritis: distinct histopathological entity?J Clin Pathol 1986; 39 : 524530.
Rate question:

Next question

Comment on this question

All contents of this site are 2012 E-Medical Revision Ltd

http://www.emrcs.com/question/question.php?q=0

Terms and Conditions

Privacy policy

1/1

13/07/2015

Reference ranges

Previous1 / 3

Question 55-57 of 284

Next

Question stats

End and review

Score: 44.6%

Average score for registered users:

1
2

Theme: Lung cancer


A. Adenocarcinoma

55

56.3%

56

56.5%

57

41.8%

B. Small cell lung cancer

3
4
5

C. Large cell lung cancer

D. Squamous cell carcinoma

Search

Please select the most likely lung cancer variant for the scenario described. Each option may
be used once, more than once or not at all.

7
Go

8
9
10
11

55.

A 73 year old heavy smoker presents with haemoptysis. On examination he is


cachectic and shows evidence of clubbing. Imaging shows a main bronchial
tumour with massive mediastinal lymphadenopathy together with widespread
visceral metastases.

12
13
14
15

Small cell lung cancer

16

Theme from April 2012


Small cell carcinoma is associated with disseminated disease at presentation in
the majority of cases. Most cases occur in the main airways and paraneoplastic
features are common.
56.

A 68 year old female who has never smoked presents with a mass at the
periphery of her right lung.

17
18
19
20
21
22

You answered Large cell lung cancer

23

The correct answer is Adenocarcinoma

24

Adenocarcinomas are the most common tumour type present in never


smokers. They are usually located at the periphery.

25-27 2 / 3
28-30 0 / 3
31

57.

An 85 year old man presents with a cough and haemoptysis. He has a modest
smoking history of 15 pack years. He is found to have a tumour located in the
right main bronchus, with no evidence of metastatic disease. He decides not
undergo any treatment and he remains well for a further 12 months before
developing symptomatic metastasis.

32-34 0 / 3
35
36
37
38

You answered Small cell lung cancer

39

The correct answer is Squamous cell carcinoma


40

Squamous cell carcinomas are reported to be more slow growing and are
typically centrally located. Small cell carcinomas are usually centrally located.
However, small cell carcinomas would seldom be associated with a survival of a
year without treatment.

41
42
43
44

Next question

45
46
47-49 2 / 3

Lung cancer

50
51

Lung cancers may be classified according to histological subtypes. The main distinction is
between small cell and non small cell lung cancer. Non small cell lung cancer is the most
common variant and accounts for 80% of all lung cancers.
Non small cell lung cancer
These share common features of prognosis and management. They comprise the following
tumours:

52
53
54
55-57 1 / 3

Squamous cell carcinoma (25% cases)


Adenocarcinoma (40% cases)
Large cell carcinoma (10% cases)
Paraneoplastic features and early disease dissemination are less likely than with small cell
lung carcinoma. Adenocarcinoma is the most common lung cancer type encountered in
never smokers.
Small cell lung carcinoma

http://www.emrcs.com/question/question.php?q=0

1/2

13/07/2015

Small cell lung carcinomas are comprised of cells with a neuro endocrine differentiation. The
neuroendocrine hormones may be released from these cells with a wide range of
paraneoplastic associations. These tumours are strongly associated with smoking and will
typically arise in the larger airways. They disseminate early in the course of the disease and
although they are usually chemosensitive this seldom results in long lasting remissions.
Rate question:

Next question

Comment on this question

All contents of this site are 2012 E-Medical Revision Ltd

http://www.emrcs.com/question/question.php?q=0

Terms and Conditions

Privacy policy

2/2

13/07/2015

Reference ranges

Question 58 of 284

Previous

Next

Question stats

End and review

Score: 46.3%
1

14.7%

38%

16.3%

A. Pappenheimer bodies

15.8%

B. Stipple cells

15.1%

C. Erythrocyte containing siderotic granules

38% of users answered this


question correctly

Which of the following is not found on a blood film post splenectomy?

D. Howell-Jolly bodies
E. Target cells

6
7
8
9

Search
Next question
Go

10
11

Stipple cells are found in lead poisoning/haemoglobinopathies.

12

Blood film in hyposplenism:

13
14

Howell-Jolly bodies
Pappenheimer bodies
Poikilocytes (Target cells)
Erythrocyte containing siderotic granules
Heinz bodies

15
16
17
18

Post splenectomy blood film changes

19
20

The loss of splenic tissue results in the inability to readily remove immature or abnormal red
blood cells from the circulation. The red cell count does not alter significantly. However,
cytoplasmic inclusions may be seen e.g. Howell-Jolly bodies.
In the first few days after splenectomy target cells, siderocytes and reticulocytes will appear
in the circulation. Immediately following splenectomy a granulocytosis (mainly composed of
neutrophils) is seen, this is replaced by a lymphocytosis and monocytosis over the following
weeks.
The platelet count is usually increased and this may be persistent, oral antiplatelet agents
may be needed in some patients.

21
22
23
24
25-27 2 / 3
28-30 0 / 3
31
32-34 0 / 3

Image showing Howell Jolly bodies (arrowed)

35
36
37
38
39
40
41
42
43
44
45
46

Image sourced from Wikipedia

47-49 2 / 3
50

Rate question:

51

Next question

52
Comment on this question

53
54
55-57 1 / 3
58

All contents of this site are 2012 E-Medical Revision Ltd

http://www.emrcs.com/question/question.php?q=0

Terms and Conditions

Privacy policy

1/1

13/07/2015

Reference ranges

Previous

Question 59 of 284

Next

A 45 year old man with long standing ulcerative colitis and rectal dysplasia presents with a
DALM lesion in the rectum. What is the most appropriate management option?

A. Snare polypectomy
B. Repeat endoscopy in 2 years

Question stats

Score: 45.6%
1

22.6%

11.3%

7.3%

19.8%

39%

39% of users answered this


question correctly

C. Discharge

End and review

D. Anterior resection

6
7
8

E. Panproctocolectomy

Search
Go

Next question

10
11
12

DALM lesions complicating ulcerative colitis should be managed with panproctocolectomy. An


anterior resection is inadequate since it will only remove the rectum and ulcerative colitis
affects the entire colon. Since many will be associated with invasion a snare polypectomy is
not sufficient either.

13
14
15

Colonic lesions - DALM

16
17
18

The term DALM lesion refers to a Dysplasia Associated Lesion or Mass.


They may complicate dysplasia occurring in patients with longstanding ulcerative
colitis.
They have a high incidence of invasive foci.
When they complicate longstanding ulcerative colitis, they should be treated by
panproctocolectomy.

19
20
21
22
23
24

Rate question:

25-27 2 / 3
28-30 0 / 3

Next question

Comment on this question

31
32-34 0 / 3
35
36
37
38
39
40
41
42
43
44
45
46
47-49 2 / 3
50
51
52
53
54
55-57 1 / 3
58
59

All contents of this site are 2012 E-Medical Revision Ltd

http://www.emrcs.com/question/question.php?q=0

Terms and Conditions

Privacy policy

1/1

13/07/2015

Reference ranges

Question 60 of 284

Previous

Next

Which of the metastatic bone tumours described below is at the greatest risk of pathological
fracture ?

A. Proximal humeral lesion from a prostate cancer


B. Vertebral body lesions from a prostate cancer

Question stats

Score: 44.3%
1

7.4%

25.7%

36%

13.8%

17.1%

36% of users answered this


question correctly

C. Peritrochanteric lesion from a carcinoma of the breast

End and review

D. Proximal humeral lesion from a carcinoma of the breast

6
7
8

E. Peritrochanteric lesion from a prostate cancer

Search
Go

Next question

10
11

Peritrochanteric lesions have the greatest risks of fracture (due to loading). The lesions from
breast cancer are usually lytic and therefore at higher risk rather than the sclerotic lesions
from prostate cancer.

12

Metastatic bone disease- risk of fracture

15

13
14

16

Metastatic bone tumours may be described as blastic, lytic or mixed. Osteoblastic metastatic
disease has the lowest risk of spontaneous fracture when compared to osteolytic lesions of a
similar size.
Lesions affecting the peritrochanteric region are most prone to spontaneous fracture
(because of loading forces at that site).
The factors are incorporated into the Mirel Scoring system to stratify the risk of spontaneous
fracture for bone metastasis of varying types.

17
18
19
20
21
22

Mirel Scoring system

23
24

Score
points

Site

Radiographic
appearance

Width of bone
involved

Pain

Upper
extremity

Blastic

Less than 1/3

Mild

Lower
extremity

Mixed

Peritrochanteric

Lytic

25-27 2 / 3
28-30 0 / 3
31
32-34 0 / 3

1/3 to 2/3

35

Moderate

36

More than 2/3

37

Aggravated by
function

38
39

Depending upon the score the treatment should be as follows:

40
41

Score

Risk of fracture

Treatment

42

9 or greater

Impending (33%)

Prophylactic fixation

43

Borderline

Consider fixation

7 or less

Not impending (4%)

Non operative management

44
45
46
47-49 2 / 3
50

Rate question:

51

Next question

52
Comment on this question

53
54
55-57 1 / 3
58
59
60

All contents of this site are 2012 E-Medical Revision Ltd

http://www.emrcs.com/question/question.php?q=0

Terms and Conditions

Privacy policy

1/1

13/07/2015

Reference ranges

Question 61 of 284

Previous

Next

A 63 year old male presents with several episodes of haematuria. He suffers from COPD
secondary to long term smoking. What is the most likely underlying cause?

A. Renal cortical adenoma


B. Renal adenocarcinoma

Question stats

Score: 43.1%
1

6.3%

17.5%

7.1%

57.4%

11.7%

57.4% of users answered this


question correctly

C. Nephroblastoma

End and review

D. Transitional cell carcinoma of the bladder

6
7
8

E. Adenocarcinoma of the bladder

Search
Next question

Go

10
11

Theme from 2009 Exam


TCC is the most common subtype and is strongly linked to smoking. The important point to
note in this question is the term most likely as renal adenocarcinoma may produce similar
symptoms but is less likely.

12
13
14
15

Bladder cancer

16
17

Bladder cancer is the second most common urological cancer. It most commonly affects
males aged between 50 and 80 years of age. Those who are current, or previous (within 20
years), smokers have a 2-5 fold increased risk of the disease. Exposure to hydrocarbons
such as 2-Naphthylamine increases the risk. Although rare in the UK, chronic bladder
inflammation arising from Schistosomiasis infection remains a common cause of squamous
cell carcinomas, in those countries where the disease is endemic.

18
19
20
21
22

Benign tumours
Benign tumours of the bladder including inverted urothelial papilloma and nephrogenic
adenoma are uncommon.
Bladder malignancies

23
24
25-27 2 / 3
28-30 0 / 3
31

Transitional cell carcinoma (>90% of cases)


Squamous cell carcinoma ( 1-7% -except in regions affected by schistosomiasis)
Adenocarcinoma (2%)

32-34 0 / 3
35
36

Transitional cell carcinomas may arise as solitary lesions, or may be multifocal, owing to the
effect of "field change" within the urothelium. Up to 70% of TCC's will have a papillary growth
pattern. These tumours are usually superficial in location and accordingly have a better
prognosis. The remaining tumours show either mixed papillary and solid growth or pure solid
growths. These tumours are typically more prone to local invasion and may be of higher
grade, the prognosis is therefore worse. Those with T3 disease or worse have a 30% (or
higher) risk of regional or distant lymph node metastasis.

37

TNM Staging

43

38
39
40
41
42

44

Stage

Description

T0

No evidence of tumour

Ta

Non invasive papillary carcinoma

T1

Tumour invades sub epithelial connective tissue

T2a

Tumor invades superficial muscularis propria (inner half)

T2b

Tumor invades deep muscularis propria (outer half)

T3

Tumour extends to perivesical fat

T4

Tumor invades any of the following: prostatic stroma, seminal vesicles, uterus, vagina

T4a

Invasion of uterus, prostate or bowel

60

T4b

Invasion of pelvic sidewall or abdominal wall

61

N0

No nodal disease

N1

Single regional lymph node metastasis in the true pelvis (hypogastric, obturator,
external iliac, or presacral lymph node)

N2

Multiple regional lymph node metastasis in the true pelvis (hypogastric, obturator,

45
46
47-49 2 / 3
50
51
52
53
54
55-57 1 / 3
58
59

http://www.emrcs.com/question/question.php?q=0

1/2

13/07/2015

external iliac, or presacral lymph node metastasis)


N3

Lymph node metastasis to the common iliac lymph nodes

M0

No distant metastasis

M1

Distant disease

Presentation
Most patients (85%) will present with painless, macroscopic haematuria. In those patients
with incidental microscopic haematuria, up to 10% of females aged over 50 will be found to
have a malignancy (once infection excluded).
Staging
Most will undergo a cystoscopy and biopsies or TURBT, this provides histological diagnosis
and information relating to depth of invasion. Locoregional spread is best determined using
pelvic MRI and distant disease CT scanning. Nodes of uncertain significance may be
investigated using PET CT.
Treatment
Those with superficial lesions may be managed using TURBT in isolation. Those with
recurrences or higher grade/ risk on histology may be offered intravesical chemotherapy.
Those with T2 disease are usually offered either surgery (radical cystectomy and ileal
conduit) or radical radiotherapy.
Prognosis
T1

90%

T2

60%

T3

35%

T4a

10-25%

Any T, N1-N2

30%

Rate question:

Next question

Comment on this question

All contents of this site are 2012 E-Medical Revision Ltd

http://www.emrcs.com/question/question.php?q=0

Terms and Conditions

Privacy policy

2/2

13/07/2015

Reference ranges

Previous0 / 3

Question 62-64 of 284

Next

Question stats

End and review

Score: 41.3%

Average score for registered users:

1
2

Theme: Neck lumps


A. Cystic hygroma

62

79.4%

63

70.5%

64

88.3%

B. Bartonella infection

3
4
5

C. Mycobacterium tuberculosis infection

D. Branchial cyst

Search

E. Thyroglossal cyst

7
Go

F. Pharyngeal pouch

8
9

G. Follicular thyroid cyst


10

H. Parathyroid adenoma
11

I. None of the above

12

Please select the most likely underlying disease process for the scenario given. Each option
may be used once, more than once or not at all.

13
14
15

62.

A 25 year old cat lover presents with symptoms of abdominal pain, lethargy and
sweats. These have been present for the past two weeks. On examination she
has lymphadenopathy in the posterior triangle.

16
17
18
19

You answered Branchial cyst

20

The correct answer is Bartonella infection

21

Theme from January 2015 Exam


Bartonella infection may occur following a cat scratch. The organism is
intracellular. Generalised systemic symptoms may occur for a week or so prior
to clinical presentation.
63.

A 25 year old lady presents with an swelling located at the anterior border of
the sternocleidomastoid muscle. The swelling is intermittent and on examination
it is soft and fluctuant.

22
23
24
25-27 2 / 3
28-30 0 / 3
31
32-34 0 / 3
35

You answered Follicular thyroid cyst

36

The correct answer is Branchial cyst


37

Branchial cysts are remnants of the branchial cleft. They may become infected.
64.

A 38 year old lady presents with a mass in the midline of the neck immediately
below the hyoid bone. It moves upwards on tongue protrusion.

38
39
40
41

You answered Cystic hygroma

42

The correct answer is Thyroglossal cyst

43

Thyroglossal cysts are usually located in the midline and are linked to the
foramen caecum and will thus move upwards on tongue protrusion.

44
45
46
47-49 2 / 3

Next question

50
51
52

Neck lumps

53

The table below gives characteristic exam question features for conditions causing neck
lumps:

54
55-57 1 / 3
58

Reactive
lymphadenopathy

By far the most common cause of neck swellings. There may be a history
of local infection or a generalised viral illness

Lymphoma

Rubbery, painless lymphadenopathy


The phenomenon of pain whilst drinking alcohol is very uncommon
There may be associated night sweats and splenomegaly

Thyroid swelling

May be hypo-, eu- or hyperthyroid symptomatically


Moves upwards on swallowing

Thyroglossal cyst

More common in patients < 20 years old

http://www.emrcs.com/question/question.php?q=0

59
60
61
62-64 0 / 3

1/2

13/07/2015

Usually midline, between the isthmus of the thyroid and the hyoid bone
Moves upwards with protrusion of the tongue
May be painful if infected
Pharyngeal
pouch

More common in older men


Represents a posteromedial herniation between thyropharyngeus and
cricopharyngeus muscles
Usually not seen, but if large then a midline lump in the neck that gurgles
on palpation
Typical symptoms are dysphagia, regurgitation, aspiration and chronic
cough

Cystic hygroma

A congenital lymphatic lesion (lymphangioma) typically found in the neck,


classically on the left side
Most are evident at birth, around 90% present before 2 years of age

Branchial cyst

An oval, mobile cystic mass that develops between the


sternocleidomastoid muscle and the pharynx
Develop due to failure of obliteration of the second branchial cleft in
embryonic development
Usually present in early adulthood

Cervical rib

More common in adult females


Around 10% develop thoracic outlet syndrome

Carotid aneurysm

Pulsatile lateral neck mass which doesn't move on swallowing

Rate question:

Next question

Comment on this question

All contents of this site are 2012 E-Medical Revision Ltd

http://www.emrcs.com/question/question.php?q=0

Terms and Conditions

Privacy policy

2/2

13/07/2015

Reference ranges

Question 65 of 284

Previous

Next

A 22 year old man presents with a discharging area on his lower back. On examination there
is an epithelial defect located 6cm proximal to the tip of his coccyx and located in the midline.
There are two further defects located about 2cm superiorly in the same position. He is
extremely hirsute. What is the most likely diagnosis?

Question stats

Score: 42.1%
1

7%

10.1%

56.5%

7.7%

18.7%

56.5% of users answered this


question correctly

A. Pre sacral tumour

End and review

B. Sacrococcygeal teratoma

6
7
8

C. Pilonidal sinus

Search

D. Fistula in ano
E. Occult spina bifida

Go

10
11

Next question

12
13

Pilonidal sinuses are extremely common in hirsute individuals and typically present as midline
sinuses in the natal cleft.

14
15

Pilonidal sinus

16
17
18

Occur as a result of hair debris creating sinuses in the skin (Bascom theory).
Usually in the natal cleft of male patients after puberty.
It is more common in Caucasians related to their hair type and growth patterns.
The opening of the sinus is lined by squamous epithelium, but most of its wall consists
of granulation tissue. Up to 50 cases of squamous cell carcinoma have been
described in patients with chronic pilonidal sinus disease.
Hairs become trapped within the sinus.
Clinically the sinus presents when acute inflammation occurs, leading to an abscess.
Patients may describe cycles of being asymptomatic and periods of pain and
discharge from the sinus.
Treatment is difficult and opinions differ. Definitive treatment should never be
undertaken when acute infection or abscess is present as this will result in failure.
Definitive treatments include the Bascom procedure with excision of the pits and
obliteration of the underlying cavity. The Karydakis procedure involves wide excision of
the natal cleft such that the surface is recontoured once the wound is closed. This
avoids the shearing forces that break off the hairs and has reasonable results.

19
20
21
22
23
24
25-27 2 / 3
28-30 0 / 3
31
32-34 0 / 3
35
36
37

Pilonidal sinuses are most commonly located in the midline of the natal cleft, as illustrated
below

38
39
40
41
42
43
44
45
46
47-49 2 / 3
50
51
52
53
54
55-57 1 / 3
58

Image sourced from Wikipedia

59
60

Rate question:

61

Next question

62-64 0 / 3
Comment on this question

65

All contents of this site are 2012 E-Medical Revision Ltd

http://www.emrcs.com/question/question.php?q=0

Terms and Conditions

Privacy policy

1/1

13/07/2015

Reference ranges

Previous

Question 66 of 284

Next

A 43 year old man from Greece presents with colicky right upper quadrant pain, jaundice and
an urticarial rash. He is initially treated with ciprofloxacin, but does not improve. What is the
most likely diagnosis?

Question stats

Score: 41.6%
1

17.5%

37.7%

10.3%

8.5%

26.1%

A. Infection with Wucheria bancrofti


37.7% of users answered this
question correctly

B. Infection with Echinococcus granulosus

End and review

6
7

C. Type III hypersensitivity reaction


8

D. Allergy to ciprofloxacin
9

Search

E. Common bile duct stones

Go

10
11

Next question

12

Infection with Echinococcus granulosus will typically produce a type I hypersensitivity reaction
which is characterised by a urticarial rash. With biliary rupture a classical triad of biliary colic,
jaundice and urticaria occurs. Whilst jaundice and biliary colic may be a feature of CBD
stones they do not produce an urticarial rash. Antibiotic sensitivity with ciprofloxacin may
produce jaundice and a rash, however it was not present at the outset and does not cause
biliary colic.

13

Hydatid cysts

18

14
15
16
17

19

Hydatid cysts are endemic in Mediterranean and Middle Eastern countries. They are caused
by the tapeworm parasite Echinococcus granulosus. An outer fibrous capsule is formed
containing multiple small daughter cysts. These cysts are allergens which precipitate a type
1 hypersensitivity reaction.

20
21
22
23

Clinical features are as follows:

24

Up to 90% cysts occur in the liver and lungs


Can be asymtomatic, or symptomatic if cysts > 5cm in diameter
Morbidity caused by cyst bursting, infection and organ dysfunction (biliary, bronchial,
renal and cerebrospinal fluid outflow obstruction)
In biliary ruputure there may be the classical triad of; biliary colic, jaundice, and
urticaria

25-27 2 / 3
28-30 0 / 3
31
32-34 0 / 3
35
36
37

CT is the best investigation to differentiate hydatid cysts from amoebic and pyogenic cysts.
Surgery is the mainstay of treatment (the cyst walls must not be ruptured during removal and
the contents sterilised first).

38
39
40

Rate question:

Next question

41

Comment on this question

42
43
44
45
46
47-49 2 / 3
50
51
52
53
54
55-57 1 / 3
58
59
60
61
62-64 0 / 3
65
66

All contents of this site are 2012 E-Medical Revision Ltd

http://www.emrcs.com/question/question.php?q=0

Terms and Conditions

Privacy policy

1/2

13/07/2015

http://www.emrcs.com/question/question.php?q=0

2/2

13/07/2015

Reference ranges

Previous

Question 67 of 284

Next

A 22 year old lady presents with an episode of renal colic and following investigation is
suspected of suffering from MEN IIa. Which of the following abnormalities of the parathyroid
glands are most often found in this condition?

Question stats

Score: 42.3%
1

8.2%

33.7%

30.4%

18.4%

9.4%

A. Hypertrophy
33.7% of users answered this
question correctly

B. Hyperplasia

End and review

6
7

C. Adenoma
8

D. Carcinoma
9

Search

E. Metaplasia

Go

Next question

10
11
12
13

MEN IIa

14

Medullary thyroid cancer


Hyperparathyroidism (usually hyperplasia)
Phaeochromocytoma

15
16
17
18

In MEN IIa the commonest lesion is medullary thyroid cancer, with regards to the parathyroid
glands the most common lesion is hyperplasia. In MEN I a parathyroid adenoma is the most
common lesion.

19
20
21

Multiple Endocrine Neoplasia

22

Multiple endocrine neoplasia (MEN) is inherited as an autosomal dominant disorder.

23
24

The table below summarises the three main types of MEN:


MEN type I

MEN type IIa

25-27 2 / 3
28-30 0 / 3

MEN type IIb

31
32-34 0 / 3

Mnemonic 'three P's':


Parathyroid (95%): Parathyroid adenoma
Pituitary (70%): Prolactinoma/ACTH/Growth
Hormone secreting adenoma
Pancreas (50%): Islet cell tumours/Zollinger
Ellison syndrome

Phaeochromocytoma
Medullary thyroid
cancer (70%)
Hyperparathyroidism
(60%)

Same as MEN IIa


with addition of:
Marfanoid body
habitus
Mucosal neuromas

36
37
38
39

also: Adrenal (adenoma) and thyroid (adenoma)


MENIN gene (chromosome 11)

35

40

RET oncogene
(chromosome 10)

RET oncogene
(chromosome 10)

Most common presentation = hypercalcaemia

41
42
43
44
45

Rate question:

Next question

46
47-49 2 / 3

Comment on this question

50
51
52
53
54
55-57 1 / 3
58
59
60
61
62-64 0 / 3
65
66
67

http://www.emrcs.com/question/question.php?q=0

1/2

13/07/2015

All contents of this site are 2012 E-Medical Revision Ltd

http://www.emrcs.com/question/question.php?q=0

Terms and Conditions

Privacy policy

2/2

13/07/2015

Reference ranges

Previous

Question 68 of 284

Next

A male infant is born prematurely at 34 weeks gestation by emergency cesarean section. He


initially appears to be stable. However, over the ensuing 24 hours he develops worsening
neurological function. Which of the following processes is most likely to have occurred?

Question stats

Score: 41.8%
1

11.6%

10.9%

9.3%

54.1%

14.1%

A. Extra dural haemorrhage


54.1% of users answered this
question correctly

B. Sub dural haemorrhage

End and review

6
7

C. Sub arachnoid haemorrhage


8

D. Intraventricular haemorrhage
9

Search

E. Arteriovenous malformation

Go

Next question

10
11
12

Theme from April 2012 Exam

13
14
15

Intraventricular haemorrhage

16

Intraventricular haemorrhage is a haemorrhage that occurs into the ventricular system of the
brain. It is relatively rare in adult surgical practice and when it does occur, it is typically
associated with severe head injuries. In premature neonates it may occur spontaneously.
The blood may clot and occlude CSF flow, hydrocephalus may result.
In neonatal practice the vast majority of IVH occur in the first 72 hours after birth, the
aetiology is not well understood and it is suggested to occur as a result of birth trauma
combined with cellular hypoxia, together the with the delicate neonatal CNS.

17
18
19
20
21
22

Treatment
Is largely supportive, therapies such as intraventricular thrombolysis and prophylactic CSF
drainage have been trialled and not demonstrated to show benefit. Hydrocephalus and rising
ICP is an indication for shunting.
Rate question:

Next question

23
24
25-27 2 / 3
28-30 0 / 3
31
32-34 0 / 3

Comment on this question

35
36
37
38
39
40
41
42
43
44
45
46
47-49 2 / 3
50
51
52
53
54
55-57 1 / 3
58
59
60
61
62-64 0 / 3
65
66
67
68

http://www.emrcs.com/question/question.php?q=0

1/2

13/07/2015

All contents of this site are 2012 E-Medical Revision Ltd

http://www.emrcs.com/question/question.php?q=0

Terms and Conditions

Privacy policy

2/2

13/07/2015

Reference ranges

Previous

Question 69 of 284

Next

A 22 year old man is admitted to hospital with a lower respiratory chest infection. He had a
splenectomy after being involved in a car accident. What is the most likely infective
organism?

Question stats

Score: 41.3%
1

67.9%

10.2%

6.4%

8.1%

7.3%

A. Haemophilus influenzae
67.9% of users answered this
question correctly

B. Staphylococcus aureus

End and review

6
7

C. Rhinovirus
8

D. Mycobacterium tuberculosis
9

Search

E. Moraxella catarrhalis

Go

Next question

10
11
12

Organisms causing post splenectomy sepsis:


Streptococcus pneumoniae
Haemophilus influenzae
Meningococci

13
14
15
16
17

Encapsulated organisms carry the greatest pathogenic risk following splenectomy. The
effects of sepsis following splenectomy are variable. This may be the result of small isolated
fragments of splenic tissue that retain some function following splenectomy. These may
implant spontaneously following splenic rupture (in trauma) or be surgically implanted at the
time of splenectomy.
Post splenectomy sepsis

18
19
20
21
22
23

Hyposplenism may complicate certain medical conditions where splenic atrophy occurs or
may be the result of medical intervention such as splenic artery embolization and
splenectomy for trauma. Diagnosis of hyposplenism is difficult and whilst there may be
peripheral markers of the splenectomised state (e.g. Howell Jolly bodies) these are neither
100% sensitive or specific. The most sensitive test is a radionucleotide labeled red cell scan.
Hyposplenism, by whatever mechanism it occurs dramatically increases the risk of post
splenectomy sepsis, particularly with encapsulated organisms. Since these organisms may
be opsonised, but this then goes undetected at an immunological level due to loss of the
spleen. For this reason individuals are recommended to be vaccinated and have antibiotic
prophylaxis.

24
25-27 2 / 3
28-30 0 / 3
31
32-34 0 / 3
35
36
37
38

Key recommendations
All those with hyposplenism or may become so (such as prior to an elective
splenectomy) should receive pneumococcal, haemophilus type b and meningococcal
type C vaccines. These should be administered 2 weeks prior to splenectomy or two
weeks following splenectomy. The vaccine schedule for meningococcal disease
essentially consists of a dose of Men C and Hib at 2 weeks and then a dose of the
MenACWY vaccine one month later. Those aged under 2 may require a booster at 2
years. A dose of pneumococcal polyvalent polysaccharide vaccine (PPV) is given at
two weeks. A conjugated vaccine (PCV) is offered to young children. The PCV is more
immunogenic but covers fewer serotypes. Boosting PPV is either guided by serological
measurements (where available) or by routine boosting doses at 5 yearly intervals.
Annual influenza vaccination is recommended in all cases
Antibiotic prophylaxis is offered to all. The risk of post splenectomy sepsis is greatest
immediately following splenectomy and in those aged less than 16 years or greater
than 50 years. Individuals with a poor response to pneumococcal vaccination are
another high risk group. High risk individuals should be counselled to take penicillin or
macrolide prophylaxis. Those at low risk may choose to discontinue therapy. All
patients should be advised about taking antibiotics early in the case of intercurrent
infections.
Asplenic individuals traveling to malaria endemic areas are at high risk and should
have both pharmacological and mechanical protection.

39
40
41
42
43
44
45
46
47-49 2 / 3
50
51
52
53
54
55-57 1 / 3
58
59
60

Dosing
Penicillin V 500mg BD or amoxicillin 250mg BD
References
Davies J et al. Review of guidelines for the prevention and treatment of infection in patients
with an absent or dysfunctional spleen: Prepared on behalf of the British Committee for
Standards in Haematology by a Working Party of the Haemato-Oncology Task Force. British
Journal of Haematology 2011 (155): 308317.

http://www.emrcs.com/question/question.php?q=0

61
62-64 0 / 3
65
66
67
68

1/2

13/07/2015
69

Rate question:

Next question

Comment on this question

All contents of this site are 2012 E-Medical Revision Ltd

http://www.emrcs.com/question/question.php?q=0

Terms and Conditions

Privacy policy

2/2

13/07/2015

Reference ranges

Question 70 of 284

Previous

Next

A 24 year old man presents with symptoms of malaise, weight loss and lymphadenopathy. A
lymph node biopsy is performed and the subsequent histology report states that there is
evidence of granuloma formation and central necrosis. What is the most likely underlying
cause?

Question stats

Score: 40.7%
1

12.1%

7.8%

9.5%

7.2%

63.4%

63.4% of users answered this


question correctly

A. Non Hodgkins lymphoma

End and review

B. Churg Strauss syndrome

6
7
8

C. Epstein Barr Virus infection

Search

D. Rheumatoid nodule
E. Infection with Mycobacterium tuberculosis

Go

10
11

Next question

12
13

These histological features are typically seen in TB. Necrosis occurring in granulomas is
usually indicative of an underlying infective cause. Churg Strauss syndrome is a form of
vasculitis, which is the usual histological finding. Granulomas are reported in the condition,
but it is rare for them to demonstrate necrosis.

14
15
16
17

Tuberculosis pathology

18
19

Is a form of primary chronic inflammation, caused by the inability of macrophages to kill


the Mycobacterium tuberculosis.
The macrophages often migrate to regional lymph nodes, the lung lesion plus affected
lymph nodes is referred to as a Ghon complex.
This leads to the formation of a granuloma which is a collection of epithelioid
histiocytes.
There is the presence of caseous necrosis in the centre.
The inflammatory response is mediated by a type 4 hypersensitivity reaction.
In healthy individuals the disease may be contained, in the immunocompromised
disseminated (miliary TB) may occur.

20
21
22
23
24
25-27 2 / 3
28-30 0 / 3
31
32-34 0 / 3

Diagnosis

35

Waxy membrane of mycobacteria prevents binding with normal stains. Ziehl - Neelsen
staining is typically used.
Culture based methods take far longer.

36
37
38
39

Image showing acid- alcohol fast mycobacteria stained using the Ziehl- Neelsen method

40
41
42
43
44
45
46
47-49 2 / 3
50
51
52

Image sourced from Wikipedia

53
54

Rate question:
Comment on this question

Next question

55-57 1 / 3
58
59
60
61
62-64 0 / 3
65
66
67
68

http://www.emrcs.com/question/question.php?q=0

1/2

13/07/2015
69
70

All contents of this site are 2012 E-Medical Revision Ltd

http://www.emrcs.com/question/question.php?q=0

Terms and Conditions

Privacy policy

2/2

13/07/2015

Reference ranges

Previous

Question 71 of 284

Next

A 20 year old man develops acute appendicitis, his appendix is removed and he makes a full
recovery. Which of the following pathological processes is least likely to be present in the
acutely inflamed tissues?

Question stats

Score: 39.8%
1

16.4%

10.6%

8.3%

55.6%

9.1%

A. Altered Starlings forces.


55.6% of users answered this
question correctly

B. Seqestration of neurophils

End and review

6
7

C. Formation of fluid exudate


8

D. Formation of granulomas
9

Search

E. None of the above

Go

Next question

10
11
12

Neutrophil polymorphs=Acute inflammation.


Granuloma = Chronic inflammation.

13
14
15

Acute inflammation:
3 phases
1. Changes in blood vessel and flow: flush, flare, wheal
2. Fluid exudates (rich in protein i.e. Ig, coagulation factors) produced via increased vascular
permeability
3. Cellular exudates mainly containing neutrophil polymorphs pass into extravascular space.
Neutrophils are then transported to tissues via:

16
17
18
19
20
21
22

a. Margination of neutrophils to the peripheral plasmatic of the vessel rather than the
central axial stream
b. Pavementing: Adhesion of neutrophils to endothelial cells in venules at site of acute
inflammation
c. Emigration: neutrophils pass between endothelial cells into the tissue

25-27 2 / 3
28-30 0 / 3

Acute inflammation

32-34 0 / 3

23
24

31
35

Inflammation is the reaction of the tissue elements to injury. Vascular changes occur,
resulting in the generation of a protein rich exudate. So long as the injury does not totally
destroy the existing tissue architecture, the episode may resolve with restoration of original
tissue architecture.

36
37
38
39

Vascular changes
40

Vasodilation occurs and persists throughout the inflammatory phase.


Inflammatory cells exit the circulation at the site of injury.
The equilibrium that balances Starlings forces within capillary beds is disrupted and a
protein rich exudate will form as the vessel walls also become more permeable to
proteins.
The high fibrinogen content of the fluid may form a fibrin clot. This has several
important immunomodulatory functions.

41
42
43
44
45
46
47-49 2 / 3

Sequelae

50

Resolution

Typically occurs with minimal initial injury


Stimulus removed and normal tissue architecture results

51
52
53

Organisation

Delayed removal of exudate


Tissues undergo organisation and usually fibrosis

54
55-57 1 / 3
58

Suppuration

Typically formation of an abscess or an empyema


Sequestration of large quantities of dead neutrophils

59
60
61

Progression to chronic
inflammation

Coupled inflammatory and reparative activities


Usually occurs when initial infection or suppuration has
been inadequately managed

62-64 0 / 3
65
66
67

Causes

http://www.emrcs.com/question/question.php?q=0

68

1/2

13/07/2015

Infections e.g. Viruses, exotoxins or endotoxins released by bacteria


Chemical agents
Physical agents e.g. Trauma
Hypersensitivity reactions
Tissue necrosis

69
70
71

Presence of neutrophil polymorphs is a histological diagnostic feature of acute


inflammation
Rate question:

Next question

Comment on this question

All contents of this site are 2012 E-Medical Revision Ltd

http://www.emrcs.com/question/question.php?q=0

Terms and Conditions

Privacy policy

2/2

13/07/2015

Reference ranges

Previous1 / 3

Question 72-74 of 284

Next

Question stats

End and review

Score: 39.5%

Average score for registered users:

1
2

Theme: Liver lesions


A. Haemangioma

72

90.8%

73

70.1%

74

46.8%

B. Hepatocellular carcinoma

3
4
5

C. Hepatic metastasis

D. Polycystic liver disease

Search

E. Simple liver cyst

7
Go

F. Hyatid cyst

8
9

G. Amoebic abscess
10

H. Mesenchymal hamartoma
11

Please select the most likely liver lesion for the scenario given. Each option may be used
once, more than once or not at all.

12
13
14
15

72.

A 42 year old lady has suffered from hepatitis C for many years and has also
developed cirrhosis. On routine follow up, an ultrasound has demonstrated a
2.5cm lesion in the right lobe of the liver.

16
17
18

Hepatocellular carcinoma

19

In patients with cirrhosis the presence of a lesion >2cm is highly suggestive of


malignancy. The diagnosis is virtually confirmed if the AFP is >400ng/mL.

20
21

73.

A 25 year old man from the far east presents with a fever and right upper
quadrant pain. As part of his investigations a CT scan shows an ill defined
lesion in the right lobe of the liver.

22
23
24

You answered Polycystic liver disease

25-27 2 / 3
28-30 0 / 3

The correct answer is Amoebic abscess

31

Amoebic abscesses will tend to present in a similar fashion to other pyogenic


liver abscesses. They should be considered in any individual presenting from a
region where Entamoeba histiolytica is endemic. Treatment with metronidazole
usually produces a marked clinical response.

32-34 0 / 3
35
36
37

74.

A 42 year old lady presents with right upper quadrant pain and a sensation of
abdominal fullness. An ultrasound scan demonstrates a 6.5 cm hyperechoic
lesion in the right lobe of the liver. Serum AFP is normal.

38
39
40

You answered Hepatic metastasis

41

The correct answer is Haemangioma

42

A large hyperechoic lesion in the presence of normal AFP is likely to be a


haemangioma. An HCC of equivalent size will almost always result in rise in
AFP.

43
44
45
46

Next question

47-49 2 / 3
50
51

Benign liver lesions

52
53
54

Benign liver lesions


Haemangioma

55-57 1 / 3

Most common benign tumours of mesenchymal origin


Incidence in autopsy series is 8%
Cavernous haemangiomas may be enormous
Clinically they are reddish purple hypervascular lesions
Lesions are normally separated from normal liver by ring of fibrous tissue
On ultrasound they are typically hyperechoic

58
59
60
61
62-64 0 / 3

Liver cell
adenoma

65

90% develop in women in their third to fifth decade


Linked to use of oral contraceptive pill
Lesions are usually solitary
They are usually sharply demarcated from normal liver although they
usually lack a fibrous capsule

http://www.emrcs.com/question/question.php?q=0

66
67
68

1/2

13/07/2015

On ultrasound the appearances are of mixed echoity and heterogeneous


texture. On CT most lesions are hypodense when imaged prior to
administration of IV contrast agents
In patients with haemorrhage or symptoms removal of the adenoma may
be required

69
70
71
72-74 1 / 3

Mesenchymal
hamartomas
Liver abscess

Amoebic
abscess

Hyatid cysts

Polycystic
liver disease

Cystadenoma

Congential and benign, usually present in infants. May compress normal liver

Biliary sepsis is a major predisposing factor


Structures drained by the portal venous system form the second largest
source
Common symptoms include fever, right upper quadrant pain. Jaundice
may be seen in 50%
Ultrasound will usually show a fluid filled cavity, hyperechoic walls may
be seen in chronic abscesses

Liver abscess is the most common extra intestinal manifestation of


amoebiasis
Between 75 and 90% lesions occur in the right lobe
Presenting complaints typically include fever and right upper quadrant
pain
Ultrasonography will usually show a fluid filled structure with poorly
defined boundaries
Aspiration yield sterile odourless fluid which has an anchovy paste
consistency
Treatment is with metronidazole

Seen in cases of Echinococcus infection


Typically an intense fibrotic reaction occurs around sites of infection
The cyst has no epithelial lining
Cysts are commonly unilocular and may grow to 20cm in size. The cyst
wall is thick and has an external laminated hilar membrane and an
internal enucleated germinal layer
Typically presents with malaise and right upper quadrant pain.
Secondary bacterial infection occurs in 10%.
Liver function tests are usually abnormal and eosinophilia is present in
33% cases
Ultrasound may show septa and hyatid sand or daughter cysts.
Percutaneous aspiration is contra indicated
Treatment is by sterilisation of the cyst with mebendazole and may be
followed by surgical resection. Hypertonic swabs are packed around the
cysts during surgery

Usually occurs in association with polycystic kidney disease


Autosomal dominant disorder
Symptoms may occur as a result of capsular stretch

Rare lesions with malignant potential


Usually solitary multiloculated lesions
Liver function tests usually normal
Ultrasonography typically shows a large anechoic, fluid filled area with
irregular margins. Internal echos may result from septa
Surgical resection is indicated in all cases

Rate question:

Next question

Comment on this question

All contents of this site are 2012 E-Medical Revision Ltd

http://www.emrcs.com/question/question.php?q=0

Terms and Conditions

Privacy policy

2/2

13/07/2015

Reference ranges

Previous

Question 75 of 284

Next

Question stats

End and review

Score: 40.2%
1

22.4%

31%

8%

A. Acute lymphoblastic leukaemia

7.3%

B. Acute myeloblastic leukaemia

31.4%

C. Acute myelomonocytic leukaemia

31.4% of users answered this


question correctly

Which of the following disorders is associated with massive splenomegaly?

D. Acute monoblastic leukaemia


E. Chronic granulocytic leukaemia

6
7
8
9

Search
Next question
Go

Chronic leukaemia is more likely to be associated with splenomegaly than acute leukaemia.

10
11
12

Spleen

13
14

The spleen is located in the left upper quadrant of the abdomen and its size can vary
depending upon the amount of blood it contains. The typical adult spleen is 12.5cm long and
7.5cm wide. The usual weight of the adult spleen is 150g.
The exact position of the spleen can vary with respiratory activity, posture and the state of
surrounding viscera. It usually lies obliquely with its long axis aligned to the 9th, 10th and
11th ribs. It is separated from these ribs by both diaphragm and pleural cavity. The normal
spleen is not palpable.

15
16
17
18
19
20

The shape of the spleen is influenced by the state of the colon and stomach. Gastric
distension will cause the spleen to resemble the shape of an orange segment. Colonic
distension will cause it to become more tetrahedral.

21
22
23

The spleen is almost entirely covered by peritoneum, which adheres firmly to its capsule.
Recesses of the greater sac separate it from the stomach and kidney. It develops from the
upper dorsal mesogastrium, remaining connected to the posterior abdominal wall and
stomach by two folds of peritoneum; the lienorenal ligament and gastrosplenic ligament. The
lienorenal ligament is derived from peritoneum where the wall of the general peritoneum
meets the omental bursa between the left kidney and spleen; the splenic vessels lie in its
layers. The gastrosplenic ligament also has two layers, formed by the meeting of the walls of
the greater sac and omental bursa between spleen and stomach, the short gastric and left
gastroepiploic branches of the splenic artery pass in its layers. Laterally, the spleen is in
contact with the phrenicocolic ligament.

24
25-27 2 / 3
28-30 0 / 3
31
32-34 0 / 3
35
36
37
38
39

Relations
Superiorly

Diaphragm

40

Anteriorly

Gastric impression

41

Posteriorly Kidney

42

Inferiorly

Colon

43

Hilum

Tail of pancreas and splenic vessels (splenic artery divides here, branches
pass to the w hite pulp transporting plasma)

44
45

Contents
White
pulp

Immune function. Contains central trabecular artery. The germinal centres are
supplied by arterioles called penicilliary radicles.

Red
pulp

Filters abnormal red blood cells.

46
47-49 2 / 3
50
51
52

Function
Filtration of abnormal blood cells and foreign bodies such as bacteria.
Immunity: IgM. Production of properdin, and tuftsin which help target fungi and bacteria
for phagocytosis.
Haematopoiesis: up to 5th month gestation or in haematological disorders.
Pooling: storage of 40% platelets.
Iron reutilisation
Storage monocytes

53
54
55-57 1 / 3
58
59
60
61
62-64 0 / 3

Disorders of the spleen


Massive splenomegaly
Myelofibrosis
Chronic myeloid leukaemia

http://www.emrcs.com/question/question.php?q=0

65
66
67
68

1/2

13/07/2015

Visceral leishmaniasis (kala-azar)


Malaria
Gaucher's syndrome

69
70
71
72-74 1 / 3

Other causes (as above plus)

75

Portal hypertension e.g. secondary to cirrhosis


Lymphoproliferative disease e.g. CLL, Hodgkin's
Haemolytic anaemia
Infection: hepatitis, glandular fever
Infective endocarditis
Sickle-cell*, thalassaemia
Rheumatoid arthritis (Felty's syndrome)

*the majority of adult patients with sickle-cell will have an atrophied spleen due to repeated
infarction
Rate question:

Next question

Comment on this question

All contents of this site are 2012 E-Medical Revision Ltd

http://www.emrcs.com/question/question.php?q=0

Terms and Conditions

Privacy policy

2/2

13/07/2015

Reference ranges

Previous

Question 76 of 284

Next

Question stats

End and review

Score: 39.8%
1

10.9%

10.9%

10.4%

A. Sarcoidosis

37.3%

B. Tuberculosis

30.6%

C. Ulcerative colitis

30.6% of users answered this


question correctly

Causes of primary chronic inflammation do not include which of the following?

D. Hip prostheses
E. Chronic cholecystitis

6
7
8
9

Search
Next question
Go

Chronic cholecystitis is caused by recurrent episodes of acute inflammation.


Prosthetic implants may be the site of primary chronic inflammation. A common example
clinically is breast implants which may become encapsulated. The subsequent fibrosis then
results in distortion and may be painful.

10
11
12
13
14
15

Chronic inflammation

16

Overview
Chronic inflammation may occur secondary to acute inflammation.In most cases chronic
inflammation occurs as a primary process. These may be broadly viewed as being one of
three main processes:
Persisting infection with certain organisms such as Mycobacterium tuberculosis which
results in delayed type hypersensitivity reactions and inflammation.
Prolonged exposure to non-biodegradable substances such as silica or suture
materials which may induce an inflammatory response.
Autoimmune conditions involving antibodies formed against host antigens.

17
18
19
20
21
22
23
24
25-27 2 / 3
28-30 0 / 3

Acute vs. Chronic inflammation


Acute inflammation

Chronic inflammation

Changes to existing vascular structure and increased


permeability of endothelial cells

Angiogenesis predominates

Infiltration of neutrophils

Macrophages, plasma cells and


lymphocytes predominate

31
32-34 0 / 3
35
36
37
38

Process may resolve with:

Healing by fibrosis is the main result


39

Suppuration
Complete resolution
Abscess formation
Progression to chronic inflammation
Healing by fibrosis

40
41
42
43

Granulomatous inflammation
A granuloma consists of a microscopic aggregation of macrophages (with epithelial type
arrangement =epitheliod). Large giant cells may be found at the periphery of granulomas.

44
45
46
47-49 2 / 3

Mediators
Growth factors released by activated macrophages include agents such as interferon and
fibroblast growth factor (plus many more). Some of these such as interferons may have
systemic features resulting in systemic symptoms and signs, which may be present in
individuals with long standing chronic inflammation.
The finding of granulomas is pathognomonic of chronic inflammation, as illustrated in this
biopsy from a patient with colonic Crohns disease

http://www.emrcs.com/question/question.php?q=0

50
51
52
53
54
55-57 1 / 3

1/2

13/07/2015
58
59
60
61
62-64 0 / 3
65
66
67
68
69
Image sourced from Wikipedia

70
71
72-74 1 / 3

Rate question:

Next question

75
76

Comment on this question

All contents of this site are 2012 E-Medical Revision Ltd

http://www.emrcs.com/question/question.php?q=0

Terms and Conditions

Privacy policy

2/2

13/07/2015

Reference ranges

Previous0 / 3

Question 78-80 of 284

Next

Question stats

End and review

Score: 37.6%

Average score for registered users:

1
2

Theme: Adrenal gland disorders


A. Nelsons syndrome

78

73.6%

79

77.8%

80

81%

B. Conns syndrome

3
4
5

C. Cushings syndrome

D. Benign incidental adenoma

Search

E. Malignant adrenal adenoma

7
Go

F. Waterhouse- Friderichsen syndrome

8
9

G. Metastatic lesion
10

H. Walker - Warburg syndrome


11

I. Phaeochromocytoma

12

Please select the most appropriate adrenal disorder for the scenario given. Each disorder
may be selected once, more than once or not at all.

13
14
15

78.

A 19 year old lady is admitted to ITU with severe meningococcal sepsis. She is
on maximal inotropic support and a CT scan of her chest and abdomen is
performed. The adrenal glands show evidence of diffuse haemorrhage.

16
17
18
19

You answered Metastatic lesion

20

The correct answer is Waterhouse- Friderichsen syndrome

21

WaterhouseFriderichsen syndrome is defined as adrenal gland failure due to


bleeding into the adrenal glands. It is caused by severe bacterial infection
(most commonly the meningococcus Neisseria meningitidis).

22
23
24

The bacterial infection leads to massive hemorrhage into one or (usually) both
adrenal glands. It is characterised by overwhelming bacterial infection
meningococcemia leading to massive blood invasion, organ failure, coma,
haemodynamic shock, disseminated intravascular coagulation with widespread
purpura, rapidly developing adrenocortical insufficiency and death.

25-27 2 / 3
28-30 0 / 3
31
32-34 0 / 3
35

79.

A 34 year old lady is admitted with recurrent episodes of non-specific


abdominal pain. On each admission all blood investigations are normal, as are
her observations. On this admission a CT scan was performed. This
demonstrates a 1.5cm nodule in the right adrenal gland. This is associated with
a lipid rich core. Urinary VMA is within normal limits. Other hormonal studies are
normal.

36
37
38
39
40

You answered Cushings syndrome

41

The correct answer is Benign incidental adenoma

42

This is typical for a benign adenoma. Benign adenomas often have a lipid rich
core that is readily identifiable on CT scanning. In addition the nodules are
often well circumscribed.
80.

A 38 year old man is noted to have a blood pressure of 175/110 on routine


screening. On examination there are no physical abnormalities of note. CT
scanning shows a left sided adrenal mass. Plasma metanephrines are
elevated.

43
44
45
46
47-49 2 / 3
50
51

You answered Conns syndrome

52
53

The correct answer is Phaeochromocytoma

54

Hypertension in a young patient without any obvious cause should be


investigated. Urinary VMA and plasma metanephrines are typically elevated.

55-57 1 / 3
58
59

Next question

60
61
62-64 0 / 3

Phaeochromocytoma and adrenal lesions

65
66

Phaeochromocytoma
Neuroendocrine tumour of the chromaffin cells of the adrenal medulla. Hypertension and
hyperglycaemia are often found.
http://www.emrcs.com/question/question.php?q=0

67
68

1/2

13/07/2015

hyperglycaemia are often found.


69

10% of cases are bilateral.


10% occur in children.
11% are malignant (higher when tumour is located outside the adrenal).
10% will not be hypertensive.

70
71
72-74 1 / 3
75
76

Familial cases are usually linked to the Multiple endocrine neoplasia syndromes (considered
under its own heading).

77
78-80 0 / 3

Most tumours are unilateral (often right sided) and smaller than 10cm.
Diagnosis
Urine analysis of vanillymandelic acid (VMA) is often used (false positives may occur e.g. in
patients eating vanilla ice cream!)
Blood testing for plasma metanephrine levels.
CT and MRI scanning are both used to localise the lesion.
Treatment
Patients require medical therapy first. An irreversible alpha adrenoreceptor blocker should
be given, although minority may prefer reversible blockade(1). Labetolol may be coadministered for cardiac chronotropic control. Isolated beta blockade should not be
considered as it will lead to unopposed alpha activity.
These patients are often volume depleted and will often require moderate volumes of intra
venous normal saline perioperatively.
Once medically optimised the phaeochromocytoma should be removed. Most
adrenalectomies can now be performed using a laparoscopic approach(2). The adrenals are
highly vascular structures and removal can be complicated by catastrophic haemorrhage in
the hands of the inexperienced. This is particularly true of right sided resections where the
IVC is perilously close. Should the IVC be damaged a laparotomy will be necessary and the
defect enclosed within a Satinsky style vascular clamp and the defect closed with prolene
sutures. Attempting to interfere with the IVC using any instruments other than vascular
clamps will result in vessel trauma and make a bad situation much worse.
Incidental adrenal lesions
Adrenal lesions may be identified on CT scanning performed for other reasons(3). Factors
suggesting benign disease on CT include(4):
Size less than 3cm
Homogeneous texture
Lipid rich tissue
Thin wall to lesion

All patients with incidental lesions should be managed jointly with an endocrinologist and full
work up as described above. Patients with functioning lesions or those with adverse
radiological features (Particularly size >3cm) should proceed to surgery.

References
1. Weingarten TN, Cata JP, O'Hara JF, Prybilla DJ, Pike TL, Thompson GB, et al.
Comparison of two preoperative medical management strategies for laparoscopic resection
of pheochromocytoma. Urology. 2010 Aug;76(2):508 e6-11.
2. Nguyen PH, Keller JE, Novitsky YW, Heniford BT, Kercher KW. Laparoscopic approach to
adrenalectomy: review of perioperative outcomes in a single center. Am Surg. 2011
May;77(5):592-6.
3. Ng VW, Ma RC, So WY, Choi KC, Kong AP, Cockram CS, et al. Evaluation of functional
and malignant adrenal incidentalomas. Arch Intern Med. 2010 Dec 13;170(22):2017-20.
4. Muth A, Hammarstedt L, Hellstrom M, Sigurjonsdottir HA, Almqvist E, Wangberg B. Cohort
study of patients with adrenal lesions discovered incidentally. Br J Surg. 2011 May 27.
Rate question:

Next question

Comment on this question

All contents of this site are 2012 E-Medical Revision Ltd

http://www.emrcs.com/question/question.php?q=0

Terms and Conditions

Privacy policy

2/2

13/07/2015

Reference ranges

Previous

Question 81 of 284

Next

A 43 year old man presents with dyspepsia and undergoes an upper GI endoscopy. During
the procedure diffuse gastric and duodenal ulcers are identified. A Clo test confirms the
presence of Helicobacter pylori infection. What is the most likely explanation for the ulcers?

Question stats

Score: 37.2%
1

7.7%

33%

16%

8.4%

34.9%

A. Decreased gastric motility


34.9% of users answered this
question correctly

B. Increased urease activity

End and review

6
7

C. Decreased release of mucous and bicarbonate


8

D. Decreased gastrin levels


9

Search

E. Increased acid production

Go

Next question

10
11
12

Theme from April 2011 Exam


H-Pylori has a number of pathological effects. In this question the main issue is by what
mechanism the organism is able to induce both gastric and duodenal ulceration. Without
modestly elevated acid levels, the duodenum would not undergo gastric metaplasia. H-Pylori
cannot colonise duodenal mucosa and therefore the development of ulcers at this site can
only occur in those who have undergone metaplastic transformation (mediated by increased
acidity).

13
14
15
16
17
18

Helicobacter Pylori

19
20

Infection with Helicobacter Pylori is implicated in many cases of duodenal ulceration and up
to 60% of patients with gastric ulceration.

21
22

It is a gram negative, helical shaped rod with microaerophillic requirements. It has the ability
to produce a urease enzyme that will hydrolyse urea resulting in the production of ammonia.
The effect of ammonia on antral G cells is to cause release of gastrin via a negative
feedback loop.
Once infection is established the organism releases enzymes that disrupt the gastric mucous
layer. Certain subtypes release cytotoxins cag A and vac A gene products. The organism
incites a classical chronic inflammatory process of the gastric epithelium. This accounts for
the development of gastric ulcers. The mildly increased acidity may induce a process of
duodenal gastric metaplasia. Whilst duodenal mucosa cannot be colonised by H-Pylori,
mucosa that has undergone metaplastic change to the gastric epithelial type may be
colonised by H- Pylori with subsequent inflammation and development of duodenitis and
ulcers.
In patients who are colonised there is a 10-20% risk of peptic ulcer, 1-2% risk gastric cancer
and <1% risk MALT lymphoma.

23
24
25-27 2 / 3
28-30 0 / 3
31
32-34 0 / 3
35
36
37
38
39
40

Rate question:
Comment on this question

Next question

41
42
43
44
45
46
47-49 2 / 3
50
51
52
53
54
55-57 1 / 3
58
59
60
61
62-64 0 / 3
65
66
67
68

http://www.emrcs.com/question/question.php?q=0

1/2

13/07/2015
69
70
71
72-74 1 / 3
75
76
77
78-80 0 / 3
81

All contents of this site are 2012 E-Medical Revision Ltd

http://www.emrcs.com/question/question.php?q=0

Terms and Conditions

Privacy policy

2/2

13/07/2015

Reference ranges

Previous

Question 82 of 284

Next

A 15 year old boy is admitted with colicky abdominal pain of 6 hours duration. On
examination he has a soft abdomen, on systemic examination he has brownish spots around
his mouth, feet and hands. His mother underwent surgery for intussusception, aged 12, and
has similar lesions. What is the most likely underlying diagnosis?

Question stats

Score: 36.8%
1

10.3%

66%

8.5%

8.8%

6.4%

66% of users answered this


question correctly

A. Li Fraumeni syndrome

End and review

B. Peutz-Jeghers syndrome

6
7
8

C. Addisons disease

Search

D. McCune -Albright syndrome


E. Appendicitis

Go

10
11

Next question

12
13

This is most likely to be Peutz-Jeghers syndrome. Addisons and McCune Albright syndrome
may produce similar skin changes but the intussusception resulting from polyps combined
with the autosomal inheritance pattern makes this the most likely diagnosis.

14
15
16

Peutz-Jeghers syndrome

17

Peutz-Jeghers syndrome is an autosomal dominant condition characterised by numerous


benign hamartomatous polyps in the gastrointestinal tract. It is also associated with
pigmented freckles on the lips, face, palms and soles. Around 50% of patients will have died
from a gastrointestinal tract cancer by the age of 60 years.

18
19
20
21
22

Genetics

23

Autosomal dominant
Responsible gene encodes serine threonine kinase LKB1 or STK11

24
25-27 2 / 3
28-30 0 / 3

Features

31

Hamartomatous polyps in GI tract (mainly small bowel)


Pigmented lesions on lips, oral mucosa, face, palms and soles
Intestinal obstruction e.g. intussusception (which may lead to diagnosis)
Gastrointestinal bleeding

32-34 0 / 3
35
36
37
38

Management

39

Conservative unless complications develop

40
41
42

Rate question:
Comment on this question

Next question

43
44
45
46
47-49 2 / 3
50
51
52
53
54
55-57 1 / 3
58
59
60
61
62-64 0 / 3
65
66
67
68

http://www.emrcs.com/question/question.php?q=0

1/2

13/07/2015
69
70
71
72-74 1 / 3
75
76
77
78-80 0 / 3
81
82

All contents of this site are 2012 E-Medical Revision Ltd

http://www.emrcs.com/question/question.php?q=0

Terms and Conditions

Privacy policy

2/2

13/07/2015

Reference ranges

Previous

Question 83 of 284

Next

What is the most likely electrolyte abnormality in a patient with diarrhoea and a soft mass felt
on digital rectal examination?

A. Hyperkalaemia
B. Hypokalaemia

Question stats

Score: 37.5%
1

9.9%

61.3%

13.8%

7.3%

7.6%

61.3% of users answered this


question correctly

C. Hyponatraemia

End and review

D. Hypernatraemia

6
7
8

E. Hypocalcaemia

Search
Next question

Go

10
11

Large villous adenomas of the rectum may have marked secretory activity and result in the
development of hypokalaemia as rectal secretions are rich in potassium.

12
13
14

Hypokalaemia

15

Potassium and hydrogen can be thought of as competitors. Hyperkalaemia tends to be


associated with acidosis because as potassium levels rise fewer hydrogen ions can enter the
cells

16

Hypokalaemia with alkalosis

19

17
18

20

Vomiting
Diuretics
Cushing's syndrome
Conn's syndrome (primary hyperaldosteronism)

21
22
23
24

Hypokalaemia with acidosis

25-27 2 / 3
28-30 0 / 3

Diarrhoea
Renal tubular acidosis
Acetazolamide
Partially treated diabetic ketoacidosis

31
32-34 0 / 3
35
36
37

Rate question:
Comment on this question

Next question

38
39
40
41
42
43
44
45
46
47-49 2 / 3
50
51
52
53
54
55-57 1 / 3
58
59
60
61
62-64 0 / 3
65
66
67
68

http://www.emrcs.com/question/question.php?q=0

1/2

13/07/2015
69
70
71
72-74 1 / 3
75
76
77
78-80 0 / 3
81
82
83

All contents of this site are 2012 E-Medical Revision Ltd

http://www.emrcs.com/question/question.php?q=0

Terms and Conditions

Privacy policy

2/2

13/07/2015

Reference ranges

Previous

Question 84 of 284

Next

Question stats

End and review

Score: 38.1%
1

10.4%

19.4%

10%

A. Phaeochromocytoma

45.1%

B. Visceral ganglioneuromas

15.2%

C. Thyroid medullary carcinoma

45.1% of users answered this


question correctly

Which of the following is not included in Multiple Endocrine Neoplasia Type 2b?

D. Zollinger Ellison syndrome


E. Marfanoid features

6
7
8
9

Search
Next question
Go

10
11

MEN IIB

12

Medullary thyroid cancer


Phaeochromocytoma
Mucosal neuroma
Marfanoid appearance

13
14
15
16
17

Multiple Endocrine Neoplasia

18

Multiple endocrine neoplasia (MEN) is inherited as an autosomal dominant disorder.

19
20

The table below summarises the three main types of MEN:

21
22

MEN type I

MEN type IIa

MEN type IIb

Mnemonic 'three P's':

Phaeochromocytoma
Medullary thyroid
cancer (70%)
Hyperparathyroidism
(60%)

Same as MEN IIa


with addition of:
Marfanoid body
habitus
Mucosal neuromas

Parathyroid (95%): Parathyroid adenoma


Pituitary (70%): Prolactinoma/ACTH/Growth
Hormone secreting adenoma
Pancreas (50%): Islet cell tumours/Zollinger
Ellison syndrome

24
25-27 2 / 3
28-30 0 / 3
31
32-34 0 / 3
35

also: Adrenal (adenoma) and thyroid (adenoma)


MENIN gene (chromosome 11)

23

36

RET oncogene
(chromosome 10)

RET oncogene
(chromosome 10)

37
38

Most common presentation = hypercalcaemia


39
40
41

Rate question:

Next question

42
43

Comment on this question

44
45
46
47-49 2 / 3
50
51
52
53
54
55-57 1 / 3
58
59
60
61
62-64 0 / 3
65
66
67
68

http://www.emrcs.com/question/question.php?q=0

1/2

13/07/2015
69
70
71
72-74 1 / 3
75
76
77
78-80 0 / 3
81
82
83
84

All contents of this site are 2012 E-Medical Revision Ltd

http://www.emrcs.com/question/question.php?q=0

Terms and Conditions

Privacy policy

2/2

13/07/2015

Reference ranges

Previous

Question 85 of 284

Next

A 25 year old male pedestrian is involved in a road traffic accident. He sustains multiple
injuries and is admitted to the intensive care unit, intubated and ventilated. Over the next
week he develops adult respiratory distress syndrome. What is the main reason for
hypoxaemia in this condition?

Question stats

Score: 37.8%
1

17.1%

43%

19.1%

10.6%

10.2%

43% of users answered this


question correctly

A. Increased lung compliance

End and review

B. Reduced diffusion

6
7
8

C. Reduced surfactant

Search

D. Reduced elastase
E. Left to right shunt

Go

10
11

Next question

12
13

Theme from September 2014 Exam


The diffuse lung injury, which is associated with loss of surfactant and increased elastase
release from neutrophils, results in fluid accumulation. This leads to reduced diffusion, which
is the main reason for hypoxaemia.

14
15
16
17

Adult respiratory distress syndrome

18

Defined as an acute condition characterized by bilateral pulmonary infiltrates and severe


hypoxemia (PaO2/FiO2 ratio < 200) in the absence of evidence for cardiogenic pulmonary
oedema (clinically or pulmonary capillary wedge pressure of less than 18 mm Hg).
In is subdivided into two stages. Early stages consist of an exudative phase of injury with
associated oedema. The later stage is one of repair and consists of fibroproliferative
changes. Subsequent scarring may result in poor lung function.

19
20
21
22
23
24

Causes
Sepsis
Direct lung injury
Trauma
Acute pancreatitis
Long bone fracture or multiple fractures (through fat embolism)
Head injury (causes sympathetic nervous stimulation which leads to acute pulmonary
hypertension)

25-27 2 / 3
28-30 0 / 3
31
32-34 0 / 3
35
36
37
38

Clinical features

39
40

Acute dyspnoea and hypoxaemia hours/days after event


Multi organ failure
Rising ventilatory pressures

41
42
43

Management

44

Treat the underlying cause


Antibiotics (if signs of sepsis)
Negative fluid balance i.e. Diuretics
Recruitment manoeuvres such as prone ventilation, use of positive end expiratory
pressure
Mechanical ventilation strategy using low tidal volumes, as conventional tidal volumes
may cause lung injury (only treatment found to improve survival rates)

45
46
47-49 2 / 3
50
51
52
53
54

Rate question:
Comment on this question

Next question

55-57 1 / 3
58
59
60
61
62-64 0 / 3
65
66
67
68

http://www.emrcs.com/question/question.php?q=0

1/2

13/07/2015
69
70
71
72-74 1 / 3
75
76
77
78-80 0 / 3
81
82
83
84
85

All contents of this site are 2012 E-Medical Revision Ltd

http://www.emrcs.com/question/question.php?q=0

Terms and Conditions

Privacy policy

2/2

13/07/2015

Reference ranges

Previous

Question 86 of 284

Next

A 24 year old male was admitted with bloody diarrhea, cramping abdominal pain and weight
loss. Colonoscopy revealed a friable, diffusely red mucosa involving the rectum and sigmoid
colon. The mucosa was normal proximal to this. The disease progressed with time to involve
most of the entire colon, but not the ileum. Many years later, a colonic biopsy shows high
grade epithelial dysplasia. What is the most likely initial diagnosis?

Question stats

Score: 37.4%
1

6.5%

7.6%

67.4%

12.1%

6.6%

67.4% of users answered this


question correctly

A. Colonic tuberculosis

End and review

6
7
8

B. Collagenous colitis
C. Ulcerative colitis

Search

D. Crohns disease

Go

E. Ischaemic colitis

10
11
12

Next question

Theme from January 2015 Exam


Ulcerative colitis spreads in a progressive distal to proximal manner. Over time a dysplastic
transformation is recognised. Such endoscopic findings mandate a minimum of close
endoscopic surveillance and if they occur in association with a colonic mass then usually a
pancproctocolectomy.

13
14
15
16
17
18

Ulcerative colitis

19
20

Ulcerative colitis is a form of inflammatory bowel disease. Inflammation always starts at


rectum, does not spread beyond ileocaecal valve (although backwash ileitis may occur) and
is continuous. The peak incidence of ulcerative colitis is in people aged 15-25 years and in
those aged 55-65 years. It is less common in smokers.
The initial presentation is usually following insidious and intermittent symptoms. Features
include:
bloody diarrhoea
urgency
tenesmus
abdominal pain, particularly in the left lower quadrant
extra-intestinal features (see below)

21
22
23
24
25-27 2 / 3
28-30 0 / 3
31
32-34 0 / 3
35
36
37

Questions regarding the 'extra-intestinal' features of inflammatory bowel disease are


common. Extra-intestinal features include sclerosing cholangitis, iritis and ankylosing
spondylitis.

38
39
40

Common to both Crohn's disease


(CD) and Ulcerative colitis (UC)

Notes

Related to
disease
activity

Arthritis: pauciarticular, asymmetric


Erythema nodosum
Episcleritis
Osteoporosis

Arthritis is the most common extraintestinal feature in both CD and UC


Episcleritis is more common in
Crohns disease

Unrelated to
disease
activity

Arthritis: polyarticular, symmetric


Uveitis
Pyoderma gangrenosum
Clubbing
Primary sclerosing cholangitis

Primary sclerosing cholangitis is


much more common in UC
Uveitis is more common in UC

41
42
43
44
45
46
47-49 2 / 3
50
51
52

Pathology

53
54

Red, raw mucosa, bleeds easily


No inflammation beyond submucosa (unless fulminant disease)
Widespread superficial ulceration with preservation of adjacent mucosa which has the
appearance of polyps ('pseudopolyps')
Inflammatory cell infiltrate in lamina propria
Neutrophils migrate through the walls of glands to form crypt abscesses
Depletion of goblet cells and mucin from gland epithelium
Granulomas are infrequent

55-57 1 / 3
58
59
60
61
62-64 0 / 3
65

Barium enema

66
67

Loss of haustrations
Superficial ulceration, 'pseudopolyps'

http://www.emrcs.com/question/question.php?q=0

68

1/2

13/07/2015

Long standing disease: colon is narrow and short -'drainpipe colon'

69
70

Endoscopy

71
72-74 1 / 3

Superficial inflammation of the colonic and rectal mucosa


Continuous disease from rectum proximally
Superficial ulceration, mucosal islands, loss of vascular definition and continuous
ulceration pattern.

75
76
77
78-80 0 / 3

Management

81
82

Patients with long term disease are at increased risk of development of malignancy
Acute exacerbations are generally managed with steroids, in chronic patients agents
such as azathioprine and infliximab may be used
Individuals with medically unresponsive disease usually require surgery- in the acute
phase a sub total colectomy and end ileostomy. In the longer term a proctectomy will
be required. An ileoanal pouch is an option for selected patients

83
84
85
86

References
Ford A et al. Ulcerative colitis. BMJ 2013 (346):29-34.
Rate question:

Next question

Comment on this question

All contents of this site are 2012 E-Medical Revision Ltd

http://www.emrcs.com/question/question.php?q=0

Terms and Conditions

Privacy policy

2/2

13/07/2015

Reference ranges

Previous

Question 87 of 284

Next

Question stats

End and review

Score: 37%
1

38.3%

14.1%

21.5%

A. Human herpes virus 8

16.3%

B. Human papillomavirus 16

9.7%

C. Human T-lymphotropic virus 1

38.3% of users answered this


question correctly

Which virus is associated with Kaposi's sarcoma?

D. Epstein-Barr virus
E. Human papillomavirus 18

6
7
8
9

Search
Next question
Go

10
11
12

Oncoviruses

13
14
15

Viruses which cause cancer


These may be detected on blood test and prevented by vaccine

16
17

These are the main types of oncoviruses and their diseases:

18
19

Oncovirus

Cancer

Epstein-Barr virus

Burkitt's lymphoma
Hodgkin's lymphoma
Post transplant lymphoma
Nasopharyngeal carcinoma

Human papillomavirus 16/18

20
21
22
23
24

Cervical cancer
Anal cancer
Penile cancer
Vulval cancer
Oropharyneal cancer

25-27 2 / 3
28-30 0 / 3
31
32-34 0 / 3

Human herpes virus 8

Kaposi's sarcoma

Hepatitis B virus

Hepatocellular carcinoma

Hepatitis C virus

Hepatocellular carcinoma

38

Human T-lymphotropic virus 1

Tropical spastic paraparesis


Adult T cell leukaemia

39

35
36
37

40
41
42

Rate question:
Comment on this question

Next question

43
44
45
46
47-49 2 / 3
50
51
52
53
54
55-57 1 / 3
58
59
60
61
62-64 0 / 3
65
66
67
68

http://www.emrcs.com/question/question.php?q=0

1/2

13/07/2015
69
70
71
72-74 1 / 3
75
76
77
78-80 0 / 3
81
82
83
84
85
86
87

All contents of this site are 2012 E-Medical Revision Ltd

http://www.emrcs.com/question/question.php?q=0

Terms and Conditions

Privacy policy

2/2

13/07/2015

Reference ranges

Previous

Question 88 of 284

Next

Question stats

End and review

Score: 36.6%
1

10.6%

20.1%

39.9%

A. May result from an axonotmesis

13.8%

B. May occur in either the central or peripheral nervous systems

15.6%

C. The axon remains excitable throughout the whole process

39.9% of users answered this


question correctly

Which of the following is not a feature of Wallerian Degeneration?

D. The distal neuronal stump is affected


E. Is a component of the healing process following neuronal injury

6
7
8
9

Search
Next question
Go

10
11

The axon loses its excitability once the process is established.

12

Wallerian degeneration

13
14

- Is the process that occurs when a nerve is cut or crushed.


- It occurs when the part of the axon separated from the neuron's cell nucleus degenerates.
- It usually begins 24 hours following neuronal injury and the distal axon remains excitable up
until this time.
- The degeneration of the axon is following by breakdown of the myelin sheath, a process
that occurs by infiltration of the site with macrophages.
- Eventually regeneration of the nerve may occur although recovery will depend on the
extent and manner of injury

15
16
17
18
19
20
21

Rate question:
Comment on this question

Next question

22
23
24
25-27 2 / 3
28-30 0 / 3
31
32-34 0 / 3
35
36
37
38
39
40
41
42
43
44
45
46
47-49 2 / 3
50
51
52
53
54
55-57 1 / 3
58
59
60
61
62-64 0 / 3
65
66
67
68

http://www.emrcs.com/question/question.php?q=0

1/2

13/07/2015
69
70
71
72-74 1 / 3
75
76
77
78-80 0 / 3
81
82
83
84
85
86
87
88

All contents of this site are 2012 E-Medical Revision Ltd

http://www.emrcs.com/question/question.php?q=0

Terms and Conditions

Privacy policy

2/2

13/07/2015

Reference ranges

Question 1 of 196

Next

A 45 year old woman complains of painful tingling in her fingers. The pain is relieved by
hanging the arm over the side of the bed. She has a positive Tinel's sign at the wrist. Which
of the following is most likely to contribute to her diagnosis?

Question stats

End and review

Score: 0%

11.3%

7.8%

21.6%

7.6%

51.7%

A. Methotrexate use
51.7% of users answered this
question correctly

B. Crohn's disease
C. Hyperthyroidism
D. Tuberculosis

Search

E. Rheumatoid arthritis

Go

Next question

This woman has a diagnosis of carpal tunnel syndrome. Rheumatological disorders are a
common cause. Clinical examination should focus on identifying stigmata of rheumatoid
arthritis, such as rheumatoid nodules, vasculitic lesions and metacarpophalangeal joint
arthritis.
Carpal tunnel syndome
Carpal tunnel syndrome is caused by compression of median nerve in the carpal tunnel
History
pain/pins and needles in thumb, index, middle finger e.g. at night
patient flicks hand to obtain relief

Examination
weakness of thumb abduction
wasting of thenar eminence (NOT hypothenar)
Tinel's sign: tapping causes paraesthesia
Phalen's sign: flexion of wrist causes symptoms

Causes of carpal tunnel syndrome


MEDIAN TRAP Mnemonic

Myxoedema
Edema premenstrually
Diabetes
Idiopathic
Acromegaly
Neoplasm
Trauma
Rheumatoid arthritis
Amyloidosis
Pregnancy

Management
Non surgical
treatment

Surgery

May resolve
spontaneously
Avoid precipitants
and reassurance
Night-time splints
Local steroid
injections

Complete division of the flexor retinaculum and decompression of the


tunnel (successful in approximately 80% of patients)

Rate question:

Next question

Comment on this question

http://www.emrcs.com/question/question.php?q=0

1/2

13/07/2015

All contents of this site are 2012 E-Medical Revision Ltd

http://www.emrcs.com/question/question.php?q=0

Terms and Conditions

Privacy policy

2/2

13/07/2015

Reference ranges

Previous

Question 2 of 196

Next

A 30 year old male presents with a painless swelling of the testis. Histologically the stroma
has a lymphocytic infiltrate. The most likely diagnosis is :

A. Differentiated teratoma
B. Malignant undifferentiated teratoma

Question stats

End and review

Score: 0%

15.2%

12.5%

44%

15.4%

13%

1
2

44% of users answered this


question correctly

C. Classical seminoma
D. Spermatocytic seminoma
E. Anaplastic seminoma

Search
Next question

Go

Seminoma is the commonest type of testicular tumour and is more common in males aged
between 30-40 years. Classical seminoma is the commonest subtype and histology shows
lymphocytic stromal infiltrate. Other subtypes include:
1. Spermatocytic: tumour cells resemble spermatocytes. Excellent prognosis.
2. Anaplastic
3. Syncytiotrophoblast giant cells: HCG present in cells
A teratoma is more common in males aged 20-30 years.
Testicular disorders
Testicular cancer
Testicular cancer is the most common malignancy in men aged 20-30 years. Around 95% of
cases of testicular cancer are germ-cell tumours. Germ cell tumours may essentially be
divided into:
Tumour type

Seminoma

Non seminomatous
germ cell tumours
(42%)
Teratoma
Yolk sac tumour
Choriocarcinoma
Mixed germ cell
tumours (10%)

Key features

Commonest
subtype
(50%)
Average
age at
diagnosis =
40
Even
advanced
disease
associated
with 5 year
survival of
73%
Younger age at
presentation =2030 years
Advanced
disease carries
worse prognosis
(48% at 5 years)
Retroperitoneal
lymph node
dissection may be
needed for residual
disease after
chemotherapy

Tumour
markers

Pathology

AFP usually
normal
HCG elevated
in 10%
seminomas
Lactate
dehydrogenase;
elevated in 1020%
seminomas (but
also in many
other
conditions)

Sheet like lobular patterns


of cells with substantial
fibrous component. Fibrous
septa contain lymphocytic
inclusions and granulomas
may be seen.

AFP elevated
in up to 70% of
cases
HCG elevated
in up to 40% of
cases
Other
markers rarely
helpful

Heterogenous texture with


occasional ectopic tissue
such as hair

Image demonstrating a classical seminoma, these tumours are typically more uniform than
teratomas

http://www.emrcs.com/question/question.php?q=0

1/3

13/07/2015

Image sourced from Wikipedia

Risk factors for testicular cancer


Cryptorchidism
Infertility
Family history
Klinefelter's syndrome
Mumps orchitis

Features
A painless lump is the most common presenting symptom
Pain may also be present in a minority of men
Other possible features include hydrocele, gynaecomastia

Diagnosis
Ultrasound is first-line
CT scanning of the chest/ abdomen and pelvis is used for staging
Tumour markers (see above) should be measured

Management
Orchidectomy (Inguinal approach)
Chemotherapy and radiotherapy may be given depending on staging
Abdominal lesions >1cm following chemotherapy may require retroperitoneal lymph
node dissection.

Prognosis is generally excellent


5 year survival for seminomas is around 95% if Stage I
5 year survival for teratomas is around 85% if Stage I

Benign disease
Epididymo-orchitis
Acute epididymitis is an acute inflammation of the epididymis, often involving the testis and
usually caused by bacterial infection.
Infection spreads from the urethra or bladder. In men <35 years, gonorrhoea or
chlamydia are the usual infections.
Amiodarone is a recognised non infective cause of epididymitis, which resolves on
stopping the drug.
Tenderness is usually confined to the epididymis, which may facilitate differentiating it
from torsion where pain usually affects the entire testis.

Testicular torsion

http://www.emrcs.com/question/question.php?q=0

2/3

13/07/2015

Twist of the spermatic cord resulting in testicular ischaemia and necrosis.


Most common in males aged between 10 and 30 (peak incidence 13-15 years)
Pain is usually severe and of sudden onset.
Cremasteric reflex is lost and elevation of the testis does not ease the pain.
Treatment is with surgical exploration. If a torted testis is identified then both testis
should be fixed as the condition of bell clapper testis is often bilateral.

Hydrocele
Presents as a mass that transilluminates, usually possible to "get above" it on
examination.
In younger men it should be investigated with USS to exclude tumour.
In children it may occur as a result of a patent processus vaginalis.
Treatment in adults is with a Lords or Jabouley procedure.
Treatment in children is with trans inguinal ligation of PPV.

Rate question:

Next question

Comment on this question

All contents of this site are 2012 E-Medical Revision Ltd

http://www.emrcs.com/question/question.php?q=0

Terms and Conditions

Privacy policy

3/3

13/07/2015

Reference ranges

Question 3 of 196

Previous

Next

A 48 year old women presents with recurrent loin pain and fevers. Investigation reveals a
staghorn calculus of the left kidney. Infection with which of the following organisms is most
likely?

Question stats

End and review

Score: 0%
1

8.4%

52.2%

9.9%

21.5%

8.1%

4 5-7 -

A. Staphylococcus saprophyticus
52.2% of users answered this
question correctly

B. Proteus mirabilis
C. Klebsiella
D. E-Coli

Search

E. Staphylococcus epidermidis

Go

Next question

Theme from April 2012 Exam


Infection with Proteus mirabilis accounts for 90% of all proteus infections. It has a urease
producing enzyme. This will tend to favor urinary alkalinisation which is a relative perrequisite for the formation of staghorn calculi.
Renal stones

Type of
stones

Features

Percentage
of all
calculi

Calcium
oxalate

Hypercalciuria is a major risk factor (various causes)


Hyperoxaluria may also increase risk
Hypocitraturia increases risk because citrate forms complexes with
calcium making it more soluble
Stones are radio-opaque (though less than calcium phosphate
stones)
Hyperuricosuria may cause uric acid stones to which calcium
oxalate binds

85%

Cystine

Inherited recessive disorder of transmembrane cystine transport


leading to decreased absorption of cystine from intestine and renal
tubule
Multiple stones may form
Relatively radiodense because they contain sulphur

1%

Uric acid

Uric acid is a product of purine metabolism


May precipitate when urinary pH low
May be caused by diseases with extensive tissue breakdown e.g.
malignancy
More common in children with inborn errors of metabolism
Radiolucent

5-10%

Calcium
phosphate

May occur in renal tubular acidosis, high urinary pH increases


supersaturation of urine with calcium and phosphate
Renal tubular acidosis types 1 and 3 increase risk of stone
formation (types 2 and 4 do not)
Radio-opaque stones (composition similar to bone)

10%

Struvite

Stones formed from magnesium, ammonium and phosphate


Occur as a result of urease producing bacteria (and are thus
associated with chronic infections)
Under the alkaline conditions produced, the crystals can
precipitate
Slightly radio-opaque

2-20%

Effect of urinary pH on stone formation


Urine pH will show individual variation (from pH 5-7). Post prandially the pH falls as purine
metabolism will produce uric acid. Then the urine becomes more alkaline (alkaline tide).
When the stone is not available for analysis the pH of urine may help to determine which
stone was present.
Stone type

Urine acidity

Mean urine pH

Calcium phosphate

Normal- alkaline

>5.5

Calcium oxalate

Variable

Uric acid

Acid

5.5

Struvate

Alkaline

>7.2

http://www.emrcs.com/question/question.php?q=0

1/2

13/07/2015

Struvate

Alkaline

>7.2

Cystine

Normal

6.5

Rate question:

Next question

Comment on this question

All contents of this site are 2012 E-Medical Revision Ltd

http://www.emrcs.com/question/question.php?q=0

Terms and Conditions

Privacy policy

2/2

13/07/2015

Reference ranges

Previous

Question 4 of 196

Next

Question stats

End and review

Score: 0%
1

33%

30.1%

14.7%

A. Amiodarone

13.5%

B. Allopurinol

8.7%

C. Sulphonamides

33% of users answered this


question correctly

Causes of granulomatous disease do not include:

D. Beryllium

4
5-7 -

E. Wegener's granulomatosis
Search
Next question
Go

Allopurinol and sulphonamides cause hepatic granulomas.


Chronic inflammation
Overview
Chronic inflammation may occur secondary to acute inflammation.In most cases chronic
inflammation occurs as a primary process. These may be broadly viewed as being one of
three main processes:
Persisting infection with certain organisms such as Mycobacterium tuberculosis which
results in delayed type hypersensitivity reactions and inflammation.
Prolonged exposure to non-biodegradable substances such as silica or suture
materials which may induce an inflammatory response.
Autoimmune conditions involving antibodies formed against host antigens.

Acute vs. Chronic inflammation


Acute inflammation

Chronic inflammation

Changes to existing vascular structure and increased


permeability of endothelial cells

Angiogenesis predominates

Infiltration of neutrophils

Macrophages, plasma cells and


lymphocytes predominate

Process may resolve with:

Healing by fibrosis is the main result

Suppuration
Complete resolution
Abscess formation
Progression to chronic inflammation
Healing by fibrosis

Granulomatous inflammation
A granuloma consists of a microscopic aggregation of macrophages (with epithelial type
arrangement =epitheliod). Large giant cells may be found at the periphery of granulomas.
Mediators
Growth factors released by activated macrophages include agents such as interferon and
fibroblast growth factor (plus many more). Some of these such as interferons may have
systemic features resulting in systemic symptoms and signs, which may be present in
individuals with long standing chronic inflammation.
The finding of granulomas is pathognomonic of chronic inflammation, as illustrated in this
biopsy from a patient with colonic Crohns disease

Image sourced from Wikipedia

http://www.emrcs.com/question/question.php?q=0

1/2

13/07/2015

Rate question:

Next question

Comment on this question

All contents of this site are 2012 E-Medical Revision Ltd

http://www.emrcs.com/question/question.php?q=0

Terms and Conditions

Privacy policy

2/2

13/07/2015

Reference ranges

Previous1 / 3

Question 5-7 of 196

Next

Question stats

End and review

Score: 14.3%

Average score for registered users:

1
2

Theme: Facial nerve palsy


A. Adenoid cystic carcinoma

57.8%

87.4%

91.7%

B. Cerebrovascular accident

3
4
5-7 1 / 3

C. Petrous temporal fracture


D. Warthins tumour

Search

E. Sarcoidosis

Go

F. Pleomorphic adenoma
G. Cholesteatoma
Please select the most likely cause of facial nerve palsy for the scenario given. Each option
may be used once, more than once or not at all.

5.

A 22 year old man presents with symptoms of lethargy and bilateral facial nerve
palsy. On examination he has bilateral parotid gland enlargement.
You answered Cerebrovascular accident
The correct answer is Sarcoidosis
Theme from 2011 Exam
Facial nerve palsy is the commonest neurological manifestation of sarcoid. It
usually resolves. The absence of ear discharge or discrete lesion on palpation is
against the other causes.

6.

A 21 year old man presents with a unilateral facial nerve palsy after being hit in
the head. On examination he has a right sided facial nerve palsy and a watery
discharge from his nose.
Petrous temporal fracture
Nasal discharge of clear fluid and recent head injury makes a basal skull
fracture the most likely underlying diagnosis.

7.

A 43 year old lady presents with symptoms of chronic ear discharge and a right
sided facial nerve palsy. On examination she has foul smelling fluid draining from
her right ear and a complete right sided facial nerve palsy.
You answered Sarcoidosis
The correct answer is Cholesteatoma
Foul smelling ear discharge and facial nerve weakness is likely to be due to
cholesteatoma. The presence of a neurological deficit is a sinister feature.

Next question

Facial nerve palsy

Sarcoid

Facial nerve palsy is the most frequent neurological manifestation of sarcoid


Affects right and left side with equal frequency, may be bilateral
Typically resolves in up to 80% of cases

Cholesteatoma

Destructive and expanding growth of keratinised squamous epithelium


Patients often complain of chronic ear discharge
Infection with Pseudomonas may occur resulting in foul smell to discharge
Aquired lesions usually arise from the Pars flaccida region of the tympanic
membrane
Surgical removal and mastoidectomy may be needed
Recurrence rates of 20% may be seen following surgery

Basal skull
fracture

History of head injury


Presence of features such as Battles sign on examination
Clinical presence of CSF leak strongly supports diagnosis
Assessment is by CT and MRI scan
Prophylactic antibiotics should be given in cases of CSF leak

http://www.emrcs.com/question/question.php?q=0

1/2

13/07/2015

Rate question:

Next question

Comment on this question

All contents of this site are 2012 E-Medical Revision Ltd

http://www.emrcs.com/question/question.php?q=0

Terms and Conditions

Privacy policy

2/2

13/07/2015

Reference ranges

Previous

Question 8 of 196

Next

A 59 year old lady is referred from the NHS breast screening program. A recent mammogram
is reported as showing linear, branching microcalcification with coarse granules. Which
disease process is the most likely underlying cause of these appearances?

Question stats

Score: 12.5%
1

16.9%

16.1%

23.5%

31.5%

12.1%

A. Invasive lobular cancer


31.5% of users answered this
question correctly

B. Lobular carcinoma in situ

End and review

4
5-7 1 / 3
8

C. Cribriform type ductal carcinoma in situ


D. Comedo type ductal carcinoma in situ
Search

E. Fibroadenosis

Go

Next question

Comedo type DCIS is usually associated with microcalcifications. Cribriform lesions are
usually multifocal but less likely to form microcalcifications. Lobular cancers and in situ
lesions rarely form microcalcifications and are difficult to detect using mammography.
Breast cancer - In situ disease
Breast cancer that has yet to invade the basement membrane is referred to as in situ
disease. Both ductal and lobular in situ variants are recognised.
Ductal carcinoma in situ
Sub types include; comedo, cribriform, micropapillary and solid
Comdeo DCIS is most likely to form microcalcifications
Cribriform and micropapillary are most likely to be multifocal
Most lesions are mixed (composed of multiple subtypes)
High nuclear grade DCIS is associated with more malignant characteristics (loss of
p53, increased erbB2 expression)
Local excision of low nuclear grade DCIS will usually produce satisfactory outcomes.
Multifocal lesions, large and high nuclear grade lesions will usually require mastectomy
Whole breast irradiation improves locoregional control when breast conserving
surgery is performed

Lobular carcinoma in situ


Much rarer than DCIS
Does not form microcalcifications
Usually single growth pattern
When an invasive component is found it is less likely to be associated with axillary
nodal metastasis than with DCIS
Low grade LCIS is usually treated by monitoring rather than excision

Rate question:

Next question

Comment on this question

All contents of this site are 2012 E-Medical Revision Ltd

http://www.emrcs.com/question/question.php?q=0

Terms and Conditions

Privacy policy

1/1

13/07/2015

Reference ranges

Previous

Question 9 of 196

Next

Question stats

End and review

Score: 11.1%
1

10.4%

57.4%

8.8%

A. The first part of the duodenum

11%

B. The second part of the duodenum

12.4%

C. The fourth part of the duodenum

57.4% of users answered this


question correctly

In patients with an annular pancreas where is the most likely site of obstruction?

D. The third part of the duodenum

4
5-7 1 / 3
8
9

E. The duodeno-jejunal flexure


Search
Next question
Go

Theme from September 2014 Exam


The pancreas develops from two foregut outgrowths (ventral and dorsal). During rotation the
ventral bud and adjacent gallbladder and bile duct lie together and fuse. When the pancreas
fails to rotate normally it can compress the duodenum with development of obstruction.
Usually occurring as a result of associated duodenal malformation. The second part of the
duodenum is the commonest site.
Pancreas
The pancreas is a retroperitoneal organ and lies posterior to the stomach. It may be
accessed surgically by dividing the peritoneal reflection that connects the greater omentum
to the transverse colon. The pancreatic head sits in the curvature of the duodenum. Its tail
lies close to the hilum of the spleen, a site of potential injury during splenectomy.
Relations
Posterior to the pancreas
Pancreatic head

Inferior vena cava


Common bile duct
Right and left renal veins
Superior mesenteric vein and artery

Pancreatic neck

Superior mesenteric vein, portal vein

Pancreatic body-

Left renal vein


Crus of diaphragm
Psoas muscle
Adrenal gland
Kidney
Aorta

Pancreatic tail

Left kidney

Anterior to the pancreas


Pancreatic head

1st part of the duodenum


Pylorus
Gastroduodenal artery
SMA and SMV(uncinate process)

Pancreatic body

Stomach
Duodenojejunal flexure

Pancreatic tail

Splenic hilum

Superior to the pancreas


Coeliac trunk and its branches common hepatic artery and splenic artery
Grooves of the head of the pancreas
2nd and 3rd part of the duodenum
Arterial supply
Head: pancreaticoduodenal artery
Rest: splenic artery

Venous drainage
Head: superior mesenteric vein
Body and tail: splenic vein

http://www.emrcs.com/question/question.php?q=0

1/2

13/07/2015

Ampulla of Vater
Merge of pancreatic duct and common bile duct
Is an important landmark, halfway along the second part of the duodenum, that marks
the anatomical transition from foregut to midgut (also the site of transition between
regions supplied by coeliac trunk and SMA).

Image sourced from Wikipedia

Rate question:

Next question

Comment on this question

All contents of this site are 2012 E-Medical Revision Ltd

http://www.emrcs.com/question/question.php?q=0

Terms and Conditions

Privacy policy

2/2

13/07/2015

Reference ranges

Previous2 / 3

Question 10-12 of 196

Next

Question stats

End and review

Score: 25%

Average score for registered users:

1
2

Theme: Chest pain


A. Achalasia

10

86.1%

11

69%

12

84.2%

B. Pulmonary embolus

3
4
5-7 1 / 3

C. Dissection of thoracic aorta

D. Boerhaaves syndrome

Search

E. Gastro-oesophageal reflux

Go

10-12 2 / 3

F. Carcinoma of the oesophagus


G. Oesophageal candidiasis
Please select the most likely cause for chest pain for the scenario given. Each option may be
used once, more than once or not at all.

10.

A 43 year old man who has a long term history of alcohol misuse is admitted
with a history of an attack of vomiting after an episode of binge drinking. After
vomiting he developed sudden onset left sided chest pain, which is pleuritic in
nature. On examination he is profoundly septic and drowsy with severe
epigastric tenderness and left sided chest pain.
Boerhaaves syndrome
In patients with Boerhaaves the rupture is often on the left side. The story here
is typical. All patients should have a contrast study to confirm the diagnosis and
the affected site prior to thoracotomy.

11.

A 22 year old man is admitted with severe retrosternal chest pain and recurrent
episodes of dysphagia. These occur sporadically and often resolve
spontaneously. On examination there are no physical abnormalities and the
patient seems well.
Achalasia
Achalasia may produce severe chest pain and many older patients may
undergo cardiac investigations prior to endoscopy.
Endoscopic injection with botulinum toxin is a popular treatment (although the
benefit is not long lasting). Cardiomyotomy together with an antireflux
procedure is a more durable alternative.

12.

An obese 53 year old man presents with symptoms of recurrent retrosternal


discomfort and dyspepsia. This is typically worse at night after eating a large
meal. On examination there is no physical abnormality to find.
You answered Oesophageal candidiasis
The correct answer is Gastro-oesophageal reflux
Patients with GORD often have symptoms that are worse at night. In this age
group an Upper GI endoscopy should probably be performed.

Next question

Surgical chest pain

Dissection of
thoracic
aorta

Diffuse
oesophageal
spasm

Gastrooesphageal
reflux

Tearing interscapular pain


Discrepancy in arterial blood pressures taken in both arms
May show mediastinal widening on chest x-ray

Spectrum of oesophageal motility disorders


Caused by uncoordinated contractions of oesphageal muscles
May show "nutcracker oesophagus" on barium swallow
Symptoms include dysphagia, retrosternal discomfort and dyspepsia

Common cause of retrosternal discomfort


Usually associated with symptoms of regurgitation, odynophagia and

http://www.emrcs.com/question/question.php?q=0

1/2

13/07/2015

reflux

Boerhaaves
syndrome

Achalasia

dyspepsia
Symptoms usually well controlled with PPI therapy
Risk factors include obesity, smoking and excess alcohol consumption

Spontaneous rupture of the oesophagus


Caused by episodes of repeated vomiting often in association with
alcohol excess
Typically there is an episode of repetitive vomiting followed by severe
chest and epigastric pain
Diagnosis is by CT and contrast studies
Treatment is surgical; during first 12 hours primary repair, beyond this
usually creation of controlled fistula with a T Tube, delay beyond 24
hours is associated with fulminent mediastinitis and is usually fatal.

Difficulty swallowing, dysphagia to both liquids and solids and


sometimes chest pain
Usually caused by failure of distal oesphageal inhibitory neurones
Diagnosis is by pH and manometry studies together with contrast
swallow and endoscopy
Treatment is with either botulinum toxin, pneumatic dilatation or
cardiomyotomy

Rate question:

Next question

Comment on this question

All contents of this site are 2012 E-Medical Revision Ltd

http://www.emrcs.com/question/question.php?q=0

Terms and Conditions

Privacy policy

2/2

13/07/2015

Reference ranges

Previous

Question 13 of 196

Next

A 32 year old woman presents with an episode of haemoptysis and is found to have
metastatic tumour present within the parenchyma of the lungs. This is biopsied and
subsequent histology shows clear cells. What is the most likely primary site?

Question stats

Score: 31.3%
1

44.1%

23.5%

7.3%

12.4%

12.6%

A. Kidney
44.1% of users answered this
question correctly

B. Breast

End and review

C. Liver

4
5-7 1 / 3
8
9
10-12 2 / 3

D. Adrenal

13
Search

E. Bone

Go

Next question

Clear cell tumours are a sub type of renal cell cancer it is associated with specific genetic
changes localised to chromosome 3.
Renal lesions

Lesion
Renal cell
carcinoma

Nephroblastoma

Neuroblastoma

Transitional cell
carcinoma

Angiomyolipoma

Disease specific features


Most present with haematuria
(50%)
Common renal tumour (85%
cases)
Paraneoplastic features include
hypertension and polycythaemia
Most commonly has
haematogenous mestastasis

Rare childhood tumour


It accounts for 80% of all
genitourinary malignancies in
those under the age of 15 years
Up to 90% will have a mass
50% will be hypertensive
Diagnostic work up includes
ultrasound and CT scanning

Most common extracranial


tumour of childhood
80% occur in those under 4
years of age
Tumour of neural crest origin (up
to 50% occur in the adrenal
gland)
The tumour is usually calcified
and may be diagnosed using
MIBG scanning
Staging is with CT

Accounts for 90% of lower urinary


tract tumour, but only 10% of
renal tumours
Males affected 3x more than
females
Occupational exposure to
industrial dyes and rubber
chemicals may increase risk
Up to 80% present with painless
haematuria
Diagnosis and staging is with CT
IVU

80% of these hamartoma type


lesions occur sporadically, the
remainder are seen in those with
tuberous sclerosis
Tumour is composed of blood
vessels, smooth muscle and fat
Massive bleeding may occur in
10% of cases

http://www.emrcs.com/question/question.php?q=0

Treatment
Usually radical or partial
nephrectomy

Surgical resection combined with


chemotherapy (usually
vincristine, actinomycin D and
doxorubicin)

Surgical resection, radiotherapy


and chemotherapy

Radical nephroureterectomy

50% of patients with lesions


>4cm will have symptoms and
will require surgical resection

1/2

13/07/2015

Rate question:

Next question

Comment on this question

All contents of this site are 2012 E-Medical Revision Ltd

http://www.emrcs.com/question/question.php?q=0

Terms and Conditions

Privacy policy

2/2

13/07/2015

Reference ranges

Previous

Question 14 of 196

Next

A laceration of the wrist produces a median nerve transection. The wound is clean and seen
immediately after injury. Collateral soft tissue damage is absent. The patient asks what the
prognosis is. You indicate that the nerve should regrow at approximately:

Question stats

Score: 35.3%
1

31.6%

44.4%

6.4%

7.2%

10.4%

A. 0.1 mm per day


44.4% of users answered this
question correctly

B. 1 mm per day

End and review

C. 5 mm per day

4
5-7 1 / 3
8
9
10-12 2 / 3

D. 1 cm per day

13
Search

E. None of the above

14
Go

Next question

Transaction of a peripheral nerve results in hemorrhage and in retraction of the several


nerve ends. Almost immediately, degeneration of the axon distal to the injury begins.
Degeneration also occurs in the proximal fragment back to the first node of Ranvier.
Phagocytosis of the degenerated axonal fragments leaves neurilemmal sheath with empty
cylindrical spaces where the axons were. Several days following the injury, axons from the
proximal fragment begin to regrow. If they make contact with the distal neurilemmal sheath,
regrowth occurs at about the rate of 1 mm/day. However, if associated trauma, fracture,
infection, or separation of neurilemmal sheath ends precludes contact between axons,
growth is haphazard and a traumatic neuroma is formed. When neural transaction is
associated with widespread soft tissue damage and hemorrhage (with increased probability
of infection), many surgeons choose to delay reapproximation of the severed nerve end for 3
to 4 weeks.
Nerve injury
There are 3 types of nerve injury:
Neuropraxia

Axonotmesis

Neurotmesis

Nerve intact but electrical conduction is affected


Full recovery
Autonomic function preserved
Wallerian degeneration does not occur

Axon is damaged and the myelin sheath is preserved. The connective


tissue framework is not affected.
Wallerian degeneration occurs.

Disruption of the axon, myelin sheath and surrounding connective tissue.


Wallerian degeneration occurs.

Wallerian Degeneration
Axonal degeneration distal to the site of injury.
Typically begins 24-36 hours following injury.
Axons are excitable prior to degeneration occurring.
Myelin sheath degenerates and is phagocytosed by tissue macrophages.

Nerve repair
Neuronal repair may only occur physiologically where nerves are in direct contact.
Where a large defect is present, the process of nerve regeneration is hampered. It
may not occur at all or result in the formation of a neuroma. Where nerve regrowth
occurs it is typically at a rate of 1mm per day.

Rate question:

Next question

Comment on this question

All contents of this site are 2012 E-Medical Revision Ltd

http://www.emrcs.com/question/question.php?q=0

Terms and Conditions

Privacy policy

1/1

13/07/2015

Reference ranges

Question 15 of 196

Previous

Next

Question stats

End and review

Score: 33.3%
1

10.3%

15%

44.8%

A. It is associated with chronic helicobacter pylori infection

9%

B. 5% of gastric malignancies are due to lymphoma

20.9%

C. In the Lauren classification the diffuse type of adenocarcinoma typically


presents as a large exophytic growth in the antrum

44.8% of users answered this


question correctly

Which of the following statements relating to gastric cancer is untrue?

D. Smoking is a risk factor

4
5-7 1 / 3
8
9
10-12 2 / 3
13

E. It is associated with acanthosis nigricans


Search

14
Next question

Go

15

Barium meal appearances of linitis plastica:


Due to the increased rigidity of the wall, the stomach cannot be adequately distended,
with only a narrow lumen identified. The normal mucosal fold pattern is absent, either
distorted, thickened or nodular.

The Lauren classification describes a diffuse type of adenocarcinoma (Linitis plastica type
lesion) and an intestinal type. The diffuse type is often deeply infiltrative and may be difficult
to detect on endoscopy. Barium meal appearances can be characteristic.
Gastric cancer
Overview
There are 700,000 new cases of gastric cancer worldwide each year. It is most common in
Japan and less common in western countries. It is more common in men and incidence rises
with increasing age. The exact cause of many sporadic cancer is not known, however,
familial cases do occur in HNPCC families. In addition, smoking and smoked or preserved
foods increase the risk. Japanese migrants retain their increased risk (decreased in
subsequent generations). The distribution of the disease in western countries is changing
towards a more proximal location (perhaps due to rising obesity).
Pathology
There is some evidence of support a stepwise progression of the disease through intestinal
metaplasia progressing to atrophic gastritis and subsequent dysplasia, through to cancer.
The favoured staging system is TNM. The risk of lymph node involvement is related to size
and depth of invasion; early cancers confined to submucosa have a 20% incidence of lymph
node metastasis. Tumours of the gastro-oesophageal junction are classified as below:
Type
1

True oesophageal cancers and may be associated with Barrett's oesophagus.

Type
2

Carcinoma of the cardia, arising from cardiac type epithelium


or short segments with intestinal metaplasia at the oesophagogastric junction.

Type
3

Sub cardial cancers that spread across the junction. Involve similar nodal stations to
gastric cancer.

Groups for close endoscopic monitoring


Intestinal metaplasia of columnar type
Atrophic gastritis
Low to medium grade dysplasia
Patients who have previously undergone resections for benign peptic ulcer disease
(except highly selective vagotomy).

Referral to endoscopy
Patients of any age with
dyspepsia and any of the
following

Patients without
dyspepsia

Worsening dyspepsia

Chronic gastrointestinal bleeding

Dysphagia

Barretts oesophagus

Dysphagia

Unexplained
abdominal pain or
weight loss

Intestinal metaplasia

http://www.emrcs.com/question/question.php?q=0

1/3

13/07/2015

Weight loss

Vomiting

Dysplasia

Iron deficiency anaemia

Upper abdominal
mass

Atrophic gastritis

Upper abdominal mass

Jaundice

Patient aged over 55 years with


unexplained or persistent
dyspepsia

Upper GI endoscopy performed for dyspepsia. The addition of dye spraying (as shown in the
bottom right) may facilitate identification of smaller tumours

Image sourced from Wikipedia

Staging
CT scanning of the chest abdomen and pelvis is the routine first line staging
investigation in most centres.
Laparoscopy to identify occult peritoneal disease
PET CT (particularly for junctional tumours)

Treatment
Proximally sited disease greater than 5-10cm from the OG junction may be treated by
sub total gastrectomy
Total gastrectomy if tumour is <5cm from OG junction
For type 2 junctional tumours (extending into oesophagus) oesophagogastrectomy is
usual
Endoscopic sub mucosal resection may play a role in early gastric cancer confined to
the mucosa and perhaps the sub mucosa (this is debated)
Lymphadenectomy should be performed. A D2 lymphadenectomy is widely advocated
by the Japanese, the survival advantages of extended lymphadenectomy have been
debated. However, the overall recommendation is that a D2 nodal dissection be
undertaken.
Most patients will receive chemotherapy either pre or post operatively.

Prognosis
UK Data
Disease extent

Percentage 5 year survival

All RO resections

54%

Early gastric cancer

91%

Stage 1

87%

Stage 2

65%

Stage 3

18%

Operative procedure
Total Gastrectomy , lymphadenectomy and Roux en Y anastomosis
General anaesthesia
Prophylactic intravenous antibiotics
Incision: Rooftop.

http://www.emrcs.com/question/question.php?q=0

2/3

13/07/2015

Perform a thorough laparotomy to identify any occult disease.


Mobilise the left lobe of the liver off the diaphragm and place a large pack over it. Insert a
large self retaining retractor e.g. omnitract or Balfour (take time with this, the set up should
be perfect). Pack the small bowel away.
Begin by mobilising the omentum off the transverse colon.
Proceed to detach the short gastric vessels.
Mobilise the pylorus and divide it at least 2cm distally using a linear cutter stapling device.
Continue the dissection into the lesser sac taking the lesser omentum and left gastric artery
flush at its origin.
The lymph nodes should be removed en bloc with the specimen where possible.
Place 2 stay sutures either side of the distal oesophagus. Ask the anaesthetist to pull back
on the nasogastric tube. Divide the distal oesophagus and remove the stomach.
The oesphago jejunal anastomosis should be constructed. Identify the DJ flexure and bring a
loop of jejunum up to the oesophagus (to check it will reach). Divide the jejunum at this point.
Bring the divided jejunum either retrocolic or antecolic to the oesophagus. Anastamose the
oesophagus to the jejunum, using either interrupted 3/0 vicryl or a stapling device. Then
create the remainder of the Roux en Y reconstruction distally.
Place a jejunostomy feeding tube.
Wash out the abdomen and insert drains (usually the anastomosis and duodenal stump).
Help the anaesthetist insert the nasogastric tube (carefully!)
Close the abdomen and skin.
Enteral feeding may commence on the first post-operative day. However, most surgeons will
leave patients on free NG drainage for several days and keep them nil by mouth.

Rate question:

Next question

Comment on this question

All contents of this site are 2012 E-Medical Revision Ltd

http://www.emrcs.com/question/question.php?q=0

Terms and Conditions

Privacy policy

3/3

13/07/2015

Reference ranges

Previous

Question 16 of 196

Next

Which of the following statements relating to Gardners syndrome variant of familial


adenomatous polyposis coli is false?

A. It is an autosomal dominant condition


B. Patients may develop retroperitoneal desmoid tumours

Question stats

Score: 36.8%
1

8.6%

8.9%

56.6%

14.6%

11.4%

56.6% of users answered this


question correctly

C. The vast majority of the polyps are benign and thus the risk of
colorectal cancer is small

End and review

4
5-7 1 / 3
8
9
10-12 2 / 3

D. Patients are at increased risk of thyroid cancer

13
Search

E. It is characterised by a mutation in the APC gene

14
Go

15

Next question

16

The multiple polyps increase the risk of malignancy and most patients should undergo a
colectomy.
Genetics and surgical disease
Some of the more commonly occurring genetic conditions occurring in surgical patients are
presented here.
Li-Fraumeni Syndrome
Autosomal dominant
Consists of germline mutations to p53 tumour suppressor gene
High incidence of malignancies particularly sarcomas and leukaemias
Diagnosed when:

*Individual develops sarcoma under 45 years


*First degree relative diagnosed with any cancer below age 45 years and another family
member develops malignancy under 45 years or sarcoma at any age
BRCA 1 and 2
Carried on chromosome 17 (BRCA 1) and Chromosome 13 (BRCA 2)
Linked to developing breast cancer (60%) risk.
Associated risk of developing ovarian cancer (55% with BRCA 1 and 25% with BRCA
2).

Lynch Syndrome
Autosomal dominant
Develop colonic cancer and endometrial cancer at young age
80% of affected individuals will get colonic and/ or endometrial cancer
High risk individuals may be identified using the Amsterdam criteria

Amsterdam criteria
Three or more family members with a confirmed diagnosis of colorectal cancer, one of whom
is a first degree (parent, child, sibling) relative of the other two.
Two successive affected generations.
One or more colon cancers diagnosed under age 50 years.
Familial adenomatous polyposis (FAP) has been excluded.
Gardners syndrome
Autosomal dominant familial colorectal polyposis
Multiple colonic polyps
Extra colonic diseases include: skull osteoma, thyroid cancer and epidermoid cysts
Desmoid tumours are seen in 15%
Mutation of APC gene located on chromosome 5
Due to colonic polyps most patients will undergo colectomy to reduce risk of colorectal
cancer
Now considered a variant of familial adenomatous polyposis coli

Rate question:

Next question

Comment on this question

http://www.emrcs.com/question/question.php?q=0

1/2

13/07/2015

All contents of this site are 2012 E-Medical Revision Ltd

http://www.emrcs.com/question/question.php?q=0

Terms and Conditions

Privacy policy

2/2

13/07/2015

Reference ranges

Previous

Question 17 of 196

Next

A 60-year-old man presents with lower urinary tract symptoms and is offered a PSA test.
Which one of the following could interfere with the PSA level?

A. Vigorous exercise in the past 48 hours


B. Poorly controlled diabetes mellitus

Question stats

Score: 35%
1

52.4%

10.3%

15.9%

11.4%

10%

52.4% of users answered this


question correctly

C. Drinking more than 4 units of alcohol in the past 48 hours

End and review

4
5-7 1 / 3
8
9
10-12 2 / 3

D. Smoking

13

E. Recent cholecystectomy

Search
14
Go

Next question

15
16
17

PSA testing
Prostate specific antigen (PSA) is a serine protease enzyme produced by normal and
malignant prostate epithelial cells. It has become an important tumour marker but much
controversy still exists regarding its usefulness as a screening tool.
The NHS Prostate Cancer Risk Management Programme (PCRMP) has published updated
guidelines in 2009 on how to handle requests for PSA testing in asymptomatic men. A recent
European trial (ERSPC) showed a statistically significant reduction in the rate of death
prostate cancer by 20% in men aged 55 to 69 years but this was associated with a high risk
of over-diagnosis and over-treatment. Having reviewed this and other data the National
Screening Committee have decided not to introduce a prostate cancer screening programme
yet but rather allow men to make an informed choice.
Age-adjusted upper limits for PSA were recommended by the PCRMP*:
{Age}

{PSA level (ng/ml)}

50-59 years

3.0

60-69 years

4.0

> 70 years

5.0

PSA levels may also be raised by**:


benign prostatic hyperplasia (BPH)
prostatitis and urinary tract infection (NICE recommend to postpone the PSA test for at
least 1 month after treatment)
ejaculation (ideally not in the previous 48 hours)
vigorous exercise (ideally not in the previous 48 hours)
urinary retention
instrumentation of the urinary tract

Poor specificity and sensitivity


around 33% of men with a PSA of 4-10 ng/ml will be found to have prostate cancer.
With a PSA of 10-20 ng/ml this rises to 60% of men
around 20% with prostate cancer have a normal PSA
various methods are used to try and add greater meaning to a PSA level including
age-adjusted upper limits and monitoring change in PSA level with time (PSA velocity
or PSA doubling time)

*aide memoire for upper PSA limit: (age - 20) / 10


**whether digital rectal examination actually causes a rise in PSA levels is a matter of debate
Rate question:

Next question

Comment on this question

All contents of this site are 2012 E-Medical Revision Ltd

http://www.emrcs.com/question/question.php?q=0

Terms and Conditions

Privacy policy

1/1

13/07/2015

Reference ranges

Previous

Question 18 of 196

Next

Question stats

End and review

Score: 33.3%
1

9.4%

37.5%

26.6%

A. Painless jaundice

7.8%

B. Hyperamylasaemia

18.8%

C. Hyperglycaemia

37.5% of users answered this


question correctly

Which of the symptoms below is least typical of pancreatic cancer?

D. Weight loss

4
5-7 1 / 3
8
9
10-12 2 / 3

E. Classical Courvoisier syndrome

13
Search
14

Next question
Go

Raised serum amylase is relatively uncommon. The typical Courvoisier syndrome typically
occurs in 20% and hyperglycaemia occurs in 15-20%.

15
16
17
18

Pancreatic cancer

Adenocarcinoma
Risk factors: Smoking, diabetes, Adenoma, Familial adenomatous polyposis
Mainly occur in the head of the pancreas (70%)
Spread locally and metastasizes to the liver
Carcinoma of the pancreas should be differentiated from other periampullary tumours
with better prognosis

Clinical features
Weight loss
Painless jaundice
Epigastric discomfort (pain usually due to invasion of the coeliac plexus is a late
feature)
Pancreatitis
Trousseau's sign: migratory superficial thrombophlebitis

Investigations
USS: May miss small lesions
CT Scanning (pancreatic protocol). If unresectable on CT then no further staging
needed
PET/CT for those with operable disease on CT alone
ERCP/ MRI for bile duct assessment
Staging laparoscopy to exclude peritoneal disease

Management
Head of pancreas: Whipple's resection (SE dumping and ulcers). Newer techniques
include pylorus preservation and SMA/ SMV resection
Carcinoma body and tail: poor prognosis, distal pancreatectomy, if operable
Usually adjuvent chemotherapy for resectable disease
ERCP and stent for jaundice and palliation
Surgical bypass may be needed for duodenal obstruction

Rate question:

Next question

Comment on this question

All contents of this site are 2012 E-Medical Revision Ltd

http://www.emrcs.com/question/question.php?q=0

Terms and Conditions

Privacy policy

1/1

13/07/2015

Reference ranges

Previous

Question 19 of 196

Next

A 53 year old man presents with dyspepsia. An upper GI endoscopy is performed and
Helicobacter pylori is identified. A duodenal ulcer is present in the first part of the duodenum.
Duodenal biopsies are taken and demonstrate epithelium that resembles cells of the gastric
antrum. Which of the following is the most likely explanation for this process?

Question stats

Score: 30.4%
1

9.7%

58.5%

16%

7%

8.7%

58.5% of users answered this


question correctly

A. Hyperplasia of the crypts of Lieberkhun

End and review

B. Duodenal metaplasia

4
5-7 1 / 3
8
9
10-12 2 / 3

C. Duodenal dysplasia

13
Search

D. Duodenal carcinoma

14

E. Hyptertrophy of Brunners glands

Go

15
16

Next question

17
18

Metaplasia = cell type conversion

19

Theme in January 2012 exam


The process involved is metaplasia. During metaplasia there is no direct carcinogenesis,
however the persistent presence of precipitants of metaplasia will lead to malignant changes
in cells.
Metaplastic changes in the duodenal cap are frequently seen in association with H-Pylori
induced ulcers. It typically resolves after ulcer healing and eradication therapy.
Metaplasia

Definition: reversible change of differentiated cells to another cell type.


May represent an adaptive substitution of cells that are sensitive to stress by cell types
better able to withstand the adverse environment.
Can be a normal physiological response (ossification of cartilage to form bone)
Most common epithelial metaplasia occurs with transformation of columnar cells to
squamous cells (smoking causes ciliated columnar cells to be replaced by squamous
epithelial cells; Schistosomiasis).
Metaplasia from squamous to columnar cells occurs in Barrett oesophagus.
If the metaplastic stimulus is removed, the cells will return to their original pattern of
differentiation. However, if the stimulus is not removed then progression to dysplasia
may occur.
Not considered directly carcinogenic, however the factors which predispose to
metaplasia, if persistent may induce malignant transformation.
The pathogenesis involves a reprogramming of stem cells that are known to exist in
normal tissues, or of undifferentiated mesenchymal cells present in connective tissue.
In a metaplastic change, these precursor cells differentiate along a new pathway.

Rate question:

Next question

Comment on this question

All contents of this site are 2012 E-Medical Revision Ltd

http://www.emrcs.com/question/question.php?q=0

Terms and Conditions

Privacy policy

1/1

13/07/2015

Reference ranges

Previous

Question 20 of 196

Next

A 22 year old lady presents with symptoms and signs of hyperthyroidism. Her diagnostic work
up results in a diagnosis of Graves disease. Which of the following best describes the
pathophysiology of the condition?

Question stats

Score: 33.3%
1

57.4%

8.6%

16.7%

9.8%

7.5%

A. Formation of IgG antibodies to the TSH receptors on the thyroid gland


57.4% of users answered this
question correctly

B. Formation of IgG antibodies to the TRH receptors on the anterior


pituitary

End and review

4
5-7 1 / 3
8
9
10-12 2 / 3

C. Formation of IgM antibodies to the TSH receptors on the thyroid gland

13
Search

D. Formation of IgA antibodies to the TSH receptors on the thyroid gland

14

E. Formation of IgM antibodies to the TRH receptors on the anterior


pituitary

Go

15
16

Next question

17
18

Usually IgG antibodies are formed against the TSH receptors on the thyroid gland. Which is
why the TSH level is often very low in Graves disease.

19
20

Thyroid hormones
Hormones of the thyroid gland
Triiodothyronine T3

Major hormone active in target cells

Thyroxine T4

Most prevalent form in plasma, less biologically active than T3

Calcitonin

Lowers plasma calcium

Synthesis and secretion of thyroid hormones


Thyroid actively concentrates iodide to twenty five times the plasma concentration.
Iodide is oxidised by peroxidase in the follicular cells to atomic iodine which then
iodinates tyrosine residues contained in thyroglobulin.
Iodinated tyrosine residues in thyroglobulin undergo coupling to either T3 or T4.
Process is stimulated by TSH, which stimulates secretion of thyroid hormones.
The normal thyroid has approximately 3 month reserves of thyroid hormones.

LATS and Graves disease


In Graves disease patients develop IgG antibodies to the TSH receptors on the thyroid
gland. This results in chronic and long term stimulation of the gland with release of thyroid
hormones. The typically situation is raised thyroid hormones and low TSH. Thyroid receptor
autoantibodies should be checked in individuals presenting with hyperthyroidism as they are
present in up to 85% cases.
Rate question:

Next question

Comment on this question

All contents of this site are 2012 E-Medical Revision Ltd

http://www.emrcs.com/question/question.php?q=0

Terms and Conditions

Privacy policy

1/1

13/07/2015

Reference ranges

Previous

Question 21 of 196

Next

Question stats

End and review

Score: 32%
1

25.1%

35.2%

22.9%

A. Thiazide use

9.2%

B. Metastatic malignancy

7.6%

C. Primary hyperparathyroidism

35.2% of users answered this


question correctly

What is the most common cause of hypercalcaemia in the UK in hospitalised patients?

D. Osteogenic sarcoma

4
5-7 1 / 3
8
9
10-12 2 / 3

E. Sarcoidosis

13
Search
14

Next question
Go

Metastatic cancer accounts for most cases of hypercalcaemia in hospitalised patients. In the
community primary hyperparathyroidism is the commonest cause.

15
16
17
18

Hypercalcaemia

19

Main causes

20

Malignancy (most common cause in hospital in-patients)


Primary hyperparathyroidism (commonest cause in non hospitalised patients)

21

Less common
Sarcoidosis (extrarenal synthesis of calcitriol )
Thiazides, lithium
Immobilisation
Pagets disease
Vitamin A/D toxicity
Thyrotoxicosis
MEN
Milk alkali syndrome

Clinical features
Stones, bones, abdominal groans, and psychic moans
High serum calcium levels result in decreased neuronal excitability. Therefore sluggish
reflexes, muscle weakness and constipation may occur.
Rate question:

Next question

Comment on this question

All contents of this site are 2012 E-Medical Revision Ltd

http://www.emrcs.com/question/question.php?q=0

Terms and Conditions

Privacy policy

1/1

13/07/2015

Reference ranges

Previous1 / 3

Question 22-24 of 196

Next

Question stats

End and review

Score: 32.3%

Average score for registered users:

1
2

Theme: Diseases affecting the great vessels


A. Aortic coarctation

22

58.3%

23

66.8%

24

56.1%

B. Cervical rib

3
4
5-7 1 / 3

C. Takayasu's arteritis

D. Subclavian steal syndrome

Search

E. Patent ductus arteriosus

Go

F. Aortic dissection

10-12 2 / 3
13
14

Please select the most likely underlying cause for the symptoms described. Each option may
be used once, more than once or not at all.

15
16
17

22.

A 24 year old lady from Western India presents with symptoms of lethargy and
dizziness, worse on turning her head. On examination her systolic blood
pressure is 176/128. Her pulses are impalpable at all peripheral sites.
Auscultation of her chest reveals a systolic heart murmur.

18
19
20
21

Takayasu's arteritis

22-24 1 / 3

Takayasu's arteritis most commonly affects young Asian females. Pulseless


peripheries are a classical finding. The CNS symptoms may be variable.
23.

A 48 year old man notices that he is becoming increasingly dizzy when he plays
squash, in addition he has also developed cramping pain in his left arm. One
day he is inflating his car tyre with a hand held pump, he collapses and is
brought to hospital.
You answered Aortic dissection
The correct answer is Subclavian steal syndrome
Subclavian steal syndrome is associated with a stenosis or occlusion of the
subclavian artery, proximal to the origin of the vertebral artery. As a result the
increased metabolic needs of the arm then cause retrograde flow and
symptoms of CNS vascular insufficiency.

24.

A 25 year old junior doctor has a chest x-ray performed as part of a routine
insurance medical examination. The x-ray shows evidence of rib notching.
Auscultation of his chest reveals a systolic murmur which is loudest at the
posterior aspect of the fourth intercostal space.
You answered Patent ductus arteriosus
The correct answer is Aortic coarctation
Coarctation of the aorta may occur due to the remnant of the ductus arteriosus
acting as a fibrous constrictive band of the aorta. Weak arm pulses may be
seen, radiofemoral delay is the classical physical finding. Collateral flow
through the intercostal vessels may produce notching of the ribs, if the disease
is long standing.

Next question

Vascular disease

Aortic
dissection

Cervical rib

Chest pain (anterior chest pain- ascending aorta, back pain descending aorta)
Widening of aorta on chest x-ray
Diagnosis made by CT scanning
Treatment is either medical (Type B disease) or surgical (Type A
disease)

Supernumery fibrous band arising from seventh cervical vertebra


Incidence of 1 in 500
May cause thoracic outlet syndrome
Treatment involves surgical division of rib

http://www.emrcs.com/question/question.php?q=0

1/2

13/07/2015

Subclavian
steal
syndrome

Takayasu's
arteritis

Patent ductus
arteriosus

Coarctation of
the aorta

Due to proximal stenotic lesion of the subclavian artery


Results in retrograte flow through vertebral or internal thoracic arteries
The result is that decrease in cerebral blood flow may occur and
produce syncopal symptoms
A duplex scan and/ or angiogram will delineate the lesion and allow
treatment to be planned

Large vessel granulomatous vasculitis


Results in intimal narrowing
Most commonly affects young asian females
Patients present with features of mild systemic illness, followed by
pulseless phase with symptoms of vascular insufficiency
Treatment is with systemic steroids

Ductus arteriosus is a normal foetal vessel that closes spontaneously


after birth
Results in high pressure, oxygenated blood entering the pulmonary
circuit
Untreated patients develop symptoms of congestive cardiac failure

Aortic stenosis at the site of the ductus arteriosus insertion


Most common in boys and girls with Turners syndrome
Patients may present with symptoms of arterial insufficiency, such as
syncope and claudication
Blood pressure mismatch may be seen, as may mismatch of pulse
pressure in the upper and lower limbs
Treatment is either with angioplasty or surgical resection (the former is
the most common)

Rate question:

Next question

Comment on this question

All contents of this site are 2012 E-Medical Revision Ltd

http://www.emrcs.com/question/question.php?q=0

Terms and Conditions

Privacy policy

2/2

13/07/2015

Reference ranges

Previous

Question 25 of 196

Next

A 25 year old man is found to have carcinoid syndrome. Which of the following hormones is
released by carcinoids?

A. Serotonin
B. Dopamine

Question stats

Score: 34.4%
1

62.9%

9%

9.6%

8.6%

9.9%

62.9% of users answered this


question correctly

C. Nor adrenaline

End and review

4
5-7 1 / 3
8
9
10-12 2 / 3

D. Adrenaline

13

E. Aldosterone

Search
14
Go

Next question

15
16
17

Rule of thirds:

18

1/3 multiple
1/3 small bowel
1/3 metastasize
1/3 second tumour

19
20
21
22-24 1 / 3
25

Carcinoids secrete serotonin. Carcinoid syndrome will only occur in the presence of liver
metastasis as the hormone released from primary lesions will typically be metabolised by the
liver.
Carcinoid syndrome

Carcinoid tumours secrete serotonin


Originate in neuroendocrine cells mainly in the intestine (midgut-distal ileum/appendix)
Can occur in the rectum, bronchi
Hormonal symptoms mainly occur when disease spreads outside the bowel

Clinical features
Onset: years
Flushing face
Palpitations
Pulmonary valve stenosis and tricuspid regurgitation causing dyspnoea
Asthma
Severe diarrhoea (secretory, persists despite fasting)

Investigation
5-HIAA in a 24-hour urine collection
Somatostatin receptor scintigraphy
CT scan
Blood testing for chromogranin A

Treatment
Octreotide
Surgical removal

Rate question:

Next question

Comment on this question

All contents of this site are 2012 E-Medical Revision Ltd

http://www.emrcs.com/question/question.php?q=0

Terms and Conditions

Privacy policy

1/1

13/07/2015

Reference ranges

Previous

Question 26 of 196

Next

Question stats

End and review

Score: 36.4%
1

9.8%

62.5%

10%

A. sis

9.6%

B. p53

8.1%

C. ras

62.5% of users answered this


question correctly

Which one of the following genes protects against neoplasms?

D. myc

4
5-7 1 / 3
8
9
10-12 2 / 3

E. src

13
Search
14

Next question
Go

p53 is a tumour supressor gene and located on chromosome 17. It plays an important role in
causing cells that are undergoing neoplastic changes to enter an apoptotic pathway.

15
16
17
18

Oncogenes

19

Oncogenes are cancer promoting genes that are derived from normal genes (protooncogenes). Proto-oncogenes play an important physiological role in cellular growth. They
are implicated in the development of up to 20% of human cancers.

20
21
22-24 1 / 3

Proto-oncogenes may become oncogenes via the following processes:

25
26

Mutation (point mutation)


Chromosomal translocation
Increased protein expression
Only one mutated copy of the gene is needed for cancer to occur - a dominant effect
Classification of oncogenes
Growth factors e.g. Sis
Transcription factors e.g. Myc
Receptor tyrosine kinase e.g. RET
Cytoplasmic tyrosine kinase e.g. Src
Regulatory GTPases e.g. Ras

Tumour supressor genes


Tumour supressor genes restrict or repress cellular proliferation in normal cells. Their
inactivation through mutation or germ line incorporation is implicated in renal, colonic, breast,
bladder and many other cancers. One of the best known tumour supressor genes is p53.
p53 gene offers protection by causing apoptosis of damaged cells. Other well known genes
include BRCA 1 and 2.
Rate question:

Next question

Comment on this question

All contents of this site are 2012 E-Medical Revision Ltd

http://www.emrcs.com/question/question.php?q=0

Terms and Conditions

Privacy policy

1/1

13/07/2015

Reference ranges

Previous

Question 27 of 196

Next

A 55 year old man with a long history of achalasia is successfully treated by a Hellers
Cardiomyotomy. Several years later he develops an oesophageal malignancy. Which of the
following lesions is most likely to be present?

Question stats

Score: 35.3%
1

34.9%

9.6%

10.4%

7.8%

37.3%

A. Adenocarcinoma
37.3% of users answered this
question correctly

B. Gastrointestinal stromal tumour

End and review

C. Leiomyosarcoma

4
5-7 1 / 3
8
9
10-12 2 / 3

D. Rhabdomyosarcoma

13
Search

E. Squamous cell carcinoma

14
Go

Next question

15
16
17

Achalasia is a rare condition. However, even once treated there is an increased risk of
malignancy. When it does occur it is most likely to be of squamous cell type.

18
19

Oesophageal cancer

20
21

Incidence is increasing
In most cases in the Western world this increase is accounted for by a rise in the
number of cases of adenocarcinoma. In the UK adenocarcinomas account for 65% of
cases.
Barretts oesophagus is a major risk factor for most cases of oesophageal
adenocarcinoma.
In other regions of the world squamous cancer is more common and is linked to
smoking, alcohol intake, diets rich in nitrosamines and achalasia.
Surveillance of Barretts is important, as it imparts a 30 fold increase in cancer risk and
if invasive malignancy is diagnosed early then survival may approach 85% at 5 years.

22-24 1 / 3
25
26
27

Diagnosis
Upper GI endoscopy is the first line test
Contrast swallow may be of benefit in classifying benign motility disorders but has no
place in the assessment of tumours
Staging is initially undertaken with CT scanning of the chest, abdomen and pelvis. If
overt metastatic disease is identified using this modality then further complex imaging
is unnecessary
If CT does not show metastatic disease, then local stage may be more accurately
assessed by use of endoscopic ultrasound.
Staging laparoscopy is performed to detect occult peritoneal disease. PET CT is
performed in those with negative laparoscopy. Thoracoscopy is not routinely
performed.

Treatment
Operable disease is best managed by surgical resection. The most standard procedure is an
Ivor- Lewis type oesophagectomy. This procedure involves the mobilisation of the stomach
and division of the oesophageal hiatus. The abdomen is closed and a right sided
thoracotomy performed. The stomach is brought into the chest and the oesophagus
mobilised further. An intrathoracic oesophagogastric anastomosis is constructed. Alternative
surgical strategies include a transhiatal resection (for distal lesions), a left thoraco-abdominal
resection (difficult access due to thoracic aorta) and a total oesophagectomy (McKeown) with
a cervical oesophagogastric anastomosis.
The biggest surgical challenge is that of anastomotic leak, with an intrathoracic anastomosis
this will result in mediastinitis. With high mortality. The McKeown technique has an
intrinsically lower systemic insult in the event of anastomotic leakage.
In addition to surgical resection many patients will be treated with adjuvant chemotherapy.
Rate question:

Next question

Comment on this question

All contents of this site are 2012 E-Medical Revision Ltd

http://www.emrcs.com/question/question.php?q=0

Terms and Conditions

Privacy policy

1/1

13/07/2015

Reference ranges

Previous

Question 29 of 196

Next

A 63 year old lady is suspected as having sarcoidosis. She is sent to the general surgeons
and a lymph node biopsy is performed. Which histological feature is most likely to be
identified in a lymph node if sarcoid is present?

Question stats

Score: 38.9%
1

18.6%

10.9%

21.6%

36.4%

12.6%

A. Psammoma bodies
36.4% of users answered this
question correctly

B. Extensive necrosis

End and review

C. Dense eosinophillic infiltrates

4
5-7 1 / 3
8
9
10-12 2 / 3

D. Asteroid bodies

13
Search

E. None of the above

14
Go

Next question

15
16

Asteroid bodies are often found in the granulomas of individuals with sarcoid. Unlike the
granulomata associated with tuberculosis the granulomas of sarcoid are rarely associated
with extensive necrosis.

17

An Asteroid body in an individual with sarcoid

20

18
19

21
22-24 1 / 3
25
26
27
28
29

Image sourced from Wikipedia

Chronic inflammation
Overview
Chronic inflammation may occur secondary to acute inflammation.In most cases chronic
inflammation occurs as a primary process. These may be broadly viewed as being one of
three main processes:
Persisting infection with certain organisms such as Mycobacterium tuberculosis which
results in delayed type hypersensitivity reactions and inflammation.
Prolonged exposure to non-biodegradable substances such as silica or suture
materials which may induce an inflammatory response.
Autoimmune conditions involving antibodies formed against host antigens.

Acute vs. Chronic inflammation


Acute inflammation

Chronic inflammation

Changes to existing vascular structure and increased


permeability of endothelial cells

Angiogenesis predominates

Infiltration of neutrophils

Macrophages, plasma cells and


lymphocytes predominate

Process may resolve with:

Healing by fibrosis is the main result

http://www.emrcs.com/question/question.php?q=0

1/2

13/07/2015

Suppuration
Complete resolution
Abscess formation
Progression to chronic inflammation
Healing by fibrosis

Granulomatous inflammation
A granuloma consists of a microscopic aggregation of macrophages (with epithelial type
arrangement =epitheliod). Large giant cells may be found at the periphery of granulomas.
Mediators
Growth factors released by activated macrophages include agents such as interferon and
fibroblast growth factor (plus many more). Some of these such as interferons may have
systemic features resulting in systemic symptoms and signs, which may be present in
individuals with long standing chronic inflammation.
The finding of granulomas is pathognomonic of chronic inflammation, as illustrated in this
biopsy from a patient with colonic Crohns disease

Image sourced from Wikipedia

Rate question:

Next question

Comment on this question

All contents of this site are 2012 E-Medical Revision Ltd

http://www.emrcs.com/question/question.php?q=0

Terms and Conditions

Privacy policy

2/2

13/07/2015

Reference ranges

Previous

Question 30 of 196

Next

Question stats

End and review

Score: 36.8%
1

10.1%

7.4%

47.9%

A. Hyperthyroidism

13.2%

B. Hypothyroidism

21.3%

C. Hyperparathyroidism

47.9% of users answered this


question correctly

Brown tumours of bone are associated with which of the following?

D. Hypoparathyroidism

4
5-7 1 / 3
8
9
10-12 2 / 3

E. Osteopetrosis

13
Search
14

Next question
Go

Brown tumors are tumors of bone that arise in settings of excess osteoclast activity, such as
hyperparathyroidism, and consist of fibrous tissue, woven bone and supporting vasculature,
but no matrix. They are radiolucent on x-ray. The osteoclasts consume the trabecular bone
that osteoblasts lay down and this front of reparative bone deposition followed by additional
resorption can expand beyond the usual shape of the bone, involving the periosteum thus
causing bone pain. They appear brown because haemosiderin is deposited at the site.

15
16
17
18
19
20
21

Primary hyperparathyroidism

22-24 1 / 3

In exams primary hyperparathyroidism is stereotypically seen in elderly females with an


unquenchable thirst and an inappropriately normal or raised parathyroid hormone level. It is
most commonly due to a solitary adenoma

25
26
27

Causes of primary hyperparathyroidism

28
29

80%: solitary adenoma


15%: hyperplasia
4%: multiple adenoma
1%: carcinoma

30

Features - 'bones, stones, abdominal groans and psychic moans'


Polydipsia, polyuria
Peptic ulceration/constipation/pancreatitis
Bone pain/fracture
Renal stones
Depression
Hypertension

Associations
Hypertension
Multiple endocrine neoplasia: MEN I and II

Investigations
Raised calcium, low phosphate
PTH may be raised or normal
Technetium-MIBI subtraction scan

Treatment
Parathyroidectomy, if imaging suggests target gland then a focused approach may be
used

Rate question:

Next question

Comment on this question

All contents of this site are 2012 E-Medical Revision Ltd

http://www.emrcs.com/question/question.php?q=0

Terms and Conditions

Privacy policy

1/1

13/07/2015

Reference ranges

Question 31 of 196

Previous

Next

A 73 year old man is recovering following an emergency Hartmans procedure performed for
an obstructing sigmoid cancer. The pathology report shows a moderately differentiated
adenocarcinoma that invades the muscularis propria, 3 of 15 lymph nodes are involved with
metastatic disease. What is the correct stage for this?

Question stats

Score: 35.9%
1

8%

5.7%

14.6%

46.9%

24.8%

46.9% of users answered this


question correctly

A. Astler Coller Stage B2

End and review

B. Dukes stage A

4
5-7 1 / 3
8
9
10-12 2 / 3

C. Dukes stage B

13
Search

D. Dukes stage C

14

E. Dukes stage D

Go

15
16

Next question

17
18

Remember that the term metastasis simply refers to spread and can include the lymph
nodes. In an examination setting marks can be lost by incorrectly selecting Dukes D
(which would be consistent with liver metastasis) rather than nodal metastasis (Dukes C).

19
20
21
22-24 1 / 3

Theme from September 2011 Exam

25

The involvement of lymph nodes makes this Dukes C. In the Astler Coller system the B and C
subsets are split to B1 and B2 and C1 and C2. Where C2 denotes involvement of the nodes
in conjunction with penetration of the muscularis propria.

26
27
28

Dukes classification

29
30

Gives the extent of spread of colorectal cancer


31

Dukes A

Tumour confined to the mucosa (90%)

Dukes B

Tumour invading bowel wall (70%)

Dukes C

Lymph node metastases (45%)

Dukes D

Distant metastases (6%)(20% if resectable)

5 year survival in brackets


Rate question:

Next question

Comment on this question

All contents of this site are 2012 E-Medical Revision Ltd

http://www.emrcs.com/question/question.php?q=0

Terms and Conditions

Privacy policy

1/1

13/07/2015

Reference ranges

Previous

Question 32 of 196

End and review

Next

A 55 year old man presents with a soft, fluctuant lesion overlying his right scapula. The
surgeon suspects the lesion may be a lipoma. Which of the following, if present, may be
indicative of an alternative diagnosis?

A. Located in superficial tissues


B. Size greater than 5cm
C. Presence of multiple similar lesions at other anatomical sites
D. Increased mobility of the lesion
E. Lobulated appearance during surgical excision
Submit answer

All contents of this site are 2012 E-Medical Revision Ltd

http://www.emrcs.com/question/question.php?q=0

Terms and Conditions

Privacy policy

1/1

13/07/2015

Reference ranges

Previous

Question 33 of 196

Next

A 55 year old man with dyspepsia undergoes an upper GI endoscopy. An irregular


erythematous area is seen to protrude proximally from the gastro-oesophageal junction.
Apart from specialised intestinal metaplasia, which of the following cell types should also be
present for a diagnosis of Barretts oesophagus to be made?

Question stats

Score: 37.5%
1

54.2%

7.9%

9.7%

15.2%

13%

54.2% of users answered this


question correctly

A. Goblet cell

End and review

B. Neutrophil

4
5-7 1 / 3
8
9
10-12 2 / 3

C. Lymphocytes

13
Search

D. Epithelial cells

14

E. Macrophages

Go

15
16

Next question

17
18

Goblet cells need to be present for a diagnosis of Barrett's oesophagus to be made.

19

Barrett's oesophagus

20
21

Barretts oesophagus is a condition characterised by the metaplastic transformation of


squamous oesophageal epithelium to columnar gastric type epithelium. Three types of this
metaplastic process are recognised; intestinal (high risk), cardiac and fundic. The latter two
categories may cause difficulties in diagnosis. The most concrete diagnosis can be made
when endoscopic features of Barretts oesophagus are present together with a deep biopsy
that demonstrates not just goblet cell metaplasia but also oesophageal glands.

22-24 1 / 3
25
26
27
28

Barrett's can be sub divided into short (<3cm) and long (>3cm). The length of the affected
segment correlates strongly with the chances of identifying metaplasia. The overall
prevalence of Barrett's oesophagus is difficult to determine but may be in the region of 1 in
20 and is identified in up to 12% of those undergoing endoscopy for reflux.

29
30
31
32

A proportion of patients with metaplasia will progress to dysplasia and for this reason
individuals identified as having Barrett's should undergo endoscopic surveillance (every 2-5
years). Biopsies should be quadrantic and taken at 1-2cm intervals. Biopsies need to be
adequate. Where mass lesions are present consideration should be given to endoscopic sub
mucosal resection. Up to 40% of patients will be upstaged from high grade dysplasia to
invasive malignancy with such techniques.

33

Treatment
Long term proton pump inhibitor
Consider pH and manometry studies in younger patients who may prefer to consider
an anti reflux procedure
Regular endoscopic monitoring (more frequently if moderate dysplasia). With
quadrantic biopsies every 2-3 cm
If severe dysplasia be very wary of small foci of cancer

References
A consensus statement of the British approach is provided by:
Bennett C et al Consensus Statements for Management of Barrett's Dysplasia and EarlyStage Esophageal Adenocarcinoma, Based on a Delphi Process. Gastroenterology Volume
143, Issue 2 , Pages 336-346, August 2012.
Rate question:

Next question

Comment on this question

All contents of this site are 2012 E-Medical Revision Ltd

http://www.emrcs.com/question/question.php?q=0

Terms and Conditions

Privacy policy

1/1

13/07/2015

Reference ranges

Previous

Question 34 of 196

Next

Question stats

End and review

Score: 39%
1

19%

9.6%

36.2%

A. Alanine

21%

B. Aspartime

14.2%

C. Glycine

36.2% of users answered this


question correctly

Which of the following amino acids is present in all types of collagen?

D. Tyrosine

4
5-7 1 / 3
8
9
10-12 2 / 3

E. Cysteine

13
Search
14

Next question
Go

Collagen has a generic structure of Glycine- X- Y, where X and Y are variable sub units. The
relatively small size of the glycine molecule enables collagen to form a tight helical structure.

15
16
17
18

Collagen

19

Collagen is one of the most important structural proteins within the extracellular matrix,
collagen together with components such as elastin and glycosaminoglycans determine the
properties of all tissues.

20
21
22-24 1 / 3

Composed of 3 polypeptide strands that are woven into a helix, usually a combination
of glycine with either proline or hydroxyproline plus another amino acid
Numerous hydrogen bonds exist within molecule to provide additional strength
Many sub types but commonest sub type is I (90% of bodily collagen), tissues with
increased levels of flexibility have increased levels of type III collagen
Vitamin c is important in establishing cross links
Synthesised by fibroblasts

25
26
27
28
29
30
31

Collagen Diseases
Disorders of collagen range from relatively common, acquired defects (typically aging),
through to rarer congenital disorders. The latter are exemplified by conditions such as
osteogenesis imperfecta and Ehlers Danlos syndromes.

32

33
34

Osteogenesis imperfecta:
-8 Subtypes
-Defect of type I collagen
-In type I the collagen is normal quality but insufficient quantity
-Type II- poor quantity and quality
-Type III- Collagen poorly formed, normal quantity
-Type IV- Sufficient quantity but poor quality
Patients have bones which fracture easily, loose joint and multiple other defects depending
upon which sub type they suffer from.
Ehlers Danlos:
-Multiple sub types
-Abnormality of types 1 and 3 collagen
-Patients have features of hypermobility.
-Individuals are prone to joint dislocations and pelvic organ prolapse. In addition to many
other diseases related to connective tissue defects.
Rate question:

Next question

Comment on this question

All contents of this site are 2012 E-Medical Revision Ltd

http://www.emrcs.com/question/question.php?q=0

Terms and Conditions

Privacy policy

1/1

13/07/2015

Reference ranges

Previous

Question 35 of 196

Next

An 8 year old boy presented with a painless swelling on the superotemporal aspect of his
orbit. It was smooth on examination, produced no visual disturbances. Following excision it
was found to be lined by squamous epithelium and hair follicles. Which of the following
lesions most closely matches these findings?

Question stats

Score: 38.1%
1

57.2%

13.1%

6.9%

15.8%

7%

57.2% of users answered this


question correctly

A. Dermoid cyst

End and review

B. Desmoid tumour

4
5-7 1 / 3
8
9
10-12 2 / 3

C. Lipoma

13
Search

D. Sebaceous cyst

14

E. Schwannoma

Go

Next question

15
16
17

Dermoid cysts are embryological remnants and may be lined by hair and squamous
epithelium (like teratomas). They are often located in the midline and may be linked to
deeper structures resulting in a dumbbell shape to the lesion. Complete excision is required
as they have a propensity to local recurrence if not excised.
Desmoid tumours are a different entity, they most commonly develop in ligaments and
tendons. They are also referred to as aggressive fibromatosis and consist of fibroblast
dense lesions (resembling scar tissue). They should be managed in a similar manner to soft
tissue sarcomas.

18
19
20
21
22-24 1 / 3
25
26
27

Skin Diseases

28

Skin lesions may be referred for surgical assessment, but more commonly will come via a
dermatologist for definitive surgical management.

29
30

Skin malignancies include basal cell carcinoma, squamous cell carcinoma and malignant
melanoma.

31
32

33

Basal Cell Carcinoma

34

Most common form of skin cancer.


Commonly occur on sun exposed sites apart from the ear.
Sub types include nodular, morphoeic, superficial and pigmented.
Typically slow growing with low metastatic potential.
Standard surgical excision, topical chemotherapy and radiotherapy are all successful.
As a minimum a diagnostic punch biopsy should be taken if treatment other than
standard surgical excision is planned.

35

Squamous Cell Carcinoma


Again related to sun exposure.
May arise in pre - existing solar keratoses.
May metastasize if left.
Immunosupression (e.g. following transplant), increases risk.
Wide local excision is the treatment of choice and where a diagnostic excision biopsy
has demonstrated SCC, repeat surgery to gain adequate margins may be required.

Malignant Melanoma
The main diagnostic features (major criteria):

Secondary features (minor criteria)

Change in size
Change in shape
Change in colour

Diameter >6mm
Inflammation
Oozing or bleeding
Altered sensation

Treatment
Suspicious lesions should undergo excision biopsy. The lesion should be removed in
completely as incision biopsy can make subsequent histopathological assessment
difficult.
Once the diagnosis is confirmed the pathology report should be reviewed to determine
whether further re-excision of margins is required (see below):

Margins of excision-Related to Breslow thickness


Lesions 0-1mm thick

1cm

http://www.emrcs.com/question/question.php?q=0

1/2

13/07/2015

Lesions 1-2mm thick

1- 2cm (Depending upon site and pathological features)

Lesions 2-4mm thick

2-3 cm (Depending upon site and pathological features)

Lesions >4 mm thick

3cm

Marsden J et al. Revised UK guidelines for management of Melanoma. Br J Dermatol 2010


163:238-256.
Further treatments such as sentinel lymph node mapping, isolated limb perfusion and block
dissection of regional lymph node groups should be selectively applied.
Kaposi Sarcoma
Tumour of vascular and lymphatic endothelium.
Purple cutaneous nodules.
Associated with immuno supression.
Classical form affects elderly males and is slow growing.
Immunosupression form is much more aggressive and tends to affect those with HIV
related disease.

Non malignant skin disease


Dermatitis Herpetiformis
Chronic itchy clusters of blisters.
Linked to underlying gluten enteropathy (coeliac disease).

Dermatofibroma
Benign lesion.
Firm elevated nodules.
Usually history of trauma.
Lesion consists of histiocytes, blood vessels and fibrotic changes.

Pyogenic granuloma
Overgrowth of blood vessels.
Red nodules.
Usually follow trauma.
May mimic amelanotic melanoma.

Acanthosis nigricans
Brown to black, poorly defined, velvety hyperpigmentation of the skin.
Usually found in body folds such as the posterior and lateral folds of the neck, the
axilla, groin, umbilicus, forehead, and other areas.
The most common cause of acanthosis nigricans is insulin resistance, which leads to
increased circulating insulin levels. Insulin spillover into the skin results in its abnormal
increase in growth (hyperplasia of the skin).
In the context of a malignant disease, acanthosis nigricans is a paraneoplastic
syndrome and is then commonly referred to as acanthosis nigricans maligna.
Involvement of mucous membranes is rare and suggests a coexisting malignant
condition.

Rate question:

Next question

Comment on this question

All contents of this site are 2012 E-Medical Revision Ltd

http://www.emrcs.com/question/question.php?q=0

Terms and Conditions

Privacy policy

2/2

13/07/2015

Reference ranges

Previous

Question 36 of 196

Next

A 55 year old man from Hong Kong presents with left sided otalgia and recurrent episodes of
epistaxis. On examination his pharynx appears normal. Examination of his neck reveals left
sided cervical lymphadenopathy. What is the most likely underlying diagnosis?

Question stats

Score: 39.5%
1

8.1%

61.1%

10.2%

9.9%

10.6%

A. Antrochoanal polyp
61.1% of users answered this
question correctly

B. Nasopharyngeal carcinoma

End and review

C. Adenocarcinoma of the tonsil

4
5-7 1 / 3
8
9
10-12 2 / 3

D. Angiofibroma

13
Search

E. Globus syndrome

14
Go

Next question

15
16
17

Given this mans ethnic origin and presenting features a nasopharyngeal carcinoma is the
most likely underlying diagnosis.

18
19

Nasopharyngeal carcinoma

20
21

Squamous cell carcinoma of the nasopharynx


Rare in most parts of the world, apart from individuals from Southern China
Associated with Epstein Barr virus infection

22-24 1 / 3
25
26
27

Presenting features

28

Systemic

Local

Cervical lymphadenopathy

Otalgia

29
30
31

Unilateral serous otitis media

32

Nasal obstruction, discharge and/ or epistaxis

33
34

Cranial nerve palsies e.g. III-VI

35
36

Imaging
Combined CT and MRI.
Treatment
Radiotherapy is first line therapy.
Rate question:

Next question

Comment on this question

All contents of this site are 2012 E-Medical Revision Ltd

http://www.emrcs.com/question/question.php?q=0

Terms and Conditions

Privacy policy

1/1

13/07/2015

Reference ranges

Previous

Question 37 of 196

Next

A women is diagnosed as having pernicious anaemia. What is the most likely underlying
explanation for this?

A. Autoimmune antibodies to parietal cells


B. Autoimmune antibodies to chief cells

Question stats

Score: 42.2%
1

57%

17.1%

9.2%

8.8%

7.9%

57% of users answered this


question correctly

C. Autoimmune antibodies to goblet cells

End and review

4
5-7 1 / 3
8
9
10-12 2 / 3

D. Autoimmune antibodies to Brunners glands

13

E. Autimmune antibodies to fundic cells

Search
14
Go

Next question

15
16

Parietal cell destruction is a major cause of pernicious anaemia and is usually autoimmune
mediated. Other conditions such as bacterial overgrowth may produce mixed patterns and
require more complex diagnostic evaluation.

17
18
19

Pernicious anaemia

20
21

Pernicious anaemia is a chronic illness caused by impaired absorption of vitamin B-12


because of a lack of intrinsic factor (IF) in gastric secretions. It occurs as a relatively
common adult form of anaemia that is associated with gastric atrophy and a loss of IF
production and as a rare congenital autosomal recessive form in which IF production is
lacking without gastric atrophy.
Classic pernicious anaemia is caused by the failure of gastric parietal cells to produce
sufficient IF (a gastric protein secreted by parietal cells) to permit the absorption of
adequate quantities of dietary vitamin B-12. Other disorders that interfere with the
absorption and metabolism of vitamin B-12 can produce cobalamin deficiency, with the
development of a macrocytic anaemia and neurologic complications. In many cases
the underlying cause is autoimmune destruction of the parietal cell mass of the gastric
antrum.
Diagnostic evaluation may include investigations to delineate the precise cause and
this may include Schilling test, trials of B12 and in some cases bone marrow sampling.
Treatment of the disorder is with 3 monthly treatment of vitamin B12 injections. Folic
acid supplementation may also be required.

22-24 1 / 3
25
26
27
28
29
30
31
32

33
34
35
36
37

Rate question:

Next question

Comment on this question

All contents of this site are 2012 E-Medical Revision Ltd

http://www.emrcs.com/question/question.php?q=0

Terms and Conditions

Privacy policy

1/1

13/07/2015

Reference ranges

Previous

Question 38 of 196

Next

Question stats

End and review

Score: 44.7%
1

9.3%

11.3%

23.7%

42.9%

A. Temporal artery biopsy is only diagnostic if there is visual loss

12.9%

B. Biopsy is typically taken from the non-symptomatic side to avoid the risk
of blindness

42.9% of users answered this


question correctly

Patients with suspected temporal arteritis are often sent for temporal artery biopsy. Which
statement is true?

C. Pre-operative localisation with duplex is mandatory

4
5-7 1 / 3
8
9
10-12 2 / 3

D. Biopsies may be non diagnostic in over 50% of cases

13
Search

E. Biopsies are usually performed under general anaesthesia

14
Go

15

Next question

16
17

Temporal artery biopsies are frequently non diagnostic. They should be taken from the
symptomatic side and though not mandatory a duplex ultrasound is a helpful investigation,
particularly if they mark the artery. It is usually performed under local anaesthetic.

18
19
20

Temporal artery biopsy

21
22-24 1 / 3

Superficial temporal artery is a terminal branch of the external carotid artery

25
26

Main indication

27

Temporal arteritis

28
29

American College of Rheumatology guidelines recommend a temporal artery biopsy if:

30
31

Age of onset older than 50 years


New-onset headache or localized head pain
Temporal artery tenderness to palpation or reduced pulsation
ESR > 50 mm/h

32

33
34
35
36

Histopathology

37

Vessel wall granulomatous arteritis with mononuclear cell infiltrates and giant cell
formation

38

Procedure
Position: supine, head 45 degrees
USS doppler to locate the superficial temporal artery or palpate
Local anaesthetic
Artery within temporoparietal fascia
Clamp and ligate the vessel
Cut 3-5cm
Ligate the remaining ends with absorbable suture
Close the skin

Contraindication
Glucocorticoid therapy > 30 days
Risks
Injury to facial or auriculotemporal nerve
Rate question:

Next question

Comment on this question

All contents of this site are 2012 E-Medical Revision Ltd

http://www.emrcs.com/question/question.php?q=0

Terms and Conditions

Privacy policy

1/1

13/07/2015

Reference ranges

Previous

Question 39 of 196

Next

Which of the following best describes the processes underpinning type IV hypersensitivity
reactions?

A. Deposition of immune complexes of IgG and antigen at the site of


inflammation

Question stats

Score: 46.9%
1

12.9%

7.5%

10.6%

11.7%

57.3%

57.3% of users answered this


question correctly

B. Deposition of IgA complexes at the site of inflammation

End and review

C. Deposition of IgM and IgG complexes at the site of inflammation

4
5-7 1 / 3
8
9
10-12 2 / 3

D. Degranulation of mast cells at the site of inflammation

13
Search

E. T cell mediated response at the site of inflammation

14
Go

15

Next question

16
17

Hypersensitivity reactions: ACID

18
19

type 1 --Anaphylactic
type 2 --Cytotoxic
type 3 --Immune complex
type 4 --Delayed hypersensitivity

20
21
22-24 1 / 3
25

T Cells are the mediators of type 4 hypersensitivity reactions which are characterised by the
absence of immune complex deposition.

26
27
28

Hypersensitivity reactions

29

The Gell and Coombs classification divides hypersensitivity reactions into 4 types

30
31

Description

Mediator

Type I

Type II

Type III

Type IV

Anaphylactic

Cytotoxic

Immune
complex

Delayed type

IgG, Ig A,
IgM

T-cells

IgE

IgG, IgM

32

33
34
35
36
37

Antigen

Exogenous

Cell surface

Soluble

Tissues

Response
time

Minutes

Hours

Hours

2-3 days

Examples

Asthma
Hay fever

Autoimmune haemolytic
anaemia
Pemphigus
Goodpasture's

Serum
sickness
SLE
Aspergillosis

Graft versus host


disease
Contact dermatitis

38

Rate question:

39

Next question

Comment on this question

All contents of this site are 2012 E-Medical Revision Ltd

http://www.emrcs.com/question/question.php?q=0

Terms and Conditions

Privacy policy

1/1

13/07/2015

Reference ranges

Previous2 / 3

Question 40-42 of 196

Next

Question stats

End and review

Score: 48.1%

Average score for registered users:

1
2

Theme: Bone tumours


A. Osteosarcoma

40

54.3%

41

59%

42

32.7%

B. Fibrosarcoma

3
4
5-7 1 / 3

C. Osteoclastoma

D. Ewings sarcoma

Search

E. Leiomyosarcoma

Go

10-12 2 / 3

F. Chondrosarcoma

13

G. Rhabdomyosarcoma

14

H. Osteoid osteoma

15

I. Malignant fibrous histiocytoma

16
17

Please select the most appropriate lesion for the clinical scenario given. Each option may be
used once, more than once or not at all.

18
19
20

40.

A 16 year-old boy presents to his GP with loss of weight, pain and fever. On
examination, a soft tissue mass is palpable over the mid-thigh region

21
22-24 1 / 3
25

You answered Fibrosarcoma

26

The correct answer is Ewings sarcoma

27

Ewing's sarcoma is a malignant round cell tumour occurring in the diaphysis of


the long bones in the children. These are not confined to the ends of long
bones. x Rays often show a large soft-tissue mass with concentric layers of new
bone formation ( 'onion-peel' sign). The ESR may be elevated, thus suggesting
an inflammatory or an infective cause such as osteomyelitis; although
osteomyelitis usually affects the metaphyseal region in children. Treatment is
with chemotherapy and surgical excision, an endoprothesis may be used to
conserve the limb.

28
29
30
31
32

33
34

41.

A 75 year old lady presents with weight loss, pain and a swelling over her left
knee. She has been treated for Pagets disease of the bone for some time.

35
36

Osteosarcoma

37

Osteosarcoma may complicate Pagets disease of bone in up to 10% cases.


Radiological appearances include bone destruction coupled with new bone
formation, periosteal elevation may also occur. Surgical resection is the main
treatment.
42.

38
39
40-42 2 / 3

A 17-year-old girl presents with weight loss, fever and a swelling over her right
knee. Movements of her knee are restricted. A plain x-ray of the affected site
shows multiple lytic and lucent lesions with clearly defined borders.
Osteoclastoma
Osteoclastoma has a characteristic appearance on x-ray with multple lytic and
lucent areas (Soap bubble) appearances. Pathological fractures may occur.
The disease is usually indolent.

Next question

Sarcomas

Malignant tumours of mesenchymal origin

Types
May be either bone or soft tissue in origin.
Bone sarcoma include:
Osteosarcoma
Ewings sarcoma (although non bony sites recognised)
Chondrosarcoma - originate from Chondrocytes

http://www.emrcs.com/question/question.php?q=0

1/2

13/07/2015

Soft tissue sarcoma are a far more heterogeneous group and include:
Liposarcoma-adipocytes
Rhabdomyosarcoma-striated muscle
Leiomyosarcoma-smooth muscle
Synovial sarcomas- close to joints (cell of origin not known but not synovium)

Malignant fibrous histiocytoma is a sarcoma that may arise in both soft tissue and bone.
Features
Certain features of a mass or swelling should raise suspicion for a sarcoma these include:
Large >5cm soft tissue mass
Deep tissue location or intra muscular location
Rapid growth
Painful lump

Assessment
Imaging of suspicious masses should utilise a combination of MRI, CT and USS. Blind biopsy
should not be performed prior to imaging and where required should be done in such a way
that the biopsy tract can be subsequently included in any resection.
Ewings sarcoma
Commoner in males
Incidence of 0.3 / 1, 000, 000
Onset typically between 10 and 20 years of age
Location by femoral diaphysis is commonest site
Histologically it is a small round tumour
Blood borne metastasis is common and chemotherapy is often combined with surgery

Osteosarcoma
Mesenchymal cells with osteoblastic differentiation
20% of all primary bone tumours
Incidence of 5 per 1,000,000
Peak age 15-30, commoner in males
Limb preserving surgery may be possible and many patients will receive chemotherapy

Liposarcoma
Malignancy of adipocytes
Rare, approximately 2.5 per 1,000,000. They are the second most common soft tissue
sarcoma
Typically located in deep locations such as retroperitoneum
Affect older age group usually >40 years of age
May be well differentiated and thus slow growing although may undergo dedifferentiation and disease progression
Many tumours will have a pseudocapsule that can misleadingly allow surgeons to feel
that they can 'shell out' these lesions. In reality, tumour may invade at the edge of the
pseudocapsule and result in local recurrence if this strategy is adopted
Usually resistant to radiotherapy, although this is often used in a palliative setting

Malignant Fibrous Histiocytoma


Tumour with large number of histiocytes
Most common sarcoma in adults
Also described as undifferentiated pleomorphic sarcoma NOS (i.e. Cell of origin is not
known)
Four major subtypes are recognised: storiform-pleomorphic (70% cases), myxoid (less
aggressive), giant cell and inflammatory
Treatment is usually with surgical resection and adjuvant radiotherapy as this reduces
the likelihood of local recurrence

Rate question:

Next question

Comment on this question

All contents of this site are 2012 E-Medical Revision Ltd

http://www.emrcs.com/question/question.php?q=0

Terms and Conditions

Privacy policy

2/2

13/07/2015

Reference ranges

Question 1 of 154

Next

A 56 year old man is diagnosed with an abdominal aortic aneurysm and undergoes a CT
scan to asses the size of the aorta. During the course of his investigations a lesion of the
adrenal gland is identified. It measures 1.5 cm in diameter and the gland is otherwise normal.
What is the most likely diagnosis?

Question stats

End and review

Score: 0%

5.1%

12.1%

23.2%

15.6%

44%

44% of users answered this


question correctly

A. Adrenal gland metastasis


B. Adrenal gland arterio-venous malformation
C. Adrenal cyst

Search

D. Phaeochromocytoma
E. Adrenal cortical adenoma

Go

Next question

25% of all adrenal lesions >4cm in diameter are malignant

Theme from 2011 Exam


Incidentalomas of the adrenal gland are common and represent the most likely lesion in this
scenario. Clearly the other lesions are all possibilities but are unlikely.
Adrenal lesions- Incidental
Incidentaloma of the adrenal glands have become increasingly common as CT scanning of
the abdomen is widely undertaken. Prevalences range from 1.5-9% in autopsy studies.
Overall, 75% will be non functioning adenomas. However, a thorough diagnostic work up is
required to exclude a more significant lesion.
Investigation
Morning and midnight plasma cortisol measurements
Dexamethasone suppression test
24 hour urinary cortisol excretion
24 hour urinary excretion of catecholamines
Serum potassium, aldosterone and renin levels

Management
The risk of malignancy is related to the size of the lesion and 25% of all masses greater than
4cm will be malignant. Such lesions should usually be excised. Where a lesion is a suspected
metastatic deposit a biopsy may be considered. Smaller, innocent lesions are usually
followed up by serial CT scans at 6, 12 and 24 months.
Rate question:

Next question

Comment on this question

All contents of this site are 2012 E-Medical Revision Ltd

http://www.emrcs.com/question/question.php?q=0

Terms and Conditions

Privacy policy

1/1

13/07/2015

Reference ranges

Previous

Question 2 of 154

Next

A 22 year old man is undergoing an abdominal ultrasound scan as part of a series of


investigations for abdominal pain. The radiologist notes that there is evidence of splenic
atrophy. What is the most likely cause?

Question stats

End and review

Score: 50%

17.9%

32.1%

12%

23.1%

14.9%

1
2

A. Letterer-Siwe disease
32.1% of users answered this
question correctly

B. Coeliac disease
C. Malaria
D. Niemann-Pick disease

Search

E. Sarcoidosis

Go

Next question

Splenic atrophy may occur in coeliac disease together with the appearance of Howell-Jolly
bodies in erythrocytes. Letterer - Siwe disease is a form of Histiocytosis X in which
macrophages proliferate.
Spleen
The spleen is located in the left upper quadrant of the abdomen and its size can vary
depending upon the amount of blood it contains. The typical adult spleen is 12.5cm long and
7.5cm wide. The usual weight of the adult spleen is 150g.
The exact position of the spleen can vary with respiratory activity, posture and the state of
surrounding viscera. It usually lies obliquely with its long axis aligned to the 9th, 10th and
11th ribs. It is separated from these ribs by both diaphragm and pleural cavity. The normal
spleen is not palpable.
The shape of the spleen is influenced by the state of the colon and stomach. Gastric
distension will cause the spleen to resemble the shape of an orange segment. Colonic
distension will cause it to become more tetrahedral.
The spleen is almost entirely covered by peritoneum, which adheres firmly to its capsule.
Recesses of the greater sac separate it from the stomach and kidney. It develops from the
upper dorsal mesogastrium, remaining connected to the posterior abdominal wall and
stomach by two folds of peritoneum; the lienorenal ligament and gastrosplenic ligament. The
lienorenal ligament is derived from peritoneum where the wall of the general peritoneum
meets the omental bursa between the left kidney and spleen; the splenic vessels lie in its
layers. The gastrosplenic ligament also has two layers, formed by the meeting of the walls of
the greater sac and omental bursa between spleen and stomach, the short gastric and left
gastroepiploic branches of the splenic artery pass in its layers. Laterally, the spleen is in
contact with the phrenicocolic ligament.

Relations
Superiorly

Diaphragm

Anteriorly

Gastric impression

Posteriorly Kidney
Inferiorly

Colon

Hilum

Tail of pancreas and splenic vessels (splenic artery divides here, branches
pass to the w hite pulp transporting plasma)

Contents
White
pulp

Immune function. Contains central trabecular artery. The germinal centres are
supplied by arterioles called penicilliary radicles.

Red
pulp

Filters abnormal red blood cells.

Function
Filtration of abnormal blood cells and foreign bodies such as bacteria.
Immunity: IgM. Production of properdin, and tuftsin which help target fungi and bacteria
for phagocytosis.
Haematopoiesis: up to 5th month gestation or in haematological disorders.
Pooling: storage of 40% platelets.
Iron reutilisation
Storage monocytes

Disorders of the spleen

http://www.emrcs.com/question/question.php?q=0

1/2

13/07/2015

Massive splenomegaly
Myelofibrosis
Chronic myeloid leukaemia
Visceral leishmaniasis (kala-azar)
Malaria
Gaucher's syndrome

Other causes (as above plus)


Portal hypertension e.g. secondary to cirrhosis
Lymphoproliferative disease e.g. CLL, Hodgkin's
Haemolytic anaemia
Infection: hepatitis, glandular fever
Infective endocarditis
Sickle-cell*, thalassaemia
Rheumatoid arthritis (Felty's syndrome)

*the majority of adult patients with sickle-cell will have an atrophied spleen due to repeated
infarction
Rate question:

Next question

Comment on this question

All contents of this site are 2012 E-Medical Revision Ltd

http://www.emrcs.com/question/question.php?q=0

Terms and Conditions

Privacy policy

2/2

13/07/2015

Reference ranges

Previous

Question 3 of 154

Next

Question stats

End and review

Score: 66.7%
1

11.7%

10.1%

22.2%

A. They are tumours of chromaffin cells in the adrenal medulla.

8.4%

B. They are bilateral in 10% of cases.

47.6%

C. When located in an extra adrenal location have a higher incidence of


malignancy.

47.6% of users answered this


question correctly

Which statement relating to phaeochromocytoma is untrue?

D. May be associated with an elevated urinary VMA.


E. Up to 40% may have a blood pressure within the normal range.
Search
Next question

Go

Normotension is seen in around 10% cases. The remainder show a degree of hypertension.
Phaeochromocytoma and adrenal lesions
Phaeochromocytoma
Neuroendocrine tumour of the chromaffin cells of the adrenal medulla. Hypertension and
hyperglycaemia are often found.
10% of cases are bilateral.
10% occur in children.
11% are malignant (higher when tumour is located outside the adrenal).
10% will not be hypertensive.

Familial cases are usually linked to the Multiple endocrine neoplasia syndromes (considered
under its own heading).
Most tumours are unilateral (often right sided) and smaller than 10cm.
Diagnosis
Urine analysis of vanillymandelic acid (VMA) is often used (false positives may occur e.g. in
patients eating vanilla ice cream!)
Blood testing for plasma metanephrine levels.
CT and MRI scanning are both used to localise the lesion.
Treatment
Patients require medical therapy first. An irreversible alpha adrenoreceptor blocker should
be given, although minority may prefer reversible blockade(1). Labetolol may be coadministered for cardiac chronotropic control. Isolated beta blockade should not be
considered as it will lead to unopposed alpha activity.
These patients are often volume depleted and will often require moderate volumes of intra
venous normal saline perioperatively.
Once medically optimised the phaeochromocytoma should be removed. Most
adrenalectomies can now be performed using a laparoscopic approach(2). The adrenals are
highly vascular structures and removal can be complicated by catastrophic haemorrhage in
the hands of the inexperienced. This is particularly true of right sided resections where the
IVC is perilously close. Should the IVC be damaged a laparotomy will be necessary and the
defect enclosed within a Satinsky style vascular clamp and the defect closed with prolene
sutures. Attempting to interfere with the IVC using any instruments other than vascular
clamps will result in vessel trauma and make a bad situation much worse.
Incidental adrenal lesions
Adrenal lesions may be identified on CT scanning performed for other reasons(3). Factors
suggesting benign disease on CT include(4):
Size less than 3cm
Homogeneous texture
Lipid rich tissue
Thin wall to lesion

All patients with incidental lesions should be managed jointly with an endocrinologist and full
work up as described above. Patients with functioning lesions or those with adverse
radiological features (Particularly size >3cm) should proceed to surgery.

http://www.emrcs.com/question/question.php?q=0

1/2

13/07/2015

References
1. Weingarten TN, Cata JP, O'Hara JF, Prybilla DJ, Pike TL, Thompson GB, et al.
Comparison of two preoperative medical management strategies for laparoscopic resection
of pheochromocytoma. Urology. 2010 Aug;76(2):508 e6-11.
2. Nguyen PH, Keller JE, Novitsky YW, Heniford BT, Kercher KW. Laparoscopic approach to
adrenalectomy: review of perioperative outcomes in a single center. Am Surg. 2011
May;77(5):592-6.
3. Ng VW, Ma RC, So WY, Choi KC, Kong AP, Cockram CS, et al. Evaluation of functional
and malignant adrenal incidentalomas. Arch Intern Med. 2010 Dec 13;170(22):2017-20.
4. Muth A, Hammarstedt L, Hellstrom M, Sigurjonsdottir HA, Almqvist E, Wangberg B. Cohort
study of patients with adrenal lesions discovered incidentally. Br J Surg. 2011 May 27.
Rate question:

Next question

Comment on this question

All contents of this site are 2012 E-Medical Revision Ltd

http://www.emrcs.com/question/question.php?q=0

Terms and Conditions

Privacy policy

2/2

13/07/2015

Reference ranges

Previous

Question 4 of 154

Next

A 69 year old man presents with a purple lesion on his forearm. It is excised and an a 3 cm
Merkel cell tumour is diagnosed. Which of the following statements relating to this diagnosis
is false?

Question stats

End and review

Score: 50%
1

16.6%

12.9%

13.5%

32.1%

25%

A. He should undergo a sentinel lymph node biopsy.


32.1% of users answered this
question correctly

B. Lymphovascular invasion is typically seen histologically


C. They are more common in immunosupressed patients
D. Histologically they may resemble pyogenic granuloma

Search

E. They are associated with visceral metastasis

Go

Next question

Merkel cell tumours are rare cutaneous tumours. Histologically they consist of sheets and
nodules of hyperchromatic epithelial cells, with high rates of mitosis and apoptosis. As such
they are relatively easy to distinguish from pyogenic granuloma which has no features of
malignancy and would not show lymphovascular invasion.
Merkel cell tumours of the skin

Rare but aggressive tumour.


Develops from intra epidermal Merkel cells.
Usually presents on elderly, sun damaged skin. The periorbital area is the commonest
site.
Histologically these tumours appear within the dermis and subcutis. The lesions
consist of sheets and nodules of small hyperchromatic epithelial cells with high rates of
mitosis and apoptosis. Lymphovascular invasion is commonly seen.
Pre-existing infection with Merkel Cell Polyomavirus is seen in 80% cases.

Treatment
Surgical excision is first line. Margins of 1cm are required. Lesions >10mm in diameter should
undergo sentinel lymph node biopsy. Adjuvant radiotherapy is often given to reduce the risk
of local recurrence.
Prognosis
With lymph node metastasis 5 year survival is 50% or less.
Small lesions without nodal spread are usually associated with a 5 year survival of
80%.

Skin biopsy demonstrating a Merkel Cell cancer. Note the hyperchromatic epithelial cells and
high mitotic rate

http://www.emrcs.com/question/question.php?q=0

1/2

13/07/2015

Image sourced from Wikipedia

Rate question:

Next question

Comment on this question

All contents of this site are 2012 E-Medical Revision Ltd

http://www.emrcs.com/question/question.php?q=0

Terms and Conditions

Privacy policy

2/2

13/07/2015

Reference ranges

Previous

Question 5 of 154

Next

A 58 year old lady undergoes a screening mammogram and appearances are suspicious for
ductal carcinoma in situ. A stereotactic core biopsy is performed. If ductal carcinoma in situ is
to be diagnosed, which of the following pathological features must not be present?

Question stats

End and review

Score: 40%
1

9.2%

10.6%

10.3%

13.2%

56.7%

A. Nuclear pleomorphism
56.7% of users answered this
question correctly

B. Coarse chromatin
C. Abnormal mitoses
D. Angiogenesis

Search

E. Dysplastic cells infiltrating the suspensory ligaments of the breast

Go

Next question

The presence of invasion is a hallmark of invasive disease and thus would not be a feature
of DCIS. Angiogenesis may occur in association with high grade DCIS.
Histopathology of malignancy

Abnormal tissue architecture


Coarse chromatin
Invasion of basement membrane*
Abnormal mitoses
Angiogenesis
De-differentiation
Areas of necrosis
Nuclear pleomorphism

*= Those features that distinguish invasive malignancy from in situ disease


Rate question:

Next question

Comment on this question

All contents of this site are 2012 E-Medical Revision Ltd

http://www.emrcs.com/question/question.php?q=0

Terms and Conditions

Privacy policy

1/1

13/07/2015

Reference ranges

Previous

Question 6 of 154

Next

Question stats

End and review

Score: 33.3%
1

7.3%

37.3%

10%

A. Plasma leakage into interstitial space

13.5%

B. Absolute polycythaemia

31.9%

C. Increased haematocrit

37.3% of users answered this


question correctly

Which of the following does not occur as a pathological response to extensive burns?

D. Keratinocyte migration during healing

E. Cardiac output reduction by 50% in first 30 minutes


Search
Next question
Go

Haemolysis is the main pathological response.


Burns pathology
Extensive burns
Haemolysis due to damage of erythrocytes by heat and microangiopathy
Loss of capillary membrane integrity causing plasma leakage into interstitial space
Extravasation of fluids from the burn site causing hypovolaemic shock (up to 48h after
injury)- decreased blood volume and increased haematocrit
Protein loss
Secondary infection e.g. Staphylococcus aureus
ARDS
Risk of Curlings ulcer (acute peptic stress ulcers)
Danger of full thickness circumferential burns in an extremity as these may develop
compartment syndrome

Healing
Superficial burns: keratinocytes migrate to form a new layer over the burn site
Full thickness burns: dermal scarring. Usually need keratinocytes from skin grafts to
provide optimal coverage.

Rate question:

Next question

Comment on this question

All contents of this site are 2012 E-Medical Revision Ltd

http://www.emrcs.com/question/question.php?q=0

Terms and Conditions

Privacy policy

1/1

13/07/2015

Reference ranges

Previous1 / 3

Question 7-9 of 154

Next

Question stats

End and review

Score: 33.3%

Average score for registered users:

1
2

Theme: Chest pain


A. Pulmonary embolism

53.5%

46.6%

82.2%

B. Acute exacerbation asthma

3
4
5

C. Physiological

D. Mitral valve stenosis

Search

E. Aortic dissection

7-9 1 / 3
Go

F. Mitral regurgitation
G. Bronchopneumonia
H. Tuberculosis
I. None of the above
What is the most likely diagnosis for the scenario given? Each option may be used once,
more than once or not at all.

7.

A 28 year old Indian woman, who is 18 weeks pregnant, presents with increasing
shortness of breath, chest pain and coughing clear sputum. She is apyrexial,
blood pressure is 140/80 mmHg, heart rate 130 bpm and saturations 94% on
15L oxygen. On examination there is a mid diastolic murmur, there are bibasal
crepitations and mild pedal oedema. She suddenly deteriorates and has a
respiratory arrest. Her chest x-ray shows a whiteout of both of her lungs.
Mitral valve stenosis
Mitral stenosis is the commonest cause of cardiac abnormality occurring in
pregnant women. Mitral stenosis is becoming less common in the UK population,
however should be considered in women from countries where there is a higher
incidence of rheumatic heart disease. Mitral stenosis causes a mid diastolic
murmur which may be difficult to auscultate unless the patient is placed into the
left lateral position. These patients are at risk of atrial fibrillation (up to 40%),
which can also contribute to rapid decompensation. Physiological changes in
pregnancy may cause an otherwise asymptomatic patient to suddenly
deteriorate. Balloon valvuloplasty is the treatment of choice.

8.

A 28 year old woman, who is 30 weeks pregnant, presents with sudden onset
chest pain associated with loss of consciousness. Her blood pressure is 170/90
mmHg, saturations on 15L oxygen 93%, heart rate 120 bpm and she is
apyrexial. On examination there is an early diastolic murmur, occasional bibasal
creptitations and mild peal oedema. An ECG shows ST elevation in leads II, III
and aVF.
You answered Mitral regurgitation
The correct answer is Aortic dissection
Aortic dissection is associated with the 3rd trimester of pregnancy, connective
tissue disorders (Marfan's, Ehlers- Danlos) and bicuspid valve. Patients may
complain of a tearing chest pain or syncope. Clinically they may be
hypertensive. The right coronary artery may become involved in the dissection,
causing myocardial infarct in up to 2% cases (hence ST elevation in the inferior
leads). An aortic regurgitant murmur may be auscultated.

9.

A 28 year old woman, who is 18 weeks pregnant, presents with sudden chest
pain. Her blood pressure is 150/70 mmHg, saturations are 92% on 15L oxygen
and her heart rate is 130 bpm. There are no murmurs and her chest is clear.
There is signs of thrombophlebitis in the left leg.
You answered Tuberculosis
The correct answer is Pulmonary embolism
Chest pain, hypoxia and clear chest on auscultation in pregnancy should lead to
a high suspicion of pulmonary embolism.

Next question

Theme question in September 2011 exam


Pregnant women can decompensate rapidly from cardiac compromise.

http://www.emrcs.com/question/question.php?q=0

1/2

13/07/2015

Chest pain in pregnancy

Aortic dissection
Predisposing factors in pregnancy are hypertension, congenital heart disease and
Marfan's syndrome
Mainly Stanford type A dissections
Sudden tearing chest pain, transient syncope
Patient may be cold and clammy, hypertensive and have an aortic regurgitation
murmur
Involvement of the right coronary artery may cause inferior myocardial infarction

Surgical management
Gestational
timeframe

Management

< 28/40

Aortic repair with the fetus kept in utero

28-32/40

Dependent on fetal condition

> 32/40

Primary Cesarean section followed by aortic repair at the same


operation

Mitral stenosis
Most cases associated with rheumatic heart disease
Becoming less common in British women; suspect in Immigrant women
Commonest cardiac condition in pregnancy
Commonly associated with mortality
Valve surgery; balloon valvuloplasty preferable

Pulmonary embolism
Leading cause of mortality in pregnancy
Half dose scintigraphy; CT chest if underlying lung disease, should aid diagnosis
Treatment with low molecular weight heparin throughout pregnancy and 4-6 weeks
after childbirth
Warfarin is contra indicated in pregnancy

References
1. Bates S.M. and Ginsberg J.S. How we manage venous thromboembolism during
pregnancy. Blood 2002 (100): 3470-3478.
2. Scarsbrook A.Fand Gleeson V. Investigating suspected pulmonary embolism in
pregnancy. BMJ 2007 (326) : 1135 doi: 10.1136/bmj.7399.1135.
3. Morley C. A. and Lim B. A. Lesson of the Week: The risks of delay in diagnosis of
breathlessness in pregnancy. BMJ 1995 (311) : 1083.
Rate question:

Next question

Comment on this question

All contents of this site are 2012 E-Medical Revision Ltd

http://www.emrcs.com/question/question.php?q=0

Terms and Conditions

Privacy policy

2/2

13/07/2015

Reference ranges

Previous

Question 10 of 154

Next

A 67 year old man is investigated for biliary colic and a 4.8 cm abdominal aortic aneurysm is
identified. Which of the following statements relating to this condition is untrue?

A. The wall will be composed of dense fibrous tissue only


B. The majority are located inferior to the renal arteries

Question stats

Score: 40%
54.1%

12.7%

9%

12.2%

12%

D. He should initially be managed by a process of active surveillance


E. Aortoduodenal fistula is a recognised complication following repair.
Next question

54.1% of users answered this


question correctly

C. They occur most often in current or former smokers

End and review

6
7-9 1 / 3
10

Search
Go

They are true aneurysms and have all 3 layers of arterial wall.
Abdominal aorta aneurysm

Abdominal aortic aneurysms are a common problem in vascular surgery.


They may occur as either true or false aneurysm. With the former all 3 layers of the
arterial wall are involved, in the latter only a single layer of fibrous tissue forms the
aneurysm wall.
True abdominal aortic aneurysms have an approximate incidence of 0.06 per 1000
people. They are commonest in elderly men and for this reason the UK is now
introducing the aneurysm screening program with the aim of performing an abdominal
aortic ultrasound measurement in all men aged 65 years.

Causes
Several different groups of patients suffer from aneurysmal disease.
The commonest group is those who suffer from standard arterial disease, i.e. Those
who are hypertensive, have diabetes and have been or are smokers.
Other patients such as those suffering from connective tissue diseases such as
Marfan's may also develop aneurysms. In patients with abdominal aortic aneurysms
the extracellular matrix becomes disrupted with a change in the balance of collagen
and elastic fibres.

Management
Most abdominal aortic aneurysms are an incidental finding.
Symptoms most often relate to rupture or impending rupture.
20% rupture anteriorly into the peritoneal cavity. Very poor prognosis.
80% rupture posteriorly into the retroperitoneal space
The risk of rupture is related to aneurysm size, only 2% of aneurysms measuring less
than 4cm in diameter will rupture over a 5 year period. This contrasts with 75% of
aneurysms measuring over 7cm in diameter.
This is well explained by Laplaces' law which relates size to transmural pressure.
For this reason most vascular surgeons will subject patients with an aneurysm size of
5cm or greater to CT scanning of the chest, abdomen and pelvis with the aim of
delineating anatomy and planning treatment. Depending upon co-morbidities, surgery
is generally offered once the aneurysm is between 5.5cm and 6cm.

A CT reconstruction showing an infrarenal abdominal aortic aneurysm. The walls of the sac
are calcified which may facilitate identification on plain x-rays

http://www.emrcs.com/question/question.php?q=0

1/3

13/07/2015

Image sourced from Wikipedia

Indications for surgery


Symptomatic aneurysms (80% annual mortality if untreated)
Increasing size above 5.5cm if asymptomatic
Rupture (100% mortality without surgery)

Surgical procedures
Abdominal aortic aneurysm repair
Procedure:
GA
Invasive monitoring (A-line, CVP, catheter)
Incision: Midline or transverse
Bowel and distal duodenum mobilised to access aorta.
Aneurysm neck and base dissected out and prepared for cross clamp
Systemic heparinisation
Cross clamp (proximal first)
Longitudinal aortotomy
Atherectomy
Deal with back bleeding from lumbar vessels and inferior mesenteric artery
Insert graft either tube or bifurcated depending upon anatomy
Suture using Prolene (3/0 for proximal , distal anastomosis suture varies according to site)
Clamps off: End tidal CO2 will rise owing to effects of reperfusion, at this point major risk of
myocardial events.
Haemostasis
Closure of aneurysm sac to minimise risk of aorto-enteric fistula
Closure: Loop 1 PDS or Prolene to abdominal wall
Skin- surgeons preference
Post operatively:
ITU (Almost all)
Greatest risk of complications following emergency repair
Complications: Embolic- gut and foot infarcts
Cardiac - owing to premorbid states, re-perfusion injury and effects of cross clamp
Wound problems
Later risks related to graft- infection and aorto-enteric fistula
Special groups
Supra renal AAA
These patients will require a supra renal clamp and this carries a far higher risk of
complications and risk of renal failure.
http://www.emrcs.com/question/question.php?q=0

2/3

13/07/2015

Ruptured AAA
Pre-operatively the management depends upon haemodynamic instability. In patients with
symptoms of rupture (typical pain, haemodynamic compromise and risk factors) then ideally
prompt laparotomy. In those with vague symptoms and haemodynamic stability the ideal test
is CT scan to determine whether rupture has occurred or not. Most common rupture site is
retroperitoneal 80%. These patients will tend to develop retroperitoneal haematoma. This
can be disrupted if Bp is allowed to rise too high so aim for Bp 100mmHg.
Operative details are similar to elective repair although surgery should be swift, blind rushing
often makes the situation worse. Plunging vascular clamps blindly into a pool of blood at the
aneurysm neck carries the risk of injury the vena cava that these patients do not withstand.
Occasionally a supracoeliac clamp is needed to effect temporary control, although leaving
this applied for more than 20 minutes tends to carry a dismal outcome.
EVAR
Increasingly patients are now being offered endovascular aortic aneurysm repair. This is
undertaken by surgeons and radiologists working jointly. The morphology of the aneurysm is
important and not all are suitable. Here is a typical list of those features favoring a suitable
aneurysm:
Long neck
Straight iliac vessels
Healthy groin vessels

Clearly few AAA patients possess the above and compromise has to be made. The use of
fenestrated grafts can allow supra renal AAA to be treated.
Procedure:
GA
Radiology or theatre
Bilateral groin incisions
Common femoral artery dissected out
Heparinisation
Arteriotomy and insertion of guide wire
Dilation of arteriotomy
Insertion of EVAR Device
Once in satisfactory position it is released
Arteriotomy closed once check angiogram shows good position and no endoleak
Complications:
Endoleaks depending upon site are either Type I or 2. These may necessitate reintervention and all EVAR patients require follow up . Details are not needed for MRCS.
References
A reasonable review is provided by:
Sakalihasan N, Limet R, Defawe O. Abdominal aortic aneurysm. Lancet 2005 (365):15771589
Rate question:

Next question

Comment on this question

All contents of this site are 2012 E-Medical Revision Ltd

http://www.emrcs.com/question/question.php?q=0

Terms and Conditions

Privacy policy

3/3

13/07/2015

Reference ranges

Previous

Question 11 of 154

Next

Question stats

End and review

Score: 45.5%
1

33.4%

12.1%

27.1%

A. It may induce necrosis of cells with non repairable DNA damage

10.3%

B. It is affected in Li Fraumeni syndrome

17.1%

C. It can induce DNA repair

33.4% of users answered this


question correctly

Which of the following statements in relation to the p53 tumour suppressor protein is false?

D. It can halt the cell cycle

6
7-9 1 / 3
10

E. It may inhibit angiogenesis

11

Search
Next question
Go

When DNA cannot be repaired it will induce cellular apoptosis (not necrosis)
Genetics and surgical disease
Some of the more commonly occurring genetic conditions occurring in surgical patients are
presented here.
Li-Fraumeni Syndrome
Autosomal dominant
Consists of germline mutations to p53 tumour suppressor gene
High incidence of malignancies particularly sarcomas and leukaemias
Diagnosed when:

*Individual develops sarcoma under 45 years


*First degree relative diagnosed with any cancer below age 45 years and another family
member develops malignancy under 45 years or sarcoma at any age
BRCA 1 and 2
Carried on chromosome 17 (BRCA 1) and Chromosome 13 (BRCA 2)
Linked to developing breast cancer (60%) risk.
Associated risk of developing ovarian cancer (55% with BRCA 1 and 25% with BRCA
2).

Lynch Syndrome
Autosomal dominant
Develop colonic cancer and endometrial cancer at young age
80% of affected individuals will get colonic and/ or endometrial cancer
High risk individuals may be identified using the Amsterdam criteria

Amsterdam criteria
Three or more family members with a confirmed diagnosis of colorectal cancer, one of whom
is a first degree (parent, child, sibling) relative of the other two.
Two successive affected generations.
One or more colon cancers diagnosed under age 50 years.
Familial adenomatous polyposis (FAP) has been excluded.
Gardners syndrome
Autosomal dominant familial colorectal polyposis
Multiple colonic polyps
Extra colonic diseases include: skull osteoma, thyroid cancer and epidermoid cysts
Desmoid tumours are seen in 15%
Mutation of APC gene located on chromosome 5
Due to colonic polyps most patients will undergo colectomy to reduce risk of colorectal
cancer
Now considered a variant of familial adenomatous polyposis coli

Rate question:

Next question

Comment on this question

All contents of this site are 2012 E-Medical Revision Ltd

http://www.emrcs.com/question/question.php?q=0

Terms and Conditions

Privacy policy

1/2

13/07/2015

http://www.emrcs.com/question/question.php?q=0

2/2

13/07/2015

Reference ranges

Previous

Question 12 of 154

Next

Question stats

End and review

Score: 41.7%
1

50.2%

12.5%

18.8%

A. Squamous cells

10%

B. Goblet cells

8.5%

C. Columnar cells

50.2% of users answered this


question correctly

Which of the following cell types is most likely to be identified in the wall of a fistula in ano?

D. Ciliated columnar cells

6
7-9 1 / 3
10

E. None of the above

11

Search
Next question

12
Go

A fistula is an abnormal connection between two epithelial lined surfaces, in the case of a
fistula in ano it will be lined by squamous cells.
Fistulas

A fistula is defined as an abnormal connection between two epithelial surfaces.


There are many types ranging from Branchial fistulae in the neck to entero-cutaneous
fistulae abdominally.
In general surgical practice the abdominal cavity generates the majority and most of
these arise from diverticular disease and Crohn's.
As a general rule all fistulae will resolve spontaneously as long as there is no distal
obstruction. This is particularly true of intestinal fistulae.

The four types of fistulae are:


Enterocutaneous
These link the intestine to the skin. They may be high (>500ml) or low output (<250ml)
depending upon source. Duodenal /jejunal fistulae will tend to produce high volume,
electrolyte rich secretions which can lead to severe excoriation of the skin. Colo-cutaneous
fistulae will tend to leak faeculent material. Both fistulae may result from the spontaneous
rupture of an abscess cavity onto the skin (such as following perianal abscess drainage) or
may occur as a result of iatrogenic input. In some cases it may even be surgically desirable
e.g. mucous fistula following sub total colectomy for colitis.
Suspect if there is excess fluid in the drain.
Enteroenteric or Enterocolic
This is a fistula that involves the large or small intestine. They may originate in a similar
manner to enterocutaneous fistulae. A particular problem with this fistula type is that bacterial
overgrowth may precipitate malabsorption syndromes. This may be particularly serious in
inflammatory bowel disease.
Enterovaginal
Aetiology as above.
Enterovesicular
This type of fistula goes to the bladder. These fistulas may result in frequent urinary tract
infections, or the passage of gas from the urethra during urination.
Management
Some rules relating to fistula management:
They will heal provided there is no underlying inflammatory bowel disease and no
distal obstruction, so conservative measures may be the best option
Where there is skin involvement, protect the overlying skin, often using a well fitted
stoma bag- skin damage is difficult to treat
A high output fistula may be rendered more easily managed by the use of octreotide,
this will tend to reduce the volume of pancreatic secretions.
Nutritional complications are common especially with high fistula (e.g. high jejunal or
duodenal) these may necessitate the use of TPN to provide nutritional support
together with the concomitant use of octreotide to reduce volume and protect skin.
When managing perianal fistulae surgeons should avoid probing the fistula where
acute inflammation is present, this almost always worsens outcomes.
When perianal fistulae occur secondary to Crohn's disease the best management
option is often to drain acute sepsis and maintain that drainage through the judicious
use of setons whilst medical management is implemented.
Always attempt to delineate the fistula anatomy, for abscesses and fistulae that have
an intra abdominal source the use of barium and CT studies should show a track. For
http://www.emrcs.com/question/question.php?q=0

1/2

13/07/2015

perianal fistulae surgeons should recall Goodsall's rule in relation to internal and
external openings.

Rate question:

Next question

Comment on this question

All contents of this site are 2012 E-Medical Revision Ltd

http://www.emrcs.com/question/question.php?q=0

Terms and Conditions

Privacy policy

2/2

13/07/2015

Reference ranges

Previous

Question 13 of 154

Next

A 22 year old man is referred to the surgical clinic. He has been complaining of varicose
veins for many years. On examination he has extensive varicosities of the right leg, there are
areas of marked port wine staining. The saphenofemoral junction is competent on doppler
assessment. The most likely underlying diagnosis is:

Question stats

Score: 46.2%
1

8.5%

46.3%

11.9%

25.7%

7.6%

46.3% of users answered this


question correctly

A. Deep vein thrombosis

End and review

B. Klippel-Trenaunay syndrome

6
7-9 1 / 3
10

C. Varicose veins due to sapheno-popliteal junction incompetence


11

Search

D. Sturge - Weber syndrome

12

E. Angiosarcoma

Go

13
Next question

Sturge - Weber syndrome is a an arteriovenous malformation affecting the face and CNS,
the peripheral vessels are not affected. Simple varicose veins should not typically be
associated with port wine staining, nor should a DVT or angiosarcoma.
Klippel-Trenaunay syndrome
Klippel-Trenaunay-Weber syndrome generally affects a single extremity, although cases of
multiple affected limbs have been reported. The leg is the most common site followed by the
arms, the trunk, and rarely the head and the neck
Signs and symptoms
The birth defect is diagnosed by the presence of a combination of these symptoms:
One or more distinctive port-wine stains with sharp borders
Varicose veins
Hypertrophy of bony and soft tissues, that may lead to local gigantism or shrinking.
An improperly developed lymphatic system
In some cases, port-wine stains (capillary port wine type) may be absent. Such cases are
very rare and may be classified as "atypical Klippel-Trenaunay syndrome".
KTS can either affect blood vessels, lymph vessels, or both. The condition most commonly
presents with a mixture of the two. Those with venous involvement experience increased pain
and complications.
Rate question:

Next question

Comment on this question

All contents of this site are 2012 E-Medical Revision Ltd

http://www.emrcs.com/question/question.php?q=0

Terms and Conditions

Privacy policy

1/1

13/07/2015

Reference ranges

Previous

Question 14 of 154

Next

A 68 year old man presents with an ulcerated lesion on his right cheek. It is excised and on
histological assessment a squamous cell carcinoma is diagnosed. It measures 25mm in
diameter and is 4mm deep. Which of the following statements relating to this condition is
false?

Question stats

Score: 42.9%
1

25.2%

15.5%

27%

13.8%

18.5%

27% of users answered this


question correctly

A. In this particular case margins of at least 6mm are required

End and review

B. Use of cryosurgery to treat this patients lesion would have been unsafe

6
7-9 1 / 3
10

C. Use of radiotherapy to treat this lesion would have been unsafe


11

Search

D. This patients local recurrence rate may approach 15%

12

E. The disease usually spreads via lymphatics

Go

13
Next question

14

Poor prognostic factors in SCC:


Size >20mm (local recurrence rate of up to 15%)
Depth greater than 4mm (risk of metastasis up to 30%)

This man has an SCC with significant risk of metastasis. Although cryotherapy may be used
to treat SCC it would be most unsafe in this setting as the lesion extends deeply. However,
radiotherapy is a safe treatment modality for SCC and may be used in selected cases. It is
unwise to use radiotherapy in areas prone to radionecrosis e.g. the nose.
Squamous cell carcinoma of the skin

Second most common skin malignancy


Derived from epidermal keratinocytes
Commonest in fair skinned individuals in sun exposed sites
May occur in perianal and genital skin especially in association with Human Papilloma
Virus 16 and 18 infections.

Groups at high risk


Renal transplant and on immunosuppression
Individuals with HIV
Those who have received psoralen UVA therapy
Chronic wounds (Marjolins ulcer)
Xeroderma pigmentosum
Oculocutaneous albinism
Prognosis
Good Prognosis

Poor prognosis

Well differentiated tumours

Poorly differentiated tumours

<20mm diameter

>20mm in diameter

<2mm deep

>4mm deep

No associated diseases

Immunosupression for whatever reason

Treatment
Surgical excision with 4mm margins if lesion <20mm in diameter. If tumour >20mm then
margins should be 6mm.
Squamous cell carcinomas of the skin typically arise in areas of sun exposure as shown here

http://www.emrcs.com/question/question.php?q=0

1/2

13/07/2015

Image sourced from Wikipedia

References
The British Association of Dermatology provides guidelines for the diagnosis and treatment
of SCC.
http://www. bad.org.uk

Rate question:

Next question

Comment on this question

All contents of this site are 2012 E-Medical Revision Ltd

http://www.emrcs.com/question/question.php?q=0

Terms and Conditions

Privacy policy

2/2

13/07/2015

Reference ranges

Previous

Question 15 of 154

Next

A 23 year old man presents with weight loss fatigue and lymphadenopathy. He is diagnosed
with tuberculosis. Which of the following processes most closely matches the underlying
pathological process?

Question stats

Score: 46.7%
1

6.9%

10.4%

13.7%

59.6%

9.4%

A. Type 1 hypersensitivity reaction


59.6% of users answered this
question correctly

B. Type 2 hypersensitivity reaction

End and review

C. Type 3 hypersensitivity reaction

6
7-9 1 / 3
10

D. Type 4 hypersensitivity reaction

11

Search

E. None of the above

12
Go

13

Next question

14
15

Granulomas (which occur in tuberculosis) are a feature of Type 4 hypersensitivity reactions.


Hypersensitivity reactions
The Gell and Coombs classification divides hypersensitivity reactions into 4 types
Type I

Type II

Type III

Type IV

Description

Anaphylactic

Cytotoxic

Immune
complex

Delayed type

Mediator

IgE

IgG, IgM

IgG, Ig A,
IgM

T-cells

Antigen

Exogenous

Cell surface

Soluble

Tissues

Response
time

Minutes

Hours

Hours

2-3 days

Examples

Asthma
Hay fever

Autoimmune haemolytic
anaemia
Pemphigus
Goodpasture's

Serum
sickness
SLE
Aspergillosis

Graft versus host


disease
Contact dermatitis

Rate question:

Next question

Comment on this question

All contents of this site are 2012 E-Medical Revision Ltd

http://www.emrcs.com/question/question.php?q=0

Terms and Conditions

Privacy policy

1/1

13/07/2015

Reference ranges

Previous

Question 16 of 154

Next

A 73 year old man undergoes an emergency amputation for severe lower limb sepsis and
gangrene. Post operatively he develops disseminated intravascular coagulation. Which of
the following clotting factors will be most rapidly consumed in this process?

Question stats

Score: 43.8%
1

53.1%

10.3%

9.1%

15.3%

12.1%

A. Factor V and VIII


53.1% of users answered this
question correctly

B. Factor I

End and review

C. Factor I and III

6
7-9 1 / 3
10

D. Factor III and VII

11

Search

E. Factor VI and VIII

12
Go

13

Next question

14
15

D-I-S-S-E-M-I-N-A-T-E-D

16

D-Dx: D dimer
I-Immune complexes
S-Snakebite, shock, heatstroke
S-SLE
E-Eclampsia, HELLP syndrome
M-Massive tissue damage
I-Infections: viral and bacterial
N-Neoplasms
A-Acute promyelocytic leukemia
T-Tumor products: Tissue Factor (TF) and TF-like factors released by carcinomas of
pancreas, prostate, lung,
colon, stomach
E-Endotoxins (bacterial)
D-Dead fetus (retained)

DIC Will tend to consume factors five and eight intially (and platelets).
Disseminated intravascular coagulation
Simultaneous coagulation and haemorrhage caused by initially formation of thrombi which
consume clotting factors (factors 5,8) and platelets, ultimately leading to bleeding
Causes include:
Infection
Malignancy
Trauma e.g. major surgery, burns, shock, dissecting aortic aneurysm
Liver disease
Obstetric complications

Key points
Clinically bleeding is usually a dominant feature, bruising, ischaemia and organ failure
Blood tests: prolonged clotting times, thrombocytopenia, decreased fibrinogen,
increased fibrinogen degradation products
Treat the underlying cause and supportive management

Rate question:

Next question

Comment on this question

All contents of this site are 2012 E-Medical Revision Ltd

http://www.emrcs.com/question/question.php?q=0

Terms and Conditions

Privacy policy

1/1

13/07/2015

Reference ranges

Previous

Question 17 of 154

Next

A pregnant women suddenly develops bilateral leg swelling. Her mother and aunt were
troubled by the same problem. What is the most likely underlying abnormality?

A. Anti endomysial antibodies


B. Anti nuclear antibodies

Question stats

Score: 47.1%
1

14.8%

16.1%

44%

10.2%

15%

44% of users answered this


question correctly

C. Anti cardiolipin antibodies

End and review

D. Anti thyroid antibodies

6
7-9 1 / 3
10

E. Anti mitochondrial antibodies

11

Search

12
Next question

Go

13
14

Anti phospholipid syndrome= following antibodies


Lupus anticoagulant
Anti-cardiolipin
Anti-2-glycoprotein

15
16
17

Theme from September 2014 Exam


Antiphospholipid syndrome, is an autoimmune, hypercoagulable state caused by
antiphospholipid antibodies. APS provokes blood clots (thrombosis) in both arteries and
veins as well as pregnancy-related complications such as miscarriage, stillbirth, preterm
delivery, or severe preeclampsia.
The diagnostic criteria requires one clinical event, i.e. thrombosis or pregnancy complication,
and two positive blood tests spaced at least 3 months apart. These antibodies are: lupus
anticoagulant, anti-cardiolipin and anti-2-glycoprotein.
Antiphospholipid syndrome can be primary or secondary. Primary antiphospholipid syndrome
occurs in the absence of any other related disease. Secondary antiphospholipid syndrome
occurs with other autoimmune diseases, such as systemic lupus erythematosus (SLE). In
rare cases, APS leads to rapid organ failure due to generalised thrombosis; this is termed
"catastrophic antiphospholipid syndrome" (CAPS) and is associated with a high risk of death.
Antiphospholipid syndrome often requires treatment with anticoagulant medication such as
heparin to reduce the risk of further episodes of thrombosis and improve the prognosis of
pregnancy. Warfarin is not used during pregnancy because it can cross the placenta, unlike
heparin, and is teratogenic.
Hypercoagulability

Type of
thrombophilia

Features

Antithrombin deficiency

Antithrombin inactivates thrombin and factor XII a, XIa, IXa and Xa


Rare defect, inherited in autosomal dominant fashion
10x increase in risk of thrombotic events
Heparin may be ineffective because it works via antithrombin

Protein C and S
deficiency

These are natural anticoagulants (vitamin K dependent synthesis)


Protein C produced by liver
Protein S produced by liver, megakaryocytes, Leydig cells and
endothelial cells
Protein C and S bind to form activated complex which binds to factor
V
Deficiency accounts for up to 5% of thrombotic episodes

Factor V Leiden

Resistance to anticoagulant effect of activated protein C


May account for up to 20% or more of thrombotic episodes
Prevalence of 7% in Europe
Most common genetic defect accounting for DVT

Antiphospholipid
syndrome

Multi organ disease


Pregnancy involvement common
Arterial and venous thromboses
Either Lupus anticoagulant or Anti cardiolipin antibodies
APTT usually prolonged
Antibodies may be elevated following surgery, drugs or malignancy
Need anticoagulation with INR between 3 and 4

http://www.emrcs.com/question/question.php?q=0

1/2

13/07/2015

Rate question:

Next question

Comment on this question

All contents of this site are 2012 E-Medical Revision Ltd

http://www.emrcs.com/question/question.php?q=0

Terms and Conditions

Privacy policy

2/2

13/07/2015

Reference ranges

Question 18 of 154

Previous

Next

A 63 year old Japanese man presents with epigastric discomfort and iron deficiency
anaemia. He undergoes an upper GI endoscopy, where the following appearances are
found:

Question stats

End and review

Score: 44.4%
1

30.7%

36.2%

10.1%

14.7%

8.3%

36.2% of users answered this


question correctly

6
7-9 1 / 3
10
11

Search

12
Go

13
14
15
16
17
18

Image sourced from Wikipedia

The most likely diagnosis is:

A. Squamous cell carcinoma


B. Linitis plastica
C. Leiomyosarcoma
D. Gastric varices
E. None of the above
Next question

During upper GI endoscopy, a linitis plastica lesion may prevent gastric distension.

Linitis plastica produces a diffuse infiltrating lesion, the stomach is fibrotic and rigid and will
not typically distend. This may be described as a 'leather bottle stomach'. Diagnosis is made
with a combination of pathology examination with endoscopy, radiological or surgical
assessment. Pathologically signet-ring cell proliferation occurs.
At the present time clinical images do not form part of the MRCS Part A exam
Gastric cancer
Overview
There are 700,000 new cases of gastric cancer worldwide each year. It is most common in
Japan and less common in western countries. It is more common in men and incidence rises
with increasing age. The exact cause of many sporadic cancer is not known, however,
familial cases do occur in HNPCC families. In addition, smoking and smoked or preserved
foods increase the risk. Japanese migrants retain their increased risk (decreased in
subsequent generations). The distribution of the disease in western countries is changing
towards a more proximal location (perhaps due to rising obesity).
Pathology
There is some evidence of support a stepwise progression of the disease through intestinal
metaplasia progressing to atrophic gastritis and subsequent dysplasia, through to cancer.
The favoured staging system is TNM. The risk of lymph node involvement is related to size
and depth of invasion; early cancers confined to submucosa have a 20% incidence of lymph
node metastasis. Tumours of the gastro-oesophageal junction are classified as below:
Type
1

True oesophageal cancers and may be associated with Barrett's oesophagus.

Type
2

Carcinoma of the cardia, arising from cardiac type epithelium


or short segments with intestinal metaplasia at the oesophagogastric junction.

Type
3

Sub cardial cancers that spread across the junction. Involve similar nodal stations to
gastric cancer.

http://www.emrcs.com/question/question.php?q=0

1/3

13/07/2015

Groups for close endoscopic monitoring


Intestinal metaplasia of columnar type
Atrophic gastritis
Low to medium grade dysplasia
Patients who have previously undergone resections for benign peptic ulcer disease
(except highly selective vagotomy).

Referral to endoscopy
Patients of any age with
dyspepsia and any of the
following

Patients without
dyspepsia

Worsening dyspepsia

Chronic gastrointestinal bleeding

Dysphagia

Barretts oesophagus

Dysphagia

Unexplained
abdominal pain or
weight loss

Intestinal metaplasia

Weight loss

Vomiting

Dysplasia

Iron deficiency anaemia

Upper abdominal
mass

Atrophic gastritis

Upper abdominal mass

Jaundice

Patient aged over 55 years with


unexplained or persistent
dyspepsia

Upper GI endoscopy performed for dyspepsia. The addition of dye spraying (as shown in the
bottom right) may facilitate identification of smaller tumours

Image sourced from Wikipedia

Staging
CT scanning of the chest abdomen and pelvis is the routine first line staging
investigation in most centres.
Laparoscopy to identify occult peritoneal disease
PET CT (particularly for junctional tumours)

Treatment
Proximally sited disease greater than 5-10cm from the OG junction may be treated by
sub total gastrectomy
Total gastrectomy if tumour is <5cm from OG junction
For type 2 junctional tumours (extending into oesophagus) oesophagogastrectomy is
usual
Endoscopic sub mucosal resection may play a role in early gastric cancer confined to
the mucosa and perhaps the sub mucosa (this is debated)
Lymphadenectomy should be performed. A D2 lymphadenectomy is widely advocated
by the Japanese, the survival advantages of extended lymphadenectomy have been
debated. However, the overall recommendation is that a D2 nodal dissection be
undertaken.
Most patients will receive chemotherapy either pre or post operatively.

Prognosis
UK Data

http://www.emrcs.com/question/question.php?q=0

2/3

13/07/2015

Disease extent

Percentage 5 year survival

All RO resections

54%

Early gastric cancer

91%

Stage 1

87%

Stage 2

65%

Stage 3

18%

Operative procedure
Total Gastrectomy , lymphadenectomy and Roux en Y anastomosis
General anaesthesia
Prophylactic intravenous antibiotics
Incision: Rooftop.
Perform a thorough laparotomy to identify any occult disease.
Mobilise the left lobe of the liver off the diaphragm and place a large pack over it. Insert a
large self retaining retractor e.g. omnitract or Balfour (take time with this, the set up should
be perfect). Pack the small bowel away.
Begin by mobilising the omentum off the transverse colon.
Proceed to detach the short gastric vessels.
Mobilise the pylorus and divide it at least 2cm distally using a linear cutter stapling device.
Continue the dissection into the lesser sac taking the lesser omentum and left gastric artery
flush at its origin.
The lymph nodes should be removed en bloc with the specimen where possible.
Place 2 stay sutures either side of the distal oesophagus. Ask the anaesthetist to pull back
on the nasogastric tube. Divide the distal oesophagus and remove the stomach.
The oesphago jejunal anastomosis should be constructed. Identify the DJ flexure and bring a
loop of jejunum up to the oesophagus (to check it will reach). Divide the jejunum at this point.
Bring the divided jejunum either retrocolic or antecolic to the oesophagus. Anastamose the
oesophagus to the jejunum, using either interrupted 3/0 vicryl or a stapling device. Then
create the remainder of the Roux en Y reconstruction distally.
Place a jejunostomy feeding tube.
Wash out the abdomen and insert drains (usually the anastomosis and duodenal stump).
Help the anaesthetist insert the nasogastric tube (carefully!)
Close the abdomen and skin.
Enteral feeding may commence on the first post-operative day. However, most surgeons will
leave patients on free NG drainage for several days and keep them nil by mouth.

Rate question:

Next question

Comment on this question

All contents of this site are 2012 E-Medical Revision Ltd

http://www.emrcs.com/question/question.php?q=0

Terms and Conditions

Privacy policy

3/3

13/07/2015

Reference ranges

Previous1 / 3

Question 19-21 of 154

Next

Question stats

End and review

Score: 42.9%

Average score for registered users:

1
2

Theme: Genetics and cancer


A. Multiple endocrine neoplasia type I

19

61.8%

20

32.3%

21

56.3%

B. Multiple endocrine neoplasia type II

3
4
5

C. Gardner's syndrome

D. Lynch Syndrome

Search

E. Kartagener's syndrome

7-9 1 / 3
Go

10

F. Neurofibromatosis Type I

11

G. Neurofibromatosis Type II

12

Please select the most likely condition for the disease process described. Each option may
be used once, more than once or not at all.

13
14
15
16

19.

A 40 year old male is found to have multiple colonic polyps during a


colonoscopy. He mentions that he has extra teeth.

17
18
19-21 1 / 3

Gardner's syndrome
Gardner's syndrome is an AD disorder, characterised by: Colonic polyps,
supernumerary teeth, jaw osteomas, congenital hypertrophy of retinal pigment.
osteomas of the skull, thyroid cancer, epidermoid cysts, fibromas and
sebaceous cysts.
20.

A 10 year old boy who has learning difficulties, reports a difference in size
between his two legs.
You answered Neurofibromatosis Type II
The correct answer is Neurofibromatosis Type I
Neurofibromatosis type I. A hallmark finding is a plexiform neurofibroma, which
is a sheet of neurofibromatosis tissue which encases major nerves. In children
this attracts extra blood circulation, which can accelerate growth of the affected
limb.
Other features include:
Schwannoma, > 6
Cafe au lait spots, axillary freckling, Lisch nodules, Optic glioma. Meningiomas,
Glioma, or Schwannoma.

21.

A 22 year old is found to have bilateral acoustic neuromas.


You answered Neurofibromatosis Type I
The correct answer is Neurofibromatosis Type II
In NF2 bilateral acoustic neuromas are characteristic with a family history of
Neurofibroma,

Next question

Genetics and surgical disease

Some of the more commonly occurring genetic conditions occurring in surgical patients are
presented here.
Li-Fraumeni Syndrome
Autosomal dominant
Consists of germline mutations to p53 tumour suppressor gene
High incidence of malignancies particularly sarcomas and leukaemias
Diagnosed when:

*Individual develops sarcoma under 45 years


*First degree relative diagnosed with any cancer below age 45 years and another family
member develops malignancy under 45 years or sarcoma at any age

http://www.emrcs.com/question/question.php?q=0

1/2

13/07/2015

BRCA 1 and 2
Carried on chromosome 17 (BRCA 1) and Chromosome 13 (BRCA 2)
Linked to developing breast cancer (60%) risk.
Associated risk of developing ovarian cancer (55% with BRCA 1 and 25% with BRCA
2).

Lynch Syndrome
Autosomal dominant
Develop colonic cancer and endometrial cancer at young age
80% of affected individuals will get colonic and/ or endometrial cancer
High risk individuals may be identified using the Amsterdam criteria

Amsterdam criteria
Three or more family members with a confirmed diagnosis of colorectal cancer, one of whom
is a first degree (parent, child, sibling) relative of the other two.
Two successive affected generations.
One or more colon cancers diagnosed under age 50 years.
Familial adenomatous polyposis (FAP) has been excluded.
Gardners syndrome
Autosomal dominant familial colorectal polyposis
Multiple colonic polyps
Extra colonic diseases include: skull osteoma, thyroid cancer and epidermoid cysts
Desmoid tumours are seen in 15%
Mutation of APC gene located on chromosome 5
Due to colonic polyps most patients will undergo colectomy to reduce risk of colorectal
cancer
Now considered a variant of familial adenomatous polyposis coli

Rate question:

Next question

Comment on this question

All contents of this site are 2012 E-Medical Revision Ltd

http://www.emrcs.com/question/question.php?q=0

Terms and Conditions

Privacy policy

2/2

13/07/2015

Reference ranges

Question 22 of 154

Previous

Next

A 52 year old lady is referred to the breast clinic with symptoms of nipple discharge. The
discharge is usually thick and green. Which of the following statements relating to the most
likely underlying diagnosis is untrue?

Question stats

Score: 40.9%
1

11.9%

40.4%

26.8%

10%

11%

A. The majority of patients will be smokers


40.4% of users answered this
question correctly

B. Typically produces blood stained nipple discharge

End and review

C. It is not associated with increased risk of breast cancer

6
7-9 1 / 3
10

D. May result in development of mammary duct fistula


E. May require total duct excision (Hadfields operation) if it fails to resolve

11

Search

12
Go

Next question

13
14
15

Blood stained nipple discharge should always be investigated.


Nipple fluid cytology is generally unhelpful.

16
17

Discharge of this type of material is most likely to be due to duct ectasia. Green or brown
discharge is most common. Blood stained discharge should raise concern of intraductal
papilloma or cancer.

18
19-21 1 / 3
22

Nipple discharge
Causes of nipple discharge
Physiological

During breast feeding

Galactorrhoea

Commonest cause may be response to emotional events, drugs such


as histamine receptor antagonists are also implicated

Hyperprolactinaemia

Mammary duct
ectasia

Carcinoma

Intraductal
papilloma

Commonest type of pituitary tumour


Microadenomas <1cm in diameter
Macroadenomas >1cm in diameter
Pressure on optic chiasm may cause bitemporal hemianopia

Dilatation breast ducts.


Most common in menopausal women
Discharge typically thick and green in colour
Most common in smokers

Often blood stained


May be underlying mass or axillary lymphadenopathy

Commoner in younger patients


May cause blood stained discharge
There is usually no palpable lump

Assessment of patients
Examine breast and determine whether there is mass lesion present
All mass lesions should undergo Triple assessment.

Reporting of investigations
Where a mass lesion is suspected or investigations are requested these are prefixed using a
system that denotes the investigation type e.g. M for mammography, followed by a numerical
code as shown below:
1

No abnormality

Abnormality with benign features

Indeterminate probably benign

Indeterminate probably malignant

Malignant

http://www.emrcs.com/question/question.php?q=0

1/2

13/07/2015

Management of non malignant nipple discharge


Exclude endocrine disease
Nipple cytology unhelpful
Smoking cessation advice for duct ectasia
For duct ectasia with severe symptoms, total duct excision may be warranted.

Rate question:

Next question

Comment on this question

All contents of this site are 2012 E-Medical Revision Ltd

http://www.emrcs.com/question/question.php?q=0

Terms and Conditions

Privacy policy

2/2

13/07/2015

Reference ranges

Previous

Question 23 of 154

Next

Question stats

End and review

Score: 39.1%
1

7.7%

16%

59.9%

A. There is necrosis with putrefaction

9.2%

B. The causative pathogens may be detected on normal perineal skin

7.1%

C. Treatment with low dose penicillin is indicated

59.9% of users answered this


question correctly

Which of the following statements relating to gas gangrene is untrue?

D. Hyperbaric oxygen may be beneficial

6
7-9 1 / 3
10

E. Clostridium perfringens is a recognised cause

11

Search
Next question

12
Go

13

Rapid surgery and high dose antibiotics are indicated in the treatment of gas gangrene.
Meleney's Gangrene and Necrotising Fasciitis

14
15
16

Necrotising fasciitis
Advancing soft tissue infection associated with fascial necrosis
Uncommon, but can be fatal
In many cases there is underlying background immunosuppression e.g. Diabetes
Caused by polymicrobial flora (aerobic and anaerobic) and MRSA is seen increasingly
in cases of necrotising fasciitis
Streptococcus is the commonest organism in isolated pathogen infection (15%)

17
18
19-21 1 / 3
22
23

Meleneys gangrene
Meleneys is a similar principle but the infection is more superficially sited than
necrotising fasciitis and often confined to the trunk

Fournier gangrene
Necrotising fasciitis affecting the perineum
Polymicrobial with E-coli and Bacteroides acting in synergy

Clinical features
Fever
Pain
Cellulitis
Oedema
Induration
Numbness
Late findings
Purple/black skin discolouration
Blistering
Haemorrhagic bullae
Crepitus
Dirty Dishwater fluid discharge
Septic shock
A typical case of gas gangrene presenting late demonstrating some of the features
described above

http://www.emrcs.com/question/question.php?q=0

1/2

13/07/2015
Image sourced from gangrene" target="_blank" style = "font-

size:11px; color:#777;" >Wikipedia


Diagnosis is mainly clinical
Management
Radical surgical debridement forms the cornerstone of management
Sterile dressing is used to dress the wound
Reconstructive surgery is considered once the infection is completely treated

Reference
Hasham S, Matteucci P, Stanley PR, Hart NB. Necrotising fasciitis. BMJ 2005;330:830-833.
Rate question:

Next question

Comment on this question

All contents of this site are 2012 E-Medical Revision Ltd

http://www.emrcs.com/question/question.php?q=0

Terms and Conditions

Privacy policy

2/2

13/07/2015

Reference ranges

Previous

Question 24 of 154

Next

A 30 year old man presents with abdominal distension, a laparotomy is performed, at


operation the abdomen is filled with a large amount of gelatinous fluid. What is the most likely
underlying diagnosis?

Question stats

Score: 37.5%
1

9.2%

53.6%

7.7%

23.8%

5.7%

A. Infection with entamoeba histolytica


53.6% of users answered this
question correctly

B. Pseudomxyoma peritonei

End and review

C. Metastatic colonic cancer

6
7-9 1 / 3
10

D. Chylous ascites

11

Search

E. None of the above

12
Go

13

Next question

14
15

Pseudomyxoma is associated with the deposition of large amounts of gelatinous material.


The appendix is the commonest organ or origin.

16
17

Pseudomyxoma Peritonei

18
19-21 1 / 3

Rare mucinous tumour


Most commonly arising from the appendix (other abdominal viscera are also
recognised as primary sites)
Incidence of 1-2/1,000,000 per year
The disease is characterised by the accumulation of large amounts of mucinous
material in the abdominal cavity

22
23
24

Treatment
Is usually surgical and consists of cytoreductive surgery (and often peritonectomy c.f
Sugarbaker procedure) combined with intra peritoneal chemotherapy with mitomycin C.
Survival is related to the quality of primary treatment and in Sugarbakers own centre 5 year
survival rates of 75% have been quoted. Patients with disseminated intraperitoneal
malignancy from another source fare far worse.
In selected patients a second look laparotomy is advocated and some practice this routinely.
Rate question:

Next question

Comment on this question

All contents of this site are 2012 E-Medical Revision Ltd

http://www.emrcs.com/question/question.php?q=0

Terms and Conditions

Privacy policy

1/1

13/07/2015

Reference ranges

Previous

Question 25 of 154

Next

A 30 year old man is suspected of having appendicitis. At operation an inflamed Meckels


diverticulum is found. Which of the following vessels is responsible for the blood supply to a
Meckels diverticulum?

Question stats

Score: 36%
1

20.8%

53.7%

11.9%

8%

5.6%

A. Right colic artery


53.7% of users answered this
question correctly

B. Vitelline artery

End and review

C. Appendicular artery

6
7-9 1 / 3
10

D. Internal iliac artery

11

Search

E. External iliac artery

12
Go

13

Next question

14
15

The vitelline arteries supply a Meckels these are usually derived from the ileal arcades.

16

Meckel's diverticulum

17
18

Congenital abnormality resulting in incomplete obliteration of the vitello-intestinal duct


Normally, in the foetus, there is an attachment between the vitello-intestinal duct and
the yolk sac.This disappears at 6 weeks gestation.
The tip is free in majority of cases.
Associated with enterocystomas, umbilical sinuses, and omphaloileal fistulas.
Arterial supply: omphalomesenteric artery.
2% of population, 2 inches long, 2 feet from the ileocaecal valve.
Typically lined by ileal mucosa but ectopic gastric mucosa can occur, with the risk of
peptic ulceration. Pancreatic and jejunal mucosa can also occur.

19-21 1 / 3
22
23
24
25

Clinical
Normally asymptomatic and an incidental finding.
Complications are the result of obstruction, ectopic tissue, or inflammation.
Removal if narrow neck or symptomatic. Options are between wedge excision or formal
small bowel resection and anastomosis.

Rate question:

Next question

Comment on this question

All contents of this site are 2012 E-Medical Revision Ltd

http://www.emrcs.com/question/question.php?q=0

Terms and Conditions

Privacy policy

1/1

13/07/2015

Reference ranges

Previous

Question 26 of 154

Next

Question stats

End and review

Score: 34.6%
1

9.7%

14.6%

49.5%

A. Afro-Caribbean skin and keloid scarring

11%

B. Extensive third degree burns and wound contraction

15.2%

C. Chemotherapy and dehisence of healed wounds

49.5% of users answered this


question correctly

Which of the following associations are incorrect?

D. Poor healing at the site of previous radiotherapy

6
7-9 1 / 3
10

E. Zinc deficiency and delayed healing

11

Search
Next question

12
Go

13
14

Wound healing

15
16

Surgical wounds are either incisional or excisional and either clean, clean contaminated or
dirty. Although the stages of wound healing are broadly similar their contributions will vary
according to the wound type.

17
18
19-21 1 / 3

The main stages of wound healing include:

22
23

Haemostasis
24

Minutes to hours following injury


Vasospasm in adjacent vessels, platelet plug formation and generation of fibrin rich
clot.

25
26

Inflammation
Typically days 1-5
Neutrophils migrate into wound (function impaired in diabetes).
Growth factors released, including basic fibroblast growth factor and vascular
endothelial growth factor.
Fibroblasts replicate within the adjacent matrix and migrate into wound.
Macrophages and fibroblasts couple matrix regeneration and clot substitution.

Regeneration
Typically days 7 to 56
Platelet derived growth factor and transformation growth factors stimulate fibroblasts
and epithelial cells.
Fibroblasts produce a collagen network.
Angiogenesis occurs and wound resembles granulation tissue.

Remodeling
From 6 weeks to 1 year
Longest phase of the healing process and may last up to one year (or longer).
During this phase fibroblasts become differentiated (myofibroblasts) and these
facilitate wound contraction.
Collagen fibres are remodeled.
Microvessels regress leaving a pale scar.

The above description represents an idealised scenario. A number of diseases may distort
this process. Neovascularisation is an important early process. Endothelial cells may
proliferate in the wound bed and recanalise to form a vessel. Vascular disease, shock and
sepsis can all compromise microvascular flow and impair healing.
Conditions such as jaundice will impair fibroblast synthetic function and immunity with a
detrimental effect in most parts of the healing process.
Problems with scars:
Hypertrophic scars
Excessive amounts of collagen within a scar. Nodules may be present histologically
containing randomly arranged fibrils within and parallel fibres on the surface. The tissue itself
is confined to the extent of the wound itself and is usually the result of a full thickness dermal
injury. They may go on to develop contractures.

http://www.emrcs.com/question/question.php?q=0

1/2

13/07/2015

Image of hypertrophic scarring. Note that it remains confined to the boundaries of the original
wound:

Image sourced from Wikipedia

Keloid scars
Excessive amounts of collagen within a scar. Typically a keloid scar will pass beyond the
boundaries of the original injury. They do not contain nodules and may occur following even
trivial injury. They do not regress over time and may recur following removal.
Image of a keloid scar. Note the extension beyond the boundaries of the original incision:

Image sourced from Wikipedia

Drugs which impair wound healing:


Non steroidal anti inflammatory drugs
Steroids
Immunosupressive agents
Anti neoplastic drugs

Closure
Delayed primary closure is the anatomically precise closure that is delayed for a few days but
before granulation tissue becomes macroscopically evident.
Secondary closure refers to either spontaneous closure or to surgical closure after
granulation tissue has formed.
Rate question:

Next question

Comment on this question

All contents of this site are 2012 E-Medical Revision Ltd

http://www.emrcs.com/question/question.php?q=0

Terms and Conditions

Privacy policy

2/2

13/07/2015

Reference ranges

Previous

Question 27 of 154

Next

A 45 year old women is identified as having a gastric gastro-intestinal stromal tumour. What
is the usual cell of origin of these lesions?

A. Brunners glands
B. Interstitial cells of Cajal

Question stats

Score: 33.3%
1

16.8%

29.8%

29.8%

6.1%

17.6%

29.8% of users answered this


question correctly

C. Primitive stem cells of the gut wall

End and review

D. Fundic glands

6
7-9 1 / 3
10

E. Antral goblet cells

11

Search

12
Go

Next question

13
14

GIST's are derived from the interstitial pacemaker cells of Cajal. This means that they are
often located extramucosally and macroscopically, demonstrate little mucosal disruption.

15
16

Gastrointestinal stromal tumour

17
18

GIST's are not common tumours (10 per million) and originate primarily from the interstitial
pacemaker cells (of Cajal). Up to 70% occur in the stomach, the remainder occurring in the
small intestine (20%) and the colon and rectum (5%). Up to 95% are solitary lesions and
most are sporadic. The vast majority express CD117 which is a transmembrane tyrosine
kinase receptor and in these there is a mutation of the c-KIT gene.

19-21 1 / 3
22
23
24

The goal of surgery is resection of the tumour with a 1-2cm margin of normal tissue. As a
result extensive resections are not required. Unfortunately there is a high local recurrence
rate, the risk of which is related to site, incomplete resections and high mitotic count. Salvage
surgery for recurrent disease is associated with a median survival of 15 months.

25
26
27

The prognosis in high risk patients is greatly improved through the use of imatinib, which in
the ACOSOG trial (imatinib vs placebo) improved relapse rates from 17% to 2%.
In the UK it is advocated by NICE for use in patients with metastatic disease or locally
unresectable disease.
Rate question:

Next question

Comment on this question

All contents of this site are 2012 E-Medical Revision Ltd

http://www.emrcs.com/question/question.php?q=0

Terms and Conditions

Privacy policy

1/1

13/07/2015

Reference ranges

Previous

Question 28 of 154

Next

A 23 year old man fractures his right tibia in a sporting accident. At which point in the healing
process is fracture callus most likely to be visible radiologically?

A. 1 day
B. 7 days

Question stats

Score: 35.7%
1

6.7%

22.8%

9.6%

21.1%

39.8%

39.8% of users answered this


question correctly

C. 8 weeks

End and review

D. 6 weeks

6
7-9 1 / 3
10

E. 3 weeks

11

Search

12
Go

Next question

13
14

Fracture callus is composed of fibroblasts and chondroblasts and the synthesis of


fibrocartilage. It is typically visible on radiographs approximately 3 weeks following injury. If
delayed then there may be risk of non union.

15
16
17

Fracture healing

18
19-21 1 / 3

Bone fracture
- Bleeding vessels in the bone and periosteum
- Clot and haematoma formation
- The clot organises over a week (improved structure and collagen)
- The periosteum contains osteoblasts which produce new bone
- Mesenchymal cells produce cartilage (fibrocartilage and hyaline cartilage) in the soft tissue
around the fracture
- Connective tissue + hyaline cartilage = callus
- As the new bone approaches the new cartilage, endochondral ossification occurs to bridge
the gap
- Trabecular bone forms
- Trabecular bone is resorbed by osteoclasts and replaced with compact bone

22
23
24
25
26
27
28

Factors affecting fracture healing


Age
Malnutrition
Bone disorders: osteoporosis
Systemic disorders: diabetes, Marfan's syndrome and Ehlers-Danlos syndrome cause
abnormal musculoskeletal healing.
Drugs: steroids, non steroidal anti inflammatory agents.
Type of bone: Cancellous (spongy) bone fractures are usually more stable, involve
greater surface areas, and have a better blood supply than cortical (compact) bone
fractures.
Degree of Trauma: The more extensive the injury to bone and surrounding soft tissue,
the poorer the outcome.
Vascular Injury: Especially the femoral head, talus, and scaphoid bones.
Degree of Immobilization
Intra-articular Fractures: These fractures communicate with synovial fluid, which
contains collagenases that retard bone healing.
Separation of Bone Ends: Normal apposition of fracture fragments is needed for union
to occur. Inadequate reduction, excessive traction, or interposition of soft tissue will
prevent healing.
Infection

Rate question:

Next question

Comment on this question

All contents of this site are 2012 E-Medical Revision Ltd

http://www.emrcs.com/question/question.php?q=0

Terms and Conditions

Privacy policy

1/1

13/07/2015

Reference ranges

Previous

Question 29 of 154

Next

Question stats

End and review

Score: 34.5%
1

10.6%

11.1%

17.6%

A. Kawasaki disease

21%

B. Systemic Lupus Erthematosus

39.6%

C. Phenytoin

39.6% of users answered this


question correctly

Of the options below, which does not cause lymphadenopathy?

D. Hydrallazine

6
7-9 1 / 3
10

E. Amiodarone

11

Search
Next question

12
Go

13
14
15

Lymphadenopathy

16
17

Lymphadenopathy in the neck, axillae, groins and abdomen


Need to note: solitary/multiple, defined/indistinct, hard/rubbery/soft, tender/painless

18
19-21 1 / 3
22

Causes of lymphadenopathy

23

Mnemonic: Hodgkins disease

24
25

H aematological: Hodgkins lymphoma, NHL, Leukaemia


O ncological: metastases
D ermatopathic lympadenitis
G aucher's disease
K awasaki disease
I nfections: TB, glandular fever, Syphilis
N iemann Pick disease
S erum sickness
D rug reaction (phenytoin)
I mmunological (SLE)
S arcoidosis
E ndocrinological (Hyperthyroidism)
A ngioimmunoplastic lymphadenopathy
S LE
E osinophilic granulomatosis
Rate question:

26
27
28
29

Next question

Comment on this question

All contents of this site are 2012 E-Medical Revision Ltd

http://www.emrcs.com/question/question.php?q=0

Terms and Conditions

Privacy policy

1/1

13/07/2015

Reference ranges

Previous

Question 30 of 154

Next

A 23 year old man is reviewed on the ward 10 days following a laparotomy. The wound is
inspected and is healing well. Which of the following processes is least likely to be occurring
in the wound at this stage?

Question stats

Score: 36.7%
1

13.4%

12.3%

45.8%

12.1%

16.3%

A. Angiogenesis
45.8% of users answered this
question correctly

B. Synthesis of collagen

End and review

C. Necrosis of fibroblasts

6
7-9 1 / 3
10

D. Secretion of matrix metalloproteinases by fibroblasts

11

Search

E. Proliferation of fibroblasts

12
Go

Next question

13
14
15

Vasculogenesis vs Angiogenesis
Vascu is new. Angi is pre

16
17
18

Vasculogenesis is new vessels developing in situ from existing mesenchyme.


Angiogenesis is vessels develop from sprouting off pre-existing arteries.

19-21 1 / 3
22
23

Fibroblasts are an important cell type in healing wounds. They typically proliferate in the
early phases of wound healing. They release matrix metalloproteinases and these facilitate in
the remodelling of the matrix within the healing wound. Necrosis in a healing wound would be
unusual as wounds will tend to show clinical evidence of angiognesis by this time.
Wound healing

24
25
26
27
28

Surgical wounds are either incisional or excisional and either clean, clean contaminated or
dirty. Although the stages of wound healing are broadly similar their contributions will vary
according to the wound type.

29
30

The main stages of wound healing include:


Haemostasis
Minutes to hours following injury
Vasospasm in adjacent vessels, platelet plug formation and generation of fibrin rich
clot.

Inflammation
Typically days 1-5
Neutrophils migrate into wound (function impaired in diabetes).
Growth factors released, including basic fibroblast growth factor and vascular
endothelial growth factor.
Fibroblasts replicate within the adjacent matrix and migrate into wound.
Macrophages and fibroblasts couple matrix regeneration and clot substitution.

Regeneration
Typically days 7 to 56
Platelet derived growth factor and transformation growth factors stimulate fibroblasts
and epithelial cells.
Fibroblasts produce a collagen network.
Angiogenesis occurs and wound resembles granulation tissue.

Remodeling
From 6 weeks to 1 year
Longest phase of the healing process and may last up to one year (or longer).
During this phase fibroblasts become differentiated (myofibroblasts) and these
facilitate wound contraction.
Collagen fibres are remodeled.
Microvessels regress leaving a pale scar.

The above description represents an idealised scenario. A number of diseases may distort
this process. Neovascularisation is an important early process. Endothelial cells may

http://www.emrcs.com/question/question.php?q=0

1/2

13/07/2015

proliferate in the wound bed and recanalise to form a vessel. Vascular disease, shock and
sepsis can all compromise microvascular flow and impair healing.
Conditions such as jaundice will impair fibroblast synthetic function and immunity with a
detrimental effect in most parts of the healing process.
Problems with scars:
Hypertrophic scars
Excessive amounts of collagen within a scar. Nodules may be present histologically
containing randomly arranged fibrils within and parallel fibres on the surface. The tissue itself
is confined to the extent of the wound itself and is usually the result of a full thickness dermal
injury. They may go on to develop contractures.
Image of hypertrophic scarring. Note that it remains confined to the boundaries of the original
wound:

Image sourced from Wikipedia

Keloid scars
Excessive amounts of collagen within a scar. Typically a keloid scar will pass beyond the
boundaries of the original injury. They do not contain nodules and may occur following even
trivial injury. They do not regress over time and may recur following removal.
Image of a keloid scar. Note the extension beyond the boundaries of the original incision:

Image sourced from Wikipedia

Drugs which impair wound healing:


Non steroidal anti inflammatory drugs
Steroids
Immunosupressive agents
Anti neoplastic drugs

Closure
Delayed primary closure is the anatomically precise closure that is delayed for a few days but
before granulation tissue becomes macroscopically evident.
Secondary closure refers to either spontaneous closure or to surgical closure after
granulation tissue has formed.
Rate question:

Next question

Comment on this question

All contents of this site are 2012 E-Medical Revision Ltd

http://www.emrcs.com/question/question.php?q=0

Terms and Conditions

Privacy policy

2/2

13/07/2015

Reference ranges

Previous

Question 31 of 154

Next

A 25 year old women presents with a slowly enlarging mass on the side of the face. Clinical
examination demonstrates that the mass is located in the tail of the parotid gland. There is
no evidence of facial nerve involvement. What is the most likely cause?

Question stats

Score: 35.5%
1

14.3%

7.9%

16.7%

7.4%

53.7%

A. Sialolithiasis
53.7% of users answered this
question correctly

B. Adenocarcinoma

End and review

C. Warthins tumour

6
7-9 1 / 3
10

D. Oncocytoma

11

Search

E. Pleomorphic adenoma

12
Go

Next question

13
14

Pleomorphic adenomas are the commonest tumours of the parotid gland and are often slow
growing, smooth and mobile. Warthins tumours are typically found in elderly males and are
composed of multiple cysts and solid components consisting of lymphoid tissue. Warthins
tumours are most often found in the tail of the parotid gland, but not in 25 year old females,
where a pleomorphic adenoma remains the most likely lesion.

15
16
17
18
19-21 1 / 3

Parotid gland clinical

22
23

Benign neoplasms
Up to 80% of all salivary gland tumours occur in the parotid gland and up to 80% of these
are benign. There is no consistent correlation between the rate of growth and the malignant
potential of the lesion. However, benign tumours should not invade structures such as the
facial nerve.
With the exception of Warthins tumours, they are commoner in women than men. The
median age of developing a lesion is in the 5th decade of life.

24
25
26
27
28
29

Benign tumour types


30

Tumour type

Features

Benign pleomorphic
adenoma or benign mixed
tumor

Most common parotid neoplasm (80%)


Proliferation of epithelial and myoepithelial cells of the ducts and an
increase in stromal components
Slow growing, lobular, and not well encapsulated
Recurrence rate of 1-5% with appropriate excision (parotidectomy)
Recurrence possibly secondary to capsular disruption during
surgery
Malignant degeneration occurring in 2-10% of adenomas observed
for long periods, with carcinoma ex-pleomorphic adenoma occurring
most frequently as adenocarcinoma

Warthin tumor (papillary


cystadenoma lymphoma
or adenolymphoma)

Second most common benign parotid tumor (5%)


Most common bilateral benign neoplasm of the parotid
Marked male as compared to female predominance
Occurs later in life (sixth and seventh decades)
Presents as a lymphocytic infiltrate and cystic epithelial
proliferation
May represent heterotopic salivary gland epithelial tissue trapped
within intraparotid lymph nodes
Incidence of bilaterality and multicentricity of 10%
Malignant transformation rare (almost unheard of)

Monomorphic adenoma

Account for less than 5% of tumours


Slow growing
Consist of only one morphological cell type (hence term mono)
Include; basal cell adenoma, canalicular adenoma, oncocytoma,
myoepitheliomas

Haemangioma

Should be considered in the differential of a parotid mass in a child


Accounts for 90% of parotid tumours in children less than 1 year of
age
Hypervascular on imaging
Spontaneous regression may occur and malignant transformation is
almost unheard of

31

Malignant salivary gland tumours


Types of malignancy
Mucoepidermoid
carcinoma

30% of all parotid malignancies


Usually low potential for local invasiveness and metastasis (depends
mainly on grade)

http://www.emrcs.com/question/question.php?q=0

1/2

13/07/2015

Adenoid cystic
carcinoma

Unpredictable growth pattern


Tendency for perineural spread
Nerve growth may display skip lesions resulting in incomplete excision
Distant metastasis more common (visceral rather than nodal spread)
5 year survival 35%

Mixed tumours

Often a malignancy occurring in a previously benign parotid lesion

Acinic cell
carcinoma

Intermediate grade malignancy


May show perineural invasion
Low potential for distant metastasis
5 year survival 80%

Adenocarcinoma

Develops from secretory portion of gland


Risk of regional nodal and distant metastasis
5 year survival depends upon stage at presentation, may be up to 75%
with small lesions with no nodal involvement

Lymphoma

Large rubbery lesion, may occur in association with Warthins tumours


Diagnosis should be based on regional nodal biopsy rather than parotid
resection
Treatment is with chemotherapy (and radiotherapy)

Diagnostic evaluation
Plain x-rays may be used to exclude calculi
Sialography may be used to delineate ductal anatomy
FNAC is used in most cases
Superficial parotidectomy may be either diagnostic of therapeutic depending upon the
nature of the lesion
Where malignancy is suspected the primary approach should be definitive resection
rather than excisional biopsy
CT/ MRI may be used in cases of malignancy for staging primary disease

Treatment
For nearly all lesions this consists of surgical resection, for benign disease this will usually
consist of a superficial parotidectomy. For malignant disease a radical or extended radical
parotidectomy is performed. The facial nerve is included in the resection if involved. The
need for neck dissection is determined by the potential for nodal involvement.
Other parotid disorders
HIV infection
Lymphoepithelial cysts associated with HIV occur almost exclusively in the parotid
Typically presents as bilateral, multicystic, symmetrical swelling
Risk of malignant transformation is low and management usually conservative

Sjogren syndrome
Autoimmune disorder characterised by parotid enlargement, xerostomia and
keratoconjunctivitis sicca
90% of cases occur in females
Second most common connective tissue disorder
Bilateral, non tender enlargement of the gland is usual
Histologically, the usual findings are of a lymphocytic infiltrate in acinar units and
epimyoepithelial islands surrounded by lymphoid stroma
Treatment is supportive
There is an increased risk of subsequent lymphoma

Sarcoid
Parotid involvement occurs in 6% of patients with sarcoid
Bilateral in most cases
Gland is not tender
Xerostomia may occur
Management of isolated parotid disease is usually conservative

Rate question:

Next question

Comment on this question

All contents of this site are 2012 E-Medical Revision Ltd

http://www.emrcs.com/question/question.php?q=0

Terms and Conditions

Privacy policy

2/2

13/07/2015

Reference ranges

Previous

Question 32 of 154

Next

Question stats

End and review

Score: 37.5%
1

10.8%

7%

60.5%

A. Lung cancer

9.8%

B. Bowel cancer

11.9%

C. Bladder cancer

60.5% of users answered this


question correctly

Beta-naphthalamine is associated with which of the following malignancies?

D. Liver cancer

6
7-9 1 / 3
10

E. Renal cancer

11

Search
Next question

12
Go

13

Beta-naphthalamine is used in the rubber industry.

14
15

The following factors are associated with the development of bladder cancer:

16

smoking
occupational: aniline dyes used in printing and textile industry, rubber manufacture
schistosomiasis
drugs: cyclophosphamide

17
18
19-21 1 / 3
22
23

Occupational cancers

24
25

Occupational cancers accounted for 5.3% cancer deaths in 2005.


In men the main cancers include:

26
27

Mesothelioma
Bladder cancer
Non melanoma skin cancer
Lung cancer
Sino nasal cancer

28
29
30
31
32

Occupations with high levels of occupational tumours include:


Construction industry
Working with coal tar and pitch
Mining
Metalworkers
Working with asbestos (accounts for 98% of all mesotheliomas)
Working in rubber industry

Shift work has been linked to breast cancer in women (Health and safety executive report
RR595).
The latency between exposure and disease is typically 15 years for solid tumours and 20 for
leukaemia.
Many occupational cancers are otherwise rare. For example sino nasal cancer is an
uncommon tumour, 50% will be SCC. They are linked to conditions such as wood dust
exposure and unlike lung cancer is not strongly linked to cigarette smoking. Another typical
occupational tumour is angiosarcoma of the liver which is linked to working with vinyl chloride.
Again in the non occupational context this is an extremely rare sporadic tumour.
Rate question:

Next question

Comment on this question

All contents of this site are 2012 E-Medical Revision Ltd

http://www.emrcs.com/question/question.php?q=0

Terms and Conditions

Privacy policy

1/1

13/07/2015

Reference ranges

Question 33 of 154

Previous

Next

A 56 year old man with Wilsons disease presents with right upper quadrant discomfort. An
ultrasound scan is performed and this demonstrates a mass lesion in the right lobe of the
liver. What is the most appropriate method of establishing the underlying diagnosis?

Question stats

Score: 36.4%
1

7.8%

23.5%

30.6%

21.4%

16.7%

A. PET CT scan
30.6% of users answered this
question correctly

B. Ultrasound guided biopsy

End and review

C. Measurement of serum alpha feto protein

6
7-9 1 / 3
10

D. MRI scan of the liver

11

Search

E. CT scan of the liver

12
Go

Next question

13
14
15

High AFP + chronic liver inflammation = Hepatocellular carcinoma.

16
17

This is likely to be a hepatocellulcar carcinoma. Diagnosis is usually made by AFP


measurement (with further imaging depending on the result). Biopsy should not be
performed as it may seed the tumour. Chronic liver diseases such as Wilsons disease
(Hepato-lenticular degeneration) increase the risk.

18
19-21 1 / 3
22
23

Liver tumours
Primary liver tumours
The most common primary tumours are cholangiocarcinoma and hepatocellular carcinoma.
Overall metastatic disease accounts for 95% of all liver malignancies making the primary liver
tumours comparatively rare.
Primary liver tumours include:
Cholangiocarcinoma
Hepatocellular carcinoma
Hepatoblastoma
Sarcomas (Rare)
Lymphomas
Carcinoids (most often secondary although primary may occur)

24
25
26
27
28
29
30
31
32
33

Hepatocellular carcinoma
These account for the bulk of primary liver tumours (75% cases). Its worldwide incidence
reflects its propensity to occur on a background of chronic inflammatory activity. Most cases
arise in cirrhotic livers or those with chronic hepatitis B infection, especially where viral
replication is actively occurring. In the UK it accounts for less than 5% of all cancers,
although in parts of Asia its incidence is 100 per 100,000.
The majority of patients (80%) present with existing liver cirrhosis, with a mass discovered on
screening ultrasound.
Diagnosis
CT/ MRI (usually both) are the imaging modalities of choice
a-fetoprotein is elevated in almost all cases
Biopsy should be avoided as it seeds tumours cells through a resection plane.
In cases of diagnostic doubt serial CT and FP measurements are the preferred
strategy.

Treatment
Patients should be staged with liver MRI and chest, abdomen and pelvic CT scan.
The testis should be examined in males (testicular tumours may cause raised AFP).
PET CT may be used to identify occult nodal disease.
Surgical resection is the mainstay of treatment in operable cases. In patients with a
small primary tumour in a cirrhotic liver whose primary disease process is controlled,
consideration may be given to primary whole liver resection and transplantation.
Liver resections are an option but since most cases occur in an already diseased liver
the operative risks and post-operative hepatic dysfunction are far greater than is seen
following metastectomy.
These tumours are not particularly chemo or radiosensitive however, both may be
used in a palliative setting. Tumour ablation is a more popular strategy.

http://www.emrcs.com/question/question.php?q=0

1/2

13/07/2015

Survival
Poor, overall survival is 15% at 5 years.
Cholangiocarcinoma
This is the second most common type of primary liver malignancy. As its name suggests
these tumours arise in the bile ducts. Up to 80% of tumours arise in the extra hepatic biliary
tree. Most patients present with jaundice and by this stage the majority will have disease that
is not resectable.
Primary sclerosing cholangitis is the main risk factor. In deprived countries typhoid and liver
flukes are also major risk factors.
Diagnosis
Patients will typically have an obstructive picture on liver function tests.
CA 19-9, CEA and CA 125 are often elevated
CT/ MRI and MRCP are the imaging methods of choice.

Treatment
Surgical resection offers the best chance of cure. Local invasion of peri hilar tumours
is a particular problem and this coupled with lobar atrophy will often contra indicate
surgical resection.
Palliation of jaundice is important, although metallic stents should be avoided in those
considered for resection.

Survival
Is poor, approximately 5-10% 5 year survival.

Rate question:

Next question

Comment on this question

All contents of this site are 2012 E-Medical Revision Ltd

http://www.emrcs.com/question/question.php?q=0

Terms and Conditions

Privacy policy

2/2

13/07/2015

Reference ranges

Previous

Question 34 of 154

Next

A 55 year old man has suffered from reflux oesophagitis for many years. During a recent
endoscopy a biopsy is taken from the distal oesophagus. The histopathology report indicates
that cells are identified with features of coarse chromatin and abnormal mitoses. The cells
are confined to the superficial epithelial layer only. Which of the following accounts for this
process?

Question stats

Score: 38.2%
1

31.2%

4.9%

6.4%

50.5%

6.9%

50.5% of users answered this


question correctly

A. Metaplasia

End and review

6
7-9 1 / 3
10

B. Apoptosis
C. Autoimmune oesophagitis

11

Search

12

D. Dysplasia

Go

13

E. Infection with Helicobacter pylori

14
Next question

15
16
17

Dysplasia = pre cancerous

18
19-21 1 / 3

Dysplasia tends to develop as a result of prolonged stimulation by precipitants. Removal of


these precipitants may possibly reverse these changes. Replacement of differentiated cells
with another cell type describes metaplasia rather than dysplasia. The absence of invasion
distinguishes this from malignancy.

22
23
24
25

Dysplasia

26
27

Premalignant condition
Disordered growth and differentiation of calls
Alteration in size, shape, and organization of cells
Features increased abnormal cell growth (increased number of mitoses/abnormal
mitoses and cellular differentiation)
Underlying connective tissue is not invaded
Causes include smoking, Helicobacter pylori, Human papilloma virus
Main differences to metaplasia is that dysplasia is considered to be part of
carcinogenesis (pre cancerous) and is associated with a delay in maturation of cells
rather than differentiated cells replacing one another
The absence of invasion differentiates dysplasia from invasive malignancy
Severe dysplasia with foci of invasion are well recognised

Rate question:

28
29
30
31
32
33
34

Next question

Comment on this question

All contents of this site are 2012 E-Medical Revision Ltd

http://www.emrcs.com/question/question.php?q=0

Terms and Conditions

Privacy policy

1/1

13/07/2015

Reference ranges

Previous

Question 35 of 154

Next

Question stats

End and review

Score: 40%
1

36.5%

18.2%

13.2%

A. Obesity

15.1%

B. Long term unfractionated heparin therapy

16.9%

C. Gastrectomy

36.5% of users answered this


question correctly

Which one of the following confers the least risk of developing osteoporosis?

D. Osteogenesis imperfecta

6
7-9 1 / 3
10

E. Diabetes

11

Search
Next question

12
Go

13

Low body weight is a risk factor for osteoporosis.

14
15

Osteoporosis

16

Risk factors

17
18

Family history
Female sex
Increasing age
Deficient diet
Sedentary lifestyle
Smoking
Premature menopause
Low body weight
Caucasians and Asians

19-21 1 / 3
22
23
24
25
26
27
28

Diseases which predispose

29

Endocrine: glucocorticoid excess (e.g. Cushing's, steroid therapy), hyperthyroidism,


hypogonadism (e.g. Turner's, testosterone deficiency), growth hormone deficiency,
hyperparathyroidism, diabetes mellitus
Multiple myeloma, lymphoma
Gastrointestinal problems: inflammatory bowel disease, malabsorption (e.g. Coeliacs),
gastrectomy, liver disease
Rheumatoid arthritis
Long term heparin therapy
Chronic renal failure
Osteogenesis imperfecta, homocystinuria

Rate question:

30
31
32
33
34
35

Next question

Comment on this question

All contents of this site are 2012 E-Medical Revision Ltd

http://www.emrcs.com/question/question.php?q=0

Terms and Conditions

Privacy policy

1/1

13/07/2015

Reference ranges

Previous

Question 36 of 154

Next

A 63 year old man has a history of claudication that has been present for many years. He is
recently evaluated in the clinic and a duplex scan shows that he has an 85% stenosis of the
superficial femoral artery. Two weeks later he presents with a 1 hour history of severe pain in
his leg. On examination he has absent pulses in the affected limb and it is much cooler than
the contra-lateral limb. Which process best accounts for this presentation?

Question stats

Score: 37.8%
1

41.3%

30.3%

12.8%

9.6%

6.1%

41.3% of users answered this


question correctly

A. Thrombosis

End and review

6
7-9 1 / 3
10

B. Embolus
C. Atheroma growth

11

Search

12

D. Sub intimal dissection

Go

13

E. Anaemia

14
Next question

15
16

Theme from April 2012 Exam


In an existing lesion a complication such as thrombosis is more likely than embolus. These
patients should receive heparin and imaging with duplex scanning. Whilst an early surgical
bypass or intra-arterial thrombolysis may be indicated, an embolectomy should not generally
be performed as the lesion is not an embolus and the operation therefore ineffective.

17
18
19-21 1 / 3
22
23

Claudication

24

Claudication is a condition in which patients develop pain in a limb during periods of exercise.
The underlying disorder is usually that of arterial insufficiency. Atheroma develops in the
arterial wall and once this occludes >50-75% of the lumenal diameter the supply to
metabolising tissues distally may become compromised. The typical claudicant complains of
calf pain that is worse on exercise and relieved by rest. This typical description assumes that
the SFA is the site of disease, more proximal disease may present with other symptoms such
as buttock claudication and impotence.

25
26
27
28
29
30

The history is usually a progressive one, patients presenting as an emergency with severe
pain, diminished sensation, pallor and absent pulses have critical limb ischaemia. This may
complicate claudication and usually indicates a plaque related complication, such as
thrombosis.

31
32
33
34

Risk factors
Risk factors for claudication include smoking, diabetes and hyperlipidaemia.

35
36

Diagnosis
Diagnostic work -up includes measurement of ankle- brachial pressure indices, duplex
scanning and formal angiography.
Treatment
Those with long claudication distances, no ulceration or gangrene may be managed
conservatively. Patients with rest pain, ulceration or gangrene will almost always require
intervention. All patients should receive an antiplatelet agent and a statin unless there are
compelling contra-indications.
Rate question:

Next question

Comment on this question

All contents of this site are 2012 E-Medical Revision Ltd

http://www.emrcs.com/question/question.php?q=0

Terms and Conditions

Privacy policy

1/1

13/07/2015

Reference ranges

Previous

Question 37 of 154

Next

The following are true of carcinoid tumours except:

A. When present in the appendix tip and measure less than 2 cm have an
excellent prognosis
B. Even when metastatic disease is present it tends to follow a protracted
course

Question stats

End and review

Score: 39.5%
1

10%

20.9%

44.8%

15.5%

8.7%

44.8% of users answered this


question correctly

C. When present in the appendix body tend to present with carcinoid


syndrome even when liver metastases are not present

6
7-9 1 / 3
10

D. May be imaged using 5 HIAA radionucleotide scanning

11

Search

12

E. Advanced appendiceal carcinoids may require right hemicolectomy

Go

13
Next question

14
15
16

Rule of thirds:

17

1/3 multiple
1/3 small bowel
1/3 metastasize
1/3 second tumour

18
19-21 1 / 3
22
23
24

Liver metastases are necessary for the presence of carcinoid syndrome.

25

Carcinoid syndrome

26
27

Carcinoid tumours secrete serotonin


Originate in neuroendocrine cells mainly in the intestine (midgut-distal ileum/appendix)
Can occur in the rectum, bronchi
Hormonal symptoms mainly occur when disease spreads outside the bowel

28
29
30
31
32

Clinical features

33

Onset: years
Flushing face
Palpitations
Pulmonary valve stenosis and tricuspid regurgitation causing dyspnoea
Asthma
Severe diarrhoea (secretory, persists despite fasting)

34
35
36
37

Investigation
5-HIAA in a 24-hour urine collection
Somatostatin receptor scintigraphy
CT scan
Blood testing for chromogranin A

Treatment
Octreotide
Surgical removal

Rate question:

Next question

Comment on this question

All contents of this site are 2012 E-Medical Revision Ltd

http://www.emrcs.com/question/question.php?q=0

Terms and Conditions

Privacy policy

1/1

13/07/2015

Reference ranges

Previous

Question 38 of 154

Next

During a difficult femoro-popliteal bypass operation the surgeon inadvertently places a clamp
across the femoral nerve. It remains there for most of the procedure. At the end of the
operation the nerve is inspected, it is in continuity but has evidence of being crushed. Which
of the following is most likely to occur over the following months?

Question stats

Score: 41%
1

44%

15.8%

23.1%

10%

7.1%

44% of users answered this


question correctly

A. Wallerian degeneration

End and review

B. Rapid restoration of neuronal function because the axon itself is intact

6
7-9 1 / 3
10

C. Normal but delayed neuronal transmission due to disruption of the


myelin

11

Search

12

D. Absence of neuroma formation

Go

13

E. None of the above

14
Next question

15
16

A neuronal injury such as this will result in Wallerian degeneration even though the nerve
remains in continuity. Neuromas may well form.

17
18

Nerve injury

19-21 1 / 3
22

There are 3 types of nerve injury:


Neuropraxia

23
24

Nerve intact but electrical conduction is affected


Full recovery
Autonomic function preserved
Wallerian degeneration does not occur

25
26
27

Axonotmesis

28

Axon is damaged and the myelin sheath is preserved. The connective


tissue framework is not affected.
Wallerian degeneration occurs.

29
30
31

Neurotmesis

Disruption of the axon, myelin sheath and surrounding connective tissue.


Wallerian degeneration occurs.

32
33
34

Wallerian Degeneration

35
36

Axonal degeneration distal to the site of injury.


Typically begins 24-36 hours following injury.
Axons are excitable prior to degeneration occurring.
Myelin sheath degenerates and is phagocytosed by tissue macrophages.

37
38

Nerve repair
Neuronal repair may only occur physiologically where nerves are in direct contact.
Where a large defect is present, the process of nerve regeneration is hampered. It
may not occur at all or result in the formation of a neuroma. Where nerve regrowth
occurs it is typically at a rate of 1mm per day.

Rate question:

Next question

Comment on this question

All contents of this site are 2012 E-Medical Revision Ltd

http://www.emrcs.com/question/question.php?q=0

Terms and Conditions

Privacy policy

1/1

13/07/2015

Reference ranges

Previous

Question 39 of 154

Next

A 38 year old lady who smokes heavily presents with recurrent episodes of infection in the
right breast. On examination she has an indurated area at the lateral aspect of the nipple
areaolar complex. Imaging shows no mass lesions. What is the most likely diagnosis?

Question stats

Score: 40%
1

25.5%

37.7%

17.6%

13%

6.2%

A. Duct ectasia
37.7% of users answered this
question correctly

B. Periductal mastitis

End and review

C. Pagets disease of the nipple

6
7-9 1 / 3
10

D. Mondors disease of the breast

11

Search

E. Radial scar

12
Go

13

Next question

14
15

Periductal mastitis is common in smokers and may present with recurrent infections.
Treatment is with co-amoxyclav. Mondors disease of the breast is a localised
thrombophlebitis of a breast vein.

16
17
18

Duct ectasia

19-21 1 / 3

Duct ectasia is a dilatation and shortening of the terminal breast ducts within 3cm of the
nipple. It is common and the incidence increases with age. It typically presents with nipple
retraction and occasionally creamy nipple discharge. It may be confused with periductal
mastitis, which presents in younger women, the vast majority of which are smokers.
Periductal mastitis typically presents with periareolar or sub areolar infections and may be
recurrent.
Patients with troublesome nipple discharge may be treated by microdochectomy (if young) or
total duct excision (if older).

22
23
24
25
26
27
28

Rate question:

29

Next question

30

Comment on this question

31
32
33
34
35
36
37
38
39

All contents of this site are 2012 E-Medical Revision Ltd

http://www.emrcs.com/question/question.php?q=0

Terms and Conditions

Privacy policy

1/1

13/07/2015

Reference ranges

Previous

Question 40 of 154

Next

Question stats

End and review

Score: 39%
1

14.9%

30.1%

32.4%

A. Mutations in oncogenes lead to cell survival

11.3%

B. Cells with oncogene mutations are always resistant to cell necrosis

11.3%

C. Oncogene mutations must typically involve two allelic mutations for


biological effects to become manifest

32.4% of users answered this


question correctly

With respect to oncogenes which statement is false?

D. May prevent cellular apoptosis

6
7-9 1 / 3
10

E. Include MYC mutations in the development of Burkitts Lymphoma

11

Search

12
Go

Next question

13
14

Tumour suppressor gene mutations tend to be recessive and thus twin allelic mutations are
required as exemplified in the Knudson two hit hypothesis in the development of
retinoblastoma. Necrosis will occur at the centre of tumours that outgrow their blood supply
and induction of angiogenesis is a key feature of tumour progression.

15
16
17
18

Oncogenes

19-21 1 / 3

Oncogenes are cancer promoting genes that are derived from normal genes (protooncogenes). Proto-oncogenes play an important physiological role in cellular growth. They
are implicated in the development of up to 20% of human cancers.

22
23
24

Proto-oncogenes may become oncogenes via the following processes:

25
26

Mutation (point mutation)


Chromosomal translocation
Increased protein expression

27
28

Only one mutated copy of the gene is needed for cancer to occur - a dominant effect

29
30

Classification of oncogenes

31

Growth factors e.g. Sis


Transcription factors e.g. Myc
Receptor tyrosine kinase e.g. RET
Cytoplasmic tyrosine kinase e.g. Src
Regulatory GTPases e.g. Ras

32
33
34
35
36

Tumour supressor genes


Tumour supressor genes restrict or repress cellular proliferation in normal cells. Their
inactivation through mutation or germ line incorporation is implicated in renal, colonic, breast,
bladder and many other cancers. One of the best known tumour supressor genes is p53.
p53 gene offers protection by causing apoptosis of damaged cells. Other well known genes
include BRCA 1 and 2.
Rate question:

37
38
39
40

Next question

Comment on this question

All contents of this site are 2012 E-Medical Revision Ltd

http://www.emrcs.com/question/question.php?q=0

Terms and Conditions

Privacy policy

1/1

13/07/2015

Reference ranges

Previous

Question 41 of 154

Next

Question stats

End and review

Score: 38.1%
1

28.1%

11.2%

10.6%

A. Chronic inflammation is mainly secondary to acute inflammation

9.7%

B. Neutrophils are the predominant cells involved

40.4%

C. Growth factors are not involved in the process

40.4% of users answered this


question correctly

Which of the following statements relating to chronic inflammation is true?

D. Appendicitis is mainly a form of chronic inflammation

6
7-9 1 / 3
10

E. Fibrosis is a macroscopic feature

11

Search
Next question

12
Go

13

Macroscopic features include:

14

Ulcers
Fibrosis
Granulomatous process

15
16
17

It most commonly occurs as a primary event rather than as a result of acute inflammation.

18
19-21 1 / 3

Chronic inflammation

22

Overview
Chronic inflammation may occur secondary to acute inflammation.In most cases chronic
inflammation occurs as a primary process. These may be broadly viewed as being one of
three main processes:

23
24
25
26

Persisting infection with certain organisms such as Mycobacterium tuberculosis which


results in delayed type hypersensitivity reactions and inflammation.
Prolonged exposure to non-biodegradable substances such as silica or suture
materials which may induce an inflammatory response.
Autoimmune conditions involving antibodies formed against host antigens.

27
28
29
30
31

Acute vs. Chronic inflammation


32

Acute inflammation

Chronic inflammation

Changes to existing vascular structure and increased


permeability of endothelial cells

Angiogenesis predominates

Infiltration of neutrophils

Macrophages, plasma cells and


lymphocytes predominate

36

Process may resolve with:

Healing by fibrosis is the main result

38

33
34
35

Suppuration
Complete resolution
Abscess formation
Progression to chronic inflammation
Healing by fibrosis

37

39
40
41

Granulomatous inflammation
A granuloma consists of a microscopic aggregation of macrophages (with epithelial type
arrangement =epitheliod). Large giant cells may be found at the periphery of granulomas.
Mediators
Growth factors released by activated macrophages include agents such as interferon and
fibroblast growth factor (plus many more). Some of these such as interferons may have
systemic features resulting in systemic symptoms and signs, which may be present in
individuals with long standing chronic inflammation.
The finding of granulomas is pathognomonic of chronic inflammation, as illustrated in this
biopsy from a patient with colonic Crohns disease

http://www.emrcs.com/question/question.php?q=0

1/2

13/07/2015

Image sourced from Wikipedia

Rate question:

Next question

Comment on this question

All contents of this site are 2012 E-Medical Revision Ltd

http://www.emrcs.com/question/question.php?q=0

Terms and Conditions

Privacy policy

2/2

13/07/2015

Reference ranges

Previous0 / 3

Question 42-44 of 154

Next

Question stats

End and review

Score: 35.6%

Average score for registered users:

1
2

Theme: Paediatric ano-rectal disorders


A. Ulcerative colitis

42

78%

43

45.8%

44

76.5%

B. Juvenile polyps

3
4
5

C. Haemorroids

D. Intussceception

Search

E. Rectal cancer

7-9 1 / 3
Go

10

F. Anal fissure

11

G. Arteriovenous malformation

12

Please select the most likely cause for the condition described. Each option may be used
once, more than once or not at all.

13
14
15
16

42.

A 4 year old boy is brought to the clinic. He gives a history of difficult, painful
defecation with bright red rectal bleeding.

17
18
19-21 1 / 3

You answered Juvenile polyps

22

The correct answer is Anal fissure

23

Theme from April 2012 Exam


Painful rectal bleeding in this age group is typically due to a fissure. Treatment
should include stool softeners and lifestyle advice.

24
25
26

43.

A 2 year old has a history of rectal bleeding. The parents notice that post
defecation, a cherry red lesion is present at the anal verge.

27
28

You answered Haemorroids

29
30

The correct answer is Juvenile polyps

31

Theme from September 2012 Exam


These lesions are usually hamartomas and this accounts for the colour of the
lesions. Although the lesions are not themselves malignant they serve as a
marker of an underlying polyposis disorder.
44.

A 12 year old is brought to the colorectal clinic with a history of rectal bleeding,
altered bowel habit, weight loss and malaise. Abdominal examination is normal.

32
33
34
35
36
37

You answered Anal fissure

38

The correct answer is Ulcerative colitis

39

The systemic features in the history are strongly suggestive of inflammatory


bowel disease rather than the other causes.

40
41
42-44 0 / 3

Next question

Paediatric proctology

Children may present with altered bowel habit and/ or rectal bleeding. Classical haemorroidal
disease is relatively rare in children. Painful bright red rectal bleeding is much more common
since constipation is a relatively common childhood disorder. The hard stool causes a tear of
the ano-rectal mucosa with subsequent fissure. The pain from the fissure must be addressed
promptly or the child will delay defecation and this fissure will worsen.
Inflammatory bowel disease may present in a similar pattern in paediatric practice with
altered bowel habit (usually diarrhoea) and bleeding. Systemic features may be present and
investigation with an endoscopy may be required.
Children with intussceception usually present at a relatively young age and the history is
usually one of colicky abdominal pain, together with a mass on clinical examination.The often
cited red current jelly type stool is a rare but classical feature.
Juvenile polyps may occur as part of the familial polyposis coli syndromes. The lesions, which
are hamartomas, are often cherry red if they protrude externally.

http://www.emrcs.com/question/question.php?q=0

1/2

13/07/2015

Rate question:

Next question

Comment on this question

All contents of this site are 2012 E-Medical Revision Ltd

http://www.emrcs.com/question/question.php?q=0

Terms and Conditions

Privacy policy

2/2

13/07/2015

Reference ranges

Previous

Question 45 of 154

Next

You review a 42-year-old woman 8 months following a renal transplant for focal segmental
glomerulosclerosis. She is on a combination of tacrolimus, mycophenolate, and
prednisolone. She has now presented with a five day history of feeling generally unwell with
jaundice, fatigue and arthralgia. On examination she has jaundice, widespread
lymphadenopathy and hepatomegaly. What is the most likely diagnosis?

Question stats

End and review

Score: 34.8%
1

8.8%

28.4%

8%

8.4%

46.5%

28.4% of users answered this


question correctly

6
7-9 1 / 3
10

A. Hepatitis C
B. Epstein-Barr virus

11

Search

C. HIV

12
Go

D. Hepatitis B

13

E. Cytomegalovirus

14
15
Next question

16
17

Post transplant complications

18
19-21 1 / 3

CMV: 4 weeks to 6 months post transplant


EBV: post transplant lymphoproliferative disease. > 6 months post transplant

22
23

Post transplant lymphoproliferative disorder is most commonly associated with Epstein-Barr


virus. It typically occurs 6 months post transplant and is associated with high dose
immunosupressant therapy. Remember cytomegalovirus presents within the first 4 weeks to
6 months post transplant.
Renal transplant:HLA typing and graft failure

24
25
26
27
28
29

The human leucocyte antigen (HLA) system is the name given to the major histocompatibility
complex (MHC) in humans. It is coded for on chromosome 6.

30

Some basic points on the HLA system

32

Class 1 antigens include A, B and C. Class 2 antigens include DP,DQ and DR


When HLA matching for a renal transplant the relative importance of the HLA antigens
are as follows DR > B > A

31

33
34
35
36

Graft survival
1 year = 90%, 10 years = 60% for cadaveric transplants
1 year = 95%, 10 years = 70% for living-donor transplants

37
38
39
40

Post-op problems
ATN of graft
Vascular thrombosis
Urine leakage
UTI

41
42-44 0 / 3
45

Hyperacute acute rejection


Due to antibodies against donor HLA type 1 antigens
Rarely seen due to HLA matching

Acute graft failure (< 6 months)


Usually due to mismatched HLA
Other causes include cytomegalovirus infection
Management: give steroids, if resistant use monoclonal antibodies

Causes of chronic graft failure (> 6 months)


Chronic allograft nephropathy
Ureteric obstruction
Recurrence of original renal disease (MCGN > IgA > FSGS)

http://www.emrcs.com/question/question.php?q=0

1/2

13/07/2015

Rate question:

Next question

Comment on this question

All contents of this site are 2012 E-Medical Revision Ltd

http://www.emrcs.com/question/question.php?q=0

Terms and Conditions

Privacy policy

2/2

13/07/2015

Reference ranges

Previous

Question 46 of 154

Next

Question stats

End and review

Score: 36.2%
1

10.9%

63.7%

10.2%

A. Apoptosis

6.6%

B. Opsonisation

8.6%

C. Proteolysis

63.7% of users answered this


question correctly

Which of the following processes facilitates phagocytosis?

D. Angiogenesis

6
7-9 1 / 3
10

E. Necrosis

11

Search
Next question

12
Go

13

Theme from September 2014 Exam


Opsonisation will facilitate phagocytosis. The micro-organism becomes coated with antibody,
C3b and certain acute phase proteins. The macrophages and neutrophils have up regulation
of phagocytic cell surface receptors in these circumstances, a process mediated by proinflammatory cytokines. These cells then engulf the micro organism.

14
15
16
17
18

Phagocytosis

19-21 1 / 3
22

Ingestion of pathogens or foreign materials by cells


First step is opsonisation whereby the organism is coated by antibody
Second step is adhesion to cell surface
Third step is pseudopodial extension to form a phagocytic vacuole
Lysosomes fuse with vacuole and degrade contents

23
24
25
26
27
28

Rate question:

Next question

29

Comment on this question

30
31
32
33
34
35
36
37
38
39
40
41
42-44 0 / 3
45
46

All contents of this site are 2012 E-Medical Revision Ltd

http://www.emrcs.com/question/question.php?q=0

Terms and Conditions

Privacy policy

1/1

13/07/2015

Reference ranges

Previous

Question 47 of 154

Next

Features which are evaluated for the grading of breast cancer include all the following,
except:

A. Tubule formation
B. Mitoses

Question stats

Score: 35.4%
1

31.8%

10.2%

8.5%

35.4%

14.1%

35.4% of users answered this


question correctly

C. Nuclear pleomorphism

End and review

D. Tumour necrosis

6
7-9 1 / 3
10

E. Coarse chromatin

11

Search

12
Go

Next question

13
14

The necrosis of a tumour may be suggestive of a high grade tumour which has out grown its
blood supply. However, the grading of breast cancer which classically follows the Bloom Richardson grading model will tend to favor nuclear appearances (which include mitoses,
coarse chromatin and pleomorphism). Tubule formation is an important marker of the degree
of differentiation with formation of tubular structures being associated with well differentiated
tumours.

15
16
17
18
19-21 1 / 3

Tumour grading

22
23

Tumours may be graded according to their degree of differentiation, mitotic activity and other
features. Grade 1 tumours are the most differentiated and grade 3 or 4 the least. The
assessment is subjective, in most cases high grade equates to poor prognosis, or at least
rapid growth.
Tumours of glandular epithelium will tend to arrange themselves into acinar type structures
containing a central lumen. Well differentiated tumours may show excellent acinar formation
and poorly differentiated tumours simply clumps of cells around a desmoplastic stroma.
Sometimes tumours demonstrate mucous production without evidence of acinar formation.
Since mucous production is evidence of a glandular function such tumours are often termed
mucinous adenocarcinoma.
Squamous cell tumours will typically produce structures resembling epithelial cell
components. Well differentiated tumours may also produce keratin (depending upon tissue
of origin).

24
25
26
27
28
29
30
31
32
33

Rate question:

34

Next question

35
Comment on this question

36
37
38
39
40
41
42-44 0 / 3
45
46
47

All contents of this site are 2012 E-Medical Revision Ltd

http://www.emrcs.com/question/question.php?q=0

Terms and Conditions

Privacy policy

1/1

13/07/2015

Reference ranges

Previous

Question 48 of 154

Next

A 34 year old man is diagnosed with an aggressive caecal adenocarcinoma. His sister died
from the same disease at 38 years of age. His mother died from endometrial cancer at the
age of 41. What is the most likely underlying abnormality?

Question stats

Score: 34.7%
1

23%

26.3%

26.9%

9.1%

14.6%

A. Familial adenopolypomatosis coli


26.9% of users answered this
question correctly

B. Gardeners syndrome

End and review

C. Mutation of mismatch repair genes

6
7-9 1 / 3
10

D. Deletion of chromosome 6

11

Search

E. MYH gene mutation

12
Go

Next question

13
14

Lynch syndrome which is characterised by aggressive right sided colonic malignancy and
endometrial cancer is caused by microsatellite instability of DNA repair genes.

15
16
17

Genetics of colorectal cancer

18

The lifetime risk of colorectal cancer in the UK population is 5%. Up to 5% of newly


diagnosed bowel cancers will be in those individuals who have a high genetically acquired
risk of bowel cancer. Cancers arising in the low-moderate genetic risk group comprise
approximately 30% of newly diagnosed bowel cancer.

19-21 1 / 3
22
23
24

Genetics of inherited colorectal cancer syndromes


Syndrome

Features

25

Genes
implicated

26

28

27

FAP

More than 100 adenomatous polyps affecting the colon and


rectum. Duodenal and fundic glandular polyps

APC (over 90%)

Gardner
syndrome

As FAP but with desmoid tumours and mandibular


osteomas

APC

Turcots
syndrome

Polyposis and colonic tumours and CNS tumours

APC +MLH1
and PMS2

32

HNPCC

Colorectal cancer without extensive polyposis. Endometrial


cancer, renal and CNS

MSH2, MLH1,
PMS2 and
GTBP

34

PeutzJeghers
syndrome

Hamartomatous polyps in GI tract and increased risk of GI


malignancy

LKB1
andSTK11 (in
up to 70%)

Cowden
disease

Multiple hamartomas (see below)

PTEN (85%)

MYH
associated
polyposis

Autosomal recessive, multiple adenomatous polyps in GI


tract, those in colon having somatic KRAS mutations

MYH

29
30
31

33

35
36
37
38
39
40
41
42-44 0 / 3
45
46

FAP
Autosomal dominant condition, affects 1 in 12,000. Accounts for 0.5% of all CRCs. Lifetime
incidence of colorectal cancer in untreated FAP =100%. Up to 25% cases are caused by denovo germ line mutations and show no prior family history. The APC tumour suppressor gene
is affected in most cases.

47
48

APC in non inherited colorectal cancer


Up to 80% of sporadic colorectal cancers will have somatic mutations that inactivate APC[1].
Both alleles are usually affected. Although the APC protein more than likely has multiple
critical cellular functions, the best-established role for APC in the cancer process is as a
major binding partner and regulator of the - catenin protein in the so-called canonical or catenin dependent Wnt signaling pathway.
HNPCC
HNPCC cancers differ from conventional tumours in a number of respects. In the colon the
tumours are more likely to be right sided, histologically they are more likely to be mucinous
and have dense lymphocytic infiltrates. To be diagnosed as having HNPCC individuals must
show typically HNPCC tumours in at least three individuals, (one of whom must be a first
degree relative to the other two). In at least two successive generations. At least one cancer
must be diagnosed under the age of 50. FAP must be excluded and tumours should be

http://www.emrcs.com/question/question.php?q=0

1/2

13/07/2015

verified by pathological identification (Amsterdam criteria). The genetic changes in HNPCC


stem primarily from microsatellite instability affecting DNA mismatch repair genes. In HNPCC
the mismatch repair genes most commonly implicated include; MSH2 and MLH1 and these
occur in up to 70% of people with HNPCC. The finding of microsatellite instability is unusual
in sporadic colorectal cancers. Approximately 60% of individuals who fulfill the Amsterdam
criteria will not be found to have evidence of mismatch repair gene defects on genetic
testing. The risk of developing colorectal cancer in those who have not demonstrated
mutation of the mis match repair genes is increased if they fulfill the Amsterdam criteria, but
not
the extent that it is increased in those who fulfill the criteria AND have evidence of mis match
repair gene defects.
KRAS Mutations
The RAS family of small G proteins act as molecular switches downstream of growth factor
receptors. KRAS and the other two members of the family; HRAS and NRAS, are the site of
mutation in approximately 40% of colorectal cancers. When adenomas are examined the
proportion of adenomas less than 1cm showing KRAS mutations was only 10% which
contrasts with 50% in those lesions greater than 1cm.
p53 mutations
The p53 protein functions as a key transcriptional regulator of genes that encode proteins
with functions in cell-cycle checkpoints at the G1/S and G2/M boundaries, in promoting
apoptosis, and in restricting angiogenesis . As such, selection for p53 defects at the
adenoma-carcinoma transition may reflect the fact that stresses on tumor cells activate cellcycle arrest, apoptotic, and antiangiogenic pathways in cells with wild-type p53 function.
Many colonic tumours will demonstrate changes in the p53 gene that may facilitate tumour
progression through from adenoma to carcinoma.
Cowden syndrome
Also known as multiple hamartoma syndrome. Rare autosomal dominant condition with
incidence of 1 in 200,000.. It is characterised by multiple mucocutaneous lesions,
trichilemmomas, oral papillomas and acral keratosis. Most often diagnosed in third decade of
life. Breast carcinoma may occur in up to 50% of patients and conditions such as fibrocystic
disease of the breast may occur in 75% of women. Thyroid disease occurs in 75% and may
include malignancy. Endoscopic screening will identify disease in up to 85% although the
small bowel is rarely involved. There is a 15-20% risk of developing colorectal cancer and
regular colonoscopic screening from age 45 is recommended.
Terminology
Oncogene

Oncogenes are genes which have the potential to induce cellular proliferation and
avoid apoptosis. Oncogene mutations are general gain of function and are
therefore dominant. Increased expression of oncogenes are found in most tumours

Tumour
suppressor
gene

These genes generally inhibit cellular proliferation or induce apoptosis. Mutations


in tumour suppressor genes are generally loss of function mutations, and are
therefore recessive. Mutations in both tumour suppressor gene alleles allow cells
to proliferate without restraint

References
1. Fearon, E.R. and B. Vogelstein, A genetic model for colorectal tumorigenesis. Cell, 1990.
61(5): p. 759-67.
Rate question:

Next question

Comment on this question

All contents of this site are 2012 E-Medical Revision Ltd

http://www.emrcs.com/question/question.php?q=0

Terms and Conditions

Privacy policy

2/2

13/07/2015

Reference ranges

Question 49 of 154

Previous

Next

Question stats

End and review

Score: 36%
1

57.9%

13%

8.3%

A. Jaundice

8.2%

B. Patients taking carbamazepine

12.6%

C. General anaesthesia using thiopentone

57.9% of users answered this


question correctly

Which of the following is associated with poor wound healing?

D. General anaesthesia using ketamine

6
7-9 1 / 3
10

E. Multiple sclerosis

11

Search
Next question

12
Go

13

Mnemonic to remember factors affecting wound healing: DID NOT HEAL

14
15

D iabetes
I nfection, irradiation
D rugs eg steroids, chemotherapy

16
17
18

N utritional deficiencies (vitamin A, C & zinc, manganese), Neoplasia


O bject (foreign material)
T issue necrosis

19-21 1 / 3
22
23

H ypoxia
E xcess tension on wound
A nother wound
L ow temperature, Liver jaundice

24
25
26
27

Multiple sclerosis is associated with pressure sores, however the cellular healing process is
not affected.
Wound healing

28
29
30
31

Surgical wounds are either incisional or excisional and either clean, clean contaminated or
dirty. Although the stages of wound healing are broadly similar their contributions will vary
according to the wound type.

32

The main stages of wound healing include:

35

33
34

36

Haemostasis
Minutes to hours following injury
Vasospasm in adjacent vessels, platelet plug formation and generation of fibrin rich
clot.

37
38
39
40

Inflammation

41
42-44 0 / 3

Typically days 1-5


Neutrophils migrate into wound (function impaired in diabetes).
Growth factors released, including basic fibroblast growth factor and vascular
endothelial growth factor.
Fibroblasts replicate within the adjacent matrix and migrate into wound.
Macrophages and fibroblasts couple matrix regeneration and clot substitution.

45
46
47
48
49

Regeneration
Typically days 7 to 56
Platelet derived growth factor and transformation growth factors stimulate fibroblasts
and epithelial cells.
Fibroblasts produce a collagen network.
Angiogenesis occurs and wound resembles granulation tissue.

Remodeling
From 6 weeks to 1 year
Longest phase of the healing process and may last up to one year (or longer).
During this phase fibroblasts become differentiated (myofibroblasts) and these
facilitate wound contraction.
Collagen fibres are remodeled.
Microvessels regress leaving a pale scar.
http://www.emrcs.com/question/question.php?q=0

1/3

13/07/2015

The above description represents an idealised scenario. A number of diseases may distort
this process. Neovascularisation is an important early process. Endothelial cells may
proliferate in the wound bed and recanalise to form a vessel. Vascular disease, shock and
sepsis can all compromise microvascular flow and impair healing.
Conditions such as jaundice will impair fibroblast synthetic function and immunity with a
detrimental effect in most parts of the healing process.
Problems with scars:
Hypertrophic scars
Excessive amounts of collagen within a scar. Nodules may be present histologically
containing randomly arranged fibrils within and parallel fibres on the surface. The tissue itself
is confined to the extent of the wound itself and is usually the result of a full thickness dermal
injury. They may go on to develop contractures.
Image of hypertrophic scarring. Note that it remains confined to the boundaries of the original
wound:

Image sourced from Wikipedia

Keloid scars
Excessive amounts of collagen within a scar. Typically a keloid scar will pass beyond the
boundaries of the original injury. They do not contain nodules and may occur following even
trivial injury. They do not regress over time and may recur following removal.
Image of a keloid scar. Note the extension beyond the boundaries of the original incision:

Image sourced from Wikipedia

Drugs which impair wound healing:


Non steroidal anti inflammatory drugs
Steroids
Immunosupressive agents
Anti neoplastic drugs

Closure
Delayed primary closure is the anatomically precise closure that is delayed for a few days but
before granulation tissue becomes macroscopically evident.
Secondary closure refers to either spontaneous closure or to surgical closure after
granulation tissue has formed.
Rate question:

Next question

Comment on this question

http://www.emrcs.com/question/question.php?q=0

2/3

13/07/2015

All contents of this site are 2012 E-Medical Revision Ltd

http://www.emrcs.com/question/question.php?q=0

Terms and Conditions

Privacy policy

3/3

13/07/2015

Reference ranges

Question 50 of 154

Previous

Next

A 55 year old man undergoes a colonoscopy and a colonic polyp is identified. It has a lobular
appearance and is located on a stalk in the sigmoid colon. Which of the processes below
best accounts for this disease?

Question stats

Score: 37.3%
1

6.6%

25.6%

52.4%

6.9%

8.5%

A. Apoptosis
52.4% of users answered this
question correctly

B. Metaplasia

End and review

C. Dysplasia

6
7-9 1 / 3
10

D. Calcification

11

Search

E. Degeneration

12
Go

13

Next question

14
15

Theme from April 2012 Exam


Most colonic polyps described above are adenomas. These may have associated dysplasia.
The more high grade the dysplasia the greater the level of clinical concern.

16
17
18

Colonic polyps

19-21 1 / 3

Colonic Polyps
May occur in isolation, or greater numbers as part of the polyposis syndromes. In FAP
greater than 100 polyps are typically present. The risk of malignancy in association with
adenomas is related to size, and is the order of 10% in a 1cm adenoma. Isolated adenomas
seldom give risk of symptoms (unless large and distal). Distally sited villous lesions may
produce mucous and if very large, electrolyte disturbances may occur.

22

Follow up of colonic polyps

27

23
24
25
26

28

Group

Features

Action

Low risk

1 or 2 adenomas less than 1cm

No follow up or recolonoscopy at 5 years

Moderate
risk

3 or 4 small adenomas or 1 adenoma greater than


1cm

Re-scope at 3 years

High risk

More than 5 small adenomas or more than 3 with 1


of them greater than 1cm

Re scope at 1 year

29
30
31
32
33
34
35

From Atkins and Saunders Gut 2002 51 (suppl V:V6-V9). It is important to stratify patients
appropriately and ensure that a complete colonoscopy with good views was performed.

36
37

Segmental resection or complete colectomy should be considered when:

38
39

1. Incomplete excision of malignant polyp


2. Malignant sessile polyp
3. Malignant pedunculated polyp with submucosal invasion
4. Polyps with poorly differentiated carcinoma
5. Familial polyposis coli
-Screening from teenager up to 40 years by 2 yearly sigmoidoscopy/colonoscopy
-Panproctocolectomy and Ileostomy or Restorative Panproctocolectomy.

40
41
42-44 0 / 3
45
46
47

Rectal polypoidal lesions may be amenable to trans anal endoscopic microsurgery.


48
49

References
Cairns S et al. Guidelines for colorectal cancer screening and surveillance in moderate and
high risk groups (update from 2002). Gut 2010;59:666-690.
Rate question:

50

Next question

Comment on this question

All contents of this site are 2012 E-Medical Revision Ltd

http://www.emrcs.com/question/question.php?q=0

Terms and Conditions

Privacy policy

1/1

13/07/2015

Reference ranges

Question 51 of 154

Previous

Next

A 56 year old lady has just undergone a colonoscopy and a 1.5cm lesion was identified in
the caecum. The histology report states that biopsies have been taken from a sessile
serrated polyp with traditional features. What is the best management option?

Question stats

Score: 36.5%
1

24.7%

31.2%

8.3%

17.8%

18%

A. Perform a right hemicolectomy


31.2% of users answered this
question correctly

B. List the patient for colonoscopic polypectomy

End and review

C. Discharge the patient

6
7-9 1 / 3
10

D. Re scope the patient in 6 months

11

Search

E. Re scope the patient at 3 years

12
Go

13

Next question

14
15

These polyps represent an alternative pathway to progression to carcinoma and may be


diagnostically confused with hyperplastic polyps. Hyperplastic polyps are more common in
the left colon and confer no increased risk. SSA's are more common in the right colon and
are usually larger. Those with "traditional features" on histology have dysplasia with
increased risk of malignant transformation.

16
17
18
19-21 1 / 3

Colonic polyps

22
23

Colonic Polyps
May occur in isolation, or greater numbers as part of the polyposis syndromes. In FAP
greater than 100 polyps are typically present. The risk of malignancy in association with
adenomas is related to size, and is the order of 10% in a 1cm adenoma. Isolated adenomas
seldom give risk of symptoms (unless large and distal). Distally sited villous lesions may
produce mucous and if very large, electrolyte disturbances may occur.

24
25
26
27
28

Follow up of colonic polyps

29

Group

Features

Action

30

Low risk

1 or 2 adenomas less than 1cm

No follow up or recolonoscopy at 5 years

31

33

Moderate
risk

3 or 4 small adenomas or 1 adenoma greater than


1cm

Re-scope at 3 years

High risk

More than 5 small adenomas or more than 3 with 1


of them greater than 1cm

Re scope at 1 year

32

34
35
36
37

From Atkins and Saunders Gut 2002 51 (suppl V:V6-V9). It is important to stratify patients
appropriately and ensure that a complete colonoscopy with good views was performed.

38
39

Segmental resection or complete colectomy should be considered when:

40

1. Incomplete excision of malignant polyp


2. Malignant sessile polyp
3. Malignant pedunculated polyp with submucosal invasion
4. Polyps with poorly differentiated carcinoma
5. Familial polyposis coli
-Screening from teenager up to 40 years by 2 yearly sigmoidoscopy/colonoscopy
-Panproctocolectomy and Ileostomy or Restorative Panproctocolectomy.

41
42-44 0 / 3
45
46
47
48
49

Rectal polypoidal lesions may be amenable to trans anal endoscopic microsurgery.

50

References
Cairns S et al. Guidelines for colorectal cancer screening and surveillance in moderate and
high risk groups (update from 2002). Gut 2010;59:666-690.
Rate question:

51

Next question

Comment on this question

All contents of this site are 2012 E-Medical Revision Ltd

http://www.emrcs.com/question/question.php?q=0

Terms and Conditions

Privacy policy

1/1

13/07/2015

Reference ranges

Previous

Question 52 of 154

Next

A 30 year old male presents with gynaecomastia. Clinically, he is noted to have a nodule in
the left testis. What is the most likely diagnosis?

A. Oestrogen abuse
B. Seminoma with syncytiotrophoblast giant cells

Question stats

Score: 35.8%
1

7.4%

28.2%

21.3%

12.3%

30.9%

30.9% of users answered this


question correctly

C. Teratoma

End and review

D. Choriocarcinoma

6
7-9 1 / 3
10

E. Leydig cell tumour

11

Search

12
Next question

Leydig cell tumours are rare testicular sex cord stromal tumours (which also include sertoli
cell tumours) which are associated with hormonal activity.
Patients with Leydig cell tumours may present with gynaecomastia before they notice
testicular enlargement.
Majority are benign
Histology: eosinophilic cells in columns

Go

13
14
15
16
17
18
19-21 1 / 3
22
23

Testicular disorders

24

Testicular cancer
Testicular cancer is the most common malignancy in men aged 20-30 years. Around 95% of
cases of testicular cancer are germ-cell tumours. Germ cell tumours may essentially be
divided into:

25
26
27
28

Tumour type

Seminoma

Key features

Commonest
subtype
(50%)
Average
age at
diagnosis =
40
Even
advanced
disease
associated
with 5 year
survival of
73%

Tumour
markers

Pathology

AFP usually
normal
HCG elevated
in 10%
seminomas
Lactate
dehydrogenase;
elevated in 1020%
seminomas (but
also in many
other
conditions)

Sheet like lobular patterns


of cells with substantial
fibrous component. Fibrous
septa contain lymphocytic
inclusions and granulomas
may be seen.

29
30
31
32
33
34
35
36
37
38
39
40
41

Non seminomatous
germ cell tumours
(42%)
Teratoma
Yolk sac tumour
Choriocarcinoma
Mixed germ cell
tumours (10%)

Younger age at
presentation =2030 years
Advanced
disease carries
worse prognosis
(48% at 5 years)
Retroperitoneal
lymph node
dissection may be
needed for residual
disease after
chemotherapy

AFP elevated
in up to 70% of
cases
HCG elevated
in up to 40% of
cases
Other
markers rarely
helpful

Heterogenous texture with


occasional ectopic tissue
such as hair

42-44 0 / 3
45
46
47
48
49
50
51
52

Image demonstrating a classical seminoma, these tumours are typically more uniform than
teratomas

http://www.emrcs.com/question/question.php?q=0

1/3

13/07/2015

Image sourced from Wikipedia

Risk factors for testicular cancer


Cryptorchidism
Infertility
Family history
Klinefelter's syndrome
Mumps orchitis

Features
A painless lump is the most common presenting symptom
Pain may also be present in a minority of men
Other possible features include hydrocele, gynaecomastia

Diagnosis
Ultrasound is first-line
CT scanning of the chest/ abdomen and pelvis is used for staging
Tumour markers (see above) should be measured

Management
Orchidectomy (Inguinal approach)
Chemotherapy and radiotherapy may be given depending on staging
Abdominal lesions >1cm following chemotherapy may require retroperitoneal lymph
node dissection.

Prognosis is generally excellent


5 year survival for seminomas is around 95% if Stage I
5 year survival for teratomas is around 85% if Stage I

Benign disease
Epididymo-orchitis
Acute epididymitis is an acute inflammation of the epididymis, often involving the testis and
usually caused by bacterial infection.
Infection spreads from the urethra or bladder. In men <35 years, gonorrhoea or
chlamydia are the usual infections.
Amiodarone is a recognised non infective cause of epididymitis, which resolves on
stopping the drug.
Tenderness is usually confined to the epididymis, which may facilitate differentiating it
from torsion where pain usually affects the entire testis.

Testicular torsion

http://www.emrcs.com/question/question.php?q=0

2/3

13/07/2015

Twist of the spermatic cord resulting in testicular ischaemia and necrosis.


Most common in males aged between 10 and 30 (peak incidence 13-15 years)
Pain is usually severe and of sudden onset.
Cremasteric reflex is lost and elevation of the testis does not ease the pain.
Treatment is with surgical exploration. If a torted testis is identified then both testis
should be fixed as the condition of bell clapper testis is often bilateral.

Hydrocele
Presents as a mass that transilluminates, usually possible to "get above" it on
examination.
In younger men it should be investigated with USS to exclude tumour.
In children it may occur as a result of a patent processus vaginalis.
Treatment in adults is with a Lords or Jabouley procedure.
Treatment in children is with trans inguinal ligation of PPV.

Rate question:

Next question

Comment on this question

All contents of this site are 2012 E-Medical Revision Ltd

http://www.emrcs.com/question/question.php?q=0

Terms and Conditions

Privacy policy

3/3

13/07/2015

Reference ranges

Question 53 of 154

Previous

Next

Question stats

End and review

Score: 38.2%
1

17%

24.7%

16.3%

A. Osteosarcoma

9%

B. Neuroblastoma

33%

C. Leukaemia

24.7% of users answered this


question correctly

What is the most common cause of osteolytic bone metastasis in children?

D. Rhabdomyosarcoma

6
7-9 1 / 3
10

E. Nephroblastoma

11

Search
Next question

12
Go

13

Neuroblastomas are a relatively common childhood tumour and have a strong tendency to
developing widespread lytic metastasis.

14
15

Secondary malignant tumours of bone

16
17

Metastatic lesions affecting bone are more common than primary bone tumours.
18
19-21 1 / 3

The typical tumours that spread to bone include:

22

Breast
Bronchus
Renal
Thyroid
Prostate

23
24
25
26
27

75% cases will affect those over the age of 50


28

The commonest bone sites affected are:

29

Vertebrae (usually thoracic)


Proximal femur
Ribs
Sternum
Pelvis
Skull

30
31
32
33
34
35

Pathological fracture
Osteolytic lesions are the greatest risk for pathological fracture
The risk and load required to produce fracture varies according to bone site. Bones with
lesions that occupy 50% or less will be prone to fracture under loading (Harrington). When
75% of the bone is affected the process of torsion about a bony fulcrum may produce a
fracture.
The Mirel scoring[1] system may be used to help determine the risk of fracture and is more
systematic than the Harrington system described above.

36
37
38
39
40
41
42-44 0 / 3
45

Mirel Scoring system

46
47

Score
points

Site

Radiographic
appearance

Width of bone
involved

Pain

Upper
extremity

Blastic

Less than 1/3

Mild

Lower
extremity

Mixed

1/3 to 2/3

Moderate

Peritrochanteric

Lytic

More than 2/3

Aggravated by
function

48
49
50
51
52
53

Depending upon the score the treatment should be as follows:


Score

Risk of fracture

Treatment

9 or greater

Impending (33%)

Prophylactic fixation

Borderline

Consider fixation

http://www.emrcs.com/question/question.php?q=0

1/2

13/07/2015

7 or less

Not impending (4%)

Non operative management

Where the lesion is an isolated metastatic deposit consideration should be given to excision
and reconstruction as the outcome is better [2].
Non operative treatments
Hypercalcaemia- Treat with re hydration and bisphosphonates.
Pain- Opiate analgesics and radiotherapy.
Some tumours such as breast and prostate will benefit from chemotherapy and or hormonal
agents.
References
1. Mirels, H. Metastatic disease in long bones. A proposed scoring system for diagnosing
impending pathologic fractures. Clin Orthop Relat Res, 1989(249): p. 256-64.
2. Mavrogenis, A.F. et al. Survival analysis of patients with femoral metastases. J Surg
Oncol, 2011.
Rate question:

Next question

Comment on this question

All contents of this site are 2012 E-Medical Revision Ltd

http://www.emrcs.com/question/question.php?q=0

Terms and Conditions

Privacy policy

2/2

13/07/2015

Reference ranges

Previous

Question 54 of 154

Next

Question stats

End and review

Score: 40.4%
1

8.7%

8.6%

10.1%

A. Complex fistula in ano

21.4%

B. Small bowel strictures

51.3%

C. Skip lesions

51.3% of users answered this


question correctly

Which of the following features are not typical of Crohns disease?

D. 'Rose thorn ulcers' on barium studies

6
7-9 1 / 3
10

E. Pseudopolyps on colonoscopy

11

Search
Next question

12
Go

13

Pseudopolyps are a feature of ulcerative colitis and occur when there is severe mucosal
ulceration. The remaining islands of mucosa may then appear to be isolated and almost
polypoidal.

14
15
16

Crohns disease

17

Crohns disease is a chronic transmural inflammation of a segment(s) of the gastrointestinal


tract and may be associated with extra intestinal manifestations. Frequent disease patterns
observed include ileal, ileocolic and colonic disease. Peri-anal disease may occur in
association with any of these. The disease is often discontinuous in its distribution.
Inflammation may cause ulceration, fissures, fistulas and fibrosis with stricturing. Histology
reveals a chronic inflammatory infiltrate that is usually patchy and transmural.

18
19-21 1 / 3
22
23
24
25

Ulcerative colitis Vs Crohns

26

Crohn's disease

Ulcerative colitis

27

Distribution

Mouth to anus

Rectum and colon

28

Macroscopic
changes

Cobblestone appearance, apthoid ulceration

Contact bleeding

Depth of
disease

Transmural inflammation

Superficial inflammation

Distribution
pattern

Patchy

Continuous

Histological
features

Granulomas (non caseating epithelioid cell


aggregates with Langerhans' giant cells)

Crypt abscesses, Inflammatory


cells in the lamina propria

29
30
31
32
33
34
35
36
37

Extraintestinal manifestations of Crohns

38
39

Related to disease extent

Unrelated to disease extent

Aphthous ulcers (10%)

Sacroiliiitis (10-15%)

Erythema nodosum (5-10%)

Ankylosing spondylitis (1-2%)

Pyoderma gangrenosum (0.5%)

Primary sclerosing cholangitis (Rare)

46

Acute arthropathy (6-12%)

Gallstones (up to 30%)

47

Ocular complications (up to 10%)

Renal calculi (up to 10%)

40
41
42-44 0 / 3
45

48
49

Diarrhoea in Crohns
Diarrhoea in Crohns may be multifactorial since actual inflammation of the colon is not
common. Causes therefore include the following:
Bile salt diarrhoea secondary to terminal ileal disease
Entero-colic fistula
Short bowel due to multiple resections
Bacterial overgrowth

50
51
52
53
54

Surgical interventions in Crohns disease


The commonest disease pattern in Crohns is stricturing terminal ileal disease and this often
culminates in an ileocaecal resection. Other procedures performed include segmental small
bowel resections and stricturoplasty. Colonic involvement in patients with Crohns is not
common and, where found, distribution is often segmental. However, despite this distribution
segmental resections of the colon in patients with Crohns disease are generally not
advocated because the recurrence rate in the remaining colon is extremely high, as a result
http://www.emrcs.com/question/question.php?q=0

1/2

13/07/2015

the standard options of colonic surgery in Crohns patients are generally; sub total colectomy,
panproctocolectomy and staged sub total colectomy and proctectomy. Restorative
procedures such as ileoanal pouch have no role in therapy.
Crohns disease is notorious for the developmental of intestinal fistulae; these may form
between the rectum and skin (peri anal) or the small bowel and skin. Fistulation between
loops of bowel may also occur and result in bacterial overgrowth and malabsorption.
Management of enterocutaneous fistulae involves controlling sepsis, optimising nutrition,
imaging the disease and planning definitive surgical management.
Rate question:

Next question

Comment on this question

All contents of this site are 2012 E-Medical Revision Ltd

http://www.emrcs.com/question/question.php?q=0

Terms and Conditions

Privacy policy

2/2

13/07/2015

Reference ranges

Question 55 of 154

Previous

Next

Question stats

End and review

Score: 41.4%
1

25.4%

10%

11.2%

A. Caecum

9.4%

B. Ascending colon

44%

C. Transverse colon

44% of users answered this


question correctly

At which of the following sites is the development of diverticulosis least likely?

D. Sigmoid colon

6
7-9 1 / 3
10

E. Rectum

11

Search
Next question

12
Go

13

Rectal involvement with diverticular disease almost never occurs.

14
15

Because the rectum has a circular muscle coat (blending of of the tenia marks the rectosigmoid junction), diverticular disease almost never occurs here. Right sided colonic
diverticular disease is well recognised (though less common than left sided).
Diverticular disease

16
17
18
19-21 1 / 3
22

Diverticular disease is a common surgical problem. It consists of herniation of colonic mucosa


through the muscular wall of the colon. The usual site is between the taenia coli where
vessels pierce the muscle to supply the mucosa. For this reason, the rectum, which lacks
taenia, is often spared.

23
24
25
26

Symptoms
Altered bowel habit
Bleeding
Abdominal pain

27
28
29
30
31

Complications
Diverticulitis
Haemorrhage
Development of fistula
Perforation and faecal peritonitis
Perforation and development of abscess
Development of diverticular phlegmon

32
33
34
35
36
37
38

Diagnosis
Patients presenting in clinic will typically undergo either a colonoscopy, CT cologram or
barium enema as part of their diagnostic work up. All tests can identify diverticular disease. It
can be far more difficult to confidently exclude cancer, particularly in diverticular strictures.
Acutely unwell surgical patients should be investigated in a systematic way. Plain abdominal
films and an erect chest x-ray will identify perforation. An abdominal CT scan (not a CT
cologram) with oral and intravenous contrast will help to identify whether acute inflammation
is present but also the presence of local complications such as abscess formation.
Severity Classification- Hinchey

39
40
41
42-44 0 / 3
45
46
47
48
49

Para-colonic abscess

II

Pelvic abscess

51

III

Purulent peritonitis

52

IV

Faecal peritonitis

50

53

Treatment

54
55

Increase dietary fibre intake.


Mild attacks of diverticulitis may be managed conservatively with antibiotics.
Peri colonic abscesses should be drained either surgically or radiologically.
Recurrent episodes of acute diverticulitis requiring hospitalisation are a relative
indication for a segmental resection.
Hinchey IV perforations (generalised faecal peritonitis) will require a resection and
usually a stoma. This group have a very high risk of post operative complications and
usually require HDU admission. Less severe perforations may be managed by
http://www.emrcs.com/question/question.php?q=0

1/2

13/07/2015

usually require HDU admission. Less severe perforations may be managed by


laparoscopic washout and drain insertion.

Rate question:

Next question

Comment on this question

All contents of this site are 2012 E-Medical Revision Ltd

http://www.emrcs.com/question/question.php?q=0

Terms and Conditions

Privacy policy

2/2

13/07/2015

Reference ranges

Previous

Question 56 of 154

Next

A 32 year old lady presents with a 1.5cm pigmented lesion on her back. The surgeon is
concerned that this may be a melanoma. What is the most appropriate course of action?

A. 2mm punch biopsy from the centre of the lesion


B. 4mm punch biopsy from the centre of the lesion

Question stats

Score: 42.4%
1

8%

8.3%

19.7%

32.1%

31.9%

32.1% of users answered this


question correctly

C. Wide excision of the lesion with 3cm margins

End and review

D. Excisional biopsy of the lesion

6
7-9 1 / 3
10

E. Wide excision of the lesion with 1cm margins

11

Search

12
Go

Next question

13
14

Suspicious naevi should NOT be partially sampled as histological interpretation is


severely compromised. Complete excision is mandatory where lesions fulfil diagnostic
criteria. However, wide excision for margins may be deferred until definitive histology is
available.

15
16
17
18
19-21 1 / 3

Lesions that are suspicious for melanoma should be excised with complete margins. Radical
excision is not routinely undertaken for diagnostic purposes and therefore if subsequent
histopathological assessment determines that the lesion is a melanoma a re-exicision of
margins may be required. Incisional punch biopsies of potential melanomas makes
histological interpretation difficult and is best avoided.

22
23
24
25

Malignant melanoma

26
27

The main diagnostic features (major criteria):

Secondary features (minor criteria)

Change in size
Change in shape
Change in colour

28

Diameter >6mm
Inflammation
Oozing or bleeding
Altered sensation

29
30
31
32
33

Treatment

34

Suspicious lesions should undergo excision biopsy. The lesion should be removed in
completely as incision biopsy can make subsequent histopathological assessment
difficult.
Once the diagnosis is confirmed the pathology report should be reviewed to determine
whether further re-excision of margins is required (see below):

35
36
37
38
39

Margins of excision-Related to Breslow thickness

40

Lesions 0-1mm thick

1cm

Lesions 1-2mm thick

1- 2cm (Depending upon site and pathological features)

Lesions 2-4mm thick

2-3 cm (Depending upon site and pathological features)

Lesions >4 mm thick

3cm

41
42-44 0 / 3
45
46
47
48

Marsden J et al Revised UK guidelines for management of Melanoma. Br J Dermatol 2010


163:238-256.

49
50

Further treatments such as sentinel lymph node mapping, isolated limb perfusion and block
dissection of regional lymph node groups should be selectively applied.

51
52

Rate question:

Next question

53
54

Comment on this question

55
56

All contents of this site are 2012 E-Medical Revision Ltd

http://www.emrcs.com/question/question.php?q=0

Terms and Conditions

Privacy policy

1/1

13/07/2015

Reference ranges

Previous

Question 57 of 154

Next

A 70 year old male presents with painless frank haematuria. Clinical examination is
unremarkable. Routine blood tests reveal a haemoglobin of 18g/dl but are otherwise normal.
What is the most likely underlying diagnosis?

Question stats

Score: 41.7%
1

14.3%

8.6%

47.8%

9.3%

20%

A. Squamous cell carcinoma of the bladder


47.8% of users answered this
question correctly

B. Adenocarcinoma of the prostate

End and review

C. Adenocarcinoma of the kidney

6
7-9 1 / 3
10

D. Wilms tumour

11

Search

E. Transitional cell carcinoma of the renal pelvis

12
Go

Next question

13
14

Theme from April 2012 Exam


Polycythaemia is a recognised feature of renal cell carcinoma. Wilms tumours most
commonly occur in children.

15
16
17
18

Haematuria

19-21 1 / 3

Causes of haematuria

22
23

Trauma

Injury to renal tract


Renal trauma commonly due to blunt injury (others penetrating
injuries)
Ureter trauma rare: iatrogenic
Bladder trauma: due to RTA or pelvic fractures

24
25
26
27
28

Infection

Remember TB

29
30

Malignancy

Renal cell carcinoma (remember paraneoplastic syndromes):


painful or painless
Urothelial malignancies: 90% are transitional cell carcinoma, can
occur anywhere along the urinary tract. Painless haematuria.
Squamous cell carcinoma and adenocarcinoma: rare bladder
tumours
Prostate cancer
Penile cancers: SCC

31
32
33
34
35
36
37

Renal disease

Glomerulonephritis

38
39

Stones

Microscopic haematuria common

40
41

Structural
abnormalities

42-44 0 / 3

Benign prostatic hyperplasia (BPH) causes haematuria due to


hypervascularity of the prostate gland
Cystic renal lesions e.g. polycystic kidney disease
Vascular malformations
Renal vein thrombosis due to renal cell carcinoma

45
46
47
48

Coagulopathy

Drugs

49

Causes bleeding of underlying lesions

50
51

Cause tubular necrosis or interstitial nephritis: aminoglycosides,


chemotherapy
Interstitial nephritis: penicillin, sulphonamides, and NSAIDs
Anticoagulants

52
53
54
55

Benign

Exercise

56
57

Gynaecological

Iatrogenic

Endometriosis: flank pain, dysuria, and haematuria that is cyclical

Catheterisation
Radiotherapy; cystitis, severe haemorrhage, bladder necrosis

http://www.emrcs.com/question/question.php?q=0

1/2

13/07/2015

Pseudohaematuria

For example following consumption of beetroot

References
Http://bestpractice.bmj.com/best-practice/monograph/316/overview/aetiology.html
Rate question:

Next question

Comment on this question

All contents of this site are 2012 E-Medical Revision Ltd

http://www.emrcs.com/question/question.php?q=0

Terms and Conditions

Privacy policy

2/2

13/07/2015

Reference ranges

Previous1 / 3

Question 58-60 of 154

Next

Question stats

End and review

Score: 40.9%

Average score for registered users:

1
2

Theme: Thyroid neoplasms


A. Follicular carcinoma

58

78.1%

59

52.8%

60

64%

B. Follicular adenoma

3
4
5

C. Papillary carcinoma

D. Papillary adenoma

Search

E. Anaplastic carcinoma

7-9 1 / 3
Go

F. Medullary carcinoma

10
11

Please select the most likely underlying diagnosis for the thyroid masses described. Each
option may be used once, more than once or not at all.

12
13
14
15

58.

A 78 year old lady presents to the surgical clinic with symptoms of both
dysphagia and dyspnoea. On examination there is a large mass in the neck
that moves on swallowing. CT scanning of the neck shows a locally infiltrative
lesion arising from the thyroid and invading the strap muscles.

16
17
18
19-21 1 / 3

You answered Follicular carcinoma

22

The correct answer is Anaplastic carcinoma

23

Marked local invasion is a feature of anaplastic carcinoma. These tumours are


more common in elderly females.
59.

A 25 year old female presents with a lump in her neck. On examination she has
a discrete nodule in the right lobe of the thyroid. A fine needle aspirate shows
papillary cells. An adjacent nodule is also sampled which shows similar well
differentiated papillary cells.

24
25
26
27
28
29
30

Papillary carcinoma
Multifocal disease is a recognised feature of papillary lesions. Papillary
adenomas are not really recognised and most well differentiated lesions are
papillary carcinomas.
60.

A 45 year old man presents with a fracture of his right humerus. On


examination there is a lytic lesion of the proximal humerus and a mass in the
neck, this moves on swallowing.

31
32
33
34
35
36
37

You answered Medullary carcinoma

38

The correct answer is Follicular carcinoma


39

Follicular carcinomas are a recognised source of bone metastasis. Up to 60%


will show vascular invasion histologically.

40
41
42-44 0 / 3

Next question

45
46
47

Thyroid neoplasms

48
49

Lesion

Common features

50
51

Follicular
adenoma

Usually present as a solitary thyroid nodule


Malignancy can only be excluded on formal histological assessment

52
53
54

Papillary
carcinoma

Follicular
carcinoma

Usually contain a mixture of papillary and colloidal filled follicles


Histologically tumour has papillary projections and pale empty nuclei
Seldom encapsulated
Lymph node metastasis predominate
Haematogenous metastasis rare
Account for 60% of thyroid cancers

55
56
57
58-60 1 / 3

May appear macroscopically encapsulated, microscopically capsular


invasion is seen. Without this finding the lesion is a follicular adenoma.
Vascular invasion predominates

http://www.emrcs.com/question/question.php?q=0

1/2

13/07/2015

Multifocal disease rare


Account for 20% of all thyroid cancers
Anaplastic
carcinoma

Medullary
carcinoma

Most common in elderly females


Local invasion is a common feature
Account for 10% of thyroid cancers
Treatment is by resection where possible, palliation may be achieved
through isthmusectomy and radiotherapy. Chemotherapy is ineffective.

Tumours of the parafollicular cells (C Cells)


C cells derived from neural crest and not thyroid tissue
Serum calcitonin levels often raised
Familial genetic disease accounts for up to 20% cases
Both lymphatic and haematogenous metastasis are recognised, nodal
disease is associated with a very poor prognosis.

Rate question:

Next question

Comment on this question

All contents of this site are 2012 E-Medical Revision Ltd

http://www.emrcs.com/question/question.php?q=0

Terms and Conditions

Privacy policy

2/2

13/07/2015

Reference ranges

Previous

Question 61 of 154

Next

Question stats

End and review

Score: 41.8%
1

21.3%

7.1%

5.7%

A. Squamous cell carcinoma

7%

B. Gastro intestinal stromal tumours

58.8%

C. Carcinoid tumours

58.8% of users answered this


question correctly

Which of these lesions is most closely associated with Barretts oesophagus?

D. Leiomyosarcoma

6
7-9 1 / 3
10

E. Adenocarcinoma

11

Search
Next question

12
Go

13

Theme from September 2014 Exam


Barretts oesophagus is most closely associated with adenocarcinoma, and it confers a 30
fold increased risk of developing the condition.

14
15
16

Oesophageal cancer

17
18

Incidence is increasing
In most cases in the Western world this increase is accounted for by a rise in the
number of cases of adenocarcinoma. In the UK adenocarcinomas account for 65% of
cases.
Barretts oesophagus is a major risk factor for most cases of oesophageal
adenocarcinoma.
In other regions of the world squamous cancer is more common and is linked to
smoking, alcohol intake, diets rich in nitrosamines and achalasia.
Surveillance of Barretts is important, as it imparts a 30 fold increase in cancer risk and
if invasive malignancy is diagnosed early then survival may approach 85% at 5 years.

19-21 1 / 3
22
23
24
25
26
27
28
29

Diagnosis

30

Upper GI endoscopy is the first line test


Contrast swallow may be of benefit in classifying benign motility disorders but has no
place in the assessment of tumours
Staging is initially undertaken with CT scanning of the chest, abdomen and pelvis. If
overt metastatic disease is identified using this modality then further complex imaging
is unnecessary
If CT does not show metastatic disease, then local stage may be more accurately
assessed by use of endoscopic ultrasound.
Staging laparoscopy is performed to detect occult peritoneal disease. PET CT is
performed in those with negative laparoscopy. Thoracoscopy is not routinely
performed.

31
32
33
34
35
36
37
38
39
40

Treatment
Operable disease is best managed by surgical resection. The most standard procedure is an
Ivor- Lewis type oesophagectomy. This procedure involves the mobilisation of the stomach
and division of the oesophageal hiatus. The abdomen is closed and a right sided
thoracotomy performed. The stomach is brought into the chest and the oesophagus
mobilised further. An intrathoracic oesophagogastric anastomosis is constructed. Alternative
surgical strategies include a transhiatal resection (for distal lesions), a left thoraco-abdominal
resection (difficult access due to thoracic aorta) and a total oesophagectomy (McKeown) with
a cervical oesophagogastric anastomosis.
The biggest surgical challenge is that of anastomotic leak, with an intrathoracic anastomosis
this will result in mediastinitis. With high mortality. The McKeown technique has an
intrinsically lower systemic insult in the event of anastomotic leakage.

41
42-44 0 / 3
45
46
47
48
49
50
51
52

In addition to surgical resection many patients will be treated with adjuvant chemotherapy.

53

Rate question:

54

Next question

55
Comment on this question

56
57
58-60 1 / 3
61

All contents of this site are 2012 E-Medical Revision Ltd

http://www.emrcs.com/question/question.php?q=0

Terms and Conditions

Privacy policy

1/1

13/07/2015

Reference ranges

Previous

Question 62 of 154

Next

A 56 year old man presents with epigastric discomfort and episodes of migratory
thrombophlebitis. On examination he is mildly jaundiced. A CT scan shows peri hilar
lymphadenopathy and a mass in the pancreatic head. Which of the following is the most
likely underlying diagnosis?

Question stats

Score: 40.6%
1

9.3%

55.1%

8.8%

16%

10.8%

55.1% of users answered this


question correctly

A. Squamous cell carcinoma of the pancreas

End and review

B. Adenocarcinoma of the pancreas

6
7-9 1 / 3
10

C. Insulinoma
11

Search

D. Glucagonoma

12

E. Gastrinoma

Go

13
14

Next question

15

Adenocarcinoma of the pancreas is the most likely diagnosis and migratory thrombophlebitis
is associated with the condition. Squamous cells carcinoma is extremely uncommon in the
pancreas. Gastrinoma are extremely rare and thus not the most likely diagnosis.

16
17
18
19-21 1 / 3

Pancreatic cancer

22
23

Adenocarcinoma
Risk factors: Smoking, diabetes, Adenoma, Familial adenomatous polyposis
Mainly occur in the head of the pancreas (70%)
Spread locally and metastasizes to the liver
Carcinoma of the pancreas should be differentiated from other periampullary tumours
with better prognosis

24
25
26
27
28

Clinical features

29
30

Weight loss
Painless jaundice
Epigastric discomfort (pain usually due to invasion of the coeliac plexus is a late
feature)
Pancreatitis
Trousseau's sign: migratory superficial thrombophlebitis

31
32
33
34
35

Investigations

36
37

USS: May miss small lesions


CT Scanning (pancreatic protocol). If unresectable on CT then no further staging
needed
PET/CT for those with operable disease on CT alone
ERCP/ MRI for bile duct assessment
Staging laparoscopy to exclude peritoneal disease

38
39
40
41
42-44 0 / 3
45

Management

46

Head of pancreas: Whipple's resection (SE dumping and ulcers). Newer techniques
include pylorus preservation and SMA/ SMV resection
Carcinoma body and tail: poor prognosis, distal pancreatectomy, if operable
Usually adjuvent chemotherapy for resectable disease
ERCP and stent for jaundice and palliation
Surgical bypass may be needed for duodenal obstruction

47
48
49
50
51
52

Rate question:

53

Next question

54
Comment on this question

55
56
57
58-60 1 / 3
61
62

All contents of this site are 2012 E-Medical Revision Ltd

http://www.emrcs.com/question/question.php?q=0

Terms and Conditions

Privacy policy

1/2

13/07/2015

http://www.emrcs.com/question/question.php?q=0

2/2

13/07/2015

Reference ranges

Previous

Question 63 of 154

Next

Question stats

End and review

Score: 41.4%
1

9.1%

7.6%

13%

A. B-HCG

54.1%

B. Histamine

16.2%

C. Chromogranin A

54.1% of users answered this


question correctly

What is the urinary diagnostic marker for carcinoid syndrome?

D. 5-Hydroxyindoleacetic acid

6
7-9 1 / 3
10

E. 5-Hydroxytryptamine

11

Search
Next question

12
Go

13

Urinary measurement of 5- HIAA is an important part of clinical follow up.

14
15

Carcinoid syndrome

16

Carcinoid tumours secrete serotonin


Originate in neuroendocrine cells mainly in the intestine (midgut-distal ileum/appendix)
Can occur in the rectum, bronchi
Hormonal symptoms mainly occur when disease spreads outside the bowel

17
18
19-21 1 / 3
22
23

Clinical features

24

Onset: years
Flushing face
Palpitations
Pulmonary valve stenosis and tricuspid regurgitation causing dyspnoea
Asthma
Severe diarrhoea (secretory, persists despite fasting)

25
26
27
28
29
30

Investigation

31

5-HIAA in a 24-hour urine collection


Somatostatin receptor scintigraphy
CT scan
Blood testing for chromogranin A

32
33
34
35
36

Treatment

37

Octreotide
Surgical removal

38
39
40

Rate question:

Next question

41
42-44 0 / 3

Comment on this question

45
46
47
48
49
50
51
52
53
54
55
56
57
58-60 1 / 3
61
62
63

http://www.emrcs.com/question/question.php?q=0

1/2

13/07/2015

All contents of this site are 2012 E-Medical Revision Ltd

http://www.emrcs.com/question/question.php?q=0

Terms and Conditions

Privacy policy

2/2

13/07/2015

Reference ranges

Previous

Question 64 of 154

Next

A 22 year old man presents with symptoms of headache, lethargy and confusion. On
examination he is febrile and has a right sided weakness. A CT scan shows a ring enhancing
lesion affecting the left motor cortex. Which of the following is the most likely diagnosis?

Question stats

Score: 42.3%
1

8.5%

58.2%

12.9%

6.8%

13.6%

A. Arteriovenous malformation
58.2% of users answered this
question correctly

B. Cerebral abscess

End and review

C. Herpes simplex encephalitis

6
7-9 1 / 3
10

D. Metastatic renal adenocarcinoma

11

Search

E. Glioblastoma multiforme

12
Go

Next question

13
14

The combination of rapidly progressive neurology, fever and headache is highly suggestive
of cerebral abscess. CT scanning will show a ring enhancing lesion because the intravenous
contrast cannot penetrate the centre of the abscess cavity. HSV encephalitis does not
produce ring enhancing lesions.

15
16
17
18
19-21 1 / 3

Brain abscess

22

CNS abscesses may result from a number of causes including, extension of sepsis
from middle ear or sinuses, trauma or surgery to the scalp, penetrating head injuries
and embolic events from endocarditis.
The presenting symptoms will depend upon the site of the abscess (those in critical
areas e.g.motor cortex) will present earlier. Abscesses have a considerable mass
effect in the brain and raised intra cranial pressure is common.
Although fever, headache and focal neurology are highly suggestive of a brain
abscess the absence of one or more of these does not exclude the diagnosis, fever
may be absent and even if present, is usually not the swinging pyrexia seen with
abscesses at other sites.
Assessment of the patient includes imaging with CT scanning.
Treatment is usually surgical, a craniotomy is performed and the abscess cavity
debrided. The abscess may reform because the head is closed following abscess
drainage.

23
24
25
26
27
28
29
30
31
32
33
34
35

Rate question:
Comment on this question

Next question

36
37
38
39
40
41
42-44 0 / 3
45
46
47
48
49
50
51
52
53
54
55
56
57
58-60 1 / 3
61
62
63
64

http://www.emrcs.com/question/question.php?q=0

1/2

13/07/2015

All contents of this site are 2012 E-Medical Revision Ltd

http://www.emrcs.com/question/question.php?q=0

Terms and Conditions

Privacy policy

2/2

13/07/2015

Reference ranges

Question 65 of 154

Previous

Next

A 56 year old man from Egypt has suffered from recurrent attacks of haematuria for many
years. He presents with suprapubic discomfort and at cystoscopy is found to have a mass
lesion within the bladder. What is the most likely diagnosis?

Question stats

Score: 41.7%
1

53.7%

23.4%

7.7%

8.4%

6.8%

A. Squamous cell carcinoma


53.7% of users answered this
question correctly

B. Transitional cell papilloma

End and review

C. Adenocarcinoma

6
7-9 1 / 3
10

D. Leiomyosarcoma

11

Search

E. Rhabdomyosarcoma

12
Go

Next question

13
14

In Egypt, Schistosomiasis is more common than in the UK and may cause recurrent episodes
of haematuria. In those affected with the condition who develop a bladder neoplasm, an SCC
is the most common type.
Bladder cancer

15
16
17
18
19-21 1 / 3

Bladder cancer is the second most common urological cancer. It most commonly affects
males aged between 50 and 80 years of age. Those who are current, or previous (within 20
years), smokers have a 2-5 fold increased risk of the disease. Exposure to hydrocarbons
such as 2-Naphthylamine increases the risk. Although rare in the UK, chronic bladder
inflammation arising from Schistosomiasis infection remains a common cause of squamous
cell carcinomas, in those countries where the disease is endemic.

22

Benign tumours
Benign tumours of the bladder including inverted urothelial papilloma and nephrogenic
adenoma are uncommon.

27

Bladder malignancies

30

23
24
25
26

28
29

31

Transitional cell carcinoma (>90% of cases)


Squamous cell carcinoma ( 1-7% -except in regions affected by schistosomiasis)
Adenocarcinoma (2%)

32
33
34

Transitional cell carcinomas may arise as solitary lesions, or may be multifocal, owing to the
effect of "field change" within the urothelium. Up to 70% of TCC's will have a papillary growth
pattern. These tumours are usually superficial in location and accordingly have a better
prognosis. The remaining tumours show either mixed papillary and solid growth or pure solid
growths. These tumours are typically more prone to local invasion and may be of higher
grade, the prognosis is therefore worse. Those with T3 disease or worse have a 30% (or
higher) risk of regional or distant lymph node metastasis.

35
36
37
38
39
40

TNM Staging

41
42-44 0 / 3

Stage

Description

T0

No evidence of tumour

Ta

Non invasive papillary carcinoma

T1

Tumour invades sub epithelial connective tissue

T2a

Tumor invades superficial muscularis propria (inner half)

50

T2b

Tumor invades deep muscularis propria (outer half)

51

T3

Tumour extends to perivesical fat

T4

Tumor invades any of the following: prostatic stroma, seminal vesicles, uterus, vagina

T4a

Invasion of uterus, prostate or bowel

T4b

Invasion of pelvic sidewall or abdominal wall

45
46
47
48
49

52
53
54
55
56

N0

No nodal disease

N1

Single regional lymph node metastasis in the true pelvis (hypogastric, obturator,
external iliac, or presacral lymph node)

N2

Multiple regional lymph node metastasis in the true pelvis (hypogastric, obturator,

57
58-60 1 / 3
61
62
63

http://www.emrcs.com/question/question.php?q=0

64

1/2

13/07/2015

external iliac, or presacral lymph node metastasis)


65

N3

Lymph node metastasis to the common iliac lymph nodes

M0

No distant metastasis

M1

Distant disease

Presentation
Most patients (85%) will present with painless, macroscopic haematuria. In those patients
with incidental microscopic haematuria, up to 10% of females aged over 50 will be found to
have a malignancy (once infection excluded).
Staging
Most will undergo a cystoscopy and biopsies or TURBT, this provides histological diagnosis
and information relating to depth of invasion. Locoregional spread is best determined using
pelvic MRI and distant disease CT scanning. Nodes of uncertain significance may be
investigated using PET CT.
Treatment
Those with superficial lesions may be managed using TURBT in isolation. Those with
recurrences or higher grade/ risk on histology may be offered intravesical chemotherapy.
Those with T2 disease are usually offered either surgery (radical cystectomy and ileal
conduit) or radical radiotherapy.
Prognosis
T1

90%

T2

60%

T3

35%

T4a

10-25%

Any T, N1-N2

30%

Rate question:

Next question

Comment on this question

All contents of this site are 2012 E-Medical Revision Ltd

http://www.emrcs.com/question/question.php?q=0

Terms and Conditions

Privacy policy

2/2

13/07/2015

Reference ranges

Previous

Question 66 of 154

Next

A newborn infant is noted to have a unilateral cleft lip only. What is the most likely
explanation for this process?

A. Incomplete fusion of the second branchial arch


B. Incomplete fusion of the nasolabial muscle rings

Question stats

Score: 42.5%
1

19%

42%

12.7%

10%

16.3%

42% of users answered this


question correctly

C. Incomplete fusion of the first branchial arch

End and review

D. Incomplete fusion of the third branchial arch

6
7-9 1 / 3
10

E. Incomplete fusion of the secondary palate

11

Search

12
Next question

Theme from April 2012 Exam


Unilateral isolated cleft lip represents a failure of nasolabial ring fusion. It is not related to
branchial arch fusion. Arch disorders have a far more profound phenotype and malformation
sequences.

Go

13
14
15
16
17
18

Cleft lip and palate

19-21 1 / 3

Cleft lip and palate are the most common congenital deformity affecting the orofacial
structures. Whilst they may be an isolated developmental malformation they are also a
recognised component of more than 200 birth defects. The incidence is as high as 1 in 600
live births. The commonest variants are:

22
23
24
25

Isolated cleft lip (15%)


Isolated cleft palate (40%)
Combined cleft lip and palate (45%)

26
27

The aetiology of the disorder is multifactorial; both genetic (affected first degree relative
increases risk) and environmental factors play a role.

28
29
30

Cleft lip
Cleft lip occurs as a result of disruption of the muscles of the upper lip and nasolabial region.
These muscles comprise a chain of muscles viz; nasolabial, bilabial and labiomental. Defects
may be unilateral or bilateral.
Cleft palate
The primary palate consists of all anatomical structures anterior to the incisive foramen. The
secondary palate lies more posteriorly and is sub divided into the hard and soft palate. Cleft
palate occurs as a result of non fusion of the two palatine shelves. Both hard and soft palate
may be involved. Complete cases are associated with complete separation of the nasal
septum and vomer from the palatine processes.

31
32
33
34
35
36
37
38
39

Treatment
Surgical reconstruction is the mainstay of management. The procedures are planned
according to the extent of malformation and child age. Simple defects are managed as a
single procedure. Complex malformations are usually corrected in stages. Affected
individuals have a higher incidence of hearing and speech problems.

40
41
42-44 0 / 3
45
46

Rate question:
Comment on this question

Next question

47
48
49
50
51
52
53
54
55
56
57
58-60 1 / 3
61
62
63
64

http://www.emrcs.com/question/question.php?q=0

1/2

13/07/2015

65
66

All contents of this site are 2012 E-Medical Revision Ltd

http://www.emrcs.com/question/question.php?q=0

Terms and Conditions

Privacy policy

2/2

13/07/2015

Reference ranges

Previous3 / 3

Question 68-70 of 154

Next

Question stats

End and review

Score: 44.2%

Average score for registered users:

1
2

Theme: Pharyngitis
A. Infectious mononucleosis

68

56%

69

80.9%

70

70.5%

B. Acute bacterial tonsillitis

3
4
5

C. Quinsy

D. Lymphoma

Search

E. Diptheria

7-9 1 / 3
Go

Please select the most likely underlying cause for the following patients presenting with
pharyngitis. Each option may be used once, more than once or not at all.

10
11
12
13
14

68.

An 8 year old child presents with enlarged tonsils that meet in the midline and
are covered with a white film that bleeds when you attempt to remove it. He is
pyrexial but otherwise well.

15
16
17

Acute bacterial tonsillitis

18

Theme from April 2012 Exam


In acute tonsillitis the tonsils will often meet in the midline and may be covered
with a membrane. Individuals who are systemically well are unlikely to have
diptheria.

19-21 1 / 3
22
23
24

69.

A 10 year old child presents with enlarged tonsils that meet in the midline.
Oropharyngeal examination confirms this finding and you also notice peticheal
haemorrhages affecting the oropharynx. On systemic examination he is noted
to have splenomegaly.

25
26
27
28

Infectious mononucleosis

29

A combination of pharyngitis and tonsillitis is often seen in glandular fever.


Antibiotics containing penicillin may produce a rash when given in this situation,
leading to a mistaken label of allergy.

30
31
32

70.

A 19 year old man has had a sore throat for the past 5 days. Over the past 24
hours he has notices increasing and severe throbbing pain in the region of his
right tonsil. He is pyrexial and on examination he is noted to have a swelling of
this area.

33
34
35
36

Quinsy

37

Unilateral swelling and fever is usually indicative of quinsy. Surgical drainage


usually produces prompt resolution of symptoms.

38
39
40

Next question

41
42-44 0 / 3

Acute tonsillitis

45
46
47

Characterised by pharyngitis, fever, malaise and lymphadenopathy.


Over half of all cases are bacterial with Streptococcus pyogenes the most common
organism
The tonsils are typically oedematous and yellow or white pustules may be present
Infectious mononucleosis may mimic the condition.
Treatment with penicillin type antibiotics is indicated for bacterial tonsillitis.
Bacterial tonsillitis may result in local abscess formation (quinsy)

48
49
50
51
52
53
54

Acute streptococcal tonsillitis

http://www.emrcs.com/question/question.php?q=0

55

1/2

13/07/2015
55
56
57
58-60 1 / 3
61
62
63
64
65
66
67
68-70 3 / 3

Image sourced from Wikipedia

Rate question:

Next question

Comment on this question

All contents of this site are 2012 E-Medical Revision Ltd

http://www.emrcs.com/question/question.php?q=0

Terms and Conditions

Privacy policy

2/2

13/07/2015

Reference ranges

Previous

Question 71 of 154

Next

A 4 year old boy is brought to the clinic by his mother who has noticed a small lesion at the
external angle of his eye. On examination there is a small cystic structure which has
obviously been recently infected. On removal of the scab, there is hair visible within the
lesion. What is the most likely diagnosis?

Question stats

Score: 44.9%
1

57.5%

11.4%

12.5%

10.2%

8.4%

57.5% of users answered this


question correctly

A. Dermoid cyst

End and review

B. Desmoid cyst

6
7-9 1 / 3
10

C. Sebaceous cyst
11

Search

D. Epidermoid cyst

12

E. Keratoacanthoma

Go

13
Next question

14
15

Dermoid cysts occur at sites of embryonic fusion and may contain multiple cell types.
They occur most often in children.

16
17
18

The lesion is unlikely to be a desmoid cyst as these are seldom located either at this site or
in this age group. In addition they do not contain hair. Sebaceous cysts will usually have a
punctum and contain a cheesy material. Epidermoid cysts contain keratin plugs.

19-21 1 / 3
22
23
24

Dermoid cysts

25

A cutaneous dermoid cyst may develop at sites of embryonic developmental fusion. They are
most common in the midline of the neck, external angle of the eye and posterior to the pinna
of the ear. They typically have multiple inclusions such as hair follicles that bud out from its
walls. They may develop at other sites such as the ovary and in these sites are synonymous
with teratomas.

26
27
28
29
30

A desmoid tumour is a completely different entity and may be classified either as low grade
fibrosarcomas or non aggressive fibrous tumours. They commonly present as large
infiltrative masses. They may be divided into abdominal, extra abdominal and intra
abdominal. All types share the same biological features. Extra abdominal desmoids have an
equal sex distribution and primarily arise in the musculature of the shoulder, chest wall, back
and thigh. Abdominal desmoids usually arise in the musculoaponeurotic structures of the
abdominal wall. Intra abdominal desmoids tend to occur in the mesentery or pelvic side walls
and occur most frequently in patients with familial adenomatous polyposis coli syndrome.

31
32
33
34
35
36
37

Rate question:
Comment on this question

Next question

38
39
40
41
42-44 0 / 3
45
46
47
48
49
50
51
52
53
54
55
56
57
58-60 1 / 3
61
62
63
64

http://www.emrcs.com/question/question.php?q=0

1/2

13/07/2015

65
66
67
68-70 3 / 3
71

All contents of this site are 2012 E-Medical Revision Ltd

http://www.emrcs.com/question/question.php?q=0

Terms and Conditions

Privacy policy

2/2

13/07/2015

Reference ranges

Previous

Question 72 of 154

Next

Which of the following tumours are most likely to give rise to para-aortic nodal metastasis
early?

A. Ovarian
B. Bladder

Question stats

Score: 43.8%
1

53.2%

10.5%

13.1%

10.1%

13.2%

53.2% of users answered this


question correctly

C. Rectal

End and review

D. Caecal

6
7-9 1 / 3
10

E. Cervical

11

Search

12
Next question

Go

13
14

Theme from 2009 Exam

15

Ovarian tumours are supplied by the ovarian vessels, these branch directly from the aorta.
The cervix drains to the internal and external iliac nodes.

16
17
18

Para-aortic lymphadenopathy

19-21 1 / 3

Organ sites that may metastasise (early) to the para-aortic lymph nodes:

22
23

Testis
Ovary
Uterine fundus

24
25

Many other organs may result in para-aortic nodal disease. However, these deposits will
represent a much later stage of the disease, since other nodal stations are involved earlier.

26
27
28

Rate question:
Comment on this question

Next question

29
30
31
32
33
34
35
36
37
38
39
40
41
42-44 0 / 3
45
46
47
48
49
50
51
52
53
54
55
56
57
58-60 1 / 3
61
62
63
64

http://www.emrcs.com/question/question.php?q=0

1/2

13/07/2015

65
66
67
68-70 3 / 3
71
72

All contents of this site are 2012 E-Medical Revision Ltd

http://www.emrcs.com/question/question.php?q=0

Terms and Conditions

Privacy policy

2/2

13/07/2015

Reference ranges

Previous

Question 73 of 154

Next

Question stats

End and review

Score: 43.2%
1

16.3%

6.4%

49.1%

A. It may be treated by extrapleural pneumonectomy.

12%

B. It is linked to asbestos exposure.

16.2%

C. It is linked to cigarette smoking independent of asbestos exposure.

49.1% of users answered this


question correctly

Which of the following statements relating to malignant mesothelioma is false?

D. It may occur intra abdominally.

6
7-9 1 / 3
10

E. It is relatively resistant to radiotherapy

11

Search
Next question

12
Go

13

It is not linked to cigarette smoking. When identified at an early stage a radical resection is
the favored option. Radiotherapy is often given perioperatively. However, it is not a
particularly radiosensitive tumour. Combination chemotherapy gives some of the best results
and most regimes are cisplatin based.

14
15
16
17

Occupational cancers

18
19-21 1 / 3

Occupational cancers accounted for 5.3% cancer deaths in 2005.


In men the main cancers include:

22
23

Mesothelioma
Bladder cancer
Non melanoma skin cancer
Lung cancer
Sino nasal cancer

24
25
26
27
28

Occupations with high levels of occupational tumours include:

29

Construction industry
Working with coal tar and pitch
Mining
Metalworkers
Working with asbestos (accounts for 98% of all mesotheliomas)
Working in rubber industry

30
31
32
33
34

Shift work has been linked to breast cancer in women (Health and safety executive report
RR595).

35
36
37

The latency between exposure and disease is typically 15 years for solid tumours and 20 for
leukaemia.

38
39

Many occupational cancers are otherwise rare. For example sino nasal cancer is an
uncommon tumour, 50% will be SCC. They are linked to conditions such as wood dust
exposure and unlike lung cancer is not strongly linked to cigarette smoking. Another typical
occupational tumour is angiosarcoma of the liver which is linked to working with vinyl chloride.
Again in the non occupational context this is an extremely rare sporadic tumour.

40
41
42-44 0 / 3
45
46

Rate question:
Comment on this question

Next question

47
48
49
50
51
52
53
54
55
56
57
58-60 1 / 3
61
62
63
64

http://www.emrcs.com/question/question.php?q=0

1/2

13/07/2015

65
66
67
68-70 3 / 3
71
72
73

All contents of this site are 2012 E-Medical Revision Ltd

http://www.emrcs.com/question/question.php?q=0

Terms and Conditions

Privacy policy

2/2

13/07/2015

Reference ranges

Previous0 / 3

Question 74-76 of 154

Next

Question stats

End and review

Score: 41.7%

Average score for registered users:

1
2

Theme: Thyroid disorders


A. Sick euthyroid

74

73.3%

75

84.6%

76

25.8%

B. Hyperthyroidism

3
4
5

C. Hypothyroidism

D. Normal euthyroid

Search

E. Anxiety state

7-9 1 / 3
Go

F. Factitious hyperthyroidism

10
11

For each of the scenarios please match the scenario with the most likely underlying
diagnosis. Each answer may be used once, more than once or not at all.

12
13
14
15

74.

A 33 year old man is recovering following a protracted stay on the intensive


care unit recovering from an anastomotic leak following a difficult trans hiatal
oesophagectomy. His progress is slow, and the intensive care doctors receive
the following thyroid function test results:
TSH

1.0 u/L

16
17
18
19-21 1 / 3

Free T4 8

22

T3

23

1.0 (1.2-3.1 normal)

24
25

You answered Hypothyroidism

26

The correct answer is Sick euthyroid

27

Theme from April 2012 Exam


Sick euthyroid syndrome is caused by systemic illness. With this, the patient
may have an apparently low total and free T4 and T3, with a normal or low
TSH. Note that the levels are only mildly below normal.
75.

A 28 year old female presents to the general practitioner with symptoms of


fever and diarrhoea. As part of her diagnostic evaluation the following thyroid
function tests are obtained:
TSH

< 0.01

29
30
31
32
33
34

Free T4 30
T3

28

35

4.0

36
37

You answered Sick euthyroid

38
39

The correct answer is Hyperthyroidism

40

The symptoms are suggestive of hyperthyroidism. This is supported by the


abnormal blood results; suppressed TSH with an elevated T3 and T4.

41
42-44 0 / 3

76.

A 19 year old lady presents with palpitations. The medical officer takes a blood
sample for thyroid function tests. The following results are obtained:
TSH

45
46

> 6.0

47
Free T4 20

48
T3

2.0

49
50

You answered Hyperthyroidism

51
52

The correct answer is Hypothyroidism

53

An elevated TSH with normal T4 indicates partial thyroid failure. This is caused
by Hashimotos, drugs (lithium, antithyroids) and dyshormogenesis.

54
55
56

Next question

57
58-60 1 / 3

Hyperthyroidism

61
62
63

Causes of hyperthyroidism include:

http://www.emrcs.com/question/question.php?q=0

64

1/2

13/07/2015

Diffuse toxic goitre (Graves Disease)


Toxic nodular goitre
Toxic nodule
Rare causes

65
66
67
68-70 3 / 3
71

Graves disease
Graves disease is characterised by a diffuse vascular goitre that appears at the same time
as the clinical manifestations of hyperthyroidism. It is commonest in younger females and
may be associated with eye signs. Thyrotoxic symptoms will predominate. Up to 50% of
patients will have a familial history of autoimmune disorders. The glandular hypertrophy and
hyperplasia occur as a result of the thyroid stimulating effects of the TSH receptor
antibodies.

72
73
74-76 0 / 3

Toxic nodular goitre


In this disorder the goitre is present for a long period of time prior to the development of
clinical symptoms. In most goitres the nodules are inactive and in some cases it is the
internodular tissue that is responsible for the goitre.
Toxic nodule
Overactive, autonomously functioning nodule. It may occur as part of generalised nodularity
or be a true toxic adenoma. The TSH levels are usually low as the autonomously functioning
thyroid tissue will exert a negative feedback effect.
Signs and symptoms
Symptoms

Signs

Lethargy

Tachycardia

Emotionally labile

Agitation

Heat intolerance

Hot, moist palms

Weight loss

Exopthalmos

Excessive appetite

Thyroid goitre and bruit

Palpitations

Lid lag/retraction

Diagnosis
The most sensitive test for diagnosing hyperthyroidism is plasma T3 (which is raised). Note in
hypothyroidism the plasma T4 and TSH are the most sensitive tests. A TSH level of <0.5U/L
suggests hyperthyroidism. TSH receptor antibodies may be tested for in the diagnosis of
Graves.
Treatment
First line treatment for Graves disease is usually medical and the block and replace regime is
the favored option. Carbimazole is administered at higher doses and thyroxine is
administered orally. Patient are maintained on this regime for between 6 and 12 months.
Attempts are then made to wean off medication. Where relapse then occurs the options are
between ongoing medical therapy, radioiodine or surgery.
Rate question:

Next question

Comment on this question

All contents of this site are 2012 E-Medical Revision Ltd

http://www.emrcs.com/question/question.php?q=0

Terms and Conditions

Privacy policy

2/2

13/07/2015

Reference ranges

Previous0 / 3

Question 74-76 of 154

Next

Question stats

End and review

Score: 41.7%

Average score for registered users:

1
2

Theme: Thyroid disorders


A. Sick euthyroid

74

73.3%

75

84.6%

76

25.8%

B. Hyperthyroidism

3
4
5

C. Hypothyroidism

D. Normal euthyroid

Search

E. Anxiety state

7-9 1 / 3
Go

F. Factitious hyperthyroidism

10
11

For each of the scenarios please match the scenario with the most likely underlying
diagnosis. Each answer may be used once, more than once or not at all.

12
13
14
15

74.

A 33 year old man is recovering following a protracted stay on the intensive


care unit recovering from an anastomotic leak following a difficult trans hiatal
oesophagectomy. His progress is slow, and the intensive care doctors receive
the following thyroid function test results:
TSH

1.0 u/L

16
17
18
19-21 1 / 3

Free T4 8

22

T3

23

1.0 (1.2-3.1 normal)

24
25

You answered Hypothyroidism

26

The correct answer is Sick euthyroid

27

Theme from April 2012 Exam


Sick euthyroid syndrome is caused by systemic illness. With this, the patient
may have an apparently low total and free T4 and T3, with a normal or low
TSH. Note that the levels are only mildly below normal.
75.

A 28 year old female presents to the general practitioner with symptoms of


fever and diarrhoea. As part of her diagnostic evaluation the following thyroid
function tests are obtained:
TSH

< 0.01

29
30
31
32
33
34

Free T4 30
T3

28

35

4.0

36
37

You answered Sick euthyroid

38
39

The correct answer is Hyperthyroidism

40

The symptoms are suggestive of hyperthyroidism. This is supported by the


abnormal blood results; suppressed TSH with an elevated T3 and T4.

41
42-44 0 / 3

76.

A 19 year old lady presents with palpitations. The medical officer takes a blood
sample for thyroid function tests. The following results are obtained:
TSH

45
46

> 6.0

47
Free T4 20

48
T3

2.0

49
50

You answered Hyperthyroidism

51
52

The correct answer is Hypothyroidism

53

An elevated TSH with normal T4 indicates partial thyroid failure. This is caused
by Hashimotos, drugs (lithium, antithyroids) and dyshormogenesis.

54
55
56

Next question

57
58-60 1 / 3

Hyperthyroidism

61
62
63

Causes of hyperthyroidism include:

http://www.emrcs.com/question/question.php?q=0

64

1/2

13/07/2015

Diffuse toxic goitre (Graves Disease)


Toxic nodular goitre
Toxic nodule
Rare causes

65
66
67
68-70 3 / 3
71

Graves disease
Graves disease is characterised by a diffuse vascular goitre that appears at the same time
as the clinical manifestations of hyperthyroidism. It is commonest in younger females and
may be associated with eye signs. Thyrotoxic symptoms will predominate. Up to 50% of
patients will have a familial history of autoimmune disorders. The glandular hypertrophy and
hyperplasia occur as a result of the thyroid stimulating effects of the TSH receptor
antibodies.

72
73
74-76 0 / 3

Toxic nodular goitre


In this disorder the goitre is present for a long period of time prior to the development of
clinical symptoms. In most goitres the nodules are inactive and in some cases it is the
internodular tissue that is responsible for the goitre.
Toxic nodule
Overactive, autonomously functioning nodule. It may occur as part of generalised nodularity
or be a true toxic adenoma. The TSH levels are usually low as the autonomously functioning
thyroid tissue will exert a negative feedback effect.
Signs and symptoms
Symptoms

Signs

Lethargy

Tachycardia

Emotionally labile

Agitation

Heat intolerance

Hot, moist palms

Weight loss

Exopthalmos

Excessive appetite

Thyroid goitre and bruit

Palpitations

Lid lag/retraction

Diagnosis
The most sensitive test for diagnosing hyperthyroidism is plasma T3 (which is raised). Note in
hypothyroidism the plasma T4 and TSH are the most sensitive tests. A TSH level of <0.5U/L
suggests hyperthyroidism. TSH receptor antibodies may be tested for in the diagnosis of
Graves.
Treatment
First line treatment for Graves disease is usually medical and the block and replace regime is
the favored option. Carbimazole is administered at higher doses and thyroxine is
administered orally. Patient are maintained on this regime for between 6 and 12 months.
Attempts are then made to wean off medication. Where relapse then occurs the options are
between ongoing medical therapy, radioiodine or surgery.
Rate question:

Next question

Comment on this question

All contents of this site are 2012 E-Medical Revision Ltd

http://www.emrcs.com/question/question.php?q=0

Terms and Conditions

Privacy policy

2/2

13/07/2015

Reference ranges

Question 1 of 78

Next

Question stats

Score: 100%

22.1%

23.3%

14.4%

A. Vitiligo

14.2%

B. Molluscum contagiosum

26%

C. Lichen planus

26% of users answered this


question correctly

Which of the following conditions is least likely to exhibit the Koebner phenomenon?

D. Psoriasis

End and review

E. Lupus vulgaris
Search
Next question
Go

Lupus vulgaris is not associated with the Koebner phenomenon.


Koebner phenomenon
The Koebner phenomenon describes skin lesions which appear at the site of injury. It is seen
in:
Psoriasis
Vitiligo
Warts
Lichen planus
Lichen sclerosus
Molluscum contagiosum

Rate question:

Next question

Comment on this question

All contents of this site are 2012 E-Medical Revision Ltd

http://www.emrcs.com/question/question.php?q=0

Terms and Conditions

Privacy policy

1/1

13/07/2015

Reference ranges

Previous

Question 3 of 78

Next

A 20 year old man is suspected of having an inflamed Meckels diverticulum. At which of the
following locations is it most likely to be found?

A. Approximately 60 cm distal to the ileo-caecal valve


B. Approximately 60 cm proximal to the ileocaecal valve

Question stats

End and review

Score: 66.7%
1

16.9%

52.4%

10%

12%

8.7%

52.4% of users answered this


question correctly

C. Approximately 200cm distal to the ileocaecal valve


D. Approximately 200cm proximal to the ileocaecal valve
E. 50cm distal to the DJ flexure

Search
Go

Next question

Rule of 2's
2% of population
2 inches (5cm) long
2 feet (60 cm) from the ileocaecal valve
2 x's more common in men
2 tissue types involved

They are typically found 2 feet proximal to the ileocaecal valve (or approximately 60cm).
Meckel's diverticulum

Congenital abnormality resulting in incomplete obliteration of the vitello-intestinal duct


Normally, in the foetus, there is an attachment between the vitello-intestinal duct and
the yolk sac.This disappears at 6 weeks gestation.
The tip is free in majority of cases.
Associated with enterocystomas, umbilical sinuses, and omphaloileal fistulas.
Arterial supply: omphalomesenteric artery.
2% of population, 2 inches long, 2 feet from the ileocaecal valve.
Typically lined by ileal mucosa but ectopic gastric mucosa can occur, with the risk of
peptic ulceration. Pancreatic and jejunal mucosa can also occur.

Clinical
Normally asymptomatic and an incidental finding.
Complications are the result of obstruction, ectopic tissue, or inflammation.
Removal if narrow neck or symptomatic. Options are between wedge excision or formal
small bowel resection and anastomosis.

Rate question:

Next question

Comment on this question

All contents of this site are 2012 E-Medical Revision Ltd

http://www.emrcs.com/question/question.php?q=0

Terms and Conditions

Privacy policy

1/1

13/07/2015

Reference ranges

Question 4 of 78

Previous

Next

A 43 year old lady presents with severe chest pain. Investigations demonstrate a dissecting
aneurysm of the ascending aorta which originates at the aortic valve. What is the optimal
long term treatment?

Question stats

End and review

Score: 50%
1

15.3%

11.2%

8.8%

14.9%

49.7%

A. Endovascular stent
49.7% of users answered this
question correctly

B. Medical therapy with beta blockers


C. Medical therapy with ACE inhibitors
D. Sutured aortic repair

Search

E. Aortic root replacement

Go

Next question

Proximal aortic dissections are generally managed with surgical aortic root replacement. The
proximal origin of the dissection together with chest pain (which may occur in all types of
aortic dissection) raises concerns about the possibility of coronary ostial involvement (which
precludes stenting). There is no role for attempted suture repair in this situation.
Aortic dissection

More common than rupture of the abdominal aorta


33% of patients die within the first 24 hours, and 50% die within 48 hours if no
treatment received
Associated with hypertension
Features of aortic dissection: tear in the intimal layer, followed by formation and
propagation of a subintimal hematoma. Cystic medial necrosis (Marfan's)
Most common site of dissection: 90% occurring within 10 centimetres of the aortic
valve

Stanford Classification
Type

Location

Treatment

Ascending aorta/ aortic root

Surgery- aortic root replacement

Descending aorta

Medical therapy with antihypertensives

DeBakey classification
Type

Site affected

Ascending aorta, aortic arch, descending aorta

II

Ascending aorta only

III

Descending aorta distal to left subclavian artery

Clinical features
Tearing, sudden onset chest pain (painless 10%)
Hypertension or Hypotension
A blood pressure difference greater than 20 mm Hg
Neurologic deficits (20%)

Investigations
CXR: widened mediastinum, abnormal aortic knob, ring sign, deviation of the
trachea/oesophagus
CT angiography of the thoracic aorta
MRI angiography
Conventional angiography (now rarely used diagnostically)

Management
Beta-blockers: aim HR 60-80 bpm and systolic BP 100-120 mm Hg
For type A dissections the standard of care is aortic root replacement

http://www.emrcs.com/question/question.php?q=0

1/2

13/07/2015

Rate question:

Next question

Comment on this question

All contents of this site are 2012 E-Medical Revision Ltd

http://www.emrcs.com/question/question.php?q=0

Terms and Conditions

Privacy policy

2/2

13/07/2015

Reference ranges

Previous

Question 5 of 78

Next

A 45 year old lady presents with a pathological fracture of her femoral shaft. She is a poor
historian, but it transpires that she underwent a thyroidectomy 1 year previously. She has no
other illness or co-morbidities. What is the most likely underlying diagnosis?

Question stats

End and review

Score: 40%
1

9.9%

8.5%

14%

58.6%

9%

A. Hyperparathyroidism
58.6% of users answered this
question correctly

B. Metastatic papillary carcinoma of the thyroid


C. Metastatic medullary carcinoma of the thyroid
D. Metastatic follicular carcinoma of the thyroid

Search

E. None of the above

Go

Next question

Follicular carcinomas are a recognised cause of bone metastasis. Papillary lesions typically
spread via the lymphatics.
Thyroid malignancy
Papillary carcinoma
Commonest sub-type
Accurately diagnosed on fine needle aspiration cytology
Histologically, they may demonstrate psammoma bodies (areas of calcification) and so
called 'orphan Annie' nuclei
They typically metastasise via the lymphatics and thus laterally located apparently
ectopic thyroid tissue is usually a metastasis from a well differentiated papillary
carcinoma

Follicular carcinoma
Are less common than papillary lesions
Like papillary tumours, they may present as a discrete nodule. Although they appear
to be well encapsulated macroscopically there is invasion on microscopic evaluation
Lymph node metastases are uncommon and these tumours tend to spread
haematogenously. This translates into a higher mortality rate
Follicular lesions cannot be accurately diagnosed on fine needle aspiration cytology
and thus all follicular FNA's (THY 3) will require at least a hemi thyroidectomy

Anaplastic carcinoma
Less common and tend to occur in elderly females
Disease is usually advanced at presentation and often only palliative decompression
and radiotherapy can be offered.

Medullary carcinoma
These are tumours of the parafollicular cells ( C Cells) and are of neural crest origin.
The serum calcitonin may be elevated which is of use when monitoring for recurrence.
They may be familial and occur as part of the MEN -2A disease spectrum.
Spread may be either lymphatic or haematogenous and as these tumours are not
derived primarily from thyroid cells they are not responsive to radioiodine.

Lymphoma
These respond well to radiotherapy
Radical surgery is unnecessary once the disease has been diagnosed on biopsy
material. Such biopsy material is not generated by an FNA and thus a core biopsy has
to be obtained (with care!).

Rate question:

Next question

Comment on this question

All contents of this site are 2012 E-Medical Revision Ltd

http://www.emrcs.com/question/question.php?q=0

Terms and Conditions

Privacy policy

1/1

13/07/2015

Reference ranges

Previous2 / 3

Question 6-8 of 78

Next

Question stats

End and review

Score: 50%

Average score for registered users:

1
2

Theme: Cardiac murmurs


A. Pulmonary stenosis

74.9%

59.6%

55.6%

B. Mitral regurgitation

3
4
5

C. Tricuspid regurgitation

6-8 2 / 3

D. Aortic stenosis

Search

E. Mitral stenosis

Go

F. Aortic sclerosis
What is the most likely cause of the cardiac murmur in the following patients? Each option
may be used once, more than once or not at all.

6.

A 35 year old Singaporean female attends a varicose vein pre operative clinic.
On auscultation a mid diastolic murmur is noted at the apex. The murmur is
enhanced when the patient lies in the left lateral position.
Mitral stenosis
Theme from September 2011 exam
A mid diastolic murmur at the apex is a classical description of a mitral stenosis
murmur. The most common cause is rheumatic heart disease. Complications of
mitral stenosis include atrial fibrillation, stroke, myocardial infarction and infective
endocarditis.

7.

A 22 year old intravenous drug user is found to have a femoral abscess. The
nursing staff contact the on call doctor as the patient has a temperature of
39oC. He is found to have a pan systolic murmur loudest at the left sternal edge
at the 4th intercostal space.
Tricuspid regurgitation
Intravenous drug users are at high risk of right sided cardiac valvular
endocarditis. The character of the murmur fits with a diagnosis of tricuspid valve
endocarditis.

8.

An 83 year old woman is admitted with a left intertrochanteric neck of femur


fracture. On examination the patient is found to have an ejection systolic murmur
loudest in the aortic region. There is no radiation of the murmur to the carotid
arteries. Her ECG is normal.
You answered Aortic stenosis
The correct answer is Aortic sclerosis
The most likely diagnosis is aortic sclerosis. The main differential diagnosis is of
aortic stenosis, however as there is no radiation of the murmur to the carotids
and the ECG is normal, this is less likely.

Next question

Cardiac murmurs

Type of Murmur

Conditions

Ejection systolic

Aortic stenosis
Pulmonary stenosis, HOCM
ASD, Fallot's

Pan-systolic

Mitral regurgitation
Tricuspid regurgitation
VSD

Late systolic

Mitral valve prolapse


Coarctation of aorta

Early diastolic

Aortic regurgitation

http://www.emrcs.com/question/question.php?q=0

1/2

13/07/2015

Graham-Steel murmur (pulmonary regurgitation)


Mid diastolic

Mitral stenosis
Austin-Flint murmur (severe aortic regurgitation)

Rate question:

Next question

Comment on this question

All contents of this site are 2012 E-Medical Revision Ltd

http://www.emrcs.com/question/question.php?q=0

Terms and Conditions

Privacy policy

2/2

13/07/2015

Reference ranges

Previous

Question 9 of 78

Next

A 55 year old lady presents with discomfort in the right breast. On clinical examination a small
lesion is identified and clinical appearances suggest fibroadenoma. Imaging confirms the
presence of a fibroadenoma alone. A core biopsy is taken, this confirms the presence of the
fibroadenoma. However, the pathologist notices that a small area of lobular carcinoma in situ
is also present in the biopsy. What is the best management?

Question stats

Score: 55.6%
1

6.6%

12.4%

13.5%

43.3%

24.2%

24.2% of users answered this


question correctly

A. Whole breast irradiation

End and review

5
6-8 2 / 3
9

B. Simple mastectomy
C. Mastectomy and sentinal lymph node biopsy

Search

D. Wide local excision and sentinel lymph node biopsy

Go

E. Breast MRI scan


Next question

Lobular carcinoma in situ has a low association with invasive malignancy. It is seldom
associated with microcalcification and therefore MRI is the best tool for determining disease
extent. Resection of in situ disease is not generally recommended and most surgeons would
simply pursue a policy of close clinical and radiological follow up.
Lobular carcinoma of the breast
Lobular breast cancers are less common than their ductal counterparts. They typically
present differently, the mass is usually more diffuse and less obvious on the usual imaging
modalities of ultrasound and mammography. This is significant since the disease may be
understaged resulting in inadequate treatment when wide local excision is undertaken.
In women with invasive lobular carcinoma it is usually safest to perform an MRI scan of the
breast, if breast conserving surgery is planned.
Lobular carcinomas are also more likely to be multifocal and metastasise to the contralateral
breast.
Lobular carcinoma in situ is occasionally diagnosed incidentally on core biopsies. Unlike
DCIS, lobular carcinoma in situ is far less strongly associated with foci of invasion and is
usually managed by close monitoring.
Rate question:

Next question

Comment on this question

All contents of this site are 2012 E-Medical Revision Ltd

http://www.emrcs.com/question/question.php?q=0

Terms and Conditions

Privacy policy

1/1

13/07/2015

Reference ranges

Previous

Question 10 of 78

Next

Which one of the following may be associated with an increased risk of venous
thromboembolism?

A. Diabetes
B. Cannula

Question stats

Score: 50%
1

16.9%

17.2%

9.5%

45.8%

10.6%

45.8% of users answered this


question correctly

C. Hyperthyroidism

End and review

D. Tamoxifen

5
6-8 2 / 3
9
10

E. Amiodarone

Search
Go

Next question

Consider thromboembolism in breast cancer patients on tamoxifen!


Venous thromboembolism: risk factors
Common predisposing factors include malignancy, pregnancy and the period following an
operation. The comprehensive list below is partly based on the 2010 SIGN venous
thromboembolism (VTE) guidelines:
General
increased risk with advancing age
obesity
family history of VTE
pregnancy (especially puerperium)
immobility
hospitalisation
anaesthesia
central venous catheter: femoral >> subclavian

Underlying conditions
malignancy
thrombophilia: e.g. Activated protein C resistance, protein C and S deficiency
heart failure
antiphospholipid syndrome
Behcet's
polycythaemia
nephrotic syndrome
sickle cell disease
paroxysmal nocturnal haemoglobinuria
hyperviscosity syndrome
homocystinuria

Medication
combined oral contraceptive pill: 3rd generation more than 2nd generation
hormone replacement therapy
raloxifene and tamoxifen
antipsychotics (especially olanzapine) have recently been shown to be a risk factor

SIGN also state that the following are risk factors for recurrent VTE:
previous unprovoked VTE
male sex
obesity
thrombophilias

Rate question:

Next question

Comment on this question

All contents of this site are 2012 E-Medical Revision Ltd

http://www.emrcs.com/question/question.php?q=0

Terms and Conditions

Privacy policy

1/1

13/07/2015

Reference ranges

Question 11 of 78

Previous

Next

A 42 year old lady is investigated for symptoms of irritability and altered bowel habit. On
examination she is noted to have a smooth enlargement of the thyroid gland. As part of her
investigations thyroid function tests are requested, these are as follows:

Question stats

End and review

Score: 45.5%
1

10.5%

9.1%

65.5%

7.5%

7.4%

TSH

0.1 mug/l

Free T4

35 pmol/l

65.5% of users answered this


question correctly

The most likely underlying diagnosis is:

5
6-8 2 / 3
9
10
11

Search

A. Multinodular goitre
B. Follicular carcinoma of the thyroid gland

Go

C. Graves disease
D. Pregnancy
E. None of the above
Next question

TSH receptor antibodies will cause stimulation of the thyroid to synthesise T4. However, this
will have a negative feedback effect on the pituitary causing decrease in TSH levels.
Where hyperthyroidism occurs secondary to pregnancy the TSH is typically elevated.
Thyroid disease
Patients may present with a number of different manifestations of thyroid disease. They can
be broadly sub classified according to whether they are euthyroid or have clinical signs of
thyroid dysfunction. In addition it needs to be established whether they have a mass or not.
Assessment
History
Examination including USS
If a nodule is identified then it should be sampled ideally via an image guided fine
needle aspiration
Radionucleotide scanning is of limited use

Thyroid Tumours
Papillary carcinoma
Follicular carcinoma
Anaplastic carcinoma
Medullary carcinoma
Lymphoma's

Multinodular goitre
One of the most common reasons for presentation
Provided the patient is euthyroid and asymptomatic and no discrete nodules are seen,
they can be reassured.
In those with compressive symptoms surgery is required and the best operation is a
total thyroidectomy.
Sub total resections were practised in the past and simply result in recurrent disease
that requires a difficult revisional resection.

Endocrine dysfunction
In general these patients are managed by physicians initially.
Surgery may be offered alongside radio iodine for patients with Graves disease that
fails with medical management or in patients who would prefer not to be irradiated (e.g.
pregnant women).
Patients with hypothyroidism do not generally get offered a thyroidectomy. Sometimes
people inadvertently get offered resections during the early phase of Hashimotos
thyroiditis, however, with time the toxic phase passes and patients can simply be
managed with thyroxine.

Complications following surgery


Anatomical such as recurrent laryngeal nerve damage.

http://www.emrcs.com/question/question.php?q=0

1/2

13/07/2015

Bleeding. Owing to the confined space haematoma's may rapidly lead to respiratory
compromise owing to laryngeal oedema.
Damage to the parathyroid glands resulting in hypocalcaemia.

Further sources of information


1. http://www.acb.org.uk/docs/TFTguidelinefinal.pdf- Association of Clinical Biochemistry
guidelines for thyroid function tests.
2. British association of endocrine surgeons website- http://www.baets.org.uk

Rate question:

Next question

Comment on this question

All contents of this site are 2012 E-Medical Revision Ltd

http://www.emrcs.com/question/question.php?q=0

Terms and Conditions

Privacy policy

2/2

13/07/2015

Reference ranges

Previous

Question 12 of 78

Next

Question stats

End and review

Score: 50%
1

26.5%

7.4%

9.8%

A. Gastrectomy

7.3%

B. Solid organ transplantation with immunosupression

48.9%

C. Intravenous drug use

48.9% of users answered this


question correctly

Which of the following is not a risk factor for developing tuberculosis?

D. Haematological malignancy

5
6-8 2 / 3
9
10

E. Amiodarone

11

Search
Next question

12
Go

Risk factors for developing active tuberculosis include:


silicosis
chronic renal failure
HIV positive
solid organ transplantation with immunosuppression
intravenous drug use
haematological malignancy
anti-TNF treatment
previous gastrectomy

Tuberculosis
Tuberculosis (TB) is an infection caused by Mycobacterium tuberculosis that most commonly
affects the lungs. Understanding the pathophysiology of TB can be difficult - the key is to
differentiate between primary and secondary disease.
Primary tuberculosis
A non-immune host who is exposed to M. tuberculosis may develop primary infection of the
lungs. A small lung lesion known as a Ghon focus develops. The Ghon focus is composed of
tubercle-laden macrophages. The combination of a Ghon focus and hilar lymph nodes is
known as a Ghon complex
In immunocompetent people the initial lesion usually heals by fibrosis. Those who are
immunocompromised may develop disseminated disease (miliary tuberculosis).
Secondary (post-primary) tuberculosis
If the host becomes immunocompromised the initial infection may become reactivated.
Reactivation generally occurs in the apex of the lungs and may spread locally or to more
distant sites. Possible causes of immunocomprise include:
immunosuppressive drugs including steroids
HIV
malnutrition

The lungs remain the most common site for secondary tuberculosis. Extra-pulmonary
infection may occur in the following areas:
central nervous system (tuberculous meningitis - the most serious complication)
vertebral bodies (Pott's disease)
cervical lymph nodes (scrofuloderma)
renal
gastrointestinal tract

Rate question:

Next question

Comment on this question

All contents of this site are 2012 E-Medical Revision Ltd

http://www.emrcs.com/question/question.php?q=0

Terms and Conditions

Privacy policy

1/1

13/07/2015

Reference ranges

Question 13 of 78

Previous

Next

Question stats

End and review

Score: 46.2%
1

14.3%

6.9%

53.4%

A. There is an association with blood group A

10%

B. Adenocarcinoma is the most common subtype

15.3%

C. Individuals with histological evidence of signet ring cells have a lower


incidence of lymph node metastasis

53.4% of users answered this


question correctly

Which of the following is not true of gastric cancer?

D. Lymphomas account for 5% cases

5
6-8 2 / 3
9
10

E. In Western Countries a more proximal disease distribution has been


noted

11

Search

12
Go

Next question

13

Signet ring cells are features of poorly differentiated gastric cancer associated with a
increased risk of metastatic disease.
Gastric cancer
Overview
There are 700,000 new cases of gastric cancer worldwide each year. It is most common in
Japan and less common in western countries. It is more common in men and incidence rises
with increasing age. The exact cause of many sporadic cancer is not known, however,
familial cases do occur in HNPCC families. In addition, smoking and smoked or preserved
foods increase the risk. Japanese migrants retain their increased risk (decreased in
subsequent generations). The distribution of the disease in western countries is changing
towards a more proximal location (perhaps due to rising obesity).
Pathology
There is some evidence of support a stepwise progression of the disease through intestinal
metaplasia progressing to atrophic gastritis and subsequent dysplasia, through to cancer.
The favoured staging system is TNM. The risk of lymph node involvement is related to size
and depth of invasion; early cancers confined to submucosa have a 20% incidence of lymph
node metastasis. Tumours of the gastro-oesophageal junction are classified as below:
Type
1

True oesophageal cancers and may be associated with Barrett's oesophagus.

Type
2

Carcinoma of the cardia, arising from cardiac type epithelium


or short segments with intestinal metaplasia at the oesophagogastric junction.

Type
3

Sub cardial cancers that spread across the junction. Involve similar nodal stations to
gastric cancer.

Groups for close endoscopic monitoring


Intestinal metaplasia of columnar type
Atrophic gastritis
Low to medium grade dysplasia
Patients who have previously undergone resections for benign peptic ulcer disease
(except highly selective vagotomy).

Referral to endoscopy
Patients of any age with
dyspepsia and any of the
following

Patients without
dyspepsia

Worsening dyspepsia

Chronic gastrointestinal bleeding

Dysphagia

Barretts oesophagus

Dysphagia

Unexplained
abdominal pain or
weight loss

Intestinal metaplasia

Weight loss

Vomiting

Dysplasia

Iron deficiency anaemia

Upper abdominal
mass

Atrophic gastritis

Upper abdominal mass

Jaundice

Patient aged over 55 years with


unexplained or persistent
dyspepsia

http://www.emrcs.com/question/question.php?q=0

1/3

13/07/2015

Upper GI endoscopy performed for dyspepsia. The addition of dye spraying (as shown in the
bottom right) may facilitate identification of smaller tumours

Image sourced from Wikipedia

Staging
CT scanning of the chest abdomen and pelvis is the routine first line staging
investigation in most centres.
Laparoscopy to identify occult peritoneal disease
PET CT (particularly for junctional tumours)

Treatment
Proximally sited disease greater than 5-10cm from the OG junction may be treated by
sub total gastrectomy
Total gastrectomy if tumour is <5cm from OG junction
For type 2 junctional tumours (extending into oesophagus) oesophagogastrectomy is
usual
Endoscopic sub mucosal resection may play a role in early gastric cancer confined to
the mucosa and perhaps the sub mucosa (this is debated)
Lymphadenectomy should be performed. A D2 lymphadenectomy is widely advocated
by the Japanese, the survival advantages of extended lymphadenectomy have been
debated. However, the overall recommendation is that a D2 nodal dissection be
undertaken.
Most patients will receive chemotherapy either pre or post operatively.

Prognosis
UK Data
Disease extent

Percentage 5 year survival

All RO resections

54%

Early gastric cancer

91%

Stage 1

87%

Stage 2

65%

Stage 3

18%

Operative procedure
Total Gastrectomy , lymphadenectomy and Roux en Y anastomosis
General anaesthesia
Prophylactic intravenous antibiotics
Incision: Rooftop.
Perform a thorough laparotomy to identify any occult disease.
Mobilise the left lobe of the liver off the diaphragm and place a large pack over it. Insert a
large self retaining retractor e.g. omnitract or Balfour (take time with this, the set up should
be perfect). Pack the small bowel away.
Begin by mobilising the omentum off the transverse colon.
Proceed to detach the short gastric vessels.
Mobilise the pylorus and divide it at least 2cm distally using a linear cutter stapling device.

http://www.emrcs.com/question/question.php?q=0

2/3

13/07/2015

Continue the dissection into the lesser sac taking the lesser omentum and left gastric artery
flush at its origin.
The lymph nodes should be removed en bloc with the specimen where possible.
Place 2 stay sutures either side of the distal oesophagus. Ask the anaesthetist to pull back
on the nasogastric tube. Divide the distal oesophagus and remove the stomach.
The oesphago jejunal anastomosis should be constructed. Identify the DJ flexure and bring a
loop of jejunum up to the oesophagus (to check it will reach). Divide the jejunum at this point.
Bring the divided jejunum either retrocolic or antecolic to the oesophagus. Anastamose the
oesophagus to the jejunum, using either interrupted 3/0 vicryl or a stapling device. Then
create the remainder of the Roux en Y reconstruction distally.
Place a jejunostomy feeding tube.
Wash out the abdomen and insert drains (usually the anastomosis and duodenal stump).
Help the anaesthetist insert the nasogastric tube (carefully!)
Close the abdomen and skin.
Enteral feeding may commence on the first post-operative day. However, most surgeons will
leave patients on free NG drainage for several days and keep them nil by mouth.

Rate question:

Next question

Comment on this question

All contents of this site are 2012 E-Medical Revision Ltd

http://www.emrcs.com/question/question.php?q=0

Terms and Conditions

Privacy policy

3/3

13/07/2015

Reference ranges

Previous

Question 14 of 78

Next

Question stats

End and review

Score: 42.9%
1

5.1%

9.1%

14.9%

A. Parapharyngeal mass

26.7%

B. Mass at anterior border of masseter

44.2%

C. Mass inferior to the angle of the mandible

26.7% of users answered this


question correctly

What is the most common presentation of a parotid gland tumour?

D. Mass behind the angle of the mandible

5
6-8 2 / 3
9
10

E. Mass anterior to the ear

11

Search
Next question

12
Go

13

Parotid tumours may present at any region in the gland. However, most lesions will be
located behind the angle of the mandible, inferior to the ear lobe. Tumours of the deep lobe
of the parotid may present as a parapharyngeal mass and large lesions may displace the
tonsil.

14

Parotid gland malignancy

Most parotid neoplasms (80%) are benign lesions


Most commonly present with painless mass around the mandible
Up to 30% may present with pain, when this is associated with a discrete mass lesion
in the parotid it usually indicates perineural invasion.
Perineural invasion is very unlikely to occur in association with benign lesions
80% of patients with facial nerve weakness caused by parotid malignancies will have
nodal metastasis and a 5 year survival of 25%

Types of malignancy
Mucoepidermoid
carcinoma

30% of all parotid malignancies


Usually low potential for local invasiveness and metastasis (depends
mainly on grade)

Adenoid cystic
carcinoma

Unpredictable growth pattern


Tendency for perineural spread
Nerve growth may display skip lesions resulting in incomplete excision
Distant metastasis more common (visceral rather than nodal spread)
5 year survival 35%

Mixed tumours

Often a malignancy occurring in a previously benign parotid lesion

Acinic cell
carcinoma

Intermediate grade malignancy


May show perineural invasion
Low potential for distant metastasis
5 year survival 80%

Adenocarcinoma

Develops from secretory portion of gland


Risk of regional nodal and distant metastasis
5 year survival depends upon stage at presentation, may be up to 75% with
small lesions with no nodal involvement

Lymphoma

Large rubbery lesion, may occur in association with Warthins tumours


Diagnosis should be based on regional nodal biopsy rather than parotid
resection Treatment is with chemotherapy (and radiotherapy)

Rate question:

Next question

Comment on this question

All contents of this site are 2012 E-Medical Revision Ltd

http://www.emrcs.com/question/question.php?q=0

Terms and Conditions

Privacy policy

1/1

13/07/2015

Reference ranges

Question 15 of 78

Previous

Next

A 52 year old male is referred to urology clinic with impotence. He is known to have
hypertension. He does not have any morning erections. On further questioning the patient
reports pain in his buttocks, this worsens on mobilising. On examination there is some muscle
atrophy. The penis and scrotum are normal. What is the most likely diagnosis?

Question stats

Score: 40%
1

48%

19.2%

15.1%

7.1%

10.6%

48% of users answered this


question correctly

A. Leriche syndrome

End and review

5
6-8 2 / 3
9

B. S3-S4 cord lesion


10

C. Pudendal nerve lesion


11

Search

D. Psychological impotence

12

E. Beta blocker induced impotence

Go

13
14

Next question

15

Leriche syndrome
Classically, it is described in male patients as a triad of symptoms:
1. Claudication of the buttocks and thighs
2. Atrophy of the musculature of the legs
3. Impotence (due to paralysis of the L1 nerve)

Leriche syndrome, is atherosclerotic occlusive disease involving the abdominal aorta and/or
both of the iliac arteries. Management involves correcting underlying risk factors such as
hypercholesterolaemia and stopping smoking. Investigation is usually with angiography.
Leriche syndrome
Atheromatous disease involving the iliac vessels. Blood flow to the pelvic viscera is
compromised. Patients may present with buttock claudication and impotence (in this
particular syndrome). Diagnostic work up will include angiography, where feasible, iliac
occlusions are usually treated with endovascular angioplasty and stent insertion.
Rate question:

Next question

Comment on this question

All contents of this site are 2012 E-Medical Revision Ltd

http://www.emrcs.com/question/question.php?q=0

Terms and Conditions

Privacy policy

1/1

13/07/2015

Reference ranges

Previous

Question 16 of 78

Next

A 23 year old man presents with diarrhoea and passage of mucous. He is suspected of
having ulcerative colitis. Which of the following is least likely to be associated with this
condition?

Question stats

Score: 37.5%
1

9.8%

8.5%

40.7%

10.9%

30%

A. Superficial mucosal inflammation in the colon


40.7% of users answered this
question correctly

B. Significant risk of dysplasia in long standing disease

End and review

C. Epsiodes of large bowel obstruction during acute attacks

5
6-8 2 / 3
9
10

D. Haemorrhage

11

Search

E. Disease sparing the anal canal

12
Go

Next question

13
14

Large bowel obstruction is not a feature of UC, patients may develop megacolon. However,
this is a different entity both diagnostically and clinically. Ulcerative colitis does not affect the
anal canal and the anal transitional zone. Inflammation is superficial. Dysplasia can occur in
2% overall, but increases significantly if disease has been present over 20 years duration.
Granulomas are features of crohn's disease.

15
16

Other features:
Disease maximal in the rectum and may spread proximally
Contact bleeding
Longstanding UC crypt atrophy and metaplasia/dysplasia

Ulcerative colitis
Ulcerative colitis is a form of inflammatory bowel disease. Inflammation always starts at
rectum, does not spread beyond ileocaecal valve (although backwash ileitis may occur) and
is continuous. The peak incidence of ulcerative colitis is in people aged 15-25 years and in
those aged 55-65 years. It is less common in smokers.
The initial presentation is usually following insidious and intermittent symptoms. Features
include:
bloody diarrhoea
urgency
tenesmus
abdominal pain, particularly in the left lower quadrant
extra-intestinal features (see below)

Questions regarding the 'extra-intestinal' features of inflammatory bowel disease are


common. Extra-intestinal features include sclerosing cholangitis, iritis and ankylosing
spondylitis.
Common to both Crohn's disease
(CD) and Ulcerative colitis (UC)

Notes

Related to
disease
activity

Arthritis: pauciarticular, asymmetric


Erythema nodosum
Episcleritis
Osteoporosis

Arthritis is the most common extraintestinal feature in both CD and UC


Episcleritis is more common in
Crohns disease

Unrelated to
disease
activity

Arthritis: polyarticular, symmetric


Uveitis
Pyoderma gangrenosum
Clubbing
Primary sclerosing cholangitis

Primary sclerosing cholangitis is


much more common in UC
Uveitis is more common in UC

Pathology
Red, raw mucosa, bleeds easily
No inflammation beyond submucosa (unless fulminant disease)
Widespread superficial ulceration with preservation of adjacent mucosa which has the
appearance of polyps ('pseudopolyps')
Inflammatory cell infiltrate in lamina propria
Neutrophils migrate through the walls of glands to form crypt abscesses
Depletion of goblet cells and mucin from gland epithelium

http://www.emrcs.com/question/question.php?q=0

1/2

13/07/2015

Granulomas are infrequent

Barium enema
Loss of haustrations
Superficial ulceration, 'pseudopolyps'
Long standing disease: colon is narrow and short -'drainpipe colon'

Endoscopy
Superficial inflammation of the colonic and rectal mucosa
Continuous disease from rectum proximally
Superficial ulceration, mucosal islands, loss of vascular definition and continuous
ulceration pattern.

Management
Patients with long term disease are at increased risk of development of malignancy
Acute exacerbations are generally managed with steroids, in chronic patients agents
such as azathioprine and infliximab may be used
Individuals with medically unresponsive disease usually require surgery- in the acute
phase a sub total colectomy and end ileostomy. In the longer term a proctectomy will
be required. An ileoanal pouch is an option for selected patients

References
Ford A et al. Ulcerative colitis. BMJ 2013 (346):29-34.
Rate question:

Next question

Comment on this question

All contents of this site are 2012 E-Medical Revision Ltd

http://www.emrcs.com/question/question.php?q=0

Terms and Conditions

Privacy policy

2/2

13/07/2015

Reference ranges

Previous

Question 17 of 78

Next

A 23 year old lady undergoes a total thyroidectomy as treatment for a papillary carcinoma of
the thyroid. The pathologist examines histological sections of the thyroid gland and identifies
a psammoma body. What are these primarily composed of?

Question stats

Score: 41.2%
1

46.3%

10.3%

16.8%

17.4%

9.2%

A. Clusters of calcification
46.3% of users answered this
question correctly

B. Aggregations of neutrophils

End and review

C. Aggregations of macrophages

5
6-8 2 / 3
9
10

D. Giant cells surrounding the tumour

11

Search

E. Clusters of oxalate crystals

12
Go

13

Next question

14
15

Psammoma bodies consist of clusters of microcalcification. They are most commonly seen in
papillary carcinomas.

16
17

Thyroid malignancy
Papillary carcinoma
Commonest sub-type
Accurately diagnosed on fine needle aspiration cytology
Histologically, they may demonstrate psammoma bodies (areas of calcification) and so
called 'orphan Annie' nuclei
They typically metastasise via the lymphatics and thus laterally located apparently
ectopic thyroid tissue is usually a metastasis from a well differentiated papillary
carcinoma

Follicular carcinoma
Are less common than papillary lesions
Like papillary tumours, they may present as a discrete nodule. Although they appear
to be well encapsulated macroscopically there is invasion on microscopic evaluation
Lymph node metastases are uncommon and these tumours tend to spread
haematogenously. This translates into a higher mortality rate
Follicular lesions cannot be accurately diagnosed on fine needle aspiration cytology
and thus all follicular FNA's (THY 3) will require at least a hemi thyroidectomy

Anaplastic carcinoma
Less common and tend to occur in elderly females
Disease is usually advanced at presentation and often only palliative decompression
and radiotherapy can be offered.

Medullary carcinoma
These are tumours of the parafollicular cells ( C Cells) and are of neural crest origin.
The serum calcitonin may be elevated which is of use when monitoring for recurrence.
They may be familial and occur as part of the MEN -2A disease spectrum.
Spread may be either lymphatic or haematogenous and as these tumours are not
derived primarily from thyroid cells they are not responsive to radioiodine.

Lymphoma
These respond well to radiotherapy
Radical surgery is unnecessary once the disease has been diagnosed on biopsy
material. Such biopsy material is not generated by an FNA and thus a core biopsy has
to be obtained (with care!).

Rate question:

Next question

Comment on this question

All contents of this site are 2012 E-Medical Revision Ltd

http://www.emrcs.com/question/question.php?q=0

Terms and Conditions

Privacy policy

1/1

13/07/2015

Reference ranges

Previous

Question 18 of 78

Next

A 52 year old woman attends clinic for investigation of abdominal pain and constipation. On
examination you note blue lines on the gum margin. She mentions that her legs have
become weak in the past few days. What is the most likely diagnosis?

Question stats

Score: 44.4%
1

24.2%

53.6%

5.7%

9.8%

6.6%

A. Acute intermittent porphyria


53.6% of users answered this
question correctly

B. Lead poisoning

End and review

C. Constipation

5
6-8 2 / 3
9
10

D. Guillan Barre syndrome

11

Search

E. Rectal carcinoma

12
Go

13

Next question

14
15

This would be an impressive diagnosis to make in the surgical out patient department! The
combination of abdominal pain and a motor periperal neuropathy, should indicate this
diagnosis. The blue line along the gum margin can occur in up to 20% patients with lead
poisoning.

16
17
18

Lead poisoning
Along with acute intermittent porphyria, lead poisoning should be considered in questions
giving a combination of abdominal pain and neurological signs
Features
abdominal pain
peripheral neuropathy (mainly motor)
fatigue
constipation
blue lines on gum margin (only 20% of adult patients, very rare in children)

Investigations
The blood lead level is usually used for diagnosis. Levels greater than 10 mcg/dl are
considered significant
Full blood count: microcytic anaemia. Blood film shows red cell abnormalities including
basophilic stippling and clover-leaf morphology
Raised serum and urine levels of delta aminolaevulinic acid may be seen making it
sometimes difficult to differentiate from acute intermittent porphyria
Urinary coproporphyrin is also increased (urinary porphobilinogen and uroporphyrin
levels are normal to slightly increased)

Management - various chelating agents are currently used:


Dimercaptosuccinic acid (DMSA)
D-penicillamine
EDTA
Dimercaprol

Rate question:

Next question

Comment on this question

All contents of this site are 2012 E-Medical Revision Ltd

http://www.emrcs.com/question/question.php?q=0

Terms and Conditions

Privacy policy

1/1

13/07/2015

Reference ranges

Previous3 / 3

Question 19-21 of 78

Next

Question stats

End and review

Score: 52.4%

Average score for registered users:

1
2

Theme: Vasculitis
A. Wegeners granulomatosis

19

74.6%

20

82.5%

21

77.7%

B. Polyarteritis nodosa

3
4
5

C. Giant cell arteritis

6-8 2 / 3

D. Takayasu's arteritis

Search

E. Buergers disease

Go

For each of the scenarios provided please select the most likely underlying diagnosis from
the list below. Each option may be used once, more than once or not at all.

10
11
12
13
14

19.

A 20 year old lady is referred to the vascular clinic. She has been feeling
generally unwell for the past six weeks. She works as a typist and has noticed
increasing pain in her forearms whilst working. On examination she has absent
upper limb pulses. Her ESR is measured and mildly elevated.

15
16
17
18

Takayasu's arteritis

19-21 3 / 3

Takayasus arteritis may be divided into acute systemic phases and the chronic
pulseless phase. In the latter part of the disease process the patient may
complain of symptoms such as upper limb claudication. In the later stages of
the condition the vessels will typically show changes of intimal proliferation,
together with band fibrosis of the intima and media.
20.

A 32 year old man presents to the vascular clinic with symptoms of foot pain
during exertion. He is a heavy smoker and has recently tried to stop smoking.
On examination he has normal pulses to the level of the popliteal. However,
foot pulses are absent. A diagnostic angiogram is performed which shows an
abrupt cut off at the level of the anterior tibial artery, together with the
formation of corkscrew shaped collateral vessels distally.
Buergers disease
Buergers disease is most common in young male smokers. This demographic
is changing in those areas where young female smokers are more common. In
the acute lesion the internal elastic lamina of the vessels is usually intact. As
the disease progresses the changes progress to hypercellular occlusive
thrombus. Tortuous corkscrew collaterals may reconstitute patent segments of
the distal tibial or pedal vessels.

21.

A 78 year old man presents with symptoms of headaches and deteriorating


vision. He notices that there is marked pain on the right hand side of his face
when he combs his hair.
Giant cell arteritis
Temporal arteritis may present acutely with symptoms of headache and visual
loss, or with a less acute clinical picture. Sight may be threatened and
treatment with immunosupressants should be started promptly. The often
requested temporal artery biopsy (which can be the bane of many surgeons) is
often non diagnostic and unhelpful.

Next question

Vasculitis

The vasculitides are a group of conditions characterised by inflammation of the blood vessel
walls. This may, in turn, compromise vessel integrity. Constitutional symptoms may be
present. Whilst certain disease subtypes are reported to affect specific vessels, there is
often a degree of overlap clinically.
Vessel diameter and vasculitis classification
Aorta and branches

http://www.emrcs.com/question/question.php?q=0

Takayasu's arteritis
Buergers disease
Giant cell arteritis

1/2

13/07/2015

Large and medium sized arteries

Medium sized muscular arteries

Small muscular arteries

Buergers disease
Giant cell arteritis
Polyarteritis nodosa

Polyarteritis nodosa
Wegeners granulomatosis

Wegeners granulomatosis
Rheumatoid vasculitis

Specific conditions
Takyasu's arteritis

Buergers disease

Giant cell arteritis

Polyarteritis
nodosa

Wegeners
granulomatosis

Inflammatory, obliterative arteritis affecting aorta and branches


Females> Males
Symptoms may include upper limb claudication
Clinical findings include diminished or absent pulses
ESR often affected during the acute phase

Segmental thrombotic occlusions of the small and medium sized


lower limb vessels
Commonest in young male smokers
Proximal pulses usually present, but pedal pulses are lost
An acuter hypercellular occlusive thrombus is often present
Tortuous corkscrew shaped collateral vessels may be seen on
angiography

Systemic granulomatous arteritis that usually affects large and


medium sized vessels
Females > Males
Temporal arteritis is commonest type
Granulomatous lesions may be seen on biopsy (although up to
50% are normal)

Systemic necrotising vasculitis affecting small and medium sized


muscular arteries
Most common in populations with high prevalence of hepatitis B
Renal disease is seen in 70% cases
Angiography may show saccular or fusiform aneurysms and
arterial stenoses

Predominantly affects small and medium sized arteries


Systemic necrotising granulomatous vasculitis
Cutaneous vascular lesions may be seen (ulceration, nodules
and purpura)
Sinus imaging may show mucosal thickening and air fluid levels

Treatment
Conditions such as Buergers disease are markedly helped by smoking cessation.
Immunosupression is the main treatment for vasculitides.
Rate question:

Next question

Comment on this question

All contents of this site are 2012 E-Medical Revision Ltd

http://www.emrcs.com/question/question.php?q=0

Terms and Conditions

Privacy policy

2/2

13/07/2015

Reference ranges

Previous

Question 22 of 78

Next

Question stats

End and review

Score: 50%
1

53.6%

8.7%

13.1%

11.7%

A. First line treatment is with pentavalent antimony.

12.9%

B. Peritoneal contamination with active daughter cysts may complicate


surgery.

53.6% of users answered this


question correctly

A 28 year old man presents with right upper quadrant pain and hydatid disease is suspected.
Which of the following statements relating to the disease is untrue?

C. CT scanning of the liver may show a floating membrane.

5
6-8 2 / 3
9
10

D. Biliary communication with the cysts may occur.

11

Search

E. It is caused by Echinococcus granulosus.

12
Go

13

Next question

14

Drug treatment is with albendazole or mebendazole. Praziquantzel may be used in the pre
operative stages.

15
16
17

Hydatid cysts

18
19-21 3 / 3

Hydatid cysts are endemic in Mediterranean and Middle Eastern countries. They are caused
by the tapeworm parasite Echinococcus granulosus. An outer fibrous capsule is formed
containing multiple small daughter cysts. These cysts are allergens which precipitate a type
1 hypersensitivity reaction.

22

Clinical features are as follows:


Up to 90% cysts occur in the liver and lungs
Can be asymtomatic, or symptomatic if cysts > 5cm in diameter
Morbidity caused by cyst bursting, infection and organ dysfunction (biliary, bronchial,
renal and cerebrospinal fluid outflow obstruction)
In biliary ruputure there may be the classical triad of; biliary colic, jaundice, and
urticaria

CT is the best investigation to differentiate hydatid cysts from amoebic and pyogenic cysts.
Surgery is the mainstay of treatment (the cyst walls must not be ruptured during removal and
the contents sterilised first).
Rate question:

Next question

Comment on this question

All contents of this site are 2012 E-Medical Revision Ltd

http://www.emrcs.com/question/question.php?q=0

Terms and Conditions

Privacy policy

1/1

13/07/2015

Reference ranges

Previous

Question 23 of 78

Next

Question stats

End and review

Score: 47.8%
1

19.4%

19.7%

14.6%

A. Produce nitrogen peroxide as a microbicidal agent

29.6%

B. Not involved in opsonisation

16.7%

C. Deficiency leads to AIDS

29.6% of users answered this


question correctly

Which of the following statements relating to neutrophil polymorphs is true?

D. Neutrophil disorders cause chronic granulomatous diseases

5
6-8 2 / 3
9
10

E. Have a lifespan of 9 hours

11

Search
Next question

12
Go

13

Neutrophils are the main cells of acute inflammation, important action against gram -ve
and +ve bacteria
Appearance of segmented nucleus and granulated cytoplasm
Have a lifespan of 1-3 days (shorter when consumed during septic process, though 9
hours is unusual)
Actions include: movement, opsonise microorganisms, phagocytosis & intracellular
killing of microorganisms via aerobic (produce HYDROGEN PEROXIDE) & anaerobic
mechanisms.
Neutrophil disorders include chronic granulomatous diseases: rare
AIDS associated with T cell deficiency

14
15
16
17
18
19-21 3 / 3
22
23

Acute inflammation
Inflammation is the reaction of the tissue elements to injury. Vascular changes occur,
resulting in the generation of a protein rich exudate. So long as the injury does not totally
destroy the existing tissue architecture, the episode may resolve with restoration of original
tissue architecture.
Vascular changes
Vasodilation occurs and persists throughout the inflammatory phase.
Inflammatory cells exit the circulation at the site of injury.
The equilibrium that balances Starlings forces within capillary beds is disrupted and a
protein rich exudate will form as the vessel walls also become more permeable to
proteins.
The high fibrinogen content of the fluid may form a fibrin clot. This has several
important immunomodulatory functions.

Sequelae
Resolution

Organisation

Suppuration

Progression to chronic
inflammation

Typically occurs with minimal initial injury


Stimulus removed and normal tissue architecture results

Delayed removal of exudate


Tissues undergo organisation and usually fibrosis

Typically formation of an abscess or an empyema


Sequestration of large quantities of dead neutrophils

Coupled inflammatory and reparative activities


Usually occurs when initial infection or suppuration has
been inadequately managed

Causes
Infections e.g. Viruses, exotoxins or endotoxins released by bacteria
Chemical agents
Physical agents e.g. Trauma
Hypersensitivity reactions
Tissue necrosis

Presence of neutrophil polymorphs is a histological diagnostic feature of acute

http://www.emrcs.com/question/question.php?q=0

1/2

13/07/2015

inflammation
Rate question:

Next question

Comment on this question

All contents of this site are 2012 E-Medical Revision Ltd

http://www.emrcs.com/question/question.php?q=0

Terms and Conditions

Privacy policy

2/2

13/07/2015

Reference ranges

Previous

Question 24 of 78

Next

A 33 year old man is involved in a road traffic accident. He is initially stable and transferred
to the accident and emergency department. On arrival he is catheterised. One minute later
he becomes hypotensive, with evidence of angioedema surrounding his penis. What is the
most likely explanation for this event?

Question stats

Score: 50%
1

8%

12.8%

7.5%

63.4%

8.3%

63.4% of users answered this


question correctly

A. Type V latex hypersensitivity reaction

End and review

5
6-8 2 / 3
9

B. Type IV latex hypersensitivity reaction


10

C. Type III latex hypersensitivity reaction


11

Search

D. Type I latex hypersensitivity reaction

12

E. Type II latex hypersensitivity reaction

Go

13
14

Next question

15

Theme from January 2013 exam


Theme from April 2012 Exam
Sudden collapse and angioedema following exposure to latex (of which most urinary
catheters are manufactured) suggests a type I hypersensitivity reaction.

16
17
18
19-21 3 / 3

Hypersensitivity reactions

22
23

The Gell and Coombs classification divides hypersensitivity reactions into 4 types
24

Type I

Type II

Type III

Type IV

Description

Anaphylactic

Cytotoxic

Immune
complex

Delayed type

Mediator

IgE

IgG, IgM

IgG, Ig A,
IgM

T-cells

Antigen

Exogenous

Cell surface

Soluble

Tissues

Response
time

Minutes

Hours

Hours

2-3 days

Examples

Asthma
Hay fever

Autoimmune haemolytic
anaemia
Pemphigus
Goodpasture's

Serum
sickness
SLE
Aspergillosis

Graft versus host


disease
Contact dermatitis

Rate question:

Next question

Comment on this question

All contents of this site are 2012 E-Medical Revision Ltd

http://www.emrcs.com/question/question.php?q=0

Terms and Conditions

Privacy policy

1/1

13/07/2015

Reference ranges

Previous

Question 25 of 78

Next

A 43 year old female develops severe chest wall cellulitis following a mastectomy. On
examination the skin is markedly erythematous. Which of the acute inflammatory mediators
listed below is least likely to produce vasodilation?

Question stats

Score: 48%
1

17.9%

21.2%

14.3%

31.4%

15.3%

A. Complement component C5a


31.4% of users answered this
question correctly

B. Lysosomal compounds

End and review

C. Histamine

5
6-8 2 / 3
9
10

D. Serotonin

11

Search

E. Prostaglandins

12
Go

Next question

13
14

Erythema is a classical feature of acute inflammation. Potent mediators of vascular dilatation


include; histamine, prostaglandins, nitric oxide, platelet activating factor, complement C5a
(and C3a) and lysosomal compounds. Although serotonin is associated with acute
inflammation it is a vasoconstrictor. The effects of serotonin are dependent upon the state of
the vessels in the tissues. Intact and healthy tissues and vessels will respond to a serotonin
infusion with vasodilation (hence the flushing seen in carcinoid syndrome). In contrast it
worsens cardiac ischaemia in myocardial infarcts when released from damaged platelets.

15
16
17
18
19-21 3 / 3
22
23

Acute inflammation

24

Inflammation is the reaction of the tissue elements to injury. Vascular changes occur,
resulting in the generation of a protein rich exudate. So long as the injury does not totally
destroy the existing tissue architecture, the episode may resolve with restoration of original
tissue architecture.

25

Vascular changes
Vasodilation occurs and persists throughout the inflammatory phase.
Inflammatory cells exit the circulation at the site of injury.
The equilibrium that balances Starlings forces within capillary beds is disrupted and a
protein rich exudate will form as the vessel walls also become more permeable to
proteins.
The high fibrinogen content of the fluid may form a fibrin clot. This has several
important immunomodulatory functions.

Sequelae
Resolution

Organisation

Suppuration

Progression to chronic
inflammation

Typically occurs with minimal initial injury


Stimulus removed and normal tissue architecture results

Delayed removal of exudate


Tissues undergo organisation and usually fibrosis

Typically formation of an abscess or an empyema


Sequestration of large quantities of dead neutrophils

Coupled inflammatory and reparative activities


Usually occurs when initial infection or suppuration has
been inadequately managed

Causes
Infections e.g. Viruses, exotoxins or endotoxins released by bacteria
Chemical agents
Physical agents e.g. Trauma
Hypersensitivity reactions
Tissue necrosis

Presence of neutrophil polymorphs is a histological diagnostic feature of acute


inflammation
Rate question:

http://www.emrcs.com/question/question.php?q=0

Next question

1/2

13/07/2015

Comment on this question

All contents of this site are 2012 E-Medical Revision Ltd

http://www.emrcs.com/question/question.php?q=0

Terms and Conditions

Privacy policy

2/2

13/07/2015

Reference ranges

Previous

Question 26 of 78

Next

A 43 year old man presents with a 3 week history of malaise, sore throat, odynophagia and
dysphagia. On examination he is found to have patchy white spots in his oropharynx. An
upper GI endoscopy is performed and similar lesions are identified in the oesophagus. Which
investigation is most likely to identify the underlying pathology in this case?

Question stats

Score: 50%
1

6.8%

22.4%

15.8%

11.9%

43.1%

43.1% of users answered this


question correctly

A. Serum urea and electrolytes

End and review

5
6-8 2 / 3
9

B. Oesophageal biopsy for culture


10

C. Oesophageal biopsy for histology


11

Search

D. Glucose tolerance testing

12

E. Viral serology

Go

13
14

Next question

15

Oesophageal candidiasis is associated with immunosupression; mainly in patients on


chemotherapy, with haematological malignancy, HIV or inhaled steroids. In patients with HIV,
oesophageal candidiasis is part of the spectrum of AIDS defining illnesses and usually
occurs when the CD4 count is less than 200. Others include PCP pneumonia and CMV
infections.

16
17
18
19-21 3 / 3
22

Oesophageal candidiasis

23
24

Characterised by white spots in the oropharynx with extension into the oesophagus.
Associated with broad spectrum antibiotic usage, immunosupression and immunological
disorders.
Patients may present with oropharyngeal symptoms, odynophagia and dysphagia.
Treatment is directed both at the underlying cause (which should be investigated for) and
with oral antifungal agents.
Rate question:

25
26

Next question

Comment on this question

All contents of this site are 2012 E-Medical Revision Ltd

http://www.emrcs.com/question/question.php?q=0

Terms and Conditions

Privacy policy

1/1

13/07/2015

Reference ranges

Previous1 / 3

Question 27-29 of 78

Next

Question stats

End and review

Score: 48.3%

Average score for registered users:

1
2

Theme: Colonic obstruction


A. Malignant obstruction

27

77.4%

28

82.4%

29

63.4%

B. Ogilvies syndrome

3
4
5

C. Volvulus

6-8 2 / 3

D. Diverticular stricture

Search

E. Ischaemic stricture

Go

Please select the most likely cause of obstruction for the situation described. Each option
may be used once, more than once or not at all.

10
11
12
13
14

27.

A 78 year old man has undergone a hemi-arthroplasty for a intracapsular hip


fracture. Post operatively he develops electrolyte derangement and receives
intravenous fluids. Over the previous 24 hours he develops marked abdominal
distension. On examination he has a tense, tympanic abdomen which is not
painful. A contrast enema shows flow of contrast through to the caecum and
through the ileocaecal valve.

15
16
17
18
19-21 3 / 3

You answered Volvulus

22
23

The correct answer is Ogilvies syndrome

24

Patients with electrolyte disturbance and previous surgery may develop colonic
pseudo-obstruction (Ogilvies syndrome). The diagnosis is made using a
contrast enema and treatment is usually directed at the underlying cause with
colonic decompression if indicated.
28.

25
26
27-29 1 / 3

A 67 year old man has had multiple episodes with fever and left iliac fossa pain.
These have usually resolved with courses of intravenous antibiotics. He is
admitted with a history of increasing constipation and abdominal distension. A
contrast x-ray is performed which shows flow of contrast to the sigmoid colon,
here the contrast flows through a long narrow segment of colon into dilated
proximal bowel.
Diverticular stricture
The long history of left iliac fossa pain and development of bowel obstruction
suggests a diverticular stricture. These may contain a malignancy and most will
require resection.

29.

A 78 year old lady from a nursing home is admitted with a 24 hour history of
absolute constipation and abdominal pain. On examination she has a
distended abdomen with a soft mass in her left iliac fossa. An x-ray is
performed which shows a large dilated loop of bowel in the left iliac fossa which
contains a fluid level.
You answered Ogilvies syndrome
The correct answer is Volvulus
Sigmoid volvulus may present with an asymmetrical mass in an elderly patient.
It may contain a fluid level, visible on plain films.

Next question

Colonic obstruction

Cause
Cancer

Features
Usually insidious onset
History of progressive constipation
Systemic features (e.g. anaemia)
Abdominal distension
Absence of bowel gas distal to site of
obstruction

Diverticular

http://www.emrcs.com/question/question.php?q=0

Treatment
Establish diagnosis (e.g.
contrast enema/
endoscopy)
Laparotomy and resection,
stenting, defunctioning
colostomy or bypass

Once diagnosis

1/2

13/07/2015

Diverticular
stricture

Volvulus

Acute
colonic
pseudoobstruction

Usually history of previous acute


diverticulitis
Long history of altered bowel habit
Evidence of diverticulosis on imaging or
endoscopy

Twisting of bowel around its mesentery


Sigmoid colon affected in 76% cases
Patients usually present with abdominal
pain, bloating and constipation
Examination usually shows asymmetrical
distension
Plain X-rays usually show massively dilated
sigmoid colon, loss of haustra and "U"
shape are typical, the loop may contain fluid
levels

Symptoms and signs of large bowel


obstruction with no lesion
Usually associated with metabolic disorders
Usually a cut off in the left colon (82%
cases)
Although abdomen tense and distended, it
is usually not painful
All patients should undergo contrast enema
(may be therapeutic!)

Rate question:

Once diagnosis
established, usually
surgical resection
Colonic stenting should not
be performed for benign
disease
Initial treatment is to
untwist the loop, a flexible
sigmoidoscopy may be
needed
Those with clinical evidence
of ischaemia should
undergo surgery
Patient with recurrent
volvulus should undergo
resection

Colonoscopic
decompression
Correct metabolic disorders
IV neostigmine
Surgery

Next question

Comment on this question

All contents of this site are 2012 E-Medical Revision Ltd

http://www.emrcs.com/question/question.php?q=0

Terms and Conditions

Privacy policy

2/2

13/07/2015

Reference ranges

Previous

Question 30 of 78

Next

A 52 year old man with dyspepsia is found to have a duodenal ulcer. A CLO test is taken and
is positive. Which statement relating to the likely causative organism is false?

A. It is a gram negative organism


B. It preferentially colonises gastric type mucosa

Question stats

Score: 46.7%
1

15.9%

22.6%

12.4%

37.7%

11.3%

37.7% of users answered this


question correctly

C. It may occupy areas of ectopic gastric metaplasia

End and review

D. In patients who are colonised there is commonly evidence of fundal


gastritis on endoscopy

5
6-8 2 / 3
9
10
11

Search

E. It produces a powerful urease that forms the basis of the Clo test

12
Go

13

Next question

14
15

Helicobacter pylori accounts for >75% cases of duodenal ulceration. It may be diagnosed
with either serology, microbiology, histology or CLO testing.

16
17
18

Theme from January 2011 Exam


Helicobacter pylori rarely produces any typical features on endoscopy. Where infection is
suspected the easiest course of action is to take an antral biopsy for Clo testing in the
endoscopy suite.

19-21 3 / 3
22
23
24

Helicobacter Pylori

25

Infection with Helicobacter Pylori is implicated in many cases of duodenal ulceration and up
to 60% of patients with gastric ulceration.

26
27-29 1 / 3
30

It is a gram negative, helical shaped rod with microaerophillic requirements. It has the ability
to produce a urease enzyme that will hydrolyse urea resulting in the production of ammonia.
The effect of ammonia on antral G cells is to cause release of gastrin via a negative
feedback loop.
Once infection is established the organism releases enzymes that disrupt the gastric mucous
layer. Certain subtypes release cytotoxins cag A and vac A gene products. The organism
incites a classical chronic inflammatory process of the gastric epithelium. This accounts for
the development of gastric ulcers. The mildly increased acidity may induce a process of
duodenal gastric metaplasia. Whilst duodenal mucosa cannot be colonised by H-Pylori,
mucosa that has undergone metaplastic change to the gastric epithelial type may be
colonised by H- Pylori with subsequent inflammation and development of duodenitis and
ulcers.
In patients who are colonised there is a 10-20% risk of peptic ulcer, 1-2% risk gastric cancer
and <1% risk MALT lymphoma.
Rate question:

Next question

Comment on this question

All contents of this site are 2012 E-Medical Revision Ltd

http://www.emrcs.com/question/question.php?q=0

Terms and Conditions

Privacy policy

1/1

13/07/2015

Reference ranges

Previous

Question 31 of 78

Next

A 34 year old male presents with painful rectal bleeding and a fissure in ano is suspected.
On examination he has an epithelial defect at the mucocutaenous junction that is located
anteriorly. Approximately what proportion of patients with fissure in ano will present with this
pattern of disease?

Question stats

Score: 45.2%
1

22.3%

39.2%

11.8%

18.8%

7.9%

39.2% of users answered this


question correctly

A. 90%

End and review

5
6-8 2 / 3
9

B. 10%
10

C. 50%
11

Search

D. 25%

12

E. 100%

Go

13
14

Next question

15

Only a minority of patients with fissure in ano will have an anteriorly sited fissure. They are
particularly rare in males and an anterior fissure in a man should prompt a search for an
underlying cause.

16
17
18
19-21 3 / 3

Anal fissure

22

Anal fissures are a common cause of painful, bright red, rectal bleeding.
Most fissures are idiopathic and present as a painful mucocutaneous defect in the posterior
midline (90% cases). Fissures are more likely to be anteriorly located in females, particularly
if they are multiparous. Multiple fissures and those which are located at other sites are more
likely to be due to an underlying cause.
Diseases associated with fissure in ano include:

23
24
25
26
27-29 1 / 3

Crohns disease
Tuberculosis
Internal rectal prolapse

30
31

Diagnosis
In most cases the defect can be visualised as a posterior midline epithelial defect. Where
symptoms are highly suggestive of the condition and examination findings are unclear an
examination under anaesthesia may be helpful. Atypical disease presentation should be
investigated with colonoscopy and EUA with biopsies of the area.
Treatment
Stool softeners are important as the hard stools may tear the epithelium and result in
recurrent symptoms. The most effective first line agents are topically applied GTN (0.2%) or
Diltiazem (2%) paste. Side effects of diltiazem are better tolerated.
Resistant cases may benefit from injection of botulinum toxin or lateral internal
sphincterotomy (beware in females). Advancement flaps may be used to treat resistant
cases.
Sphincterotomy produces the best healing rates. It is associated with incontinence to flatus in
up to 10% of patients in the long term.
Rate question:

Next question

Comment on this question

All contents of this site are 2012 E-Medical Revision Ltd

http://www.emrcs.com/question/question.php?q=0

Terms and Conditions

Privacy policy

1/1

13/07/2015

Reference ranges

Previous

Question 32 of 78

Next

Question stats

End and review

Score: 46.9%
1

44%

8.5%

29%

A. Osteoclast function

9.4%

B. PTH receptors

9.1%

C. Osteoblast function

44% of users answered this


question correctly

The pathogenesis of osteopetrosis is best explained by a defect in which of the following?

D. Calcium resorption in proximal tubule

5
6-8 2 / 3
9
10

E. Calcium absorption

11

Search
Next question

12
Go

13
14
15

Osteopetrosis

16

Overview

17
18

Also known as marble bone disease


Rare disorder of defective osteoclast function resulting in failure of normal bone
resorption
Stem cell transplant and interferon-gamma have been used for treatment

19-21 3 / 3
22
23
24
25

Rate question:

Next question

26
27-29 1 / 3

Comment on this question

30
31
32

All contents of this site are 2012 E-Medical Revision Ltd

http://www.emrcs.com/question/question.php?q=0

Terms and Conditions

Privacy policy

1/1

13/07/2015

Reference ranges

Previous

Question 33 of 78

Next

A 13 month old boy is brought to the surgical clinic by his mother because his left testicle is
not located in the scrotum. At which of the following sites would the testicle be located if it
were an ectopic testis?

Question stats

Score: 45.5%
1

17.4%

25%

15.3%

32.1%

10.1%

A. Canalicular
32.1% of users answered this
question correctly

B. Inguinal

End and review

C. External inguinal ring

5
6-8 2 / 3
9
10

D. Superficial inguinal pouch

11

Search

E. High scrotal

12
Go

Next question

13
14

Theme from September 2011 Exam


Theme from January 2012 Exam
September 2013
Ectopic testes are those that come to lie outside the normal range of embryological descent
(i.e. in the superficial inguinal pouch). Other sites of ectopic testes include; base of penis,
femoral and perineal.

15
16
17
18
19-21 3 / 3
22

Testicular disorders-paediatric

23

Testicular disorders
Testicular disorders are some of the commonest conditions present in paediatric urological
practice.

24
25
26
27-29 1 / 3

Cryptorchidism

30

The embryological descent of the testicle from within the abdominal cavity may be
subject to a number of variations. Distinctions need to be made clinically from a non
descended testis and a testis that is retractile.
Testis that lie outside the normal path of embryological descent are termed ectopic
testis. Undescended testis occurs in 1% of male infants. Where the testis does not lie
in an intra scrotal location, its location should be ascertained. Where both testes are
absent, the infant may be intersex.
MRI scanning may reveal intra-abdominal testes; however a GA is often needed to
perform this investigation in this age group.
Testes that are undescended should be placed in the scrotum after 1 year of age, as
the testosterone surge that may facilitate descent occurs at 6 months of age.
Where the testes lie distally e.g. Superficial inguinal pouch an open orchidopexy is the
procedure of choice.
With abdominal testes a laparoscopy should be performed. The risk of seminoma is
increased in individuals with a non descended testes and this risk is not reduced by
orchidopexy.

31
32
33

Testicular torsion
Typically the patient has severe sudden onset of scrotal pain. The difficulty in
paediatric practice is the lack of clear history.
On examination the testis is tender and enlarged.
Management is by surgical exploration.
Delay beyond 6 hours is associated with low salvage rates.
A torted hyatid produces pain that is far more localised and the testis itself should feel
normal. However, diagnostic doubt often exists and in such cases surgical exploration
is warranted.

Hydrocele
Occur secondary to patent processus vaginalis
Present as fluid filling in scrotum or as cyst of the spermatic cord
Communicating hydroceles are treated by a trans inguinal ligation of the PPV
Cystic hydroceles in older children may be treated with scrotal exploration

Rate question:

Next question

Comment on this question

http://www.emrcs.com/question/question.php?q=0

1/2

13/07/2015

All contents of this site are 2012 E-Medical Revision Ltd

http://www.emrcs.com/question/question.php?q=0

Terms and Conditions

Privacy policy

2/2

13/07/2015

Reference ranges

Previous

Question 34 of 78

Next

Which of the following is seen more commonly with Crohns disease rather than ulcerative
colitis?

A. Mucosal islands at endoscopy


B. Goblet cell depletion on biopsy

Question stats

Score: 47.1%
1

21.7%

12.6%

36.4%

19.2%

10.1%

36.4% of users answered this


question correctly

C. Fat wrapping of the terminal ileum

End and review

D. Attenuated symptoms in smokers

5
6-8 2 / 3
9
10

E. Toxic megacolon

11

Search

12
Next question

Go

13
14

Crohns disease is worse in smokers and smoking is an independent risk factor for
disease recurrence following resection.

15
16
17

Fat wrapping of the terminal ileum is commonly seen in patients with ileal disease (the
commonest disease site). The mesenteric fat in patients with IBD is often dense, hard and
prone to considerable haemorrhage during surgery. At endoscopy, the mucosa in patients
with Crohns disease is said to resemble cobblestones, mucosal islands (pseudopolyps) are
seen in ulcerative colitis.

18
19-21 3 / 3
22
23
24

Crohns disease

25

Crohns disease is a chronic transmural inflammation of a segment(s) of the gastrointestinal


tract and may be associated with extra intestinal manifestations. Frequent disease patterns
observed include ileal, ileocolic and colonic disease. Peri-anal disease may occur in
association with any of these. The disease is often discontinuous in its distribution.
Inflammation may cause ulceration, fissures, fistulas and fibrosis with stricturing. Histology
reveals a chronic inflammatory infiltrate that is usually patchy and transmural.

26
27-29 1 / 3
30
31
32
33

Ulcerative colitis Vs Crohns


34

Crohn's disease

Ulcerative colitis

Distribution

Mouth to anus

Rectum and colon

Macroscopic
changes

Cobblestone appearance, apthoid ulceration

Contact bleeding

Depth of
disease

Transmural inflammation

Superficial inflammation

Distribution
pattern

Patchy

Continuous

Histological
features

Granulomas (non caseating epithelioid cell


aggregates with Langerhans' giant cells)

Crypt abscesses, Inflammatory


cells in the lamina propria

Extraintestinal manifestations of Crohns


Related to disease extent

Unrelated to disease extent

Aphthous ulcers (10%)

Sacroiliiitis (10-15%)

Erythema nodosum (5-10%)

Ankylosing spondylitis (1-2%)

Pyoderma gangrenosum (0.5%)

Primary sclerosing cholangitis (Rare)

Acute arthropathy (6-12%)

Gallstones (up to 30%)

Ocular complications (up to 10%)

Renal calculi (up to 10%)

Diarrhoea in Crohns
Diarrhoea in Crohns may be multifactorial since actual inflammation of the colon is not
common. Causes therefore include the following:
Bile salt diarrhoea secondary to terminal ileal disease
Entero-colic fistula
Short bowel due to multiple resections
Bacterial overgrowth

http://www.emrcs.com/question/question.php?q=0

1/2

13/07/2015

Surgical interventions in Crohns disease


The commonest disease pattern in Crohns is stricturing terminal ileal disease and this often
culminates in an ileocaecal resection. Other procedures performed include segmental small
bowel resections and stricturoplasty. Colonic involvement in patients with Crohns is not
common and, where found, distribution is often segmental. However, despite this distribution
segmental resections of the colon in patients with Crohns disease are generally not
advocated because the recurrence rate in the remaining colon is extremely high, as a result
the standard options of colonic surgery in Crohns patients are generally; sub total colectomy,
panproctocolectomy and staged sub total colectomy and proctectomy. Restorative
procedures such as ileoanal pouch have no role in therapy.
Crohns disease is notorious for the developmental of intestinal fistulae; these may form
between the rectum and skin (peri anal) or the small bowel and skin. Fistulation between
loops of bowel may also occur and result in bacterial overgrowth and malabsorption.
Management of enterocutaneous fistulae involves controlling sepsis, optimising nutrition,
imaging the disease and planning definitive surgical management.
Rate question:

Next question

Comment on this question

All contents of this site are 2012 E-Medical Revision Ltd

http://www.emrcs.com/question/question.php?q=0

Terms and Conditions

Privacy policy

2/2

13/07/2015

Reference ranges

Previous

Question 35 of 78

Next

Question stats

End and review

Score: 45.7%
1

7.2%

25.6%

7.7%

A. Pneumococcus

48.5%

B. Klebsiella

11%

C. Haemophilus influenzae

48.5% of users answered this


question correctly

A splenectomy increases the risk of infection from all the following organisms except?

D. Staphylococcus aureus

5
6-8 2 / 3
9
10

E. Neisseria meningitidis

11

Search
Next question

12
Go

13

Theme from 2010 Exam

14
15

Staphylococcus aureus infection following splenectomy is no more common than in non


splenectomised individuals. The other organisms are encapsulated, which is why they are
more likely to cause overwhelming post splenectomy sepsis.

16
17
18

Post splenectomy sepsis

19-21 3 / 3

Hyposplenism may complicate certain medical conditions where splenic atrophy occurs or
may be the result of medical intervention such as splenic artery embolization and
splenectomy for trauma. Diagnosis of hyposplenism is difficult and whilst there may be
peripheral markers of the splenectomised state (e.g. Howell Jolly bodies) these are neither
100% sensitive or specific. The most sensitive test is a radionucleotide labeled red cell scan.
Hyposplenism, by whatever mechanism it occurs dramatically increases the risk of post
splenectomy sepsis, particularly with encapsulated organisms. Since these organisms may
be opsonised, but this then goes undetected at an immunological level due to loss of the
spleen. For this reason individuals are recommended to be vaccinated and have antibiotic
prophylaxis.

22
23
24
25
26
27-29 1 / 3
30
31
32

Key recommendations

33

All those with hyposplenism or may become so (such as prior to an elective


splenectomy) should receive pneumococcal, haemophilus type b and meningococcal
type C vaccines. These should be administered 2 weeks prior to splenectomy or two
weeks following splenectomy. The vaccine schedule for meningococcal disease
essentially consists of a dose of Men C and Hib at 2 weeks and then a dose of the
MenACWY vaccine one month later. Those aged under 2 may require a booster at 2
years. A dose of pneumococcal polyvalent polysaccharide vaccine (PPV) is given at
two weeks. A conjugated vaccine (PCV) is offered to young children. The PCV is more
immunogenic but covers fewer serotypes. Boosting PPV is either guided by serological
measurements (where available) or by routine boosting doses at 5 yearly intervals.
Annual influenza vaccination is recommended in all cases
Antibiotic prophylaxis is offered to all. The risk of post splenectomy sepsis is greatest
immediately following splenectomy and in those aged less than 16 years or greater
than 50 years. Individuals with a poor response to pneumococcal vaccination are
another high risk group. High risk individuals should be counselled to take penicillin or
macrolide prophylaxis. Those at low risk may choose to discontinue therapy. All
patients should be advised about taking antibiotics early in the case of intercurrent
infections.
Asplenic individuals traveling to malaria endemic areas are at high risk and should
have both pharmacological and mechanical protection.

34
35

Dosing
Penicillin V 500mg BD or amoxicillin 250mg BD
References
Davies J et al. Review of guidelines for the prevention and treatment of infection in patients
with an absent or dysfunctional spleen: Prepared on behalf of the British Committee for
Standards in Haematology by a Working Party of the Haemato-Oncology Task Force. British
Journal of Haematology 2011 (155): 308317.

Rate question:

Next question

Comment on this question

All contents of this site are 2012 E-Medical Revision Ltd

http://www.emrcs.com/question/question.php?q=0

Terms and Conditions

Privacy policy

1/2

13/07/2015

http://www.emrcs.com/question/question.php?q=0

2/2

13/07/2015

Reference ranges

Question 36 of 78

Previous

Next

Question stats

End and review

Score: 47.2%
1

14.2%

21%

10.2%

A. Iritis

42.2%

B. Clubbing

12.4%

C. Aphthous ulcers

42.2% of users answered this


question correctly

Which of the following is not an extraintestinal feature Crohns disease?

D. Erythema multiforme

5
6-8 2 / 3
9
10

E. Pyoderma gangrenosum

11

Search
Next question

12
Go

13

Extraintestinal manifestation of inflammatory bowel disease: A PIE SAC

14
15

Aphthous ulcers
Pyoderma gangrenosum
Iritis
Erythema nodosum
Sclerosing cholangitis
Arthritis
Clubbing

16
17
18
19-21 3 / 3
22
23
24
25

Crohns disease

26
27-29 1 / 3

Crohns disease is a chronic transmural inflammation of a segment(s) of the gastrointestinal


tract and may be associated with extra intestinal manifestations. Frequent disease patterns
observed include ileal, ileocolic and colonic disease. Peri-anal disease may occur in
association with any of these. The disease is often discontinuous in its distribution.
Inflammation may cause ulceration, fissures, fistulas and fibrosis with stricturing. Histology
reveals a chronic inflammatory infiltrate that is usually patchy and transmural.

30
31
32
33
34
35

Ulcerative colitis Vs Crohns


Crohn's disease

Ulcerative colitis

Distribution

Mouth to anus

Rectum and colon

Macroscopic
changes

Cobblestone appearance, apthoid ulceration

Contact bleeding

Depth of
disease

Transmural inflammation

Superficial inflammation

Distribution
pattern

Patchy

Continuous

Histological
features

Granulomas (non caseating epithelioid cell


aggregates with Langerhans' giant cells)

Crypt abscesses, Inflammatory


cells in the lamina propria

36

Extraintestinal manifestations of Crohns


Related to disease extent

Unrelated to disease extent

Aphthous ulcers (10%)

Sacroiliiitis (10-15%)

Erythema nodosum (5-10%)

Ankylosing spondylitis (1-2%)

Pyoderma gangrenosum (0.5%)

Primary sclerosing cholangitis (Rare)

Acute arthropathy (6-12%)

Gallstones (up to 30%)

Ocular complications (up to 10%)

Renal calculi (up to 10%)

Diarrhoea in Crohns
Diarrhoea in Crohns may be multifactorial since actual inflammation of the colon is not
common. Causes therefore include the following:
Bile salt diarrhoea secondary to terminal ileal disease
Entero-colic fistula
Short bowel due to multiple resections
http://www.emrcs.com/question/question.php?q=0

1/2

13/07/2015

Bacterial overgrowth

Surgical interventions in Crohns disease


The commonest disease pattern in Crohns is stricturing terminal ileal disease and this often
culminates in an ileocaecal resection. Other procedures performed include segmental small
bowel resections and stricturoplasty. Colonic involvement in patients with Crohns is not
common and, where found, distribution is often segmental. However, despite this distribution
segmental resections of the colon in patients with Crohns disease are generally not
advocated because the recurrence rate in the remaining colon is extremely high, as a result
the standard options of colonic surgery in Crohns patients are generally; sub total colectomy,
panproctocolectomy and staged sub total colectomy and proctectomy. Restorative
procedures such as ileoanal pouch have no role in therapy.
Crohns disease is notorious for the developmental of intestinal fistulae; these may form
between the rectum and skin (peri anal) or the small bowel and skin. Fistulation between
loops of bowel may also occur and result in bacterial overgrowth and malabsorption.
Management of enterocutaneous fistulae involves controlling sepsis, optimising nutrition,
imaging the disease and planning definitive surgical management.
Rate question:

Next question

Comment on this question

All contents of this site are 2012 E-Medical Revision Ltd

http://www.emrcs.com/question/question.php?q=0

Terms and Conditions

Privacy policy

2/2

13/07/2015

Reference ranges

Previous

Question 37 of 78

Next

Which of the following is not considered a risk factor for the development of oesophageal
malignancy?

A. Oesophageal metaplasia
B. Smoking

Question stats

Score: 48.6%
1

9.1%

7.8%

7.3%

14.1%

61.8%

61.8% of users answered this


question correctly

C. Excessive intake of alcoholic spirits

End and review

D. Achalasia

5
6-8 2 / 3
9
10

E. Blood group O

11

Search

12
Go

Next question

13
14

Blood group O is not a risk factor for oesophageal cancer. Achalasia is associated with the
risk of developing squamous cell carcinoma of the oesophagus.

15
16

Oesophageal cancer

17
18

Incidence is increasing
In most cases in the Western world this increase is accounted for by a rise in the
number of cases of adenocarcinoma. In the UK adenocarcinomas account for 65% of
cases.
Barretts oesophagus is a major risk factor for most cases of oesophageal
adenocarcinoma.
In other regions of the world squamous cancer is more common and is linked to
smoking, alcohol intake, diets rich in nitrosamines and achalasia.
Surveillance of Barretts is important, as it imparts a 30 fold increase in cancer risk and
if invasive malignancy is diagnosed early then survival may approach 85% at 5 years.

19-21 3 / 3
22
23
24
25
26
27-29 1 / 3
30
31

Diagnosis

32
33

Upper GI endoscopy is the first line test


Contrast swallow may be of benefit in classifying benign motility disorders but has no
place in the assessment of tumours
Staging is initially undertaken with CT scanning of the chest, abdomen and pelvis. If
overt metastatic disease is identified using this modality then further complex imaging
is unnecessary
If CT does not show metastatic disease, then local stage may be more accurately
assessed by use of endoscopic ultrasound.
Staging laparoscopy is performed to detect occult peritoneal disease. PET CT is
performed in those with negative laparoscopy. Thoracoscopy is not routinely
performed.

34
35
36
37

Treatment
Operable disease is best managed by surgical resection. The most standard procedure is an
Ivor- Lewis type oesophagectomy. This procedure involves the mobilisation of the stomach
and division of the oesophageal hiatus. The abdomen is closed and a right sided
thoracotomy performed. The stomach is brought into the chest and the oesophagus
mobilised further. An intrathoracic oesophagogastric anastomosis is constructed. Alternative
surgical strategies include a transhiatal resection (for distal lesions), a left thoraco-abdominal
resection (difficult access due to thoracic aorta) and a total oesophagectomy (McKeown) with
a cervical oesophagogastric anastomosis.
The biggest surgical challenge is that of anastomotic leak, with an intrathoracic anastomosis
this will result in mediastinitis. With high mortality. The McKeown technique has an
intrinsically lower systemic insult in the event of anastomotic leakage.
In addition to surgical resection many patients will be treated with adjuvant chemotherapy.
Rate question:

Next question

Comment on this question

All contents of this site are 2012 E-Medical Revision Ltd

http://www.emrcs.com/question/question.php?q=0

Terms and Conditions

Privacy policy

1/1

13/07/2015

Reference ranges

Previous2 / 3

Question 38-40 of 78

Next

Question stats

End and review

Score: 50%

Average score for registered users:

1
2

Theme: Thyroid blood testing


A. Measurement of antibodies to TSH receptor

38

87%

39

64.3%

40

61%

B. Thyroid peroxidase antibodies

3
4
5

C. Thyroglobulin antibodies

6-8 2 / 3

D. Serum calcitonin

Search

Please select the blood test most commonly performed for the diagnosis or assessment of
the thyroid disorder described. Each answer may be used once, more than once or not at all.

Go

10
11
12
13

38.

A 32 year old lady is diagnosed with Medullary carcinoma of the thyroid and
has undergone resection of the tumour.

14
15
16

Serum calcitonin
Measurement of basal or stimulated calcitonin concentrations is used to assess
the completeness of surgical resection, and is of use in detecting diseases
recurrences during follow up.

17
18
19-21 3 / 3
22

39.

A 20 year old lady has undergone a total thyroidectomy for a well differentiated
papillary carcinoma. She attends clinic and is well and the surgeon wishes to
screen for disease recurrence.

23
24
25

You answered Thyroid peroxidase antibodies

26

The correct answer is Thyroglobulin antibodies

27-29 1 / 3

Antibodies to thyroglobulin, the major constituent of colloid and precursor of


thyroid hormones may be elevated in those with metastatic or recurrent thyroid
cancer. Results may be erronoeous in those with other thyroid disorders.
40.

A 33 year old lady presents with a recently diagnosed goitre and a diagnosis of
Hashimotos thyroiditis is suspected.

30
31
32
33
34
35

Thyroid peroxidase antibodies

36

Antibodies to thyroid peroxidase are found in most patients with Graves


disease or Hashimotos thyroiditis.

37
38-40 2 / 3

Next question

Blood testing in thyroid disease

Assay

Usage

Thyroid peroxidase
(microsomal)
antibodies

Found in autoimmune disease affecting the thyroid (Hashimotos


100%) and Graves (70%)

Antibodies to TSH
receptor

Individuals with Graves disease (95%)

Thyroglobulin
antibodies

Calcitonin

Not useful for clinically distinguishing between different types of


thyroid disease, may be used as part of thyroid cancer follow up

Released from the parafollicular cells


Usually found in patients with medullary carcinoma of the thyroid

Rate question:

Next question

Comment on this question

http://www.emrcs.com/question/question.php?q=0

1/2

13/07/2015

All contents of this site are 2012 E-Medical Revision Ltd

http://www.emrcs.com/question/question.php?q=0

Terms and Conditions

Privacy policy

2/2

13/07/2015

Reference ranges

Previous

Question 41 of 78

Next

Question stats

End and review

Score: 48.8%
1

10%

22.8%

10.7%

A. Right heart failure

18.9%

B. Aortic regurgitation

37.7%

C. Eisenmenger's complex

37.7% of users answered this


question correctly

Which one of the following complications is least associated with ventricular septal defects?

D. Infective endocarditis

5
6-8 2 / 3
9
10

E. Atrial fibrillation

11

Search
Next question

12
Go

13

Atrial fibrillation is associated more with atrial septal defects

14
15

Ventricular septal defect

16

Ventricular septal defects are the most common cause of congenital heart disease. They
close spontaneously in around 50% of cases. Non-congenital causes include post
myocardial infarction

17
18
19-21 3 / 3

Features

22
23

classically a pan-systolic murmur which is louder in smaller defects

24
25

Complications

26

aortic regurgitation*
infective endocarditis
Eisenmenger's complex
right heart failure

27-29 1 / 3
30
31
32

*aortic regurgitation is due to a poorly supported right coronary cusp resulting in cusp
prolapse

33
34
35

Rate question:

Next question

36

Comment on this question

37
38-40 2 / 3
41

All contents of this site are 2012 E-Medical Revision Ltd

http://www.emrcs.com/question/question.php?q=0

Terms and Conditions

Privacy policy

1/1

13/07/2015

Reference ranges

Question 42 of 78

Previous

Next

A 24 year old man from Sub Saharan Africa presents with a lymphadenopathy and weight
loss. A diagnosis of tuberculosis is suspected and a lymph node biopsy is performed.
Staining with which of the agents below is most likely to facilitate identification of the
causative organism?

Question stats

Score: 50%
1

8%

72.3%

5.7%

7.4%

6.6%

72.3% of users answered this


question correctly

A. Gram stain

End and review

5
6-8 2 / 3
9

B. Ziehl-Neelsen stain
10

C. Von Kossa stain


11

Search

D. Van Gieson stain

12

E. Masson Trichrome stain

Go

13
14

Next question

15

Ziehl-Neelsen stain is typically used to identify mycobacteria. They are not stained in the
Gram staining process. Van Gieson and Masson trichrome are histological staining methods
for identification of connective tissues. The Von Kossa technique is useful for identifying
tissue mineralisation.

16
17
18
19-21 3 / 3

Tuberculosis pathology

22
23

Is a form of primary chronic inflammation, caused by the inability of macrophages to kill


the Mycobacterium tuberculosis.
The macrophages often migrate to regional lymph nodes, the lung lesion plus affected
lymph nodes is referred to as a Ghon complex.
This leads to the formation of a granuloma which is a collection of epithelioid
histiocytes.
There is the presence of caseous necrosis in the centre.
The inflammatory response is mediated by a type 4 hypersensitivity reaction.
In healthy individuals the disease may be contained, in the immunocompromised
disseminated (miliary TB) may occur.

24
25
26
27-29 1 / 3
30
31
32
33
34

Diagnosis

35
36

Waxy membrane of mycobacteria prevents binding with normal stains. Ziehl - Neelsen
staining is typically used.
Culture based methods take far longer.

37
38-40 2 / 3
41

Image showing acid- alcohol fast mycobacteria stained using the Ziehl- Neelsen method

42

Image sourced from Wikipedia

Rate question:

Next question

Comment on this question

All contents of this site are 2012 E-Medical Revision Ltd

http://www.emrcs.com/question/question.php?q=0

Terms and Conditions

Privacy policy

1/1

13/07/2015

Reference ranges

Previous

Question 43 of 78

Next

Question stats

End and review

Score: 48.8%
1

39.7%

14%

8.8%

A. Resemblance to ductal epithelial cells

26%

B. Angiogenesis

11.5%

C. Nuclear pleomorphism

26% of users answered this


question correctly

Which of the following is not a pathological feature of breast cancer?

D. Metastatic calcification

5
6-8 2 / 3
9
10

E. Vascular invasion

11

Search
Next question

12
Go

13

Dystrophic calcification may be present in breast malignancy and is the basis for the breast
screening programme. Metastatic calcification is calcification which occurs in otherwise
normal tissues, usually as a result of hypercalcaemia. Invasive ductal carcinoma is the most
common type of breast cancer, unless the tumour is very poorly differentiated there is
usually some resemblance to ductal epithelial cells.

14
15
16
17
18

Breast cancer pathology

19-21 3 / 3

The histological features of breast cancer depend upon the underlying diagnosis. The
invasive component is usually comprised of ductal cells (unless it is an invasive lobular
cancer). In situ lesions may co-exist (such as DCIS).

22
23
24

Typical changes seen in conjunction with invasive breast cancer include:


1. Nuclear pleomorphism
2. Coarse chromatin
3. Angiogenesis
4. Invasion of the basement membrane
5. Dystrophic calcification (may be seen on mammography)
6. Abnormal mitoses
7. Vascular invasion
8. Lymph node metastasis

25
26
27-29 1 / 3
30
31
32
33
34

The primary tumour is graded on a scale of 1-3 where 1 is the most benign lesion and 3 the
most poorly differentiated.

35
36

Immunohistochemistry for oestrogen receptor and herceptin status is routinely performed.

37
38-40 2 / 3

The grade, lymph node stage and size are combined to provide the Nottingham prognostic
index.

41
42

Rate question:

43

Next question

Comment on this question

All contents of this site are 2012 E-Medical Revision Ltd

http://www.emrcs.com/question/question.php?q=0

Terms and Conditions

Privacy policy

1/1

13/07/2015

Reference ranges

Previous0 / 3

Question 44-46 of 78

Next

Question stats

End and review

Score: 42.9%

Average score for registered users:

1
2

Theme: Renal lesions


A. Renal cell carcinoma

44

73.4%

45

73.8%

46

48%

B. Renal transitional cell carcinoma

3
4
5

C. Nephroblastoma

6-8 2 / 3

D. Neuroblastoma

Search

E. Angiomyolipoma

Go

10

F. Renal squamous cell carcinoma

11

G. Retroperitoneal fibrosis

12

For each scenario please select the most likely underlying diagnosis. Each option may be
used once, more than once or not at all.

13
14
15
16

44.

A 69 year old male presents with haematuria. He worked in the textile industry.
He has a left flank mass. A CT IVU shows a lesion of the left renal pelvis.

17
18
19-21 3 / 3

You answered Renal cell carcinoma

22

The correct answer is Renal transitional cell carcinoma

23

TCC is a rare form of renal cancer, accounting for approximately 7% of all renal
tumours. Risk factors include exposure to chemicals in the textile, plastic and
rubber industry.

24
25
26

45.

A 2 year old boy presents with a right renal mass. On examination he has an
irregular mass arising from the right flank and is hypertensive. A CT scan
shows a non calcified irregular lesion affecting the apex of the right kidney and
the right adrenal gland.

27-29 1 / 3
30
31
32

You answered Neuroblastoma

46.

33

The correct answer is Nephroblastoma

34

Wilm's tumour of the kidney is the most common renal tumour in children. Both
nephroblastoma and neuroblastoma may occupy the adrenal and apex of the
kidney. In the case of neuroblastoma the lesion will have arisen from the
adrenal, in the case of nephroblastoma the lesion will have arisen from the
kidney. Hypertension is more commonly associated with nephroblastoma.
Neuroblastomas are usually calcified, whereas nephroblastomas are not and
this may be of diagnostic usefulness pre operatively.

35

A 35 year old male presents with haematuria. He is found to have bilateral


masses in the flanks. He has a history of epilepsy and learning disability.

36
37
38-40 2 / 3
41
42
43
44-46 0 / 3

You answered Retroperitoneal fibrosis


The correct answer is Angiomyolipoma
This patient has tuberous sclerosis. This is associated with angiomyolipoma,
which is present in 60-80% patients. It is a benign lesion.

Next question

Renal lesions

Lesion
Renal cell
carcinoma

Disease specific features


Most present with haematuria
(50%)
Common renal tumour (85%
cases)
Paraneoplastic features include
hypertension and polycythaemia
Most commonly has
haematogenous mestastasis

http://www.emrcs.com/question/question.php?q=0

Treatment
Usually radical or partial
nephrectomy

1/2

13/07/2015

Nephroblastoma

Neuroblastoma

Transitional cell
carcinoma

Angiomyolipoma

Rare childhood tumour


It accounts for 80% of all
genitourinary malignancies in
those under the age of 15 years
Up to 90% will have a mass
50% will be hypertensive
Diagnostic work up includes
ultrasound and CT scanning

Most common extracranial


tumour of childhood
80% occur in those under 4
years of age
Tumour of neural crest origin (up
to 50% occur in the adrenal
gland)
The tumour is usually calcified
and may be diagnosed using
MIBG scanning
Staging is with CT

Accounts for 90% of lower urinary


tract tumour, but only 10% of
renal tumours
Males affected 3x more than
females
Occupational exposure to
industrial dyes and rubber
chemicals may increase risk
Up to 80% present with painless
haematuria
Diagnosis and staging is with CT
IVU

80% of these hamartoma type


lesions occur sporadically, the
remainder are seen in those with
tuberous sclerosis
Tumour is composed of blood
vessels, smooth muscle and fat
Massive bleeding may occur in
10% of cases

Rate question:

Surgical resection combined with


chemotherapy (usually
vincristine, actinomycin D and
doxorubicin)

Surgical resection, radiotherapy


and chemotherapy

Radical nephroureterectomy

50% of patients with lesions


>4cm will have symptoms and
will require surgical resection

Next question

Comment on this question

All contents of this site are 2012 E-Medical Revision Ltd

http://www.emrcs.com/question/question.php?q=0

Terms and Conditions

Privacy policy

2/2

13/07/2015

Reference ranges

Previous

Question 47 of 78

Next

An 18 month old boy presents with recurrent urinary tract infections. As part of the diagnostic
work-up he is noted to have abnormal renal function. An ultrasound scan is performed and
shows bilateral hydronephrosis. What is the most likely underlying diagnosis?

Question stats

Score: 42%
1

55.4%

10.2%

6.2%

21.2%

7%

A. Urethral valves
55.4% of users answered this
question correctly

B. Meatal stenosis

End and review

C. Hydronephrosis

5
6-8 2 / 3
9
10

D. Pelvico-ureteric junction obstruction

11

Search

E. Benign prostatic hyperplasia

12
Go

13

Next question

14
15

Theme from April 2012 Exam


A posterior urethral valve is an obstructive, developmental uropathy that usually affects male
infants (incidence 1 in 8000). Diagnostic features include bladder wall hypertrophy,
hydronephrosis and bladder diverticula.

16
17
18
19-21 3 / 3

Urethral valves

22

Posterior urethral valves are the commonest cause of infravesical outflow obstruction in
males. They may be diagnosed on ante natal ultrasonography. Because the bladder has to
develop high emptying pressures in utero, the child may develop renal parenchymal
damage. This translates to renal impairment noted in 70% of boys at presentation.
Treatment is with bladder catheterisation. Endoscopic valvotomy is the definitive treatment of
choice with cystoscopic and renal follow up.

23
24
25
26
27-29 1 / 3
30

Rate question:

Next question

31

Comment on this question

32
33
34
35
36
37
38-40 2 / 3
41
42
43
44-46 0 / 3
47

All contents of this site are 2012 E-Medical Revision Ltd

http://www.emrcs.com/question/question.php?q=0

Terms and Conditions

Privacy policy

1/1

13/07/2015

Reference ranges

Question 48 of 78

Previous

Next

At which of the following anatomical sites does dormant tuberculosis most frequently
reactivate?

A. Apex of the lung


B. Base of the lung

Question stats

Score: 43.1%
1

64.8%

10.3%

7.1%

8.4%

9.4%

64.8% of users answered this


question correctly

C. Brain

End and review

D. Terminal ileum

5
6-8 2 / 3
9
10

E. Lumbar spine

11

Search

12
Go

Next question

13
14

TB reactivation most commonly occurs at the lung apex. This site is better oxygenated than
elsewhere allowing the mycobacteria to multiply more rapidly and then spread both locally
and distantly.

15
16
17

Tuberculosis pathology

18
19-21 3 / 3

Is a form of primary chronic inflammation, caused by the inability of macrophages to kill


the Mycobacterium tuberculosis.
The macrophages often migrate to regional lymph nodes, the lung lesion plus affected
lymph nodes is referred to as a Ghon complex.
This leads to the formation of a granuloma which is a collection of epithelioid
histiocytes.
There is the presence of caseous necrosis in the centre.
The inflammatory response is mediated by a type 4 hypersensitivity reaction.
In healthy individuals the disease may be contained, in the immunocompromised
disseminated (miliary TB) may occur.

22
23
24
25
26
27-29 1 / 3
30
31
32

Diagnosis

33

Waxy membrane of mycobacteria prevents binding with normal stains. Ziehl - Neelsen
staining is typically used.
Culture based methods take far longer.

34
35
36
37

Image showing acid- alcohol fast mycobacteria stained using the Ziehl- Neelsen method

38-40 2 / 3
41
42
43
44-46 0 / 3
47
48

Image sourced from Wikipedia

Rate question:

Next question

Comment on this question

All contents of this site are 2012 E-Medical Revision Ltd

http://www.emrcs.com/question/question.php?q=0

Terms and Conditions

Privacy policy

1/1

13/07/2015

Reference ranges

Previous

Question 49 of 78

Next

Question stats

End and review

Score: 44.2%
1

9.1%

70.7%

7.3%

A. Squamous cell carcinoma

7.1%

B. Adenocarcinoma

5.8%

C. Lymphoma

70.7% of users answered this


question correctly

What is the commonest tumour type encountered in the colon?

D. Anaplastic carcinoma

5
6-8 2 / 3
9
10

E. Sarcoma

11

Search
Next question

12
Go

13

Adenocarcinoma are the most common and typically arise as a result of the adenoma carcinoma sequence.

14
15

Colorectal cancer

16
17
18

Annually about 150,000 new cases are diagnosed and 50,000 deaths from the
disease
About 75% will have sporadic disease and 25% will have a family history
Colorectal tumours comprise a spectrum of disease ranging from adenomas through
to polyp cancers and frank malignancy.
Polyps may be categorised into: neoplastic polyps, adenomatous polyps and non
neoplastic polyps.
The majority of adenomas are polypoidal lesions, although flat lesions do occur and
may prove to be dysplastic.
Non-neoplastic polyps include hyperplastic, juvenile, hamartomatous, inflammatory,
and lymphoid polyps, which have not generally been thought of as precursors of
cancer.
Three characteristics of adenomas that correlate with malignant potential have been
characterised. These include increased size, villous architecture and dysplasia. For
this reason most polyps identified at colonoscopy should be removed.
The transformation from polyp to cancer is described by the adenoma - carcinoma
sequence and its principles should be appreciated. Essentially genetic changes
accompany the transition from adenoma to carcinoma; key changes include APC, cmyc, K RAS mutations and p53 deletions.

19-21 3 / 3
22
23
24
25
26
27-29 1 / 3
30
31
32
33
34
35
36
37
38-40 2 / 3

Rate question:

41

Next question

42
Comment on this question

43
44-46 0 / 3
47
48
49

All contents of this site are 2012 E-Medical Revision Ltd

http://www.emrcs.com/question/question.php?q=0

Terms and Conditions

Privacy policy

1/1

13/07/2015

Reference ranges

Question 50 of 78

Previous

Next

Which of the following changes are most likely to be identified in the aortic wall of a 38 year
old lady with a Marfans syndrome and a dissecting aortic aneurysm?

A. Transmural aortitis
B. Cystic medial necrosis

Question stats

Score: 45.3%
1

21.5%

35.3%

14.4%

17.4%

11.4%

35.3% of users answered this


question correctly

C. Foamy macrophages

End and review

D. Dense dystrophic calcification

5
6-8 2 / 3
9
10

E. None of the above

11

Search

12
Next question

Cystic medial necrosis ( or cystic medial degeneration) occurs when basophils and mucoid
material lie in between the intimal elastic fibres of the aorta. It is typically found in the aortic
degeneration of Marfans syndrome, but may also be seen in aortic degeneration in older
adults.

Go

13
14
15
16
17
18

Aortic dissection

19-21 3 / 3
22

More common than rupture of the abdominal aorta


33% of patients die within the first 24 hours, and 50% die within 48 hours if no
treatment received
Associated with hypertension
Features of aortic dissection: tear in the intimal layer, followed by formation and
propagation of a subintimal hematoma. Cystic medial necrosis (Marfan's)
Most common site of dissection: 90% occurring within 10 centimetres of the aortic
valve

23
24
25
26
27-29 1 / 3
30
31

Stanford Classification

32

Type

Location

Treatment

Ascending aorta/ aortic root

Surgery- aortic root replacement

Descending aorta

Medical therapy with antihypertensives

33
34
35
36
37

DeBakey classification
Type

Site affected

Ascending aorta, aortic arch, descending aorta

II

Ascending aorta only

III

Descending aorta distal to left subclavian artery

38-40 2 / 3
41
42
43
44-46 0 / 3
47
48
49

Clinical features

50

Tearing, sudden onset chest pain (painless 10%)


Hypertension or Hypotension
A blood pressure difference greater than 20 mm Hg
Neurologic deficits (20%)

Investigations
CXR: widened mediastinum, abnormal aortic knob, ring sign, deviation of the
trachea/oesophagus
CT angiography of the thoracic aorta
MRI angiography
Conventional angiography (now rarely used diagnostically)

Management
Beta-blockers: aim HR 60-80 bpm and systolic BP 100-120 mm Hg
For type A dissections the standard of care is aortic root replacement

Rate question:
http://www.emrcs.com/question/question.php?q=0

Next question

1/2

13/07/2015

Comment on this question

All contents of this site are 2012 E-Medical Revision Ltd

http://www.emrcs.com/question/question.php?q=0

Terms and Conditions

Privacy policy

2/2

13/07/2015

Reference ranges

Previous

Question 51 of 78

Next

A 58 year old man undergoes an upper GI endoscopy for the investigation of odynophagia.
At endoscopy a reddish area is seen to protrude up into the oesophagus from the
gastrooesophageal junction. Which of the following pathological events is most likely to
explain this process?

Question stats

Score: 44.4%
1

47.9%

7.2%

25.5%

6.4%

13%

47.9% of users answered this


question correctly

A. Metaplasia

End and review

5
6-8 2 / 3
9

B. Anaplasia
10

C. Dysplasia
11

Search

D. Hypoplasia

12

E. Hyperplasia

Go

13
14

Next question

15

This is most likely to represent Barretts oesphagus and is thus metaplasia. Dysplasia is less
likely in this setting although biopsies are mandatory.

16
17
18

Barrett's oesophagus

19-21 3 / 3

Barretts oesophagus is a condition characterised by the metaplastic transformation of


squamous oesophageal epithelium to columnar gastric type epithelium. Three types of this
metaplastic process are recognised; intestinal (high risk), cardiac and fundic. The latter two
categories may cause difficulties in diagnosis. The most concrete diagnosis can be made
when endoscopic features of Barretts oesophagus are present together with a deep biopsy
that demonstrates not just goblet cell metaplasia but also oesophageal glands.

22
23
24
25
26
27-29 1 / 3

Barrett's can be sub divided into short (<3cm) and long (>3cm). The length of the affected
segment correlates strongly with the chances of identifying metaplasia. The overall
prevalence of Barrett's oesophagus is difficult to determine but may be in the region of 1 in
20 and is identified in up to 12% of those undergoing endoscopy for reflux.

30
31
32
33

A proportion of patients with metaplasia will progress to dysplasia and for this reason
individuals identified as having Barrett's should undergo endoscopic surveillance (every 2-5
years). Biopsies should be quadrantic and taken at 1-2cm intervals. Biopsies need to be
adequate. Where mass lesions are present consideration should be given to endoscopic sub
mucosal resection. Up to 40% of patients will be upstaged from high grade dysplasia to
invasive malignancy with such techniques.

34
35
36
37
38-40 2 / 3

Treatment

41

Long term proton pump inhibitor


Consider pH and manometry studies in younger patients who may prefer to consider
an anti reflux procedure
Regular endoscopic monitoring (more frequently if moderate dysplasia). With
quadrantic biopsies every 2-3 cm
If severe dysplasia be very wary of small foci of cancer

42
43
44-46 0 / 3
47
48
49

References
A consensus statement of the British approach is provided by:
Bennett C et al Consensus Statements for Management of Barrett's Dysplasia and EarlyStage Esophageal Adenocarcinoma, Based on a Delphi Process. Gastroenterology Volume
143, Issue 2 , Pages 336-346, August 2012.
Rate question:

50
51

Next question

Comment on this question

All contents of this site are 2012 E-Medical Revision Ltd

http://www.emrcs.com/question/question.php?q=0

Terms and Conditions

Privacy policy

1/1

13/07/2015

Reference ranges

Previous

Question 52 of 78

Next

A male infant is born by emergency cesarean section at 39 weeks gestation for foetal
distress. Soon after the birth the baby becomes progressively hypoxic and on examination is
found to have a scaphoid abdomen. What is the most likely underlying diagnosis?

Question stats

Score: 45.5%
1

13.6%

11.5%

45.8%

14.6%

14.6%

A. Intestinal malrotation
45.8% of users answered this
question correctly

B. Hiatus hernia

End and review

C. Foramen of Bochdalek hernia

5
6-8 2 / 3
9
10

D. Foramen of Morgagni hernia

11

Search

E. Tracheo-oesphageal fistula

12
Go

Next question

13
14

The finding of a scaphoid abdomen and respiratory distress suggests extensive intra
thoracic herniation of the abdominal contents. This is seen most frequently with Bochdalek
hernias. Morgagni hernias seldom present in such a dramatic fashion. The other options do
not typically present with the symptoms and signs described.

15
16
17
18

Embryology of the diaphragm and diaphragmatic hernia

19-21 3 / 3
22

Embryology
The diaphragm is formed between the 5th and 7th weeks of gestation through the
progressive fusion of the septum transversum, pleuroperitoneal folds and via lateral
muscular ingrowth. The muscular origins of the diaphragm are somites located in cervical
segments 3 to 5, which accounts for the long path taken by the phrenic nerve. The
components contribute to the following diaphragmatic segments:
Septum transversum - Central tendon
Pleuroperitoneal membranes - Parietal membranes surrounding viscera
Cervical somites C5 to C7 - Muscular component of the diaphragm

23
24
25
26
27-29 1 / 3
30
31
32

Diaphragmatic hernia

33
34

Type of hernia

Features

Morgagni

Anteriorly located
Minimal compromise on lung development
Minimal signs on antenatal ultrasound
Usually present later
Usually good prognosis

Bochdalek hernia

Posteriorly located
Larger defect
Often diagnosed antenatally
Associated with pulmonary hypoplasia
Poor prognosis

35
36
37
38-40 2 / 3
41
42
43
44-46 0 / 3
47
48

The posterior hernias of Bochdalek are the most common type and if not diagnosed
antenatally will typically present soon after birth with respiratory distress. The classical
finding is that of a scaphoid abdomen on clinical examination because of herniation of the
abdominal contents into the chest. Bochdalek hernias are associated with a number of
chromosomal abnormalities such as Trisomy 21 and 18. Infants have considerable
respiratory distress due to hypoplasia of the developing lung. Historically this was considered
to be due to direct compression of the lung by herniated viscera. This view over simplifies the
situation and the pulmonary hypoplasia occurs concomitantly with the hernial development,
rather than as a direct result of it. The pulmonary hypoplasia is associated with pulmonary
hypertension and abnormalities of pulmonary vasculature. The pulmonary hypertension
renders infants at risk of right to left shunting (resulting in progressive and worsening
hypoxia).
Diagnostic work up of these infants includes chest x-rays/ abdominal ultrasound scans and
cardiac echo.
Surgery forms the mainstay of treatment and both thoracic and abdominal approaches may
be utilised. Following reduction of the hernial contents a careful search needs to be made for
a hernial sac as failure to recognise and correct this will result in a high recurrence rate.
Smaller defects may be primarily closed, larger defects may require a patch to close the
defect. Malrotation of the viscera is a recognised association and may require surgical
correct at the same procedure (favoring an abdominal approach).
The mortality rate is 50-75% and is related to the degree of lung compromise and age at
presentation (considerably better in infants >24 hours old).

http://www.emrcs.com/question/question.php?q=0

49
50
51
52

1/2

13/07/2015

Rate question:

Next question

Comment on this question

All contents of this site are 2012 E-Medical Revision Ltd

http://www.emrcs.com/question/question.php?q=0

Terms and Conditions

Privacy policy

2/2

13/07/2015

Reference ranges

Previous

Question 53 of 78

Next

A 72 year old lady falls and lands on her left hip. She attends the emergency department
and is given some paracetamol by the junior doctor and discharged. Several months later
she presents with ongoing pain and discomfort of the hip. Avascular necrosis of the femoral
head is suspected. Which of the following features is least likely to be present?

Question stats

Score: 44.6%
1

14.7%

33.8%

11.8%

13.9%

25.8%

25.8% of users answered this


question correctly

A. Non union of the fracture

End and review

5
6-8 2 / 3
9

B. Angiogenesis at the fracture site


10

C. Increased numbers of fibroblasts at the fracture site


11

Search

D. Osteochondritis dissecans

12

E. Apoptosis of osteoblasts

Go

13
14

Next question

15

Apoptosis is not a feature of necrotic cell death. By this stage there would usually be
attempted repair so angiogenesis and proliferation of fibroblasts would be expected. These
cells may differentiate further to become osteoblasts which in turn will lay down new matrix.

16
17
18
19-21 3 / 3

Avascular necrosis

22
23

Cellular death of bone components due to interruption of the blood supply, causing
bone destruction
Main joints affected are hip, scaphoid, lunate and the talus.
It is not the same as non union. The fracture has usually united.
Radiological evidence is slow to appear.
Vascular ingrowth into the affected bone may occur. However, many joints will develop
secondary osteoarthritis.

24
25
26
27-29 1 / 3
30
31

Causes
P ancreatitis
L upus
A lcohol
S teroids
T rauma
I diopathic, infection
C aisson disease, collagen vascular disease
R adiation, rheumatoid arthritis
A myloid
G aucher disease
S ickle cell disease

32
33
34
35
36
37
38-40 2 / 3
41
42
43
44-46 0 / 3

Presentation
Usually pain. Often despite apparent fracture union.

47
48

Investigation
MRI scanning will show changes earlier than plain films.

49
50

Treatment
In fractures at high risk sites anticipation is key. Early prompt and accurate reduction is
essential.

51
52
53

Non weight bearing may help to facilitate vascular regeneration.


Joint replacement may be necessary, or even the preferred option (e.g. Hip in the elderly).
Rate question:

Next question

Comment on this question

All contents of this site are 2012 E-Medical Revision Ltd

http://www.emrcs.com/question/question.php?q=0

Terms and Conditions

Privacy policy

1/1

13/07/2015

Reference ranges

Previous

Question 54 of 78

Next

Which one of the following is least associated with the development of colorectal cancer in
patients with ulcerative colitis?

A. Unremitting disease
B. Disease duration > 10 years

Question stats

Score: 45.6%
1

14.8%

9.2%

13.3%

12.2%

50.5%

50.5% of users answered this


question correctly

C. Onset before 15 years old

End and review

D. Poor compliance to treatment

5
6-8 2 / 3
9
10

E. Disease confined to the rectum

11

Search

12
Go

Next question

13
14
15

Ulcerative colitis and colorectal cancer

16
17

Overview
18

risk of colorectal cancer is 10-20 times that of general population


the increased risk is mainly related to chronic inflammation
worse prognosis than patients without ulcerative colitis (partly due to delayed
diagnosis)
lesions may be multifocal

19-21 3 / 3
22
23
24
25

Factors increasing risk of cancer

26
27-29 1 / 3

disease duration > 10 years


patients with pancolitis
onset before 15 years old
unremitting disease
poor compliance to treatment

30
31
32
33
34
35

Rate question:

Next question

36

Comment on this question

37
38-40 2 / 3
41
42
43
44-46 0 / 3
47
48
49
50
51
52
53
54

All contents of this site are 2012 E-Medical Revision Ltd

http://www.emrcs.com/question/question.php?q=0

Terms and Conditions

Privacy policy

1/1

13/07/2015

Reference ranges

Previous1 / 3

Question 55-57 of 78

Next

Question stats

End and review

Score: 45%

Average score for registered users:

1
2

Theme: Causes of chest pain


A. Pulmonary embolism

55

51.3%

56

76.3%

57

77%

B. Anterior myocardial infarction

3
4
5

C. Inferior myocardial infarction

6-8 2 / 3

D. Proximal aortic dissection

Search

E. Distal aortic dissection

Go

10

F. Boerhaave Syndrome

11

G. Mallory weiss tear

12

H. Perforated gastric ulcer

13

Please select the most likely cause of chest pain for the scenario given. Each option may be
used once, more than once or not at all.

14
15
16
17

55.

A 52 year old male presents with tearing central chest pain. On examination he
has an aortic regurgitation murmur. An ECG shows ST elevation in leads II, III
and aVF.

18
19-21 3 / 3
22

You answered Inferior myocardial infarction

23
24

The correct answer is Proximal aortic dissection

25

Theme from 2011 Exam


An inferior myocardial infarction and AR murmur should raise suspicions of an
ascending aorta dissection rather than an inferior myocardial infarction alone.
Also the history is more suggestive of a dissection. Other features may include
pericardial effusion, carotid dissection and absent subclavian pulse.

26
27-29 1 / 3
30
31

56.

A 52 year old male presents with central chest pain and vomiting. He has drunk
a bottle of vodka. On examination there is some mild crepitus in the epigastric
region.

32
33
34

Boerhaave Syndrome

35

The Mackler triad for Boerhaave syndrome: vomiting, thoracic pain,


subcutaneous emphysema. It commonly presents in middle aged men with a
background of alcohol abuse.
57.

A 52 year old male presents with central chest pain. On examination he has an
mitral regurgitation murmur. An ECG shows ST elevation in leads V1 to V6.
There is no ST elevation in leads II, III and aVF.

36
37
38-40 2 / 3
41
42
43
44-46 0 / 3

You answered Pulmonary embolism

47

The correct answer is Anterior myocardial infarction

48

The most likely diagnosis is an anterior MI. As there are no ST changes in the
inferior leads, aortic dissection is less likely.

49
50
51

Next question

52
53
54

Chest pain

55-57 1 / 3

Aortic dissection
This occurs when there is a flap or filling defect within the aortic intima. Blood tracks
into the medial layer and splits the tissues with the subsequent creation of a false
lumen. It most commonly occurs in the ascending aorta or just distal to the left
subclavian artery (less common). It is most common in Afro-carribean males aged 5070 years.
Patients usually present with a tearing intrascapular pain, which may be similar to the
pain of a myocardial infarct.
The dissection may spread either proximally or distally with subsequent disruption to
the arterial branches that are encountered.
In the Stanford classification system the disease is classified into lesions with a
proximal origin (Type A) and those that commence distal to the left subclavian (Type

http://www.emrcs.com/question/question.php?q=0

1/2

13/07/2015

B).
Diagnosis may be suggested by a chest x-ray showing a widened mediastinum.
Confirmation of the diagnosis is usually made by use of CT angiography
Proximal (Type A) lesions are usually treated surgically, type B lesions are usually
managed non operatively.

Pulmonary embolism
Typically sudden onset of chest pain, haemoptysis, hypoxia and small pleural effusions
may be present.
Most patients will have an underlying deep vein thrombosis
Diagnosis may be suggested by various ECG findings including S waves in lead I, Q
waves in lead III and inverted T waves in lead III. Confirmation of the diagnosis is
usually made through use of CT pulmonary angiography.
Treatment is with anticoagulation, in those patients who develop a cardiac arrest or
severe compromise from their PE, consideration may be given to thrombolysis.

Myocardial infarction
Traditionally described as sudden onset of central, crushing chest pain. It may radiate
into the neck and down the left arm. Signs of autonomic dysfunction may be present.
The presenting features may be atypical in the elderly and those with diabetes.
Diagnosis is made through identification of new and usually dynamic ECG changes
(and cardiac enzyme changes). Inferior and anterior infarcts may be distinguished by
the presence of specific ECG changes (usually II, III and aVF for inferior, leads V1-V5
for anterior).
Treatment is with oral antiplatelet agents, primary coronary angioplasty and/ or
thrombolysis.

Perforated peptic ulcer


Patients usually develop sudden onset of epigastric abdominal pain, it may be soon
followed by generalised abdominal pain.
There may be features of antecendant abdominal discomfort, the pain of gastric ulcer
is typically worse immediately after eating.
Diagnosis may be made by erect chest x-ray which may show a small amount of free
intra-abdominal air (very large amounts of air are more typically associated with
colonic perforation).
Treatment is usually with a laparotomy, small defects may be excised and overlaid with
an omental patch, larger defects are best managed with a partial gastrectomy.

Boerhaaves syndrome
Spontaneous rupture of the oesophagus that occurs as a result of repeated episodes
of vomiting.
The rupture is usually distally sited and on the left side.
Patients usually give a history of sudden onset of severe chest pain that may
complicate severe vomiting.
Severe sepsis occurs secondary to mediastinitis.
Diagnosis is CT contrast swallow.
Treatment is with thoracotomy and lavage, if less than 12 hours after onset then
primary repair is usually feasible, surgery delayed beyond 12 hours is best managed
by insertion of a T tube to create a controlled fistula between oesophagus and skin.
Delays beyond 24 hours are associated with a very high mortality rate.

Rate question:

Next question

Comment on this question

All contents of this site are 2012 E-Medical Revision Ltd

http://www.emrcs.com/question/question.php?q=0

Terms and Conditions

Privacy policy

2/2

13/07/2015

Reference ranges

Question 58 of 78

Previous

Next

A 78 year old lady presents with a tender swelling in her right groin. On examination there is
a tender swelling that lies below and lateral to the pubic tubercle. It has a cough impulse.
What is the most likely underlying diagnosis?

Question stats

Score: 44.3%
1

5.9%

63.6%

10%

12.6%

8%

A. Thrombophlebitis of the great saphenous vein


63.6% of users answered this
question correctly

B. Femoral hernia

End and review

C. Thrombophlebitis of saphena varix

5
6-8 2 / 3
9
10

D. Inguinal hernia

11

Search

E. Obturator hernia

12
Go

Next question

13
14

Theme from April 2012 Exam


Whilst a thrombophlebitis of a saphena varix may cause a tender swelling at this site, it would
not usually be associated with a cough impulse.
Femoral canal

15
16
17
18
19-21 3 / 3

The femoral canal lies at the medial aspect of the femoral sheath. The femoral sheath is a
fascial tunnel containing both the femoral artery laterally and femoral vein medially. The
canal lies medial to the vein.

22
23
24

Borders of the femoral canal


Laterally

Femoral vein

25
26
27-29 1 / 3

Medially

Lacunar ligament

Anteriorly

Inguinal ligament

Posteriorly

Pectineal ligament

30
31
32
33

Image showing dissection of femoral canal

34
35
36
37
38-40 2 / 3
41
42
43
44-46 0 / 3
47
48
49
50
51
52
53
54
55-57 1 / 3
58

Image sourced from Wikipedia

Contents
Lymphatic vessels
Cloquet's lymph node

Physiological significance
Allows the femoral vein to expand to allow for increased venous return to the lower limbs.
Pathological significance
As a potential space, it is the site of femoral hernias. The relatively tight neck places these at
http://www.emrcs.com/question/question.php?q=0

1/2

13/07/2015

high risk of strangulation.


Rate question:

Next question

Comment on this question

All contents of this site are 2012 E-Medical Revision Ltd

http://www.emrcs.com/question/question.php?q=0

Terms and Conditions

Privacy policy

2/2

13/07/2015

Reference ranges

Previous

Question 59 of 78

Next

A 3 month old boy is suspected of having hypospadias. At which of the following locations is
the urethral opening most frequently located in boys suffering from the condition?

A. On the distal ventral surface of the penis


B. On the proximal ventral surface of the penis

Question stats

Score: 45.2%
1

48.1%

16.1%

19.5%

10.1%

6.3%

48.1% of users answered this


question correctly

C. On the distal dorsal surface of the penis

End and review

D. On the proximal dorsal surface of the penis

5
6-8 2 / 3
9
10

E. At the base of the scrotum

11

Search

12
Go

Next question

13
14

The defect is located ventrally and most often distally. Proximally located urethral openings
are well recognised. Circumcision may compromise reconstruction.

15
16

Hypospadias

17
18

The urethral meatus opens on the ventral surface of the penis. There is also a ventral
deficiency of the foreskin. The urethral meatus may open more proximally in the more severe
variants. However, 75% of the openings are distally located. The incidence is 1 in 300 male
births.

19-21 3 / 3
22
23
24

Features include:

25
26

Absent frenular artery


Ventrally opened glans
Skin tethering to hypoplastic urethra
Splayed columns of spongiosum tissue distal to the meatus
Deficiency of the foreskin ventrally

27-29 1 / 3
30
31
32
33

Management:

34
35

No routine cultural circumcisions


Urethroplasty
Penile reconstruction

36
37
38-40 2 / 3
41

The foreskin is often utilised in the reconstructive process. In boys with very distal disease no
treatment may be needed.

42
43
44-46 0 / 3

Rate question:

Next question

47
48

Comment on this question

49
50
51
52
53
54
55-57 1 / 3
58
59

All contents of this site are 2012 E-Medical Revision Ltd

http://www.emrcs.com/question/question.php?q=0

Terms and Conditions

Privacy policy

1/1

13/07/2015

Reference ranges

Previous

Question 60 of 78

Next

A 52 year old male attends for a preoperative assessment for an inguinal hernia repair. You
notice that the chest x-ray shows a loculated left pleural effusion. On further questioning the
patient reports that he worked as a builder 30 years ago. What is the most likely cause for
the effusion?

Question stats

Score: 44.4%
1

29.2%

6.8%

43.9%

13.8%

6.3%

43.9% of users answered this


question correctly

A. Asbestosis

End and review

5
6-8 2 / 3
9

B. Pneumonia
10

C. Mesothelioma
11

Search

D. Silicosis

12

E. Left ventricular failure

Go

13
14

Next question

15

This patient has a risk of asbestos exposure through his occupation as a builder. As there a
is latent period of 30 years and a complicated effusion, the most likely cause is
mesothelioma.

16
17
18
19-21 3 / 3

Mesothelioma

22

Features

23

Dyspnoea, weight loss, chest wall pain


Clubbing
30% present as painless pleural effusion
Only 20% have pre-existing asbestosis
History of asbestos exposure in 85-90%, latent period of 30-40 years

24
25
26
27-29 1 / 3
30
31

Basics

32

Malignancy of mesothelial cells of pleura


Metastases to contralateral lung and peritoneum
Right lung affected more often than left

33
34
35

Management

36
37

Investigation: pleural biopsy, CT Scanning, (PET Scanning if surgery considered)


Symptomatic
Industrial compensation
Chemotherapy, Surgery if operable
Prognosis poor, median survival 12 months

38-40 2 / 3
41
42
43
44-46 0 / 3
47

Rate question:

Next question

48
49

Comment on this question

50
51
52
53
54
55-57 1 / 3
58
59
60

All contents of this site are 2012 E-Medical Revision Ltd

http://www.emrcs.com/question/question.php?q=0

Terms and Conditions

Privacy policy

1/1

13/07/2015

Reference ranges

Previous

Question 61 of 78

Next

A 64-year-old woman who is reviewed due to multiple non-healing leg ulcers. She reports
feeling generally unwell for many months. Examination findings include a blood pressure of
138/72 mmHg, pulse 90 bpm, pale conjunctivae and poor dentition associated with bleeding
gums. What is the most likely underlying diagnosis?

Question stats

Score: 45.3%
1

7.1%

14.8%

62.5%

8.4%

7.3%

62.5% of users answered this


question correctly

A. Thyrotoxicosis

End and review

5
6-8 2 / 3
9

B. Vitamin B12 deficiency


10

C. Vitamin C deficiency
11

Search

D. Diabetes mellitus

12

E. Sarcoidosis

Go

13
14

Next question

15

Bleeding gums and poor healing are suggestive of vitamin C deficiency.

16
17

Vitamin C deficiency

18
19-21 3 / 3

Vitamin C deficiency (scurvy) leads to defective synthesis of collagen resulting in capillary


fragility (bleeding tendency) and poor wound healing

22
23

Features
24

gingivitis, loose teeth


poor wound healing
bleeding from gums, haematuria, epistaxis
general malaise

25
26
27-29 1 / 3
30
31

Rate question:

32

Next question

33
Comment on this question

34
35
36
37
38-40 2 / 3
41
42
43
44-46 0 / 3
47
48
49
50
51
52
53
54
55-57 1 / 3
58
59
60
61

All contents of this site are 2012 E-Medical Revision Ltd

http://www.emrcs.com/question/question.php?q=0

Terms and Conditions

Privacy policy

1/1

13/07/2015

Reference ranges

Previous

Question 62 of 78

Next

Question stats

End and review

Score: 44.6%
1

8.6%

46.1%

13.7%

A. Transient delay in neuronal transmission

14.1%

B. Axonal degeneration distal to the site of injury

17.5%

C. Absence of neuroma formation

46.1% of users answered this


question correctly

Which of the following is not a typical feature of neuropraxia?

D. Preservation of autonomic function

5
6-8 2 / 3
9
10

E. Absence of axonal degeneration proximal to the site of injury

11

Search
Next question

12
Go

13

Full recovery may occur 6-8 weeks after nerve injury in neuropraxia.
Wallerian degeneration does not usually occur in simple neuropraxia.
Autonomic function is usually preserved.

14
15
16

Nerve injury

17
18

There are 3 types of nerve injury:

19-21 3 / 3

Neuropraxia

22

Nerve intact but electrical conduction is affected


Full recovery
Autonomic function preserved
Wallerian degeneration does not occur

23
24
25

Axonotmesis

26

Axon is damaged and the myelin sheath is preserved. The connective


tissue framework is not affected.
Wallerian degeneration occurs.

27-29 1 / 3
30
31

Neurotmesis

Disruption of the axon, myelin sheath and surrounding connective tissue.


Wallerian degeneration occurs.

32
33
34

Wallerian Degeneration

35
36

Axonal degeneration distal to the site of injury.


Typically begins 24-36 hours following injury.
Axons are excitable prior to degeneration occurring.
Myelin sheath degenerates and is phagocytosed by tissue macrophages.

37
38-40 2 / 3
41
42

Nerve repair

43

Neuronal repair may only occur physiologically where nerves are in direct contact.
Where a large defect is present, the process of nerve regeneration is hampered. It
may not occur at all or result in the formation of a neuroma. Where nerve regrowth
occurs it is typically at a rate of 1mm per day.

44-46 0 / 3
47
48
49
50

Rate question:

51
Next question

52
Comment on this question

53
54
55-57 1 / 3
58
59
60
61
62

All contents of this site are 2012 E-Medical Revision Ltd

http://www.emrcs.com/question/question.php?q=0

Terms and Conditions

Privacy policy

1/1

13/07/2015

Reference ranges

Previous

Question 63 of 78

Next

A 44 year old lady presents with a pathological fracture of the left femur. She has previously
undergone a renal transplant for end stage renal failure. Her blood test results are as
follows:
Serum Ca2+
PTH

Question stats

Score: 45.5%
1

8.1%

18.3%

55.3%

11.6%

6.6%

2.80
55.3% of users answered this
question correctly

88pg/ml

End and review

5
6-8 2 / 3
9
10

Phosphate

0.30

11

Search

A surgeon decides to perform a parathyroidectomy on the basis of these results. When the
glands are assessed histologically, which of the appearances is most likely to be identified?

12
Go

13
14

A. Metaplasia the gland

15

B. Hypertrophy of the gland

16

C. Hyperplasia of the gland

17
18

D. Parathyroid carcinoma

19-21 3 / 3

E. Necrosis of the parathyroid gland

22
Next question

23
24

This is likely to be a case of tertiary hyperparathyroidism (high Calcium, high PTH, low
phosphate). Therefore the glands will be hyperplastic. Hypertrophy is not correct as this
implies an increase in size without an increase in cellularity. This mistake has cost many
candidates marks in the MRCS exams over the years!

25
26
27-29 1 / 3
30

Parathyroid glands and disorders of calcium metabolism

31
32

Hyperparathyroidism
Disease type

Hormone profile

Clinical
features

Cause

May be
asymptomatic if
mild
Recurrent
abdominal pain
(pancreatitis,
renal colic)
Changes to
emotional or
cognitive state

Most cases due to solitary


adenoma (80%), multifocal
disease occurs in 10-15%
and parathyroid carcinoma in
1% or less

33
34
35

Primary
hyperparathyroidism

PTH
(Elevated)
Ca2+
(Elevated)
Phosphate
(Low)
Urine
calcium :
creatinine
clearance
ratio > 0.01

36
37
38-40 2 / 3
41
42
43
44-46 0 / 3
47
48

Secondary
hyperparathyroidism

PTH
(Elevated)
Ca2+ (Low or
normal)
Phosphate
(Elevated)
Vitamin D
levels (Low)

May have few


symptoms
Eventually
may develop
bone disease,
osteitis fibrosa
cystica and soft
tissue
calcifications

Parathyroid gland hyperplasia


occurs as a result of low
calcium, almost always in a
setting of chronic renal failure

49
50
51
52
53
54
55-57 1 / 3

Tertiary
hyperparathyroidism

Ca2+
(Normal or
high)
PTH
(Elevated)
Phosphate
levels
(Decreased
or Normal)
Vitamin D
(Normal or
decreased)
Alkaline
phosphatase
(Elevated)

http://www.emrcs.com/question/question.php?q=0

Metastatic
calcification
Bone pain
and / or fracture
Nephrolithiasis
Pancreatitis

Occurs as a result of ongoing


hyperplasia of the parathyroid
glands after correction of
underlying renal disorder,
hyperplasia of all 4 glands is
usually the cause

58
59
60
61
62
63

1/2

13/07/2015

Differential diagnoses
It is important to consider the rare but relatively benign condition of benign familial
hypocalciuric hypercalcaemia, caused by an autosomal dominant genetic disorder. Diagnosis
is usually made by genetic testing and concordant biochemistry (urine calcium : creatinine
clearance ratio <0.01-distinguished from primary hyperparathyroidism).
Treatment
Primary hyperparathyroidism
Indications for surgery
Elevated serum Calcium > 1mg/dL above normal
Hypercalciuria > 400mg/day
Creatinine clearance < 30% compared with normal
Episode of life threatening hypercalcaemia
Nephrolithiasis
Age < 50 years
Neuromuscular symptoms
Reduction in bone mineral density of the femoral neck, lumbar spine, or distal radius of
more than 2.5 standard deviations below peak bone mass (T score lower than -2.5)

Secondary hyperparathyroidism
Usually managed with medical therapy.
Indications for surgery in secondary (renal) hyperparathyroidism:
Bone pain
Persistent pruritus
Soft tissue calcifications

Tertiary hyperparathyroidism
Allow 12 months to elapse following transplant as many cases will resolve
The presence of an autonomously functioning parathyroid gland may require surgery. If the
culprit gland can be identified then it should be excised. Otherwise total parathyroidectomy
and re-implantation of part of the gland may be required.
References
1. Pitt S et al. Secondary and Tertiary Hyperparathyroidism, State of the Art Surgical
Management. Surg Clin North Am 2009 Oct;89(5):1227-39.
2. MacKenzie-Feder J et al. Primary Hyperparathyroidism: An Overview. Int J Endocrinol
2011; 2011: 251410.
Rate question:

Next question

Comment on this question

All contents of this site are 2012 E-Medical Revision Ltd

http://www.emrcs.com/question/question.php?q=0

Terms and Conditions

Privacy policy

2/2

13/07/2015

Reference ranges

Previous

Question 64 of 78

Next

A 78 year old man is referred to the clinic by his general practitioner. For many years he
noticed a smooth swelling approximately 5cm anterior to the tragus of his right ear. Apart
from being a heavy smoker he has no co-morbidities. What is the most likely diagnosis?

Question stats

Score: 46.3%
1

33.5%

8.1%

38.4%

12.1%

7.9%

A. Pleomorphic adenoma
38.4% of users answered this
question correctly

B. Liposarcoma

End and review

C. Warthins tumour

5
6-8 2 / 3
9
10

D. Adenocarcinoma

11

Search

E. None of the above

12
Go

Next question

13
14

Warthins tumours are most common in elderly smokers. They have a relatively benign and
indolent course. They are usually well circumscribed as illustrated below:

15
16
17
18
19-21 3 / 3
22
23
24
25
26
27-29 1 / 3
30
31
32
33
34
35

Image sourced from Wikipedia

36
37
38-40 2 / 3

Parotid gland clinical

41

Benign neoplasms
Up to 80% of all salivary gland tumours occur in the parotid gland and up to 80% of these
are benign. There is no consistent correlation between the rate of growth and the malignant
potential of the lesion. However, benign tumours should not invade structures such as the
facial nerve.
With the exception of Warthins tumours, they are commoner in women than men. The
median age of developing a lesion is in the 5th decade of life.

42
43
44-46 0 / 3
47
48
49
50

Benign tumour types


Tumour type

Features

Benign pleomorphic
adenoma or benign mixed
tumor

Most common parotid neoplasm (80%)


Proliferation of epithelial and myoepithelial cells of the ducts and an
increase in stromal components
Slow growing, lobular, and not well encapsulated
Recurrence rate of 1-5% with appropriate excision (parotidectomy)
Recurrence possibly secondary to capsular disruption during
surgery
Malignant degeneration occurring in 2-10% of adenomas observed
for long periods, with carcinoma ex-pleomorphic adenoma occurring
most frequently as adenocarcinoma

51
52

Warthin tumor (papillary


cystadenoma lymphoma
or adenolymphoma)

Second most common benign parotid tumor (5%)


Most common bilateral benign neoplasm of the parotid
Marked male as compared to female predominance
Occurs later in life (sixth and seventh decades)
Presents as a lymphocytic infiltrate and cystic epithelial
proliferation
May represent heterotopic salivary gland epithelial tissue trapped
within intraparotid lymph nodes
Incidence of bilaterality and multicentricity of 10%
Malignant transformation rare (almost unheard of)

http://www.emrcs.com/question/question.php?q=0

53
54
55-57 1 / 3
58
59
60
61
62
63
64

1/3

13/07/2015

Monomorphic adenoma

Account for less than 5% of tumours


Slow growing
Consist of only one morphological cell type (hence term mono)
Include; basal cell adenoma, canalicular adenoma, oncocytoma,
myoepitheliomas

Haemangioma

Should be considered in the differential of a parotid mass in a child


Accounts for 90% of parotid tumours in children less than 1 year of
age
Hypervascular on imaging
Spontaneous regression may occur and malignant transformation is
almost unheard of

Malignant salivary gland tumours


Types of malignancy
Mucoepidermoid
carcinoma

30% of all parotid malignancies


Usually low potential for local invasiveness and metastasis (depends
mainly on grade)

Adenoid cystic
carcinoma

Unpredictable growth pattern


Tendency for perineural spread
Nerve growth may display skip lesions resulting in incomplete excision
Distant metastasis more common (visceral rather than nodal spread)
5 year survival 35%

Mixed tumours

Often a malignancy occurring in a previously benign parotid lesion

Acinic cell
carcinoma

Intermediate grade malignancy


May show perineural invasion
Low potential for distant metastasis
5 year survival 80%

Adenocarcinoma

Develops from secretory portion of gland


Risk of regional nodal and distant metastasis
5 year survival depends upon stage at presentation, may be up to 75%
with small lesions with no nodal involvement

Lymphoma

Large rubbery lesion, may occur in association with Warthins tumours


Diagnosis should be based on regional nodal biopsy rather than parotid
resection
Treatment is with chemotherapy (and radiotherapy)

Diagnostic evaluation
Plain x-rays may be used to exclude calculi
Sialography may be used to delineate ductal anatomy
FNAC is used in most cases
Superficial parotidectomy may be either diagnostic of therapeutic depending upon the
nature of the lesion
Where malignancy is suspected the primary approach should be definitive resection
rather than excisional biopsy
CT/ MRI may be used in cases of malignancy for staging primary disease

Treatment
For nearly all lesions this consists of surgical resection, for benign disease this will usually
consist of a superficial parotidectomy. For malignant disease a radical or extended radical
parotidectomy is performed. The facial nerve is included in the resection if involved. The
need for neck dissection is determined by the potential for nodal involvement.
Other parotid disorders
HIV infection
Lymphoepithelial cysts associated with HIV occur almost exclusively in the parotid
Typically presents as bilateral, multicystic, symmetrical swelling
Risk of malignant transformation is low and management usually conservative

Sjogren syndrome
Autoimmune disorder characterised by parotid enlargement, xerostomia and
keratoconjunctivitis sicca
90% of cases occur in females
Second most common connective tissue disorder
Bilateral, non tender enlargement of the gland is usual
Histologically, the usual findings are of a lymphocytic infiltrate in acinar units and
epimyoepithelial islands surrounded by lymphoid stroma
Treatment is supportive
There is an increased risk of subsequent lymphoma

Sarcoid
Parotid involvement occurs in 6% of patients with sarcoid

http://www.emrcs.com/question/question.php?q=0

2/3

13/07/2015

Bilateral in most cases


Gland is not tender
Xerostomia may occur
Management of isolated parotid disease is usually conservative

Rate question:

Next question

Comment on this question

All contents of this site are 2012 E-Medical Revision Ltd

http://www.emrcs.com/question/question.php?q=0

Terms and Conditions

Privacy policy

3/3

13/07/2015

Reference ranges

Previous

Question 65 of 78

Next

A 56 year old man presents with lethargy, haematuria and haemoptysis. On examination he
is hypertensive and has a right loin mass. A CT scan shows a lesion affecting the upper pole
of the right kidney, it has a small cystic centre. Which of the options below is the most likely
diagnosis?

Question stats

Score: 45.6%
1

10.7%

10%

51.6%

9.2%

18.6%

51.6% of users answered this


question correctly

A. Squamous cell carcinoma of the kidney

End and review

5
6-8 2 / 3
9

B. Nephroblastoma
10

C. Renal adenocarcinoma
11

Search

D. Transitional cell carcinoma of the kidney

12

E. Polycystic kidney disease

Go

13
Next question

14
15

Renal adenocarcinoma are the most common renal tumours. These will typically affect the
renal parenchyma. Transitional cell carcinoma will usually affect urothelial surfaces.
Nephroblastoma would be very rare in this age group. Renal adenocarcinoma may produce
cannon ball metastasis in the lung which cause haemoptysis, this is not a feature of PKD.

16
17
18
19-21 3 / 3

Renal tumours

22
23

Renal cell carcinoma


Renal cell carcinoma is an adenocarcinoma of the renal cortex and is believed to arise from
the proximal convoluted tubule. They are usually solid lesions, up to 20% may be multifocal,
20% may be calcified and 20% may have either a cystic component or be wholly cystic. They
are often circumscribed by a pseudocapsule of compressed normal renal tissue. Spread may
occur either by direct extension into the adrenal gland, renal vein or surrounding fascia.
More distant disease usually occurs via the haematogenous route to lung, bone or brain.
Renal cell carcinoma comprise up to 85% of all renal malignancies. Males are more
commonly affected than females and sporadic tumours typically affect patients in their sixth
decade.
Patients may present with a variety of symptoms including; haematuria (50%), loin pain
(40%), mass (30%) and up to 25% may have symptoms of metastasis.Less than 10% have
the classic triad of haematuria, pain and mass.
Investigation
Many cases will present as haematuria and be discovered during diagnostic work up. Benign
renal tumours are rare, so renal masses should be investigated with multislice CT scanning.
Some units will add and arterial and venous phase to the scan to demonstrate vascularity
and evidence of caval ingrowth.

24
25
26
27-29 1 / 3
30
31
32
33
34
35
36
37
38-40 2 / 3
41
42

CT scanning of the chest and abdomen to detect distant disease should also be undertaken.

43
44-46 0 / 3

Routine bone scanning is not indicated in the absence of symptoms.


Biopsy should not be performed when a nephrectomy is planned but is mandatory before
any ablative therapies are undertaken.
Assessment of the functioning of the contra lateral kidney.

47
48
49
50
51

Management
T1 lesions may be managed by partial nephrectomy and this gives equivalent oncological
results to total radical nephrectomy. Partial nephrectomy may also be performed when there
is inadequate reserve in the remaining kidney.

52
53
54
55-57 1 / 3

For T2 lesions and above a radical nephrectomy is standard practice and this may be
performed via a laparoscopic or open approach. Preoperative embolisation is not indicated
nor is resection of uninvolved adrenal glands. During surgery early venous control is
mandatory to avoid shedding of tumour cells into the circulation.

58
59
60
61

Patients with completely resected disease do not benefit from adjuvant therapy with
either chemotherapy or biological agents. These should not be administered outside the
setting of clinical trials.

62
63
64

Patients with transitional cell cancer will require a nephroureterectomy with disconnection of
the ureter at the bladder.

65

References
Lungberg B et al. EAU guidelines on renal cell carcinoma: The 2010 update. European
Urology 2010 (58): 398-406.
http://www.emrcs.com/question/question.php?q=0

1/2

13/07/2015

Rate question:

Next question

Comment on this question

All contents of this site are 2012 E-Medical Revision Ltd

http://www.emrcs.com/question/question.php?q=0

Terms and Conditions

Privacy policy

2/2

13/07/2015

Reference ranges

Previous

Question 66 of 78

Next

A 34-year-old man is taken immediately to theatre with aortic dissection. You note he is tall
with pectus excavatum and arachnodactyly. His condition is primarily due to a defect in which
one of the following proteins?

Question stats

Score: 44.9%
1

7.5%

27.8%

23.5%

21.2%

20%

A. Polycystin-1
27.8% of users answered this
question correctly

B. Fibrillin

End and review

C. Type IV collagen

5
6-8 2 / 3
9
10

D. Type I collagen

11

Search

E. Elastin

12
Go

13

Next question

14
15

Although fibrillin is the primary protein affected (due to a defect in the fibrillin-1 gene) it
should be noted that fibrillin is used as a substrate of elastin.

16
17

Marfan's syndrome

18
19-21 3 / 3

Marfan's syndrome is an autosomal dominant connective tissue disorder. It is caused by a


defect in the fibrillin-1 gene on chromosome 15 and affects around 1 in 3,000 people.

22
23

Features
24

tall stature with arm span to height ratio > 1.05


high-arched palate
arachnodactyly
pectus excavatum
pes planus
scoliosis of > 20 degrees
heart: dilation of the aortic sinuses (seen in 90%) which may lead to aortic aneurysm,
aortic dissection, aortic regurgitation, mitral valve prolapse (75%),
lungs: repeated pneumothoraces
eyes: upwards lens dislocation (superotemporal ectopia lentis), blue sclera, myopia
dural ectasia (ballooning of the dural sac at the lumbosacral level)

25
26
27-29 1 / 3
30
31
32
33
34
35
36

The life expectancy of patients used to be around 40-50 years. With the advent of regular
echocardiography monitoring and beta-blocker/ACE-inhibitor therapy this has improved
significantly over recent years. Aortic dissection and other cardiovascular problems remain
the leading cause of death however.

37
38-40 2 / 3
41
42

Rate question:

Next question

43
44-46 0 / 3

Comment on this question

47
48
49
50
51
52
53
54
55-57 1 / 3
58
59
60
61
62
63
64
65
66

All contents of this site are 2012 E-Medical Revision Ltd

http://www.emrcs.com/question/question.php?q=0

Terms and Conditions

Privacy policy

1/2

13/07/2015

http://www.emrcs.com/question/question.php?q=0

2/2

13/07/2015

Reference ranges

Previous

Question 67 of 78

Next

Question stats

End and review

Score: 44.3%
1

50%

9.5%

8%

A. It is inherited in an autosomal recessive manner

15.5%

B. Affected patients are more likely to develop right colon mucinous


tumours than the general population

17%

Which of the following are not typical of Lynch syndrome?

C. Affected individuals have an 80% lifetime risk of colon cancer

50% of users answered this


question correctly

D. Endometrial cancer is seen in 80% of women

5
6-8 2 / 3
9
10

E. Gastric cancers are more common

11

Search

12
Next question

Lynch syndrome is inherited in an autosomal dominant fashion. It is characterised by


microsatellite instability in the DNA mismatch repair genes. Colonic tumours in patients with
Lynch syndrome are more likely to be right sided tumours and to be poorly differentiated.

Go

13
14
15
16
17

Genetics and surgical disease

18

Some of the more commonly occurring genetic conditions occurring in surgical patients are
presented here.

19-21 3 / 3
22
23

Li-Fraumeni Syndrome

24

Autosomal dominant
Consists of germline mutations to p53 tumour suppressor gene
High incidence of malignancies particularly sarcomas and leukaemias
Diagnosed when:

25
26
27-29 1 / 3
30

*Individual develops sarcoma under 45 years


*First degree relative diagnosed with any cancer below age 45 years and another family
member develops malignancy under 45 years or sarcoma at any age

31

BRCA 1 and 2

34

Carried on chromosome 17 (BRCA 1) and Chromosome 13 (BRCA 2)


Linked to developing breast cancer (60%) risk.
Associated risk of developing ovarian cancer (55% with BRCA 1 and 25% with BRCA
2).

32
33

35
36
37
38-40 2 / 3
41

Lynch Syndrome

42
43

Autosomal dominant
Develop colonic cancer and endometrial cancer at young age
80% of affected individuals will get colonic and/ or endometrial cancer
High risk individuals may be identified using the Amsterdam criteria

44-46 0 / 3
47
48
49

Amsterdam criteria
Three or more family members with a confirmed diagnosis of colorectal cancer, one of whom
is a first degree (parent, child, sibling) relative of the other two.
Two successive affected generations.
One or more colon cancers diagnosed under age 50 years.
Familial adenomatous polyposis (FAP) has been excluded.

50
51
52
53
54

Gardners syndrome

55-57 1 / 3

Autosomal dominant familial colorectal polyposis


Multiple colonic polyps
Extra colonic diseases include: skull osteoma, thyroid cancer and epidermoid cysts
Desmoid tumours are seen in 15%
Mutation of APC gene located on chromosome 5
Due to colonic polyps most patients will undergo colectomy to reduce risk of colorectal
cancer
Now considered a variant of familial adenomatous polyposis coli

58
59
60
61
62
63
64
65
66

Rate question:

Next question

67

Comment on this question

http://www.emrcs.com/question/question.php?q=0

1/2

13/07/2015

All contents of this site are 2012 E-Medical Revision Ltd

http://www.emrcs.com/question/question.php?q=0

Terms and Conditions

Privacy policy

2/2

13/07/2015

Reference ranges

Previous

Question 68 of 78

Next

A 23 year old man suffers a thermal injury to his left hand. It becomes red and painful. Which
of the following mediators are not involved in this process?

A. Histamine
B. Free radicals

Question stats

Score: 43.7%
1

11.5%

29.9%

8%

9.1%

41.4%

29.9% of users answered this


question correctly

C. Prostaglandins

End and review

D. Leukotrienes

5
6-8 2 / 3
9
10

E. Serotonin

11

Search

12
Next question

Go

13
14

Acute inflammation is not mediated by free radicals

15
16

Chemical mediators facilitate the spread of inflammation into normal tissue


Chemical mediators include:
Lysosomal compounds
Chemokines such as serotinin and histamine (released by platelets and mast cells)

17
18
19-21 3 / 3
22
23

Other enzyme cascades producing inflammatory mediators include:


Complement, kinin, coagulation system and fibrinolytic system

24
25
26
27-29 1 / 3
30

Acute inflammation

31

Inflammation is the reaction of the tissue elements to injury. Vascular changes occur,
resulting in the generation of a protein rich exudate. So long as the injury does not totally
destroy the existing tissue architecture, the episode may resolve with restoration of original
tissue architecture.

32
33
34
35

Vascular changes
36

Vasodilation occurs and persists throughout the inflammatory phase.


Inflammatory cells exit the circulation at the site of injury.
The equilibrium that balances Starlings forces within capillary beds is disrupted and a
protein rich exudate will form as the vessel walls also become more permeable to
proteins.
The high fibrinogen content of the fluid may form a fibrin clot. This has several
important immunomodulatory functions.

37
38-40 2 / 3
41
42
43
44-46 0 / 3
47

Sequelae
Resolution

48

Typically occurs with minimal initial injury


Stimulus removed and normal tissue architecture results

49
50
51

Organisation

Delayed removal of exudate


Tissues undergo organisation and usually fibrosis

52
53
54

Suppuration

Typically formation of an abscess or an empyema


Sequestration of large quantities of dead neutrophils

55-57 1 / 3
58
59

Progression to chronic
inflammation

Coupled inflammatory and reparative activities


Usually occurs when initial infection or suppuration has
been inadequately managed

60
61
62
63

Causes
Infections e.g. Viruses, exotoxins or endotoxins released by bacteria
Chemical agents
Physical agents e.g. Trauma
Hypersensitivity reactions
Tissue necrosis

http://www.emrcs.com/question/question.php?q=0

64
65
66
67
68

1/2

13/07/2015

Presence of neutrophil polymorphs is a histological diagnostic feature of acute


inflammation
Rate question:

Next question

Comment on this question

All contents of this site are 2012 E-Medical Revision Ltd

http://www.emrcs.com/question/question.php?q=0

Terms and Conditions

Privacy policy

2/2

13/07/2015

Reference ranges

Previous

Question 69 of 78

Next

An enthusiastic medical student approaches you with a list of questions about blood
transfusion reactions. Which of her following points is incorrect?

A. Graft versus host disease involves neutrophil proliferation


B. Thrombocytopaenia may occur in women with a prior pregnancy

Question stats

Score: 43.1%
1

36%

11.7%

27.3%

16.2%

8.9%

36% of users answered this


question correctly

C. IgA antibodies may cause blood pressure compromise during


transfusion

End and review

5
6-8 2 / 3
9
10

D. Hypocalcaemia can occur

11

Search

E. Iron overload can be avoided by chelation therapy

12
Go

Next question

13
14
15

Mnemonic for transfusion reactions:


Got a bad unit

16
17
18

G raft vs. Host disease


O verload
T hrombocytopaenia

19-21 3 / 3
22
23

A lloimmunization
B lood pressure unstable
A cute haemolytic reaction
D elayed haemolytic reaction

24
25
26
27-29 1 / 3
30

U rticaria
N eutrophilia
I nfection
T ransfusion associated lung injury

31
32
33
34

GVHD results from lymphocytic proliferation. The patient's own lymphocytes are similar to the
donor's lymphocytes, therefore don't perceive them as being foreign. The donor
lymphocytes, however, sees the recipient lymphocytes as being foreign. Therefore they
proliferate causing severe complications.

35
36
37
38-40 2 / 3

Thrombocytopaenia occurs a few days after transfusion and may resolve spontaneously.
Patients with IGA antibodies need IgA deficient blood transfusions.

41
42
43

Blood transfusion reactions

44-46 0 / 3
47

Acute transfusion reactions present as adverse signs or symptoms during or within 24 hours
of a blood transfusion. The most frequent reactions are fever, chills, pruritus, or urticaria,
which typically resolve promptly without specific treatment or complications. Other signs
occurring in temporal relationship with a blood transfusion, such as severe dyspnoea,
pyrexia, or loss of consciousness may be the first indication of a more severe potentially fatal
reaction.
The causes of adverse reactions are multi-factorial. Immune mediated reactions, some of the
most feared, occur as a result of component mismatch, the commonest cause of which is
clerical error. More common, non immune mediated, complications may occur as a result of
product contamination, this may be bacterial or viral.
Transfusion related lung injury is well recognised and there are two proposed mechanisms
which underpin this. One involves the sequestration of primed neutrophils within the recipient
pulmonary capillary bed. The other proposed mechanism suggests that HLA mismatches
between donor neutrophils and recipient lung tissue is to blame.
The table below summarises the main types of transfusion reaction.

48
49
50
51
52
53
54
55-57 1 / 3
58
59
60
61

Immune mediated

Non immune mediated

Pyrexia

Hypocalcaemia

Alloimmunization

CCF

Thrombocytopaenia

Infections

Transfusion associated lung injury

Hyperkalaemia

62
63
64
65
66
67

http://www.emrcs.com/question/question.php?q=0

68

1/2

13/07/2015
69

Graft vs Host disease


Urticaria
Acute or delayed haemolysis
ABO incompatibility
Rhesus incompatibility

Rate question:

Next question

Comment on this question

All contents of this site are 2012 E-Medical Revision Ltd

http://www.emrcs.com/question/question.php?q=0

Terms and Conditions

Privacy policy

2/2

13/07/2015

Reference ranges

Previous

Question 70 of 78

Next

An 82 year old lady presents with a carcinoma of the caecum. Approximately what proportion
of patients presenting with this diagnosis will have synchronous lesions?

A. <1%
B. 60%

Question stats

Score: 42.5%
1

12.8%

10.5%

10%

27.9%

38.8%

38.8% of users answered this


question correctly

C. 50%

End and review

D. 20%

5
6-8 2 / 3
9
10

E. 5%

11

Search

12
Next question

Go

13
14

Synchronous colonic tumours are seen in 5% cases and all patients having a flexible
sigmoidoscopy should have completion colonoscopy if tumours or polyps are found

15
16
17

Synchronous lesions may occur in up to 5% of patients with colorectal cancer. A full and
complete lumenal study with either colonoscopy, CT cologram or barium enema is mandatory
in all patients being considered for surgery.

18
19-21 3 / 3
22

Colorectal cancer

23
24

Annually about 150,000 new cases are diagnosed and 50,000 deaths from the
disease
About 75% will have sporadic disease and 25% will have a family history
Colorectal tumours comprise a spectrum of disease ranging from adenomas through
to polyp cancers and frank malignancy.
Polyps may be categorised into: neoplastic polyps, adenomatous polyps and non
neoplastic polyps.
The majority of adenomas are polypoidal lesions, although flat lesions do occur and
may prove to be dysplastic.
Non-neoplastic polyps include hyperplastic, juvenile, hamartomatous, inflammatory,
and lymphoid polyps, which have not generally been thought of as precursors of
cancer.
Three characteristics of adenomas that correlate with malignant potential have been
characterised. These include increased size, villous architecture and dysplasia. For
this reason most polyps identified at colonoscopy should be removed.
The transformation from polyp to cancer is described by the adenoma - carcinoma
sequence and its principles should be appreciated. Essentially genetic changes
accompany the transition from adenoma to carcinoma; key changes include APC, cmyc, K RAS mutations and p53 deletions.

25
26
27-29 1 / 3
30
31
32
33
34
35
36
37
38-40 2 / 3
41
42
43
44-46 0 / 3
47

Rate question:

Next question

48
49

Comment on this question

50
51
52
53
54
55-57 1 / 3
58
59
60
61
62
63
64
65
66
67
68

http://www.emrcs.com/question/question.php?q=0

1/2

13/07/2015
69
70

All contents of this site are 2012 E-Medical Revision Ltd

http://www.emrcs.com/question/question.php?q=0

Terms and Conditions

Privacy policy

2/2

13/07/2015

Reference ranges

Question 1 of 8

Next

A 22 year old man undergoes a splenectomy for an iatrogenic splenic injury. On the second
post operative day a full blood count is performed. Which of the following components of the
full blood count is the first to be affected ?

Question stats

End and review

Score: 0%

18.8%

49.5%

8.7%

11.2%

11.9%

A. Erythrocyte count
49.5% of users answered this
question correctly

B. Reticulocyte count
C. Eosinophil count
D. Monocyte count

Search

E. Lymphocyte count

Go

Next question

Theme from January 2012 Exam


The granulocyte and platelet count are the first to be affected following splenectomy. Then
reticulocytes increase. Although a lymphocytosis and monocytosis are reported, these take
several weeks to develop.
Post splenectomy blood film changes
The loss of splenic tissue results in the inability to readily remove immature or abnormal red
blood cells from the circulation. The red cell count does not alter significantly. However,
cytoplasmic inclusions may be seen e.g. Howell-Jolly bodies.
In the first few days after splenectomy target cells, siderocytes and reticulocytes will appear
in the circulation. Immediately following splenectomy a granulocytosis (mainly composed of
neutrophils) is seen, this is replaced by a lymphocytosis and monocytosis over the following
weeks.
The platelet count is usually increased and this may be persistent, oral antiplatelet agents
may be needed in some patients.
Image showing Howell Jolly bodies (arrowed)

Image sourced from Wikipedia

Rate question:

Next question

Comment on this question

All contents of this site are 2012 E-Medical Revision Ltd

http://www.emrcs.com/question/question.php?q=0

Terms and Conditions

Privacy policy

1/1

13/07/2015

Reference ranges

Previous

Question 2 of 8

Next

A 28 year old lady presents with benign cyclical mastalgia. Which of the following is not a
recognised treatment for the condition?

A. Evening primrose oil


B. Bromocriptine

Question stats

End and review

Score: 0%

10%

10%

52.8%

7.8%

19.5%

1
2

52.8% of users answered this


question correctly

C. Methotrexate
D. Danazol
E. Tamoxifen

Search
Go

Next question

Surgical excision of tender breast tissue is inappropriate

Methotrexate is used for the treatment of breast cancer. Whilst the use of tamoxifen is of
benefit other agents such as flaxseed oil or evening primrose oil should be tried first.
Danazol is effective, but many women dislike the side effects.
Benign cyclical mastalgia
Benign cyclical mastalgia is a common cause of breast pain in younger females. It varies in
intensity according to the phase of the menstrual cycle. It is not associated with point
tenderness of the chest wall (more likely to be Tietze's syndrome).
The underlying cause is difficult to pinpoint, examination should focus on identifying focal
lesions (such as cysts) that may be treated to provide symptomatic benefit. Women should
be advised to wear a supportive bra. Conservative treatments include flax seed oil and
evening primrose oil. There is slightly more evidence in favor of flax seed oil, though neither
has performed much better than placebo in RCT's.
Hormonal agents such as bromocriptine and danazol may be more effective. However, many
women discontinue these therapies due to adverse effects.
Rate question:

Next question

Comment on this question

All contents of this site are 2012 E-Medical Revision Ltd

http://www.emrcs.com/question/question.php?q=0

Terms and Conditions

Privacy policy

1/1

13/07/2015

Reference ranges

Previous

Question 4 of 8

Next

In patients with multiple endocrine neoplasia type IIb which of the following clinical
appearances is the patient most likely to display?

A. Acromegalic facies
B. Turners type features

Question stats

End and review

Score: 0%
1

13.1%

6.9%

7.7%

10.9%

61.5%

61.5% of users answered this


question correctly

C. Profound kyphoscoliosis
D. Multiple bony exostoses
E. Marfanoid features

Search
Go

Next question

Patients with MEN IIb may display Marfanoid features. It is unclear at the present time
whether they have discrete changes in the microfibrils of elastic fibres that are present in
Marfans.
Multiple Endocrine Neoplasia
Multiple endocrine neoplasia (MEN) is inherited as an autosomal dominant disorder.
The table below summarises the three main types of MEN:
MEN type I

MEN type IIa

MEN type IIb

Mnemonic 'three P's':

Phaeochromocytoma
Medullary thyroid
cancer (70%)
Hyperparathyroidism
(60%)

Same as MEN IIa


with addition of:
Marfanoid body
habitus
Mucosal neuromas

RET oncogene
(chromosome 10)

RET oncogene
(chromosome 10)

Parathyroid (95%): Parathyroid adenoma


Pituitary (70%): Prolactinoma/ACTH/Growth
Hormone secreting adenoma
Pancreas (50%): Islet cell tumours/Zollinger
Ellison syndrome
also: Adrenal (adenoma) and thyroid (adenoma)
MENIN gene (chromosome 11)
Most common presentation = hypercalcaemia

Rate question:

Next question

Comment on this question

All contents of this site are 2012 E-Medical Revision Ltd

http://www.emrcs.com/question/question.php?q=0

Terms and Conditions

Privacy policy

1/1

13/07/2015

Reference ranges

Previous

Question 5 of 8

Next

A 32 year old man undergoes an appendicectomy. A large carcinoid tumour is identified and
a completion right hemicolectomy is performed. He is well for several months and then
develops symptoms of palpitations and facial flushing. Which of the following diagnostic
markers should be requested?

Question stats

End and review

Score: 0%
1

7.9%

63.4%

8.3%

14%

6.4%

63.4% of users answered this


question correctly

A. Alpha feto protein


B. Urinary 5-Hydroxyindoleacetic acid measurements
C. Urinary catecholamines

Search

D. Urinary VMA measurements


E. None of the above

Go

Next question

5 HIAA is the most commonly used diagnostic marker for carcinoid syndrome, it is measured
in a 24 hour urine collection.
Carcinoid syndrome

Carcinoid tumours secrete serotonin


Originate in neuroendocrine cells mainly in the intestine (midgut-distal ileum/appendix)
Can occur in the rectum, bronchi
Hormonal symptoms mainly occur when disease spreads outside the bowel

Clinical features
Onset: years
Flushing face
Palpitations
Pulmonary valve stenosis and tricuspid regurgitation causing dyspnoea
Asthma
Severe diarrhoea (secretory, persists despite fasting)

Investigation
5-HIAA in a 24-hour urine collection
Somatostatin receptor scintigraphy
CT scan
Blood testing for chromogranin A

Treatment
Octreotide
Surgical removal

Rate question:

Next question

Comment on this question

All contents of this site are 2012 E-Medical Revision Ltd

http://www.emrcs.com/question/question.php?q=0

Terms and Conditions

Privacy policy

1/1

13/07/2015

Reference ranges

Previous

Question 6 of 8

Next

A 20 year old male is referred to the clinic. He has undergone genetic testing because his
father died from colorectal cancer at the age of 21. His testing revealed a mutation of the
APC gene. A colonoscopy is proposed. What is the most likely finding?

Question stats

Score: 16.7%
1

15.2%

9.5%

7.9%

38.8%

28.6%

A. Multiple colonic hamartomas


38.8% of users answered this
question correctly

B. Carpet villous adenoma of the rectum

End and review

C. Caecal carcinoma
D. Multiple colonic adenomas
Search

E. Multiple colonic hyperplastic polyps

Go

Next question

APC mutations are found in familial adenomatous polyposis coli. These have multiple colonic
adenomas.
Polyposis syndromes

Syndrome

Genetic defect

Features

Screening and
management

Associated
disorders

Familial
adenomatous
polyposis

Mutation of APC
gene (80%)
cases, dominant

Typically over
100 colonic
adenomas
Cancer risk of
100%
20% are new
mutations

If known to be at
risk then predictive
genetic testing as
teenager
Annual flexible
sigmoidoscopy from
15 years
If no polyps found
then 5 yearly
colonoscopy started
at age 20
Polyps found =
resectional surgery
(resection and
pouch Vs sub total
colectomy and IRA)

Gastric fundal
polyps (50%).
Duodenal
polyps 90%.
If severe
duodenal
polyposis
cancer risk of
30% at 10
years.
Abdominal
desmoid
tumours.

MYH
associated
polyposis

Biallelic mutation
of mut Y human
homologue (MYH)
on chromosome
1p, recessive

Multiple colonic
polyps
Later onset right
sided cancers
more common
than in FAP
100% cancer
risk by age 60

Once identified
resection and
ileoanal pouch
reconstruction is
recommended
Attenuated
phenotype - regular
colonoscopy

Duodenal
polyposis in
30%
Associated
with increased
risk of breast
cancer (self
examination)

Peutz Jeghers
syndrome

STK11 (LKB1)
mutation on
chromosome 19 in
some (but not all)
cases, dominant

Multiple benign
intestinal
hamartomas
Episodic
obstruction and
intussceception
Increased risk
of GI cancers
(colorectal
cancer 20%,
gastric 5%)
Increased risk
of breast,
ovarian, cervical
pancreatic and
testicular
cancers

Annual examination
Pan intestinal
endoscopy every 23 years

Malignancies
at other sites
Classical
pigmentation
pattern

Cowden
disease

Mutation of PTEN
gene on
chromosome
10q22, dominant

Macrocephaly
Multiple
intestinal
hamartomas
Multiple
trichilemmomas
89% risk of
cancer at any
site
16% risk of
colorectal
cancer

Targeted
individualised
screening

Breast cancer
(81% risk)
Thyroid
cancer and
non toxic
goitre
Uterine
cancer

http://www.emrcs.com/question/question.php?q=0

1/2

13/07/2015

HNPCC
(Lynch
syndrome)

Germline
mutations of DNA
mismatch repair
genes

Rate question:

Colo rectal
cancer 30-70%
Endometrial
cancer 30-70%
Gastric cancer
5-10%
Scanty colonic
polyps may be
present
Colonic tumours
likely to be right
sided and
mucinous

Colonoscopy every
1-2 years from age
25
Consideration of
prophylactic surgery
Extra colonic
surveillance
recommended

Extra colonic
cancers

Next question

Comment on this question

All contents of this site are 2012 E-Medical Revision Ltd

http://www.emrcs.com/question/question.php?q=0

Terms and Conditions

Privacy policy

2/2

13/07/2015

Reference ranges

Previous

Question 7 of 8

Next

Which of the following breast tumours is most commonly associated with a risk of metastasis
to the contralateral breast?

A. Invasive ductal carcinoma


B. Invasive lobular carcinoma

Question stats

Score: 28.6%
1

17.9%

55.3%

11.3%

8.4%

7.2%

55.3% of users answered this


question correctly

C. Phyllodes tumour

End and review

6
7

D. Pagets disease of the breast


E. Atypical ductal hyperplasia

Search
Go

Next question

Risk of metastasis to the contralateral breast is a classical feature of invasive lobular


carcinoma.
Lobular carcinoma of the breast
Lobular breast cancers are less common than their ductal counterparts. They typically
present differently, the mass is usually more diffuse and less obvious on the usual imaging
modalities of ultrasound and mammography. This is significant since the disease may be
understaged resulting in inadequate treatment when wide local excision is undertaken.
In women with invasive lobular carcinoma it is usually safest to perform an MRI scan of the
breast, if breast conserving surgery is planned.
Lobular carcinomas are also more likely to be multifocal and metastasise to the contralateral
breast.
Lobular carcinoma in situ is occasionally diagnosed incidentally on core biopsies. Unlike
DCIS, lobular carcinoma in situ is far less strongly associated with foci of invasion and is
usually managed by close monitoring.
Rate question:

Next question

Comment on this question

All contents of this site are 2012 E-Medical Revision Ltd

http://www.emrcs.com/question/question.php?q=0

Terms and Conditions

Privacy policy

1/1

13/07/2015

Reference ranges

Previous

Question 8 of 8

Question stats

End and review

Score: 25%
1

7.9%

5.7%

9.4%

A. Stipple cell

71.2%

B. Tear drop cell

5.7%

C. Reticulocytes

71.2% of users answered this


question correctly

Which is the characteristic finding on a blood film post splenectomy?

D. Howell-Jolly bodies
E. Schistocyte

6
7
8

Search

Blood film in hyposplenism:

Go

Howell-Jolly bodies
Pappenheimer bodies
Poikilocytes (Target cells)
Erythrocyte containing siderotic granules
Heinz bodies
Splenectomy
Indications
Trauma: 1/4 are iatrogenic
Spontaneous rupture: EBV
Hypersplenism: hereditary spherocytosis or elliptocytosis etc
Malignancy: lymphoma or leukaemia
Splenic cysts, hydatid cysts, splenic abscesses

Splenectomy
Technique
Trauma
GA
Long midline incision
If time permits insert a self retaining retractor (e.g. Balfour/ omnitract)
Large amount of free blood is usually present. Pack all 4 quadrants of the abdomen.
Allow the anaesthetist to 'catch up'
Remove the packs and assess the viability of the spleen. Hilar injuries and extensive
parenchymal lacerations will usually require splenectomy.
Divide the short gastric vessels and ligate them.
Clamp the splenic artery and vein. Two clamps on the patient side are better and allow
for double ligation and serve as a safety net if your assistant does not release the
clamp smoothly.
Be careful not to damage the tail of the pancreas, if you do then this will need to be
formally removed and the pancreatic duct closed.
Wash out the abdomen and place a tube drain to the splenic bed.
Some surgeons implant a portion of spleen into the omentum, whether you decide to
do this is a matter of personal choice.
Post operatively the patient will require prophylactic penicillin V and pneumococcal
vaccine.

Elective
Elective splenectomy is a very different operation from that performed in the emergency
setting. The spleen is often large (sometimes massive). Most cases can be performed
laparoscopically. The spleen will often be macerated inside a specimen bag to facilitate
extraction.
Complications
Haemorrhage (may be early and either from short gastrics or splenic hilar vessels
Pancreatic fistula (from iatrogenic damage to pancreatic tail)
Thrombocytosis: prophylactic aspirin
Encapsulated bacteria infection e.g. Strep. pneumoniae, Haemophilus influenzae and
Neisseria meningitidis

Post splenectomy changes


Platelets will rise first (therefore in ITP should be given after splenic artery clamped)
Blood film will change over following weeks, Howell Jolly bodies will appear
Other blood film changes include target cells and Pappenheimer bodies
http://www.emrcs.com/question/question.php?q=0

1/2

13/07/2015

Other blood film changes include target cells and Pappenheimer bodies
Increased risk of post splenectomy sepsis, therefore prophylactic antibiotics and
pneumococcal vaccine should be given.

Post splenectomy sepsis


Typically occurs with encapsulated organisms
Opsonisation occurs but then not recognised

Rate question:
Comment on this question

All contents of this site are 2012 E-Medical Revision Ltd

http://www.emrcs.com/question/question.php?q=0

Terms and Conditions

Privacy policy

2/2

Das könnte Ihnen auch gefallen